You are on page 1of 496

MIR

PUBL1SHF..RS

r. c.
C. M.

c.

OeMU008U'l" B. A
Et/JuMeHf(O,
Jl JlyHU,. E cP nOpwHeea, E
Cbllieea,
B. et>PO/lOB, P. fl. lllocmaK, A. P. f/nnoAbcKuiJ.

napaHeHK08.. B.

KOcUH,

n.

3AJlAlIH 11 ynPA)I(HEHI1S1

no
MATEMATHLJECKOMY
AHAJll13Y

nOG penaKt4lJl't1
B.

n. A

E M 1-1

A 0 B M q A

rocyiJapcmseHHoe U3aameAbCmpo
AU mepDlIlYPbl

t/JU3UKO-ManteMamU'l.eCKOil

OCK.6Q

G. Baranenkov, B. Drm;dovich V. Epmenko, S. Kogan,


G. Lunt~) E. Por~hnlva, E. ~!/(:h(ta, S. fl0lCJv, R. ~ho~takt
A. Yanpolsky

PROBLEMS
IN
MATHEMATICAL
ANALYSIS
Under the editorship

of

B. DEMI DOVICH
Translated /rorn the Russian.
by
G. YANKOVSKY

MI~

PUBLISHERS
Moscow

TO THE READER

M/R Publishers would be glad to have yo"r


opinion of the translatton and the design of this
book.
P lease send your suggest tons to 2. Pervy Rlzh~kg
Pereulok, Mo~(.ow, u. S. S. R.

Second PrintiRR

Printed In

the Union

of

Soviet Socialist

Republics

CONTENTS
Preface .

Cizapter 1. INTRODUCTION TO ANALYSIS


Sec.
Sec.
Sec.
Sec.
Sec.

1. Pu nctions

2
3
4
5.

. . . . . .
Graphs of Elementary Functions
Li mits
.
Infinitely Small and Large Quantities.
Continuity of Functions . . . . . . . . . . .

11
16

22
33

36

Chapter II DIFFERENTIATION Of FUNCTIONS


Sec 1. Calculating Derivatives Directly . . . . . . . . .
Sec 2 Tabular Differentiation
Sec. 3 The Dprivat'ves of Functions Not Represented Explicitly
Sec. 4. Geometrical and Mechanical Applications of the Derivative .
Sec 5 DeflvatlveC) of Hig ler Orders
.....
Sec 6 DifTerE:'ntials of First and Higher Orders
Sec 7 Mean Value Theorems .
Sec. 8 Taylor'\) Formula
Sec 9 The L'Hospital-Bernoulli Rule for Evaluating Indeterminate
Forms
,.
. .....

42
46
56
60
66

11
15
17
78

Chapter III THE EXTREMA OF A FUNCTION AND THE GEOMETRIC


APPLICATIONS OF A DER IVATIVE
Sec.
Sec.
Sec
Sec
Sec.

1. The Extrema of a Function of One Argument

2
3
4.
5.

Chapter IV
Sec. 1
Sec 2
Sec 3
Sec. 4
Sec. 5.

83
The Dirfct ion of Concavity Points of Inflection
91
AsymptoteC)
. .
93
Graph Ing Functions by Characteristic Points
96
DilTerential of an Arc Curvature . .
. 101
I~DEFINI"E

INTEGRALS

Direct lntellration . .
by Substitution
Integrat 1 0n by Part~
. . . . .
Standard Integral'\ ContaininR a Quadratic Trinornlal
Integration of Rational Functions

Inte~ration

.
.
.

107
113
116

118
121

Contents
Sec. 6. Init'grating Certain IrratIonal Functions
125
~c 7. lr;t~rating 1 rifonrrretric Functions . .
128
~ec. 8 Integrat ion of H Yferbolic Functions . . .
. . 133
Sec 9. USIng I ngonometric and Hyperbolic Substitutions for Finding

Integrals of the Form ~ R (x. Vaxl+bx+cl dx. Where R


tional
Sec 10
Sec 11
Sec. 12.

FunctIon . . . . . . . . . . . . . . . . . . . .
I ntf,:rat ion of Varlouc; Transcendental Functions
Using Reduction Formulas.
.....
MIscellaneous Examples on Integration .

IS

a Ra133

135
135
136

Chapter V DEFINITE INTEGRALS


Sec. 1. The Denn ite I ntegral as the Limit of a Sunl . 138
~c 2 Evaluatll";~ [(tirite Intfgrals by N.eans of lndeflniteIntegrals 140
Sec. 3 Improper Integrals . . . . . . . . . . .
143
Sec 4 Charge ~f Vari2ble in a Dffinite Integral
146
~ec. 5. In1f~ration by Parts . . . .
149
Sec 6 Mean- Value Theort'm . .
150
Sec. 7. The Areas of Plant' Fi~ures
153
Sec 8. The Arc Length of a Curve
158
Sec 9 Volumes of Sol ids . . . .
161
5ec 10 The Area of a ~urface of RE'volution
166
Sec 11 Norrfnts Centres of Cravity CJuldln's Thforems
168
~ec 12. Applyir.g [ef!r.ite Intgrals to the Solution of Physical Prcblems . . . . .
. . 173

Chapter VI. FUNCTIONS OF SEVERAL VARIABLES


Sec.
Sec.
Sec
Sec
Sec
Sec.
Stc.
Sec
Sec
Sec
Sec.
Sec
Sec.
Sec
Sec
Sec
Sec.

1. Basic Notions
2. Contlnulty

3
4
5
6.

180

P.artlal Derivatives
Total DlfJerential of a Function
...
Dlffprpntiation of Comroslte F-unctions

Dt-f1Vatlve In a GIven DirtAct10n and the Gradient of a Function


7 H I~lfel -Crder Cenvat lVfS and Differentials
8 I ntegration of 10tal D;f;erentials

9 Dif.erentiation of I mpllclt functions.

10 Chanf!e of Variables . . . . . . . . .

11. The Tangent Plant' and the Normal to a Surface


12 1 aylor'~ Formula for a F-un('tion of 5pveral Variables
13 The Ex tremum of a FunctIon of Several Variables

14 Fir dlf:~ 1ht- Createst and c. n~allt"st Values of Functions


15 Slnl'ular POInts ot Plane Curves
16 Envelope
.

17. Arc Length of a Space Curve ,


.

184
185

181
190
193
197

202
205
211
217
220
222
227
230

2.32
234

Contents
S~c. 18. The Vector Function of a Scalar Argument
Sec. 19 The Natural Trihedrnn of a Space Curve
Sec. 20. Curvature and Torsion of a Space Curve

7
2JS
238

242

Chapter VII. MULTIPLE AND LINE INTEGRALS


Sec. 1 The Double Integral in Rectangular Coordinates .
. . 246
Sec. 2 Chan~e of Variables in a Double Integral
252
Sec. 3. Com pu ti ng Areas .
256
Sec. 4. Computing Volumes . . .
258
Sec. 5. Computing the Areas of Surfac~s . . . .
259
Sec. 6 Applications of the Double Integral in Mechanics
230
Sec. 7. Triple Integrals
.
262
Sec. 8. Improper Integrals Dependent on a Paralneter. Improper
Multi lIe Integrals. .
269
Sec. 9 Line Integrals . . . . .
273
Sec. 10. Surface Integrals . . . . . .
284
Sec. 11. The Ostrogradsky-Gauss Formula
286
Sec. 12. Fundamentals of Field Theory
288

Chapter VIII. SERIES


Sec.
Sec.
Sec.
Sec.

1. Number Series
2. Functional Series
3. Taylor's Series
4. Fourier's Series

293

304
. . . . . 311
318

Chapter IX DIFFERENTIAL EQUATIONS


Sec. 1. Verifying Solutions. Forming Differential Equations of FaOli
lies of Curves. Initial Conet itions . . . . . . . . . . . .
Sec. 2 First-Order Differential Equations . . . . .
Sec. 3. First-Order Diflerential Equations with Variables Separable.
Orthogonal Trajer.tories . . . . . . . . . . . . . . . . .
Sec. 4 First-Order Homo~eneous Differential Equ:Itlons . .
Sec. 5. First-Order Linear
Differential
Equations.
Bernoulli's
Equation . . . . . . . . . . . . . . . . . .
Sec. 6 Exact Differential Equations. Integrating Factor . .
Sec 7 First-Ordpr DIfferentIal Equations not Solved for the Derivative
Sec. 8. The Lagrange and Clairaut Equations
.
Sec. 9. Miscellaneous Exercises on First-Order Differential Equations
Sec. 10. I-ligher-Order Differential Equations .
Sec. 11. Linear Differential Equations . . .
Sec. 12. LInear Differential Equations of Second Order with Constant
Coefficients

322
324
327
330
3::12
335

337
339
340
345

349
351

Contents

Sec. 13. Linear Differential Equations of Order Higher than Two


with Constant CoefficIents
.....
. .
Sec 14. Euler's Equations . . . . . . . .
Sec 15. Systems of Diff'l'rentlal Equa tions . . . . . . . . . . . .
Sec. 16. Integration of Dit'ien'ntial Equations by Means of Power Series . . . . . . . . . . .
Sec 17. Problenls on Fourier's Method

356
357
359
361

363

Chapter X. APPROXIMATE CALCULATIONS


Sec. 1 Operations on Approxarnatc NUlllbers
Sec. 2. Interpolation of Functions . . . . .
Sec. 3. ConlputtT1f.! thc,Rcnl Roots of EquCltions .
~ec. 4 NUlllprital, In tet!ration of FUllct ions . . . . . .
Sec. 5. f\un encal Inte~'r(ltion of Crdlrary Dtf'1t:rtntial
Sec. 6. ApprO}dITlating P<. tlfllr'S Ccefficlents

.
.

. . "

Equations
.

367
372


APPENDIX . . . . . .

I. Grrcl< A1ph a bpt .

11. SOlne COl1~tants .

III. Inverse Quantities, Powers, Roots, Lo~arithlns .

IV Trigollollletric Funct ton~ . .


.
V. EXrorel~ tIaI, 11 y ~erbolic and Trigon01l1etric Functions
VI. Some Curves . . . . . . . . . . . .

396

ANSWERS

.....

376
382
384
3Q3

475
475
475

476
478
479

480

PREFACE
This collection of problems and exercises in mathematical analysis covers the maximum requirements of general courses in
higher mathematics for higher technical schools. It contains over
3,000 problems sequentially arranged in (~hapters I to X covering
all branches of higher mathematics (with the exception of analytical geometry) given in college courses. Particular attention is
given to the most important sections of the course that require
established skills (the finding of limits, differentiation techniques,
the graphing of functions, integration techniques, the applications
of definite integrals, series, the sol ut ion of differential equations).
Since some institutes have extended courses of mathematics,
the authors have included problems on field theory, the Fourier
method, and approxima1e calculations. Experience sho\vs that
the nunlber of problclns given in this book not only fully satisfies
the requirclT:n s of the student, as far as practical 11las~ering of
the various sections of the course goes, but also enables the instructor to supply a varied choIce of problems in each section
and to select problenls for tests and examinations.
Each chap.er begins with a brief theoretical introduction that
covers the basic definitions and formulas of that section of the
course. Here the rnost inlportant typical problems are worked out
in full. We beli('ve that this will greatly Silllplify the work of
the student. Answers are given to all computational problems;
one asterisk indicate~ that hints to the solution are given in
the answers, two asterisks, that the solution is given. The
problems are frequently illustrated by drawings.
This collection of problems is the result of many years of
teaching higher mathelnatics in the technical schools of the Soviet
Union. It includes, in addition to original problems and exam
pies, a large nunlber of commonly used problelTIs.

Chapter I

INTRODUCTION TO ANALYSIS

Sec. 1. Functions
1. Real nurrl:ers. Rational and irrational numbers are collectively known
1 he ab\olutf value of a real nurnber a IS undt'rstood to be
the nonnegative numb~r I a, defined by the conditions I a I = a if a ~ 0, and
la , = - a if a < o. lhe following in(qual1ty holds for all real numbers CJ

as

real numbers

and b:

'a+bl~lal+'bl.

2. Definition of a function. If to every val ue *) of a variable x, wh ich


belongs to SOllie collect Ion (set) E, there corresponds one and onl y one finite
value of the quantity y, 1hen y is said to be a funcllon (~lnRle-valued) of x
or a dependent 1 artable def! ned on the ~et E, x is the argument or tndependent vanable The fact that II IS a functIon of x IS expressed In brief form
by the notation y = f (x) or y == F (.\), and the I'ke
If to every value of x belonging to somt set E there corresponds one or
several values of the vanabl~ !i, then y is called a multIple-valued functton
of x defined on E. Fronl now on we shall u~e the \\ford "functIon" only in
the meaning of a ~ln[!./e-vQlupd functIon, 11 not other\vl~e stated
3 The domain of deltnition of a function. The collection of values of x for
which the gIven functlon IS dt.\Hned IS called the dOTnaln 01 de{tTulton (or the
domain) of thiS function. In the slnlplest cases, the' domaIn of a function IS
either a clofied Interval [a, bl, which is the set of n~al nUlnbprs x that satisfy
the inequal itJes a ~ ~ ~ b, or an open tnterval (a. b). wh Ich :s the set of real
nuolhers that satisfy the In('quallLes a < x < b. AI~o pOSSible IS a ITIOre complex structure of the dOllJ3111 01 defInition of 3 function (see, for Instance, Probleln 21)
Exanlple 1. Determine the donlaln of definItion of the funchon
I

Y=

V x2-1 .

Solution. The function is defined if


x 2 -1

> 0,

that is, if , x I> 1. Thus, the domaIn of the function is a set of two intervals: - 00 < x < - I ~nd I < x < + 00
4. Inverse functions. If the equation y = f (x) may be solved unIquely for
the variable x, that is, if there is a function x =g (y) such that y == f [g (y,
*) Hencelorth all values will be conSIdered

stated.

8S

real, if not otherwise

Introduction to Analysis

12

(Ch. 1

then the function x = g (y), or. in standard notation, y =g (x), is the tnverse
of Y=f(x). Obviou~lv. g(f(x))=:x, that is, the function ,(x) is the Inverse
of g (x) (a nd v ice versa).
In 11 r J ereJ al ca~e, tre rquation y = f (x) refines a multiple-valued Inver~e fupct Ion x = f- J (y) ~uch that y == f If-) (y)J for all Y that are values of
the function f (x)
I:xhn Jjle 2. Cetennine the inverse of the funchon
y= 1-2- x .

(1)

Solution. Solving equation (1) for x, we have


2- x =1-y
and
x=-

IO~~~;Y) *).

(2)

tl-'e dotr'ain of cefinition of the function (2) Is - co < Y < 1.


5. Corrros it e and ireplicit functicns. A function Y of x defined by a ~e
ries of equalit1t~sy=f(u). whereu=cp(x), etc., is called a comoosite function,
or a function of a fun~tiol,.
A function dfined by an fqu~tion not solved for the derencent variable
is calh.:o an ImplIcit function. For xam~le, the equation x' + yl= 1 defines
y as an implicIt function of x.
6. The graph of a function. A set of poi nts (x, y) in an xy-plane, whose
coordinates are connectEd by the equation y
(x), is caIJed the graph of
the given funct~on.
Obviou~ly,

=,

1**. Prove that if a and b are real numbers then

Ilal-l b II ~ la-b I ~ lal+lb (.


2. Prove the following equalities:
a) Iab I = I a 11 b I;
b)

laI =a
2

2
;

c)

I~ I= :::

d)

Vi2=-.:\al.

(b =1= 0);

3. Solve the inequalities:


a) Ix-II<3; c) 12x+ 11< I;
b) Ix--t-1/>2; d) Ix-II<lx+ll.
4. Find /(-1), [(0),1(1), {(2), 1(3), {(4), if f(x)=x ' -6xl +
Ilx-6.
5. Find 1(0), I( - ~), {(-x),/( ~), f~X)' if l(x)=V1+x 2

6. f (x) = arc cos (log x). Find f (~ ), f (1), 1(10).


7. The function {(x) is linear. Find this function, if /(-1)=2
and f (2) = - 3.
) Log x is the logarithm of the number x 10 the base 10.

Functions

Sec 1)

13

8. Find the rational integral function f (x) 01 degree two, if


f(O)==1, /(1)=0 and f(3)=5.
9. Given that f (4) == - 2, f (5) = 6. Approximate the value of
1(4, 3) if we consider the function J (x) on the interval 4 ~ x ~ 5
linear (linear interpolation 01 a function).
10. Write the function

f (x) =

{ 0,

~f x ~ 0,

x, If x>O

as a single formula using the absolute-value sign.


Determine the domains of definition of the following functions:
11. a) Y=

12.

Vx+ 1;

16. Y=

b) Y= ~ x+ 1.
Y-=4--2'

17. Y= log ~+~

V -V

18. y==log

13. a) Y =
x 2 -2;
2
b) y~x x -2.

14**. y=V2+x-x

3 +2
x - xl
.
x-l2

2x

19. y=arccos l+x'


l

1
Y= V -x+ Y2+x'

15.

V X-Xl.

X)

20. Y= arc SIn log 10

21. Determine the domain of definition of the function

Y= Vsin2x.

22. f(x)=2x -3x ' -5x 2 +6x-l0. Find


4

q> (x)

= 2" (I (x) -t- I(- x) ) and

'1'( x) = 2" [/ (x) - f ( - x) J.

f (x) defined in a symmetric region - l < x < I


f (- x) = I (x) and odd if f (-x) == - I (x).

23. A function

is called even if

Determine which of the following functions are even and which


are odd:
a) f (x) = ~ (a~ +a- X );
b) f (x) = VI

c) I(x)=
d)

+ x +x'- Vl-x +x

V(x+ 1)1+ V(x-I)I;

l+x
f (x) = log -1-x;

e) l(x)=log(x+Vl+x

J
).

24. Prove that any function f (x) defined in the interval


- l < x < 1 may be represented in the fOfln of the SUln of an
even function and an odd function.

(Ch. I

Introduction to Analysis

14

25. Prove that the product of two even functions or of two odd
functions is an even fl'nctlon, and that the product of an even
function by an odd function is an odd function.
26. A function f (x) IS called periodic if there exists a positive
numter T (the period of the tunction) such that f (x + T) = f (x)
for all valres of x within the de main of definition of f (x).
Ce1ermine \\hirh of the rol1ewing functions are perIodic. and
for tLe perlcdic ft:ncticns flnd tt.eir least period T:
a)

b)
c)

f (x) = 10 sin 3 x,
f (x) = a sin AX + b cos AX;
f (x) = Vtan x;

d)

f (x) =

e) / (x)

sin l x;

= sin (VX).

27. Express the length of the segment y.= M N and the area S
of the figure AA1N as a fun~tl0n of x= AM (Fig 1). Construct
the graphs of these functions.
D
C
28. The linear density (that is,
mass per unit length) of a rod AB = I
b (Fig. 2) on the segments AC = I"
CD == 12 and DB == la (L \ + I" + La ==- l)
A
is equal to q J' q 2 and q a' respec/3

c
~--a--~

Fig. 2

Fig. 1

tively. Express the mass m of a variable segn;ent AM =X of this


rod as a function of x. Construct the graph of this function.
29. Find q;'W(x)) and ;q;(x)), if <p(X)=K and 'I'(x)=2 x .
30. Find f tf If (xlI}, if f (x) = 1 ~x
31. Find f(x+ 1), if f(x-l)=x l
32. Let f (n) be the sum of n terms of an arithmetic progression.

Show that

/ (n + 3)-3/ (n

+ 2) + 3/ (n,- 1)-1 (n) = O.

33. Show that if

f (x) =kx+ b
and the numbers
the nurn ters

gression.

Xl' XI'

J (Xl)'

x. form an arithmetic progression, then

/ (XI) and

f (XI) likewise form such a

pro-

Sec. J]

Functions

15

f (x) is an exponential function, that is,


and the numters x" XI' XI form an arithmetic
progression, then the numbers f (Xl)' f (x 2 ) and {(x,) form a geometric progression.
35. Let
34. Prove that if

f (x) = aX (a > 0),

l+x
f(x)= log .
i-x

Show that

f (x) + f (y) = f ( ;:~)


1

36. Let cp (x) = 2" (aX + a-X) and

Show that

'P (x) = 21 (aX _a-X).

rp (x + y) = cp (x) <p (y) + 'P (x) '" (y)

and

'" (x

+ y) == cp (x) 'I' (y) +- cp (y) 'I' (x).

37. Find /(-1), {(O), f(1) if

f (x) =

arc sin x for-t ~ t" ~ 0,


arctanxforO<x ':+00.

38. Determine the roots (zeros) of the region of positivity and


of the region of negativity of the function y if:
a) y == I -t- x;
d) y = x - 3x;
b) y = 2 -t- x - Xl ;
2x
e) y- Iog-c) y = I-x -t-

Xl;

1 +x

39. F:nd the inverse of the function y if:


a) y = 2x + 3;
b) y=x 2 -1;
.i/-l- 3 .
c ) Y=v - X ,

d) y = log ~ ;
2

c) y=arctan3x.

In what regions will these inverse functions be defined?


40. Find the inverse of the function
I X, if x~O,
y =) Xl, if x >

o.

41. Write the given functions as a series of equalities each


member of which contains a simple elementary function (po\ver;
exponential, trIgonometric, and the like):
a) y = (2x-5)10;
b) y = 2cos x;

c) y = log tan 2 ;
d) y = arc sin (3 -X2 ).

I_n_t,_od_u_c_t,_on_t_o_A_n_a~ly:-.s_,s

16

.....:[::.-C_h_._I

42. Write as a single equation the composite functions represented as a series of equalities:
8) y=U', U= sin x;
b) y=arctanu,
v=logx;
c) = { 2u, if u ~ 0,
y
0, if u>O;

u=Vv,

tt=x'-l.
43. Write, explicitly, functions of y defined by the equations:
a)
arc cos y =---1(,;
b) lOx + 1()Y = 10;

r-

c)

x+1YI=2y.

Find the domains of definition of the given implicit functions.


Sec. 2. Graphs of Elementary Functions
Graphs of functions Y= f (x) are l11ajnly constructed by marking a sufficiently dtln~e net of points Nli(xit I/i), where Y,=f(xj) (i==-O, 1,2, ... ) and
by connecting the pOInts with a line that takes account of intermediate pOints.
Calculations are best done by a ~ltde rule.

-... ... ......


. . ..........y,

...

Fig. 3
Graphs of the basic eleJnentary functions (see Ap pendix VI) are readily
learned through their construction. Proceeding from the graph of

Y=f (x),

(f)

we get the graphs of the following functions by means of simple geometric


construct ions:
1) YI = - f (x) is the mirror image of the graph r about the x-axis;
2) !I. = f (-I') is the mirror image of the graph r about the y-axis;

Graphs of Elementary P'J.nct,oni

Sec. 2)

17

3) Y. = f (x-a) Is the r graph displaced along th? x-axis by an amount a;

4) Y4 = b I (x) is the r graph displaced along the y-axis by an amount"


(Fig. 3).
Example. Construct the graph of the function

y = sin ( x - : ) .

Solution. The desired line Is a sine curve g = sin x displaced along the x-axis
to the: right by an amount

(Fig. 4)

x
Fig. 4

Construct the graphs of the

following

linear functions

(straight I inps):

44. y=kx, if k=O, I) 2,1/2, -1, -2.


45. Ij=x+ b, if b=O, 1,2, -1, -2.
46. y = 1.5'x -}- 2.
Construct the graphs of rat ional integral fun.tions of degree
two (parabolas).
47. y=ax 2 , if Q= 1,2, 1/2, -I, -2, O.
48. Y = x 2 + C, if c = 0, 1) 2, - 1.
49. y=(X-x o )2, ii xo=O, 1,2, -1.
50. Y = Yo + (x - 1)2, if Yo = 0, 1, 2, - I.
51*. y=axl-~-bx+c, if: I) a=l, b=-2, c=3; 2) a=-2,
b=6, c=O.
52. y = 2 t- x-x2 Fi nd the points ot intersect ion of th is parabola with the x-axis.
Construct the graphs of the following rational integral tunction~ of degree above two:
53*. Y = x 3 (cub ic parabola).
54. y=2+-(x-I)'.
55. y = x'- 3x -t- 2.
56. Y = x 4

57. y=2x l -x4..


Construct the graphs of the following linear fractional functions (hyperbolas):
68*,

y=+,

Intr,duct,on to Analysis

18
1

59. y= I-x.
x-2

60. y= x+2.

61*. y=y o -+.. x~,


if xo=l, yo::::a-l, m=6.
X

2x-3
62 * Y=3x+2"

Construct the graphs of the fractional rational functions:


1
63. y=x+-.
x

65*.

x2

+ 1
!I = ;.

64. y= x

66. y=
x .

~.l~ 1 (W itch of Agnesi).


68. y = x.2~ 1 (Newton's serpentine).
1
69. y= x+ 2x .
70. y = Xl +!.
(trident of Newton).
x
67*. Y =

Construct the graphs of the irrational functions:


71*. y=
J/72. '1= v x.
73*. y= ViZ (Niele's parabola).
74. y= x
(c;emicubical parabola).

Vi

VX-

75*. y.

76. y=

f V25-x

V xZ-l

(elltpse).

(hyperbola).

77. y= V 1- Xl
78*. y=

Vb

(cis.<;oid of Diodes).

79. y= xV 25-x 2
Conslruct the graphs of the trigonometric functions:
80*. Y = sin x.
83*. y =- cot x.
81*. y=cosx.
84*. y=sec x.
82*. y=-tanx.
85*. y=cosec x.
86. y= A sin x, if A = 1, 10, 1/2, -2.
87*. y= sin nx, if n= 1, 2, 3, 1/2.
88. y=sin(x-q, if q>=O, ~,
n, - : .
89*. y = 5 sin (2x-3).

3;,

[eh. 1

Sec. 21

Graphs of Elementary ftlnctions

19

90*. Y= a sin x+ b COS X, if Q= 6, b -= -8.


91. Y = sin x -1- cos x.
96. y = 1-2 cos x.
92*, y=cos1x.
97. g= sinx-{ sin3x.
93*. Y= x+ sin x.

98. Y= cos x+ 2 cos 2x.


n

99*. Y = cos -x .
95. y = tan! x.
100. y = sin x.
Construct the graphs of the exponential and logarithmic runetions:
94*. Y = x sin x.

lOt. g=a", if a=2, ;, e(e=2, 718, .. )*).


102*. g= lo&z x, if a= 10. 2, ; , e.
103*. y=sinhx, where sinhx=lj2(eX-e-~.
104*. y=co~hx, where co~hx=1/2(ex+e-X).
sinh x

105*. Y = tanh x, where tan h x = -cos


h- .
x
1

106. Y= lOx.
J07*. Y = e- x2 (probab ility curve).
108. y=2-

109. y=logx 2
110. Y = logl x.
lIt. y=log(logx).

1
113. y=log-.
x

114. y=log(-x).
115. Y = log! (l + x).
116. y=log(cosx).
1
112. .l j = -og
l- .
117. y=2- x sinx.
x
Construct the graphs of the inverse trigonometric
lt8*. y=arc sinx.
122. y=arcsin...!...
x
I

119*. Y = arc cos x.


123. y=arccos-.
x
120*. y=arctanx.
124. y=x+arccotx.
121 *. y = arc cot x.
Construct the graphs of the functions:
125. Y=lxl.
1
126. y = 2 (x
x I)

+,

127. a) y=xlxl;
b) y=logv7Ixl.
128. a) y=sinx+lsinxl;
b) y=slnx-lsinxl.
3-X2 \vhen Ixf~ I.
129. y= { ,:, when Ixl> 1.
*) About the number e see p. 22 for more details.

functions~

Introduction to Anall/sis

20

(Ch. 1

130. a) y=[x], b) y=x-[x], where [xl is the In~egral part


of the nurnLer x, that is, the grea~est in:eger less than or equal
to x.

Constrt:ct the graphs of the following functions in the polar


coordina.e system (r, <p) (r ~ 0):
131. r = 1.

=!

132*. r
(spiral of Archimedes).
133*. r":= e<P (Iog'lrithmic spiral).
134*. r =!:. (hyperbol ic sptral).
q>

135. r = 2 cos 'P (cirlle).


136. r = _1- (straight line).
Sin

q>

137. r=sec! ~ (parabola).

13S*. r = 10 sin 3cp (three-leafed rose)


139*. r=a(l +-coscp) (a>O) (cardioid).
143*. ,1 = a l eos 2cp (a > 0) (lemniscate).

CJns~ruct

the graphs of the functions represented parametri-

cally:
141*. x = t a, y = ('
142*. X= 10 cos t,
143*. x = 10 cos' t,
144*. x=a(cost+

(sernicubical parabola).
y= sin t (ellipse).
Y = 10 sin a t (astroid).
t slnt), y=a(slnt-tcost) (involute of a

circle ).
145*.

X=l~tal

Y=l~:a (folium of Descartes).

. . I)
se"LtCtrc
e .
1
t
147. x~~t +2-', y=2 _2- (branch of a hyperbola).
149. x=2co~Zt, y=2s1n 2 t (segntent of a straight line).
149. x==t-t 2 , y=t 2 __ t a
150. x==-~J2cost-co~2t), y=a(2sint-sin2t) (cardIoid).
CJnstruct the graphs of the following functions defined implicitly:
t 51 *.Xl + yl = 25 (c i r (l e) .
15l. xy= 12 (hyperbGla).
153*0 yJ=2x (parabola).

146

rot

Ir
, Y=
r 1 + /2

y2

at

I + tl

154. 100+64=1 (elllPse).


155. y! = Xl ( 100 - X
Z

).

156*. x ... + y"3 = a3" (astroid).


151*. x+y=-lOlogy.
158.. Xl = co~ y.

Sec. 2)

2t

Graphs of Elementary Functions

VX -t- y" = earc tan .!.!.-x


Z

(logarlthmtc spiral).
160*. X 8 +y3_3x!J=0 (folium of Descartes).
161. Derive the conversion fornlula trom the Celsius scale (C)
to the Fahrenheit scale (F) if it is known that OC corresponds
to 32P and 1000e corresponds to 212F.
Construct the graph of the function obtained.
162. Inscribed in a triangle (base b == 10, altitude h = 6) is a
rectangle (Fig. 5). Express the area of the rectangle y as a func-tion of the base x.
159*.

'--'-----~ 8

Fig. 5

FIg 6

Construct the graph of this function and find its greatest


value.
163. Given a triangle ACB with BC=a, AC=b and a variable
angle ?J ACB = x (Fig. 6).
Express y = area 6. ABC as a function of x. Plot the grapn
of this function and find its greatest value.
164. Give a graphic solution of the equations:
a) 2x"-5x-r2=O;

b) XS + x-I = 0;
c) logx=O.lx;

d) lO-x=x;
e) x = I -+ 0 5 sin x;
f) cot x=x (O<x<:t).

165. Solve the systems of equations graphically:


a) xy = 10, x -t y = 7;
b) xy = 6, XZ y" = 13;
c) x!-x+y=4, y!-2x=O;
d) x" y= 10, x+ y"= 6;

e) Y= sinx. y=cosx,

(O<x<2n).

Introd'lction to Analysis

(Ch. 1

Sec. 3. Limits
XI'

to. The limit of a sequence. The number a is the limit of


xJ ' . , x". . . . , or

tJ

sequence

lim xn=a,
n + C1J

If for any e > 0 there is a number N = N


Ixn-al<e when n>N.

(8) such that

Example 1. Show that


Ihn 2n

+ 1=2.

(I)

n+l

n-+ct)

Solution. Form the difference


2ft

+ 1_ 2 =

__
1_

n+1

n+l'

Evaluating the absolute value of this difference, \ve have:

2n 1
1
j n + 1 - 2 = n + 1 < e,

if

> -e

-1

=N

(2)

(e).

Thus, for every positive nUlnber e there will be a number N =.!-l such
e

that for n > N we will have irequality (2) Consequently, the number 2 is
the limit of the sequence x n = (2n-t- l)/(n + 1), henc~t formula (I) is true.
2. The limit of a function. We say that a function f (x) ...... A as x ~ a
(A and a are numbers), or
lim f (x):= A,
x -+a

if tor every e > 0 we have ~ = ~ (e) > 0 such that


If (x)- A I < e for 0 < I x-a I < ~.
Similarly:

if

I f (x) -

linl f (x) = A,
-+

00

A I < e for

The following conventional notation


lim

x ..... a

IS

f (x) =

I x I > tv (8).
also used:
CO,

which means that I f (x) I > E for 0 < , x-a I < f) (E), where E i~ an arbitrary
positive number
3. One-sided limits. If x < a and x -+ a, then we write conventionally
x -+ a-O; s!ml1arly, if x > a and x a, then we Wrtte x a +0. The numbers
f (a-O) = lim f (x) and J (a + 0) = lim f (x)
-P

a -- 0

-?

a+0

.are catted, respectively, the ltmlt on the left of the function f (x) at the point a
and thf limIt on the f,ght of the function f lX) at the point a (if these
nuolbers eXist).

23

Limits

Sec. 3)

For the existence of the linlit of a function f (X) as x


and sufficient to have the follcw lng equal ity:

a, it Is necessary

f (a-O) =:-f (a+O).


I f the linlits lim f 1 (x) and lim f I (x) exist, then the following theorems.
x-.a

x~a

1old:
1) lim [f. (x)
x~a

2)

lim [f. (x)


x~a

3)

litn

+ f2 (x)] =

'2 (xli

lim f. (x) + lim f2 (x);


x~a

x~a

lim ,. (x) lim


x~a

If. (x)! f 2 (x) J = lim f.

x~a

f" (x);

x~a

(x)! Brn

x~a

f 2 (x)

(lim

x~a

f 2 (X) :I: 0).

x~a

The following two limits are frequently used:


lim sin x = 1
-+

and
1

Jim (1
X-+(;I)

+..!..)X
=
X

Jim (1
-+0

+ ala =e=2 71828

..

Example 2. Find the limits on the right and left of the functioD
1

f (x) =arc tanx


as x

-+

o.

Solution. We have

'(+0)= lim (arctan


x ~ +0

and

-!..)=~
X
2

(aJctan..!..)=-~.

f(-O)= lim

x-+ - 0

Obviously. the function f (x) in this case has no limit as

-..0.

166. Prove that as n - + 00 the linlit of the sequence


1

1, 4

1
nz'
...

is equal to zero. For which values of n will we have the inequality


1
nz

<

(8 is an arbitrary positive number)?

Calculae numerically for a) 8=0.1; b) 8=0.01; c)


161. Prove that the limit of the sequence
n

x"=n+l (n=1,2, ... )

8=O.OOl~

24

Introduction to Analysis

(Cia. I

as n - t o 00 is unity. For which values of n> N will we have


the inequality

Ixn-ll< e

(e is an arbitrary positive number)?


Find N for a) 8=0.1; b) 8=0.01: c) 8=0.001.
168. Prove that

How should one choose, for a given positive number


positive number 6 so that the inequality

8,

some

Ix -41< 8
l

should follow from

Ix- 2 1<cS?

Compute 6 for a) 8=0.1; b) 8=0.01; c) 8=0.001.


169. Give the exact meaning of the following notations:
a) liin log x = - 00; b) lim 2x = + 00; c) Iinl f (x) = 00.
x

-~

+0

...

+ 00

00

170. Find the limits of the sequences:


1

a) 1, -"2' 3'
2

b)

(- I)n-l
n
, . .. ,

- 4-' ... ,

2n

T' "3' '5' ... , 2n=1' ... ;

V2. -( 2 Vf,
2 -.( 2 V'2,
d) 0.2 0.23, 0.233, 0.2333, ...

c)

'"

Find the limits:


(
171 Iun

1
2
3
2+2-12n
n

n~oo.n

n-l) .
+ ... +-21l

172. lirn (n+l)(n~2)(n+3) .


n

n~oo

173. liln

l!..:t3+5+7+"'+'(2n-I)_2n+IJ
n+l

n-+oo

174 I1m

n+(-I)n
-(-1)".
n-+oon

175. lim

n~oo

2n + 1 +3 1Z + 1
2n +,n
~

}i~ ( ~ + ~ + ~ + ... + ~ ) .

[
1
I
1
( _ 1) n - ']
177. 21~ 1-"3+ 9-27+ ... + 3"176.

178. lim 1 +2 +3 : n .... <


n

+n

Seco 3]

L_l_om_i_t.s

(V n + 1- Vn).

179. lim

n-fJ(J)

n "ill nl

-r-+
1
n

180. htll
n

-+J

When seekipg the lilnit of a ratio of two integra t polynomials In x as


x -+ 00, it is useful first to divide both ternlS of the ratio by x n , where n Is
the highest de~ree of these l\oly~omia!so
A si 1T1 ilar procedure is also possi ble in nlany cases for fractioos eontaiQfng irrational terms.
Example 1.

linl

(2x-3) (3t + 5) (4x-6)

3x'+x-1

5)( 4 - 6)
3+(2 - 3)(
x =2.3.4c= 8.
x'

-=-=- !ill)

3-~-~-~3

t+7fJ

x2

Example 2o

x
1IIII
x~rJ::3/~\3+10
V

181.
182.

1lIn
x~oo

(x+ 1)2

hIll

.r

=::

OCt

2 _L
I-

184. 11111
t -+-

rJ::

.t -+

-1- V x
J

x2

-~ oc 10

+x V "t

- +1

X2

x~oo

00

+1

189 1lIn V
--1-.

+5

3_8'\:

,.,

2.t+3

188. HIn

(2x 3)3 (3x-2)!


---5---

185. 11111

00

x + 00 x

2~2-x-1X

187 1lIn

x2-5t+ 1
183. 11111 - 3..l~7
X

2x~-3x-4
11In --;r -::---==4

x+ooX-

00

1--1->.J

lOOOx
I 1:11 -.2-1
~

186.

-1

3V-TO == .

-t- 5

x+

190.
t

Ii 111
~ too

Y:-==:x====~

-V x + V x -f

V-;-

If P (x) and Q (x) arp integral polynomials and P (0) '# 0 or <J
then the lilnlt of the rational fraction

lll)

:f: 0,

lim P (x)
t

~a

Q (x)

is obtained directly.
But if P (a) = Q (a) =0, then it is advisable to laJllel the binonllal

out of the fraction

~ ~:~

once or several times.

Example 3.

lIru
11

-+ ..tl

x -4
-

3~

+2

z::

lim (z-2) (x
. -+ 2

(x - 2) (x -

t- 2) =
1)

lim x +2 c::.ft
~ 2 x-I

Ai&a-Q

Int,oduction to Analysis

(Cho J

x'-3x+ 2
11m
195. X-+l~
4-4 x + 3 '

, x2_(a+
196 IlIn
3

x -a

x ..... a

197 I1m
h

-+

I) x+a
a

(X+h)'_X'
h

198. lim (I I - I 3 .).


x -.

The exrressions containing irrational terms are In many cases rational-

tzed by introducing a new variableo


Example 4. Find

lim Vf+X-I
.r~oVI+x_I

Solution. Putting
I +x=y',

"We have

Yf+X-l

Urn

199o 1lIn
0

-+ J

200. I1m
x

V" 1 + x-I

-+.

-"14

Vx-I

Jim

y:- 1=

Y-+ly-I

lim y!+Y+I=~.
y+1
2

Y~l

2O1. 11m

I'

xYx -8

X-+l

V-x -4

202. 1lIn

x-.

-x - I
V
V x-I
V-x + 1
V - (-2-1)2
Xl

Another way of finding the litnit of an irrational expression is to transfer the Irrational term froln the nUlnerator to the denominator, or vice versa,
froln the denominator to the numerator.
Example 5.

lim
~~Q

VX" - Va _
x-a

Um
x-a
-,
--%-+Q(x-a)(Yx+Ya)
lim

%-+aVx+Va

2- Vx-3
x2 - 4 9 '
204. I.TI x-h
x ... 8
x -2

203.

1m

-+,

205.

10

V-

Vx-I

1m ~--.

~-+IV..t-l

206.

Va

(a>

r1m 3-Y5+x
v--.

x-+41-

5-x

VI +x- Yl-x
207. lim
x
%-+0

208. lirn YX+hh,


11-+0

VX .

0),

Limits

~')

Sec.

209.

v-

r1m

x+hh

h-.o

210.
211.

V-x

.-++ aJ
2

In)

JC~3

. -++

x-x+ .

iim

(V x

x-.-t aJ

(V x! -5x -+ 6-x).
IL11 x <V x! + I-x).

213. ILn

y x -2t+6- y x + 2x-6

1.

(V x (x + a)-x).

212. lim

214.

t a- Vx).

IS)

~+oo

V 1-

215. liln (x -t

Xl).

x~eX)

The formula
Urn
x

Sitl

-+ 0

x= I

i~ frequently u~E'd when solvin~ the following examples. It is taken for


granted that lin] Sin x = Sin a and lim cos x ==C<JS a_
-'~"

x-+a

Example 6.

lim sin 5x = liIn (c;{n 5x .5) = 1.5== 5.


X
x.....,. Q \ 5x

JC-t> 0

216. a) lim

b) 1Lll
.

7 a) IlIn x sIn
- -I ;
22 J.

si fl x ;

sin x
-.
t

b) Ij'n x sin
x.....,. eX)

x -+ Cf)
sin 3x

sin 5x

229. lLn cot


x .... o

.c .... l

Sill 311X

liin

(n sin ~).
n

n-+cx>

.
221. 1Inl

230. lim
-.

.t~o

232. lim

sin x-sin a

x-a
tan 3tX

1
22 4 ,,_~C~2
x+ 2

x2

sin (x

r
in]

11

~~-

art' sin.x
234 . 11 : 1 1 - - .
.t -+ Q

+ h)-sin x
h

I) -+-0

22 6.

cos mx-cf'S nx

233.

cC"s x - CPs a

225. 1un

2
3t-x
1- 2 C0S x
11- 3x

x-a

223. ILIl
.1-+

11

x ..... -

222 11m - - - ...

231 I l~n

1- c('sx

2x cot ( ; - x) .

I-sin~

S(fl1(X

219. I I~n--.
220.

X-+I

218 1lin -2- .


Sill

-!.x .

228. lim (1 -- x) tan nx

217. I en - - .
X-+ 0
x
. -.0

x~o

X -+2

sin X-CflS x
i-tan x

arc tcln 2t
23 5. I1m - - 3 - .
r ... O

4)Ul

l-x2
236 . IIrn - - .
.t --1> 1 Sll1llX

Introduction to Analysis

11m

237

x-c;in 2x

:r .... 0 X

+ S1Il 3.
X

1-

239. lim

cos-

liIn

240.

~.
~ +11x

lim

x...,. 0

VCOS"X
x2

x -+0

1(X

238.

[Ch. I

YI +sinxx

Y I-sjnx

When taking limits of the form


linl (cp (x)]~
JC

(X)

= C

(3)

-+a

()ne should bear in mind that:


1) If there are final limits

lim cp (x) = A and lim '" (x) = B,

X-+Q

~-+a

then C= An;
2) if lim <p (x) = A #= 1 and lim 'l' (x)::..:
(

X -+

"1>Q

00,

then the problem of finding

(J

the linllt of (3) is 'iolved in straightforward fashion;


3) if linl q> (x) = 1 and lun 'i' (x) = 00, then we
x-+a

where a (x)

c=

lim
l'

-+

x-+a

0 as x

-+

'1' (X)

1
lim a (XI
{[I+a(x)]a(x)~~(.~)~(X)=ex-+a

lim [~(x) -IJ


=~-+a

+a

where e = 2.718 ... is Napier's number.


I

Eample 7. Find

sin 2x \ I+~
Urn ( - )
,
X

x-+o

Solution. Here,

lim (Sin 2X) =2 and Urn (1 +x) = I;

~-+o

bence.
11m

x-.o

(Sin 2X)1 +x =2 =2.


1

%-+0

Example 8. Find
lim
x-+ c

(X+ 1 )XJ
2x

+1

Solution. We have

1+ 1
x+l
1
11m - - = Urn --=~ -+ 2x + 1 x -+ 00 2 + l
2
x

fI:J

aod

put q> (x) = I + a (x).

a and, hen\,.e,

'1' (Xl

29

Limits

Sec 3)

1"here{ore,

.J(X+I\X
---)
2x + 1

11m
x-.

=0.

CX)

Example 9. Find

litn
% ..... 00

(X-I
)'x .
x+l

Solution. We have

t--!..

lim x-I = lim _ _


x =1
x-+CI>x+l
('~oo l+-!.

Transforming, as indicated above. \\'e have


I"1m
x-.

(X-I)~
- - = I"1m
x+1

(II

x -+ (YJ

l + (X-J
-+I

)J" =

-1

X+

!X

-2)1 - 2} I+x = eX l:n"" ;/,x _ e\x+1

= lim {[ I-t- ('


%-+ct'>

In this case it is ealijer to find the lilnlt without rec;orting to the general
procedure:

lim
JC -+ '"

(X-l)Xo=:
X+1

lim (1-+
X+ '" (

1+

'
rr J[I~mct 1( 1-++f"r
r
x

~I~

=e-l=e.' I ,

Generally, it is useful to remernber that


lim
%-+

(2+X)"~

241. Itm 3 c.....o \


- x

CJ)

t ..... z.:

244 I

--;-)~ .
x+

(X_')~+J

249. (~i~ x+3

243. lim ( I
X-+>D
X

248. lim (

242. 1i~n(XI..!J)x+l.
%-+.

(l+~)x=efl

)-i+I .

_X)n

250. lim ( 1 + n

n-+QD

(xl-2.t+3)

sln%

;~~ xZ -3x+2

251. lirn (1
-.o

+ sin x) .c

. (Xl +2 ).t
245. :~~ 2xl + 1

252**. a) lim (cos x) ~ :

..!.)II
n

b) lim (cos x)iI.

246. lim ( 1 n.... QD

247 lin) (1
~QI

+ !)J& .
%

%-+0
I

...o

Introduction to Analysis

30

[Ch. 1

When solving the problems that follow, it is useful to know that if the

limit lim f (x) exists and is positive, then


.c-.a

Um [In f (x)] = In rUm f (x)].

x-.a

x.... g

Example 10. Prove that


Urn In (1 +x)
x

x-+o

1.

(*)

Solution. We have

Formula (*) is frequently used in the solution of probleuls.

253. Iim[ln(2x+l)-ln(x+2)].
. lOR ( 1 + lOx)
254. I1m
x

K-+'

255.

~~ ( ~ In y: ~;) .

260*. lim n (Via -1)

eaX_e fJX

256. lim x (In (x+ 1) -In xl.

261. bIn

, In (Cf'S x)
257 I1m
2

262. lim~.

..... +GID

X-+O

K....O

259*. lim
%-+0

-=-

-x

SlIl

%-+0

oX

aX

263. a) lim ~irh x ;

258*. lim ,x_I.


" .... 0

-+O

(a>O),

n-+oo

(a

b) Iimcosh~-l

>0).

~ ... o

(see Problems 103 and 104).


Find the' following limits that occur on one side:
x

264. a) liln
% -+ - .... OD

b) lim
%

V ~! + 1

OD

Vx + 1
2

265. a) Ii.n tanh x;


-+-:
-++QD

266. a) lim - I-J ',


""'-1
1 +e"

,t-++o

1 +e
In (1 +e-, .
a) lim

x
x-+ -00
In (I + eX)
b) lirn
x
JC-++oo
K

267.

b) lim tanh x,
eX _e- JI
where tanh x = e-x--+e- Jt

b) lim

I ~tn-x ;
J
268. a) rlin x-+ -

b) lim 'sin x I
-.+o

Limits
x-I

270. a) lim~2 ;

269. a) 11m -- ,
I I;
0

I-

%-+2-0

X-+I-O

81

x-I
b)X-+I+O
1m x

x-

b) 1im~2.

'--=-11

%-+2+0

x-

Construct the graphs of the following functions:

271 **. Y = lim (eOS Zn x)o


n-+~

272*. y=lim
n-+oo

273. Y = lim

I+

(x~O)_

n
X

Vx-z-+-a,-z.

a-+o

274. Y = lLn (arc tan nx).


n-+(J)

275. y = lim

V 1 +xn

(x ~ 0).

n-+~

276. Transform the following mixed periodic fraction into


a common fraction:
a = 0.13555...

Regard it as the limit of the corresponding finite fraction.


277. What will happen to the roots of the quadratic equation
ax! + bx +c=O,

if the cotfficient a approaches zero while the coefficients band c


are constant, and b#=O?
278. Find the ltmit of the interior angle of a regular n-gon
as n --. 00.
279. Find the limit of the perimeters of regular n-gons inscribed
in a circle of radius R and circumscribed about it as n - 00.
2E:O. Find the limit of the sum of the lengths of the ordinates
of the curve
y = e- x cos nx,
dra\vn at the poi nts x = 0, 1, 2. .
n, as n -+ 00.
~f 1. Find the limit of the ~um of the areas of the squares
constructed on the ordinates of the curve
0

y=2 1 -

.,

ba~es, where x=: 1, 2, 3, ... , n, provided that n ~ 00.


282. ~ind the limit of the perimeter of a broken line MoM I M"

as on

inscrIbed in a logarithmic spiral


I

-=- e-CP

Int ,oduct,on to Analysis

(Ch. 1

(as 11 - . (0), if the vertices of this broken line have, respectively,


the polar angles
PI
283. A segment

= O,

q:>1

AB = a

nn

= 2"'

... , q>tJ = 2

(Fi~. 7)

is divided into n equal parts,


isoscelf's triangle \\'lth base
angles u .::~- 45. Sho\v th:lt the linlit of the perulletcr of tl1(~ hrok(,ll line thus fortned ciilTers frOl11 the
len~~th of AB despite the fact that in
the linllt the broken line "geornetrlcal1y
nl{'r!~eS \Vlt~ the segrnent AB".
ench pnrt serving ns the bClse of

Q~

cHl

Fig 8

Fig. 7

284. The point C1 divides a

sc~nlent

AB ---l in hCllf; the

point C 2 divides a segnlent AC, in half; the point C~ divides a


spglnent C 2 C, in half; the point C.. divides C:!.C 3 in half, and so
on. Dctpfl11inc the linuting position of the pOInt e,. \vhen 11-'" cu.
285. The side a of a right triangle is divided into n equal

pnrts, on each of \vhich is constructed an inscri bed rectangle


(rJig. 8). Deternlinc the linlit of the area of the step-like figure

thus formed if n ~ (X).


286. Find the constants k and b fronl the equation

:c~r~(kX+b-.::t: )=0.

(1)

\Vhat is the geolnetric I11eaning of (I)?


287*. A certain chemical process proceeds in such fashion
that the increase in quantity of a substance during each interval
of ti nle t' out of the in fi nite sequence of intervals (tT, (i -t I) i)
(i ==- 0, 1, 2, ... ) is proportional to the quantity of the substance
available at the comlnencement of each interval and to the length
of the interval. Assuming that the quantity of substance at the
initial time is QQ' deternline the quantity of substance Q~n) after
the elapse of time t if the increase takes place each nth part of
the time interval T = !.n
Find Q. = li'n Q~n).

n-..

Sec.

41

Infinitely Small and Large Quantities

33

Sec. 4. Infinitely Small and Large Quantities


1. Infinitely small quantities (Infinlteslmals). If

lin1 a (x) = 0,
x-+a

Le., if

Ia

(x)

1< e when 0 < I t-a I <

infinitesimal as x

as x

---+

a.

{j (e), then the function a (x) is an


In 51ltlilar fashion we define the infinitesImal a (x)

00.

The sum and product of a linlited number of infinitesimals as x ---. a are


also infinitesimals as x -+a.
If a (x) and p (x) are infinlteslnlals as x --+ a and
liJn a (x)_C
x-+a

P(xJ -

where C is sonle number diff~rent frolll zero, then the functions a (x) and P(x)
are called infinitesimals of the same order; but if C =0, then we say that the
function a (x) is an in fin ttesitnal of higher order than ~ (x). The function
u (x) is called an in{initesit1lal of order t1 cotnpared with the function p (x) if
Ihn
where 0 < J C 1<
If

,"~a

+ 00.

a (x)

IP (x)]n

=c

lim a (x)_1
,"-+Q ~

then the function" a

P(x)

are called
a (x) - p (x).
For exan1ple. for x --. 0 we have
(x)

and

(x) -

equit'alent

functions as x -+- a:

sin x-x; tan x-x; In (1 +x)-x


and so forth.
The sUln of t\\10 infinitesimals of different orders is equivalent to the
tenn whose order is lo\\'er.
The Unlit of a ratio of two infinitesimals remains unchanged if the tenns
of the ratio are replaced by equivalent quantities. By virtue of this theoreln,
when taking the limit of a fraction
lim a (x)
~-+a ~

(x) ,

where a (x) - + 0 and P(x) ~ 0 as x --+ a, we can subtract fronl (or add to)
the numerator or denominator infinitesitnals of higher orders chosen so that
the resultant quantities should be equivalent to the original quantities.
Example 1.
~2(~
.
1
lIn1
11m - - = %-+0 In (1 + 2x)
x-+o 2x
2

. Vii

VX"'

2. Inftnltely large quantities (InOnltes). If for an arbitrarily large number N there exists a ~ (N) such that when 0 < I x-a I < ~ (N) we have the
inequality
If (x) I> Nt

then the function


2-1900

f (x)

is called an infinite as x --+ Q.

Introduction to Analysis

34

[Ch. 1

The definition of an infinite f (x) as x ~ Q() is analogous. As in the case


of infinitesimals, we introduce the concept of infinites of different orders.

288. Prove that the function

f (x) =
is an infinitesimal as x ~
quality

00.

sin x
x

For what values of x is the ine-

If (x) 1<8

fulfilled if 8 is an arbitrary number?


Calculate for: a) 8 = 0.1; b) 8=0.01; c) 8=-0.001.
289. Prove that the function

f (x) =
is an infinitesimal for
quality

x~

i-Xl

1. For what values of x is the ine-

I f (x) 1<8

fuliilled if 8 is an arbitrary positive number? Calculate numerically for: a) 8=0.1; b) 8=0.01; c) 8=0.001.
290. Prove that the function
1

f (x) = x-2
is an infinite for x ~ 2. In what neighbourhoods of Ix - 21 < 6 is
the inequality
I f(x) I>N
fulfilled if N is an arbitrary positive number?
Find 6 if a) N = 10; b) N = 100; ,
o
c) N = 1000.
A ",..
...----.,;~
291. Determine the order of smallness
B of: a) the surface of a sphere, b) the volume
of a sphere if the radius of the sphere r
is an infinitesimal of order one. What
will the orders be of the radius of the
sphere and the volume of the sphere with
respect to its surface?
292. Let the central angle a of a cirFig. 9
cular sector ABO (Fig. 9) with radius R
tend to zero. Determine the orders of
the infinitesimals relative to the infinitesimal a: a) of the
chord AB; b) of the line CD; c) of the area of d ABD.

Infinitely Small and Large Quantities

Sec. 4]

35

293. For x---+ 0 determine the orders of smallness relative to


x of the functions:
2x
d) 1- cos x;
a) T+X;
e) tan x- sin x.
b)

Vx+ Vx;

c)

Vii-Vi';

294. Prove that the length of an infinitesimal arc of a circle


of constant radius is equivalent to the length of its chord.

295. Can we say that an infinit~simally small segment and


an infinitesimally small semicircle constructed on this segment
as a diameter are equivalent?
Using the theorem of the ratio of two infinitesimals, find
298 1 ~
296 I sin 3xsin 5x
x-.o
1m

297. lim

(x-x1)2

arc

x~0

S1n ..

1m I -x

X-+l

299. lim CO~ x-cos2x


%-.0

I-Xl

-cos X

1n (1 -- x)

-i

300. Prove that when x~O the quantities


and VI +x~i
are equivalent. Using this result, demonstrate that when I x t is
small we have the approximate equality
VI+x~I+;

(I)

Applying formula (1), approximate the following:


a) Vl.06; b) VO.97; c) VI0; d) V120

and compare the val ues obtained with tabular data.


301. Prove that when x --+ 0 we have the following approximate equalities accurate to terms of order Xl:
a) _1_ ~

I-x'
x
a -t- x~a + 2a

l+x

b)

(a

> 0);

c) (1 +x)n~1 +nx (n is a positive integer);


d) log (1 + x) = Mx,
where M=10ge=O.43429...
Using these formulas, approximate:
1

1) 1.02; 2) 0.97; 3) 105; 4)

V15;

5) 1.04'; 6) 0.93 4 ; 7) log 1.1-

Compare the values obtained with tabular data.


2*

36

ICh. 1

Introduction to Analysis

302. Show that for x ~ 00 the rational integral function


p (x) = aox rl + a1xn - I -~ +an (ao::;bO)
is an infinitely large quantity equivalent to the term of highest
degree aoxn
303. Let X--'(X). Taking x to bean infinite of the first order,
determine the order of growth of the functions:
a) x!-lOOx-l,OOO;

c)Vx+VX;

Xl

b)

x,-2 ;

d)

V x-2x

Sec. 6. Continuity of Functions


1. Definition of continuity. A function f (x) is continuous when X= 6
(or ccat the point ~"), if: 1) this function is defined at the point ~, that is,
there exists a number f (~); 2) there exists a finite limit Urn f (x); 3) this lim-

it is equal to the value of the function at the point

x-..;

~,

Le.,

lim f (x) = f (~).

(1)

X-+;

Putting

x=~+L\~,

where L\~ ---+- 0, condition (1) may be rewritten as


lim L1f (~) = lim (f (~+ L\~)-f (~)) = 0,

(2)

~;-+o

d;-+O

or the function f (x) is continuous at the point ~ if (and only if) at this point
to an infinitesimal increment in the argument there corresponds an infinitesi..
mal increment in the function.
If a function is continuous at every point of some region (interval, etc.),
then it is said to be continuous in this region.
Example 1. Prove that the function

y=sinx
ts continuous for every value of the argument x.
Solution. We have
~

Ay = sin

(x + Ax) -

sin

L1x

2 sin

~x cos ( x+ ~x) = Sl:x2

cos (

x+ ~x) . Ax.

"2

Since
L\x

Silly

lim - - = 1 and Icos(x+

dX-+O

L\x

L\2X)I~I,

it fol lows that for any x we have


lim /).y=O.
dX-+O

Hence, the function sin x is continlJOUS when - oo<x<

+ co.

Sec. 5J

37

Continuity of Functions

2. Points of discontinuity of a function. We say that a function

f (x) has

a dIscontinuity lat x=xo (or at the point xo) within the domain of definition
of the function or on the boundary of this domain if
continuity of the function at this point.
Example 2. The lunct'ion

f (x) = (1

1x)t

(Fig.

th~re

10 a)

is a break in the
is

discontinuous

when x= 1. This function is not defined at the point x= 1, and no matter

y
y=(x)
2

---~

I
__ ~

I
I

2
(b)

(a)

-I

(c)
FIg. 10

how we choose the number f (1). the redefined function f (x) will not be continuous for x = 1.
If the function f (x) has finite limits:
11m f (x) = f (xo-D) and lim f (x) = f (xo D),
X-+Xo - 0
X-+Xo +0

and not all three numbers f (xo)' f (xo-D), f (xo + D) are equal, then Xo is called
a discontinuity of the first kind. In particular, if

f (xo-O)=f (xo+O).
X o is called a removable discont inuit y.
For continuity of a function f (x) at a point xo, it is necessary and sufficient that

then

(CIl. 1

38

Example 3. The function

f (x) = ~I: ~

has a discontinuity of the first kind

at x=O. Indeed, here,

f (+ 0) = lim
x-++o

sin x =
x

+1

and
'(-0)= lim sin x = - t .
.1-+-0

-x

Example 4. The ~unction y = E (x), where E (x) denotes the integral part
of the number x [i.e., E (x) is an integer that satisfies the equal ity x = E(x) q.

where O<;q

<

I}, is discontinuous (Fig.

tOb) at every integral point: x=O,

I, 2, ... , and all the discontinuities are of the first kind.

Indeed, if n is an integer, then E (n-O):.= n-I and E (n 0) = n. At an


other points this funct ion is, obviously, continuous.
Discont inuities of a function that are not of the first kind are called
discont inuities of the second kind.
Infinite discontinuities also belong to discontinuities of the second

kind.
These ar.e points Xo such that at least one of the one-sided limits, f (xo-O) or
f (xo+O), is equal to Q() (see Example 2).

Example 5. The function y=cos ~ (Fig. 10c) at the point x=O has a
x
discontinuity of the second kind, since both one-sided limits are nonexistent
here:
lim cos ~

%-+-0

and

lim cos ~

x-++o

3. Properties of continuous functions. When testing functions for conti-

nu ity, bear in mind the following theorems:


I) the sum and product of a limited number of functions continuous in
some region is a function that is continuous in this region;
2) the quotient of two functions continuous in some region is a continuous
function for all values of the argument of this region that do not make the
divisor zero;
3) if a function f (x) is continuous in an interval (a, b), and a set of its
values is cont~fned in the interval (A, B), and a function cp (x) is continuous
in (A, B), then the cOlnposite function cp (f (x)] is continuous in (a, b).
A function f (x) continuous in an interval [a, b) has the following properties:
1) f (x) is bOl'fnded on [a, b), Le., there is some number M such that
If(x) f<M when a~x<;b;
2) f (x) has a minimum ann a maximum value on [a, b);
3) l (x) takes on all interrrlediate values between the two given values;
~hat is, if f (a) = A and f (~) = B (a ~ a < ~ ~ b), then no matter what the
number C between A and B, there will be at least one valuex=v (a<v<~)
6uch that f (V)::::IC.
In particular, if l(a) f (~)<O, then the equation
[(x) =0

has at least one real root in the interval (a,

~).

304. Show that the function y =


of the argument x.

is continuous for any value

Xl

Sec. 5)

Continuity of Functions

39

305. Prove that the rational integral tunction


p (x) = aox n + a1xn - 1 + ... -t- a n
is continuous for any value of x.
306. Prove that the rational fractional function

R( )X -

aox n+ a.x n-l+ . . . + an


boxm + b.x m- 1
+ b rn

+ ...

is cont in uous for all val ues of x except those that make the denominator zero.
307*. Prove that the function y= Vi is continuous for x ~O.
308. Prove that if the function f (x) is continuous and nonnegative in the interval (a, b), then the function
F (x) = V! (x)
is likewise continuous in this interval.
309*. Prove that the function y == cos x is continuous for any x.
310. For what values of x are the functions a) tan x and
b) cot x continuous?
311 *. Show that the function y = I x I is continuous. Plot the
graph of this function.
312. Prove that the absolute value of a continuous function
is a continuous function.
313. A function is defined by the formulas

f (x) ==

"
t

x2-4
--2
x-

for X=F 2,

for x=2.

How should one choose the val ue of the function A = f (2) so


that the thus redefined function f (x) is continuous for x = 2?
Plot the graph of the function y = f (x).
314. The right side of the equation

f (x) =

I-x sin-!.x

is meaningless for x=O. How should one choose the value {(O)
so that f (x) is continuous for x == O?
315. The function
1

f (x) = arc tan x-2


is meaningless for x = 2. Is it possi ble to define the val ue of f(2)
in such a way that the redefined function should be continuous
for x = 2?

Introduction to Analysis

40

50

(Ch. 1

316. The function f (x) is not defined for x = O. Define f (0)


that f (x) is continuous for x= 0, if:
a) f(x)

= (1 +x)n-l
x

(n

is a positive integer);

b) f(x)= I-XC1osx ;

c)

f (x) = In (I +X)-; III (I-x) _ ;

d)

f (x) =

eX _e- X
x

f (x) = x sin 1.-x ;


f) f (x) = x cot x.
2

e)

Investigate the following functions for continuity:


1

317.

Y=XX 2 .

318.

Y= 1 +x

319.

Y=

320.

y= iXl

1 +x'

324. y=lnltanil.

325. y=arc tan

V~;3
x

326. y=(1 +x)arctan I I x

327. y = e X+l

322.

321. a) y= sin -; ;
b) y=

x1

-i2

328. Y= e

x sin ~.
x

329.

Y=

- - - J-

J +e 1 -

x .
Y= -:-.stn x

323. y = In (cos x).


330. y= {

X2.

2x+l

for x ~ 3,
for x>3.

Plot the graph of this function.

331. Prove that the Dirichlet function X(x), which is zero for
irrational x and unity for rational x, is discontinuous for every
value of x.
Investigate the following functions for continuity and construct
their graphs:

332. y= lim 1+1


n-+<

(x ~ 0).

333. y == lim (x arc tan nx).


n~CD

S~c.

5]

Continuity of Functions

41

334. a) y = sgn x, b) y = x sgn x, c) y == sgn (sin x), where the


function sgn x is defined by the formulas:
+ 1, if x >0,
sgnx=
O,~fx=O,
{ -I,
If x<o.
335. a) y=x-E(x), b) y=xE(x), where E (x) is the integral
part of the number x.
336. Give an example to show that the sum of two discontinuous functions may be a continuous function.
337*. Let a be a regular positive fraction tending to zero
(O<a< I). Can we put the limit of a into the equality
E (I + a) = E (l-Q) + 1,
which is true for all values of a?
338. Show that the equation
x'-3x.+ 1 =0
has a real root in the interval (1,2). Approximate this root.
339. Prove that any pol ynomial P (x) of odd power has at
least one real root.
340. Prove that the equation
tan x=x
has an infinite number of real roots.

Chapter II

DI FFERENTIATION OF FUNCTIONS

Sec. 1. Calculating Derivatives Directly


1. Increment of the argument and increment of the function. If x and Xl
are values of the argurnent x, and Y=f (x) and Yl =f (Xl) are corresponding
values of the function y = f (x). then

Ax=xl-x
is called the increment of the argument

in the interval (x, Xl)' and

/1Y=YI-Y

or

AY=f (x 1 )-f (x) = f (x

+ Ax) -f (x)

\)

Fig. 11

Is called the increment of the function y in the same interval (x, xJ) (Fig. 1t,
where l\x=l.1A and Ay=AN). The ratio

:~=tana
js the slope of the secant MN of the graph of the function Y= f (x) (Fig. 11)
and is called the mean rate of change -of the function y over the interval
(x, x+ Ax).
Example 1. For the function

y=x l -5x+6

Calculating Derivatives Directly

Sec. 1]

43

calculate ~x and ~y, corresponding to a change in the argument:


a} from x= 1 to x= 1.1;
b) fromx=3 to x=2.
Solution. We have

Ax= 1.1-1=0.1,
= (1.1 2 -5.1.1 + 6)-(1 2 -5.1 + 6) = - 0.29;
b} Ax=2-3 = - 1,
t,.y = (2:1-5 2 + 6) - (31 -5.3 + 6) :::::0.
a}

Ay

1
Example 2. In the case of the hyperbola y= - . find the slope of the
x

"3,
1)

Solution.
Here,
fiy
1
k= lix=- 30

fix

= 10-3=7

t ) .
and N (10, 10

secant passing through the points M ( 3,

and

fiY=10-a=-M.

Hence,

2. Tbe derivative. The derivative y' = ~ of a function y = f (x) with reo

spect to the argument

JC

is the limit of the ratio

~~ when

Ax approaches zero;

that is.

The magnitude of the derivative yields the slope of the


graph of the function y = f (x) at the point x (Fig. 11):
y' =tan lp.

tan~ent

MT fo the

Finding the derivative y' is usually called diOerentiallon of the functlon. The
deriv at ive y' = f' (x) is the rate of change of the functton at the point x.
Example 3. Find the derivative of the function
y=x l

Solution. From formula (I) we have


fig = (x

+ fiX)t-xl =-2xAx4- (Ax)'

and
fiy
~x = 2x+ fix.

Hence,
y'= 11m liy= lim (2x+Ax)=2..,.
~x -+ 0 Ax 6.x -+ 0

ae

One-sided derivatives. The expressions

t'- (x) =

lim

f (x+ Ax)-f (x)

6.%-+-0

fix

and

I'
+

(x) = 11m
AX ~+Q

f (x+ fix)-f (x)


~x

44

Differentiation of Function"s

[Ch. 2

are called, respectively, the left-hand or rtght-hand derivative of the function


x. For f' (x) to exist, it is necessary and sufficient that

f (x) at the point

t'- (x) = f~ (x).


Example 4 Find f~ (0) and f~ (0) of the function

f (x)

=,

x I.

Solution. By the definition we have

f~(O)= lim lL\xl=_l,


Ax

L\x -+ -0

f~
4

(0) =

Il:!xl= 1.

lim
L\x -+ +0

I:!x

Infinite derivative. If at some point we have


lim

f (x+ I:!x)-f (x) =

ClO,

~x

AX-+- 0

then we say that the continuous function f (x) has an infinite derivative at x.
In this case, the tangent to the graph of the function y = f (x) is perpendicular to the x-axis.
Example 5. Pi nd I' (0) of the funct ion
y=

Solution. We have

Vi:

Vf:j.x

f' (0)= lim -A-= lim


L\x -+ 0

uX

L\x -+ 0

1
V-=oo.
L\x
2

341. Find the increment of the function y == x 2 that corresponds


to a change in argument:
a) from x = I to Xl = 2;
b) from x = I to Xl = 1.1;
c) from x = I to Xl == I + h.
342. Find ~y of the function y ~

Vx if:

a) x=O, ~x~O.OOl;
b) x=8, L\x==-9;
c) x=a, Ilx=h.
343. Why can
increment 6.y if
Ax = 5, while for
344. Find the
tions:

a)

Y=(X t _2)2

b) y==

Vi

c) y = log x

we, for the function y = 2x + 3, determine the


all we know is the corresponding increlnent
the function y = Xl this cannot be done?
increment tJ.y and the ratio ~y for the funcx

forx=l
forx=O
for x= 100,000

and~x=0.4;

and ~x==O.OOOI;
and t!.x = - 90,000.

Sec, 1]

45

Calculating Derivatives Directly

345. Find Ay and ~~ which correspond to a change in argument from x to x-~ ~x for the functions:
a)y=ax+b;
b) y === x' ;
1

c) Y == ~2

d)y=Vx;

e) y = 2x ;
f) y = In x.

346. Find the slope of the secant to the parabola


y=2x-x 2 ,
jf the abscissas of the points of intersection are equal:
a)x,~I,xl=2;

b) xl==l, x 2 =0.9;
Xl ~ 1, x t = I + h.

c)

To what limit does the slope of the secant tend in the latter case
if h ~O?
347. What is the mean rate of change of the function y = Xl
in the interval 1 ~ x ~ 4?
348. I'he law of motion of a point is s = 2t 2 + 3t + 5, where
the distance s is given in centimetres and the time t is in seconds.
What is the average velocity of the point over the interval of
time fron] t=1 to t=5?
349. Find the mean rise of the curve y = 2x in the interval
1~x~5.

350. Find the mean rise of the curve y = f (x) in the interval

lx, x-t-l\x].

351. What is to be understood by the rise of the curve y = f (x)


at a given point x?
352. Define: a) the tnean rate of rotation; b) the instantaneous
rate of rotation.
353. A hot body placed in a medium of lower temperature
cools ott. What is to be understood by: a) the mean rate of
cooling; b) the rate of cooling at a given instant?
354. What is to be understood by the rate of reaction of a su bstance in a chemical reaction?
355. Let In = f (x) be the mass of a non-homogeneous rod over
the interval (0, Xl. What is to be understood by: a) the Inean
linear density of the rod on the interval [x, x + l\x]; b) the linear
density of the rod at a point x?
356. Find the ratio ~~ of the function y =
at the point
x=2, if: a) l\x=l; b) l\x=O.l; c) l\x=O.Ol.Whatisthederivative y' when x~: 2?

[Ch.2

DitJerentiation of Functions

46

357**. Find the derivative of the function y = tan x.


358. Find y' = lim ~ of the functions:
~X-+ 0

a) y = Xl;
1

b) Y= z;
x

c)

y=Vx;

d) y = cot x.

V-

359. Calculate f' (8), if f (x) = x.


360. Find f' (0), f' (1), f' (2), if t(x)= x(x-I)I(x-2)1.
361. At what points does the derivative of the function
f (x) = x' coincide numerically with the value of the function itself,
that is, f (x) = I' (x)?
362. The law of motion of a point is s = 5t l , where the dis..
tance s is in metres and the time t is in seconds. Find the speed
at t = 3.
363. Find the slope of the tangent to the curve Y= O.Ix'
drawn at a point with abscissa x = 2.
364. Find the slope of the tangent to the curve y = sin x at
the point (n, 0).
365. Find the value of the derivative of the function f (x) = ~
x
at the point x = X o (x o =/= 0).
366*. What are the slopes of the tangents to the curves y =.!.
x
and y = Xl at the point of their intersection? Find the angle between these tangents.
367**. Show that the following functions do not have finite
derivatives at the indicated points:
a) y= V Xl
b)y= x-I
c) Y = I cos xl

V-

at x=O;
at x = I;
2k+l
at x=--rn, k=O, I, 2, ..

Sec. 2. Tabular Differentiation


v=

1. Basic rules for finding a derivative. If c is a constant and u = q> (x),

'i' (x) are functions that have derivatives, then


1) (c)' =0;

5) (uv)' = u'v+ v'u;

2) (x)' = 1;

(!!..)'
V

VU' -v'U
Vi

(v ,t: 0);

3) (u

v)'=u' v';

4) (cu)' =cu';

7) (

~ )' =

~~'

(v

~ 0).

Sec. 2]

Tabular DltJerentiation

47

2. Table of derivatives of basic functions


I. (x n )' = nxn - I

II. (yX)' =

,I
(x
2 fr x
III. (sin x)' = cos x.
IV. (cos x)' = - sin x.
V. (tan x)' = _11_

>

0).

cos x
-1

VI. (cot x)' = sin


-.-1-x .

y I-xl
1
(I x 1< 1).

VII. (arc sin x)' =

y -1
(I x I <
I-Xl

VIII. (arc cos x)'

IX. (cue

lijl

x)

I).

= 1+x

-I

X. (arc cot x)' = x2+ 1


XI. (aX)' = aX In a.
XII. (ex)' = eX.

XIII. (1" x)' =-!.


x

(x

> 0).

X IV. (loga x)' = -1-- = loga e (x


x "a
x
XV. (sinh x)' = cosh x.
XVI. (cosh x)' = sinh x.

> 0,

> 0).

XVII. (tanh X)'=-!.-h

cos 2 x
XVIII.

-1

(cothx)'=~hl .

sIn

XIX. (Mcsinhx)'=

~.
1 +x

XX. (arc cosh x)'

x2 -1

( I x, > 1).

XX I. (arc tanh x)' == 1--1 (I x I < I).


-x
-1

XXII. (arc coth x)' = xl-I (I x r > 1).


3. Rule for differentiating a composite function. If y=f(u) and u=q>(x).
that is, y = f [q> (x)], where the functions y and u have derivatives, then
Y~=Yu U x
(1)
or in other notations
dy_dy du

dx- du dx
This rule extends to a series of any finite number of differentiable functions.

D'Uerentiation of Functions

48

[Ch.2

Example l. Find the derivative of the function

Y= (xl

2x+ 3)5.

Solution. Putting y=u', where u=(xl -2x+3), by formula (1) we will

have

y' = (U5)~ (x 2 _ 2x + 3)~ = 5u 4 (2x - 2) = 10 (x-I) (x 2 -2x + 3)'.


Example 2. Find the derivative of the function
y=sin' 4x.
Solution. Putting

y=u';

u=sinv;

v=4x,

we find
y' = 3u 2 cos tJ4 = 12 sin:! 4x cos 4x.

Find the derivatives of the following functions (the rule for


differentiating a composite function is not used in problems
368-408).

A. Algebraic Functions
368. y=x' -4x'

369. y=i--

+ 2x-3.

~ x+r-O.5x.

+ bx+c.

370. y=ax 2

-5x
371. y =a- .

377.

+ bt
.ax'+b

373. Y=

Va

-t-X-'.

Vx
a
b
y= V-x - V-
x

376*. Y =

372 Y = at m

375. y=3x3 -2x t


l

Xl

a+bx

378. Y=c+dx.
IR

n
+

+1J2'

374. y=~~ln2.
x

2x+3

379. y=x 2 -Sx+S .


/I

380. y=2x-l-x.
_I + Vi
381 Y.r1- f Z

B. Inverse Circular and Trigonometric Functions


382. y = 5 sin x+ 3 cos x.
386. y= arc tan x + arc cot x.
383. y=tanx-cotx.
387. y=xcotx.
384. Y = sin x+ cos x

388 Y =

385. y=-2t sin t-W-2) cost.

389. y= (I +x

sin x--cos x

arc sin x.
2
)

ar; tan x-x.

Sec. 2]

49

Tabular DiOerentlation

c.

Exponential and Logarithmic Functions

390. y==x 7 .eX

396. Y = eX arc sin x.

391. y=(x-l)e".

397. Y=-l
nx

x2

eX

Xl

398. y=x Inx- 3 .

392. Y=i.
x
Xl

1nx
399. y=.!.+21nx_
.
x
x
400. y=lnxlogx-lna log a x.

393. Y=-x.
e

394. f (x) == eX cos x.


39ii. Y= (x l --2x + 2)e".

D. Hyperbolic and Inverse Hyperbolic Functions


405. Y = arc tan x- arc tanh x.

401. y=x51nhx.
Xl

402. y=-h-.
cos x

406. y = arc sin x arc 51 nh x.

403. Y= tanh x-x.

407

404

408

=3cothx
lnx

_ arc cosh x

y Y

= arc coth x

I-x!'

E. Composite Functions
In problems 409 to 466, use the rule for differentiating a C0111posite function with one intennediate argument.
Find the derivatives of the following [unctions:

409**. y=(1+3x--5x2 )'o.


Solution. Denote 1

+ 3x-5x! =

u; then y = ulO We have:

y~=30ul8;

u~=3-10x;

lI~ ~=30u28.(3-10x)=30(1 +3x-5x 2 )18.(3-IOx).

410. Y=

(UX/b)'.

411. f(y)=(2a+3by)l.

412. y=(3+2xl )4.


413. Y=56(2:-l)7

24(2~-l)'

414. y= Vf=XI.
415. y= Va+bx' .
416. y=(a2/1_x3/1)'/a.

40 (2x -1)1

50

[Ch. 2

DifJerenttation of Functions

417. y=(3-2sinx)'.
Solution.
y' = 5 (3-2 sin x)' (3-2 sin x)' = 5 (3-2 sin x)' (- 2 cos x) ==
- 10 cos x (3-2 sin x).

y=tanx- ~ tan1x+ ~ tan'x.

418.

419. Y= Vcotx-

Vcota.

423. Y= - I-a - _1 .

420. Y= 2x + 5 cos' x.

421 *.

424. Y=

= cosec t + sec t.

3 cos x cos X
, 13 sin x-2 cos x

1
_,

425. Y= Vsin! x+_

cos x

422. !(x)=-6(1-3Cosx)l.

426. Y= VI +arc sinx.


427. Y = Varc tan x- (arc sin x)'.
428.
429.
430.
431.

y arc tan x
Y= V xe" +x.
Y= V2e"-2" + I + In l x.
y=sin3x+cos ~ +tanVx.

Solution. y'=cos3x(3x)'-sin

~(i)' +cos.lyx(YX)'=3COS3X-

1 . x +
1
...... -5 sIn -5 - ..----r---'1-r-
2 ,. X cost ,. X

432. Y= sin (x l -5x+ 1) + tan ~

x
433. {(Xl = cos (ax + P).
434. f (t) = sin t sin (t + q.
435. y =
436.
437.

~ ~~~: ~ .

f (x) = a cot!-.
a
1
1
Y = - 20 cos (5xl ) - 4 cos Xl.

438. Y = arc sin 2x.


Solution. y'

439.

YI _(2X)1
y= arc sin ~
.
x

440.

f (x) = arc cos Vi.

(2x)' =

2
.
Vl-4x 2

441. Y = arc tan..!..


.
x
442. Y = arc cot l
l

+x

-x

Tabular DitJerentiation

Sec. 2]

51

443. Y= 5e- xa
444.
445.
446.
452.

453.
454.

447. Y= arc cos eX.


448. y=In(2x+7).
Y= .
sX
449. Y = log sin x.
y=x"IO"x.
450. y=In(l-xl ).
f(t)=tsin2 t
451. y=Inlx-ln(Inx).
y = In (eX + 5 sin x- 4 arc sin x).
y = arc tan (In x) + In (arc tan x).
y= Vln x+ I + In
I).
1

evx+

F. Miscellaneous Functions
5x cos " '3.
x
4u115** Y = sin
11
4
456. y=-2{x-2)I-x-2.
15

10
3 (X-3)1

457. y=- 4 (X-3)4

1
2{x-3)1-

458. y = 8 (1 -Xl).

- 2x l -2x+ 1
459 yx

460. Y=

al

x
y-_.
a -1- x
2

Xl

461. Y=
462. Y=
463.

Y (1 -}-

A 2)1

~ VXi+~xVX+~ x V?+l~X- Vi

Y=~ V (l
4

+ XS)I- ~ V (l +X)I.

VX-l

464. Y=3
x+2
4
465. Y = x (a - 2X S)1
a +bXn)",

466. y= ( a-bx n
9
3
467. Y = 5 {x+2)S- (x + 2)

+ (x+ 2)' -2" {X+2)1

469.

= (a + x) Vli=X.
y = V(x+ a) (x-t- b) (x+ c).

470.

z=Vy+Vy.

468. y

471. f(t)=(2t+l)(3t+2)V3t+2.

62

D"Oerentiation of Functions

[Chi 2

472. x= ..yr~'
2ay- y l
473. y = In (VI +ex-I)-ln (VI +ex
474. y = 1~ cos' x (3 cos' x-5).
_ (tan! x-l)(tan 4 x + 10 tan 2 x +])
475 Y 3 tan' x

+ I).

476. Y = tanS 5x.

x2 -1

485. y=arc sln~.

477. Y = ~ sin (x').


478. y = sin l (t 3 ),

479. Y = 3 sin x cos x + sin' x.


1

480. y="3 tan 3 x-tanx+x.


4

cos x

481. y = - .3
a +-3 cotx.
SlIl X

482. y= Va sin I x+ Pcos l


483. Y = arc sin Xl + arc cos

x.

x.

491. y=arcsin(1-x)+V2x-r.

484. y=}(arcsinx)'arccosx.
492. y=

v x+ 21V
x-x'.
(,x- 21) arcsin 1/--

493. y = 1n (arc sin 5x).


494. y = arc sin (In x).
x sin a

495. y =r.:: arc tan 1 -xcosa


2

496. y = ~ arc tan

x
5tan 2 +4

497. y=3b'arctan

Yb

498. y=- V2 arc cot

Ve

x-(3b +2x) Vbx-x'.

tanx

V2- x .

ax

499. y=

500. y = es1n2 x,

501. F(x)=(2ma mx +b)p.


502. F(t)=eCltcos~t.
503 = (a sin ~x - p cos ~x) e'J.x
y
a2 -1- ~2

Tabular DiOerentiation

Sec. 2]

504. Y = 110 r

53

(3 sin 3x-cos 3x). 507. Y = 3cot

xi

505. y=xna-
506. y = V cos xaYC"Oi'X.

+.

508. y=In(ax!+bx-J-c).
509. y === In (x -1- Val + Xl).

510. y=x-2Vx+2In(1-l-VX)
511. y=In(a-t-x-1-V2ax+x l ).
1
512. Y=~l
nI x

514* y=In(x-2)S

515 y --I n

x-I
5 13. Y = I n cos -x- .

517.

y= ~ Vxl-a l _

516
a
;

Y- -

In(x+ Vxl-a l ),

518. Y= In In (3-2x').
519. Y = 51n s (ax + b).

520. y=ln

y~+x.
y x -i-a! -x
2

52 I. Y = ~ In (x 2 - at) -l- ~ In ~ .
2a

522.

x+a

y=x.sin(lnx-~).
x

1 cos x

523. Y = -2 In tan -2 --2 sin


--2-
x

524. f (x)

V7tl -In 1+ YX2+1


,
x
x 2 -2x -+-1

525. Y=3 1n .\I-+-x+l


526. Y = 2arc sin IX -1- (1- arc cos 3X)I.
sin ax

527. Y =

3cOS b~ +

~ ~~:.~ .

tan ~+ 2- Y3
528. y===~ln __2 _ - - -

Va

tan~+
2

2-1-

Y3

529. Y = arc tan In x.

530. y= In arc

sinx+~ In x-t arc sin Inx.


2

531. y = arc tan In -.


x
532. y

Y2

-3- arc tan

x-I

y'2' + (fIn X'D

(x 1)'
(X-I)3(X-2)
-~----..;.

x-3

2 sin! x

I t

+ n anx.

54

DiOerentiation of Functions

533. Y= In

[Ch.2

1+ V'
VSinx
V-.+2 are tan sm x.
1-

sin x

Xl

+I

x-I

534. Y=4 1n xl _1 +41nx+l+"2arctanx.


I
1 I
I
tan 2xY3
-I
535. f (x)=2"ln(l
+x)-6
n(xI -x+ I )+ (:rare

536.

f (x) =

VIC sin x + In V I-xl.


i-Xl

037. Y = sinh' 2x.


538. y = erJx cosh px.
539. y = tanh' 2x.

542. y = arc cosh In x.


543. y = arc tanh (tan x).
544. y = arc coth (sec x).

540. y = In sinh 2x.

545. y = arc tanh I ~XI

541. y = arc sinh


547. Y =

(~

546. Y= 2 (xl-I) arc tanh

al'

x+ 2 x.

+ ~ ) arc sinh x- ~ x V I + Xl

Xl

048. Find y'

if:

a) Y= Ix I;
b) y=xlxl.

Construct the graphs of the functions y and y' .


.549. Find y' if
y=lnlxl (x*O).
550. Fin d f' (x) if
f (x) = { 1- x for x ~ 0,
e- x
for x>O.
551. Calculate I' (0) if
, (x) = e- x cos 3x.
Solution. l' (x) =e- X (-3 sin 3x)-e- X cos 3x;
l' (0) = eO (-3 sin O)-eO cos 0 =-1 .

..

552. f (x) = In (l +x) + arc sin ~.


553. Y = tan' ';;. Find

(:~

t=,

Find f' (I).

554. Find t+(O) and ''-(0) of the functions:


a)
b)

f (x) = V sin (Xl) ;


aa -x
+x
f (x) = arc sin

c) f(x) =_x,
1
\-

1 +e ~

d)

2 ;

f (x) = Xl sin..!.x ,

e) f (x) = x sin

x=t=O; 1(0)=0;

x:

x~O;
%

=F 0;

f (0) = 0;
f (0) = 0

Tabular DifJerentiation

Sec. 2]

55

+
+

555. Find f (0) xl' (0) of the function f (x) = e- x


556. Find f (3) (x-3) I' (3) of the function f (x) = VI -+ x.
557. Given the functions f (x) = tanx and cp (x) = In (I-x).

.
l' (0)
find cp' (0)

558. Given the functions


.

f (x) = I-x and cp (x) =

1- sin ~x ,

cp' (I)

find f'

(1)

559. Prove that the derivative of an even function is an odd

function, and the derivative of an odd function is an even function.


560. Prove that the derivative of a periodic function is also
a periodic function.
561. Show that the function y = xe- x satisfies the equation
xy' = (I-x) y.
x2

562. Show that the funct ion y = xe-;: sat isfies the equat ion
xy' == (I-xl) y.
1

563. Show that the function y = 1+x+ln x satisfies the equation xy' = y (y In x-I).

G. Logarithmic Derivative
A logarithmic derifJatit1e of a function Y=f (x) is the derivative of the
logarithm of this function; that is,
(In y)' ==!L = f' (x)
y f(x)
Finding the derivative is sometimes simplified by first taking logs of the function.
Example. Find the deri vative of the exponential function
y=u'D,

where u = cp (x) and v = W(x).


Solution. Taking logarithms we get
In y == v In u.
DifTerentiate both sides of this equation with respect to x:

(Iny)'=v' In u+v (In u)',


or
1

-y'=v'lnu+v -u',
y
u
whence
y'=y

(V'lnu+: u').

[Ch.2

DiOerentiation of Functions

56
or

y'=Uf1(V'lnu+~ u')
564. Find y', if
y=

- I-x
V x2 1 + x2 sin' x cost x.

2
Solution. In y =31n x + In (l-x)-ln (1 +x2 ) + 3 In sin x +2 In cos x;

y y

'=~-.!-

3 x

whence

(-1) __~

+ I-x

Y'=Y(3:-1

3_1_ cosx_2s1nx .

l+x 2 + sinx

cosx

1 X-l~XI+3cotx-2tanx).

565. Find y', if y=(sinx)".

-.!-y y' = In sin x + x cot x;


y' = (sin x)" (In sin x + x cot x).

Solution. In y = x In sin x;

.In the following problems find y' after first taking logs of the
funet ion y = f (x):
566. Y= (x+ 1) (2x+ 1) (3x+ 1).

567. Y=

574. Y=

(X+2)2

Vx.

575. y=x

(x+l)8(x+3)4.

v-

%.

568. Y = ' / x (x - I)

x-2

VX2+
3

569. Y = x

570. Y =

x2

577. y=xs1nx

(X-2)9

578. Y = (cos x)Sin x.

Y(x-I)S (X-3)1I

vr

Yx-l

579. Y=

571. Y=
x +2)1 V(x+3)'
572. y=x".
573. y=xx3

(1 +~

r.

580. y= (are tan x)..

Sec. 3. The Derivatives of Functions Not Represented Explicitly


1. The derivative of an inverse function. II a function Y =f (x) has a
derivative Y~:I= 0, then the derivative of the inverse function X=/-1 (y) is
1

XII

=-,

Yx

Sec. 3]

The Derivatives of Functions Not Represented ExplIcitly

51

or
dx
1
dy= dye
dx

Example 1. Find the derivative x~, if


y=x+ In x.
1 x+ 1
x
Solution. We have Yx = 1+-=--; hence, x y =-+ .
x
x
x 1
2. The derivatives of functions represented parametrically. If a function y
Is reI ated to an argu ment x by means of a parameter t,
I

x==- q> (t),


{ y= 'I' (t),

then

or, in other notation,

Example 2. Find

:~.

dy
dy di
dx= dx .
dt

if
x == a cos
y=a sin

Solution. We find

:~ = -a sin t

t!J!.= _
dx

t, }
t

and

~ = a cos t.

Whence

a cos t =-cot t.
a sin t

3. The derivative of an implicit function. If the relationship between x


and y is gJven in implicit form,
F (x, y) = 0,

(1)

then to find the derivative y~ == y' in the simplest cases it is sufficient: 1) tocalculate the derivative, with respect to x, of the left side of equation (I),
taking y as a function of x; 2) to equate this derivative to zero, that is, to put
d
dx F (x, y) =0,

(2)

and 3) to solve the resulting equation for y'.


Example 3. Find the derivative y~ if

x+y-3axy=0.

(3)

Solution. Forming the derivative of the left side of (3) and equating it.
to zero, we get
3x 3yy' - 3a (y XI/') = 0,

DiOerentiation of Functions

58

[Ch. 2

whence

581. Find the derivative x~ if


a) y=3x+x l

b)

y=x-~ sinx;
x

c) y=O.lx+ea .

In the following problems, find the derivative


functions y represented parametrically:

89 f x=a cos i,
5 . '\
.

x=2t-l,
y=t.

5 8{2 . ,
X

583.

585.

(_t_)'

1+ t

= 1+ tS ,

~=

Vi ,
VI.

592.

Y=

{ x = e- ,
t

693.
594.

, 1

+t

y=elt

{x = a( .In tan {+ cos t- sin t ) ,


y=a(slnt+cost).

x=a(cost+tsint),
when t =

~ if

x= a (t- sin t),


{ y=a(l-cost).
dy
Solution. dx

Yeos 2t '
sinS t
V cos 2t

Y = arc sin .. r - .
2

588. {
Y= a (sin t-t cos t).

:~

1
\

Yt:l+l

595. Calculate

( X = arc cos .r 1
,
,. 1 + t Z

{X =" ~T'
y

3at 2
+ t'

t.

cos't
X

591. if

3at

Y=

Sin

590. \ y= b sin' t.

a (1-12)

y=b

J x = a cos' t

'

= I ~Itl ,

586. \ Y =
587.

t+ 1

y
X

584.

= t~I

y' = :~ of the

a sin t
sin t
a (l-cos t) = I-cos t

Sec. 3]

The Derivatives of Functions Not Represented Explicitly

59

and

( dx

1t

sin "2

d Y)

n: =

t=-;:-

l-cos-

x=tlnt,

596. Find ddYx when t


d dy
7 F In
59.
d-x

1.

= 1 if {
In t
Y=-t'
t
w hen t __ .::.. I' f { X = et co. 5 t,
4
Y =e sIn t.

598. Prove that a function y represented pararrletricaJly by the


equations

satisfies the equation


Y=

(dY)'
(dy)1
dx
+2 dx

599. When x = 2 the following equation is true:

x' = 2x.
Does it follow from this that
(Xl)'

= (2x)'

when x == 2?
600. Let y= Val-xl. Is it possible to perform term-by-term
different iat ion of
Xl + yl == al?
In the examples that follow it is required to find the derivative y' =:~ of the implicit functions y.
601. 2x-5y+lO=O.

609. acosl(x+y)=b.

602. ::+~:= 1.

610. tany=xy.
x

603. x+y=a'.
611. xy=arctan-.

I
l
O
Y
64
o. x -}-xy+y =
612. arctan
(x+y)=x.
605.

Vx-t-VY=Vli.

606.

Vx -f-V y 2=Va
l

x-y

607. y'=-.
x+y

608. y-O.3 siny=x.

613. e>'=x+y.
11
l

614. Inx+e-X- =c.

615. lny+~=c.
Y

616. arc tan

~ = ~ In (x' + y').

60

DiUerentiation of Functions

617.

Vx 1+y2=carctan lLx .

lCJ1. 2

618. x'=yx.

619. Find y' at the point M (1,1), if


2y:=; 1 + xy.

Solution. Differentiating, we get 2y' = y' 3xy 2 y'. Putting x = 1 and


y=l, we obtain 2y'=1+3y', whence y'=-l.

620. Find the derivatives y' of specified functions y at the


indicated points:

a) (x + y)' = 27 (x-y)
b) ye' = eX+ J
c) yl ==

+ In JLx

for x=2 and Y= I;


for x=o and Y= I;
for x=1 and y= 1.

Sec. 4. Geometrical and Mechanical Applications of the Derivative


1. Equations of the tangent and the normal. From the geometric significance of a denvative it follows that the equation of the tangent to a curve
:1=/ (x) or F (x, y) =0 at a point M (x o, Yo) will be
.

\vhere y~ is the value of the derivative y' at the point M (x o, Yo)' The straight
line passing through the point of tangency perpendicularly to the tang~nt is
called the normal to the curve. For the
normal we have the equation
8

x-xo+Y: (y-Yo)=O.
2. The angle between curves. The
angle between the curves
Y=/l (x)

and

Y=f2 (x)

at their common point M o (x o' Yo) (Fig. 12)


is the angle ro between the tan~ents
MoA and MoB to these curves at the
point Mo.
Using a familiar formula of analytic geometry, we get
F' 12
Ig.

3. Segments associated with the tangent and the normal in a rectangu lar
coordinate system. The tangent and the normal determine the following four

Sec 4]

Geometrical and Mechanical Applications of the Deritatit'e

segnlents (Fig. 13):


t = TM
St=TK
n == N M
Sn = KN

61

is the so-called segment of the tangent,


is the subtangent,
is the segment of t he normal,
is the subnormal.

If Sn N

St

Fig. 13
Since KM = , Yo

t = TM=

and tan q> = Yo, it follows that

I~o~ V + (Y~)! I;

St=TK=/

I VI + (Y~)! I;

n = N M = Yo

~:I; Sn=IYoy~l

4. Segments associated with the tangent and the normal in a polar system of coordinates. If a curve is given In polar coordinates by the equation r=f(cp), then the angle J1
fornled by the tangent MT and the
radius vector , = OM (Fig. 14), is
defined by the following fonnula:
dq>

tanll.=r
-=-.
r
dr
r'
The tangent MT and the normal MN
at the point M together with the radius vector of the point of ta ngency
a nd with the perpendicular to the
radius vector drawn through the pole
o determine the following four segments (see Fig. 14):

x
T
Fig. 14

t = MT is the segment of t he polar tangent,


n = MN is the segment of the polar nornzal,
St=OT is the polar subtangent,
Sn = ON is the polar subnormal.

[Ch. 2

Differentiation of FunctIons

62

These segments are expressed by the following fornlutas:


,

2
t=MT=-,
,r , Yr +(r')I;

n=MN=Yrl+(r')I;

rl

St=OT=-1r-'I;
Sn=ON=lr'l

621. What angles cp are formed with the x-axis by the tangents

to the curve y =

X-Xl

at points with abscissas:


a) x=O; b) x=1/2; c) x=l?
Solution. We have y' = 1-2x. Whence
a) tan q> = 1, q> =45; b) tan q> =0, q> =0;
c) tan q> =-1, cp = 135 (Fig. 15).

622. At what angles do the sine


curves y = sin x and y = sin 2x intersect the axis of abscissas at the
origin?
Fig. 15
623. At what angle does the tangent curve y = tan x intersect the
axis of abscissas at the origin?
624. At what angle does the curve y = eO. IX intersect the
straight line x=2?
625. Find the points at which the tangents to the curve
y==3x 4 +4x'-12xl +20 are parallel to the x-axis.
626. At what point is the tangent to the parabola

y=x l -7x+3
parallel to the straight line 5x+ y-3 = O?
627. Find the equation of the parabola y = Xl + bx+c that is
tangent to the straight line x=y at the point (1,1).
628. Deter4mine the slope of the tangent to the curve Xl + yl - xy-7=O at the point (1,2).
629. A t what point of the curve yl = 2x' is the tangent perpendicular to the straight line 4x-3y+2=O?
630. Write the equation of the tangent and the nornlal to the
parabola
Y=
x
at the point with abscissa x = 4.

v-

Solution. We have y' =


k = [y'l." = 4 =

~_;

whence

the

slope

of

the tangent

is

Since the ;ointof tangency has coordi nates x = 4, Y = 2, It


follows that the equation of the tangent is y-2=lj4(x-4) or x-4y+4=O.
Since the slope of the normal must be perpendicular.
k 1 =-4;
whence the equation of the normal: y-2=-4(x-4) or 4x+y-18==O.

Sec. 4J

Geometrical and Mechanical Applications of the Derivative

63

631. Write the equations of the tangent and the normal to the
curve y=x'-l-2x l -4x-3 at the point (-2,5).
632. Find the equations of the tangent and the normal to the
curve
Y= x-I
at the point (1,0).
633. Form the equations of the tangent and the normal to the
curves at the indicated points:
a) y = tan 2x at the origin;
b) y=arc sin x 2 I at the point of intersection with the
x-axis;
c) y=arccos3x at the point of intersection with the y-axis;
d) y = In x at the point of intersection with the x-axis;
e) y=e1 - x2 at the points of intersection with the straight
line y = 1.
634. Write the equations of the tangent and the normal at the
point (2,2) to the curve

V-

1 +t

x=-ta ,
3

Y=2t 2 + 2t.

635. Write the equations of the tangent to the curve

x = t cos t,

Y = t sin t

at the origin and at the point t = ~ .


636. Write the equations of the tangent and the normal to the
curve X l .t- y 2+2x-6=O at the point with ordinate y=3.
637. Write the equation of the tangent to the curve X S + yS_
--2xy==0 at the point (1,1).
638. Write the equations of the tangents and the normals to
the curve y = (x-I) (x-2) (x - 3) at the points of its intersection
with the x-axis.
639. Write the equations of the tangent and the normal to the
curve y4=4x4+6xy at the point (1,2).
640. Show that the segment of the tangent to the hyperbola
xy==a 2 (the segment lies between the coordinate axes) is divided
in two at the point of tangency.
641. Show that in the case of the astroid x 2/1 + ylll = a2/1 the
segment of the tangent between the coordinate axes has a constant value equal to a.

DIOerentiatlon of Functions
[Ch. ~
64
--.
- - - - ,-_---=:.._----=-----------=-------

642. Show that the normals to the involute of the circle

x=a(cost+t sint), y=a(sint-tcost)


are tangents to the circle x l +yl=a 2.
643. Find the angle at which the parabolas y = (X-2)2 and
Y= --4 +6x-x 2 intersect.
644. At what angle do the parabolas y = Xl and y = Xl intersect?
645. Show that the curves y=4x l +2x-8 and y=xs-x+lO
are tangent to each other at the point (3,34). Will we have the
saIne thing at (-2,4)?
646. Show that the hyperbolas
XY= a 2 ;

x 2 _y= bl

intersect at a right angle.


647. Given a parabola y2=4x. At the point (1,2) evaluate the
lengths of the segments of the subtangent, subnormal, tangent,
and normal.
648. Find the length of the segment of the subtangent of the
curve y =-= 2x at any point of it.
649. Show that in the equilateral hyperbola X 2_ y 2=a 2 the
length of the normal at any point is equal to the radius vector
of this point.
650. Show that the length of the segment of the subnormal
in the hyperbola Xl_yl = a2 at any point is equal to the abscissa
of this point.
651. Show that the segments of the subtangents of the ellipse
x2
(i2+ fj2y2 = I and the circle x 2.+ yl = at at points with the same
abscissas are equal. What procedure of construction of the tangent to the ellipse follows from this?
652. Find the length of the segment ot the tangent, the nor..
mal, the subta..ngent, and the subnormal of the cycloid

= a (t - sin t),
y = a (1 - cos t)

X
{

at an arbitrary point t = to.


653. Find the angle between the tangent and the radius vector

of the point of tangency in the case of the logarithmic spiral

r=ae kCP
654. Find the angle between the tangent and the radius vector of the point of tangency in the case of the lemniscate
r l = a l cos 2q>.

Sec. 4]

Geometrical and Mechanical Applications of the Derivative

65

655. Find the lengths of the segments of the polar subtangent,


subnormal, tangent and normal, and also the angle between the
tangent and the radius vector of the point of tangency in the
case of the spiral of Archimedes

r=acp
at a point with polar angle <p=2n.
656. Find the lengths of the segments of the polar subtangent,
subnormal, tangent, and normal, and also the angle between the tan
gent and the radius vector in the hyperbolic spiral r = ..!:.. at an
cp

arbitrary point cp = CPo; r = r o.


657. The law of ITIotion of a point on the x-axis is

x=3t-t'.
Find the velocity of the point at to=O, t 1 =1, and /2=2 (x IS
in centimetres and t is in seconds).
658. Moving along the x-axis are two points that have the
following laws of motion: x= 100+5t and x= 1/2/ 1 , where t~O.
With what speed are these points receding from each other at
the time of encounter (x is in centinletres and t is in seconds)?
659. The end-points of a segment AB =-~ 5 nl are sl iding along
the coordinate axes OX and OY (Fig. 16). A is moving at 2ol/sec.
y

Fig. 16

Fig. 17

What is the rate of motion of B when A is at a distance OA = 3 m


from the origin?
660*. The law of motion of a material point thrown up at an
angle a to the horizon with initial velocity V o (in the vertical
plane OXY in Fig. 17) is given by the formulas (air resistance is
3-1900

[Ch.2

Difjerentlation of Functions

66

disregarded):
X= V o t

coso,

where t is the time and g is the acceleration of gravity. Find the


trajectory of motion and the distance covered. Also determine the
speed of motion and its direction.
661. A point is in motion along a hyperbola y = 10
so that its
x
abscissa x increases uniformly at a rate of 1 unit per second.
What is the rate of change of its ordinate when the point passes
through (5,2)?
662. At what point of the parabola y2 = IBx does the ordinate
increase at twice the rate of the abscissa?
663. One side of a rectangle, a = 10 em, is of constant length,
while the other side, b, increases at a constant rate of 4 cnl,'sec.
At what rate are the diagonal of the rectangle and its area increasing when b = 30 cm?
664. The radius of a sphere is increasing at a unifortTI rate
of 5 cm/sec. At what rate are the area of the surface of the
sphere and the volunle of the sphere increasing when the radius
becomes 50 cm?
665. A point is in motion along the spiral of Archimedes
r=acp
(a = 10 cm) so that the angular velocity of rotation of its radius
vector is constant and equal to 60 per second. Determine the rate
of elongation of the radius vector r when r = 25 cm.
666. A nonhomogeneous rod AB is 12 cm long. The mass of a
part of it, AM, increases with the square of the distance of the
moving point, M from the end A and is 10 gm when AM =- 2 em.
Find the mass of the entire rod AB and the linear density at
any point M. What is the linear density of the rod at A and B?
Sec. 5. Derivatives of Higher Orders
to. Definition of higher derivatives. A derivatzve of the second order, or
lhe second derivative, of the function y= f (x) is the derivative of its derivalive; that is,

y" = (y')'.

The second derivative may be denoted as


1
d2 y

II,

If

X=

f (I)

or

d,x2'

or

f" (x).

Is the law of rectilinear motion of a point, then

eration of thIs motion.

:;~

is the accel-

Derivatives of Higher Orders

Sec. 5]

67

Generally, the nth derivative of a function y=/ (x) is the derivative of


derivative of order (n-l). For the nth derivative we use the notation

y (n),

dny

or

dx n

f (n) (x).

or

'

Example l. Find the second derivative of the function


y=ln(l-x).
'
1
,
,
(
- I )'
1
I Ii on. Y=l-x; Y= I-x
Sou
=(I-x)2'
2. Leibniz rule. If the functions u=<p (x) and v ='1' (x) have derivatives
up to the nth order inclusive, then to evaluate the nth derivative of a product of these functions we can use the Leibl1iz rule (or formula):
(uv) (n) = u(n)v

+ n.u (n-I, v' + n (~~ I) UC-

2)

v"

+ ... + uv(n).

of functions represented parametrical ~j'. if

3. Higher-order derivatives

x=<p(t),
{ Y='i'(t),

then the derivatives

y;= :~ ,

can successively be calculated

by the formulas

" - ( ')'x -- ( y'~);


,

y xx -- y x

(Y:x)~

II'

y xxx = - - , - and so iorth.

x,

Xt

For a second derivative we have the formula


"

Yxx=

, "
x t Ytt - XttY,
( x~)a

Example 2. F\nd y", if


X=Q
{

cos t,

Y =- b sin

t.

Solution. We have
y'=

(b sln

t) ~

,=

(a cost),

b . cos t
b
= - - cot t.
-a SUI t
Q

and

'l'

-~ colt ):
(a cos t);

b
-1
--a.
Sfi1i7

-a sin t

=- -

at sin' t

68

[Ch.

Differentiation of Functions

A. Higher-Order Derivatives of Explicit Functions


In the examples that follow, find the second derivative of thl
given function.
667. y=x l +7x 8 -5x+4.
671. y=ln(x+Va 2 +x Z ).
668. Y = eX!.
672. f (x) = (1 + Xl). arc tan x.
673. y=(arcsinx)l.
669. y = sin l x.
670. y= In

VI +x

674. y=acosh~ .

675. Show that the function y = Xl + 2x


2 + 2 sa tIS fles th e dlfTer
ential equation 1 +y,z=-=2yy".
676. Show that the function y = -} x'e x satisfies the differen
tial equation y"-2y' +y=e x .
677. Show that the function y == C 1e- x -1- C 2 e- 1X satisfies th
equation y" +3y' -1-2y=0 for all constants C 1 and Cz.
678. Show that the function y = eZx sin 5x satisfies the equa
tion y" -4y' + 29y = O.
679. Find y"', if y=x 3 -5x 2 -f-7x-2.
680. Find f"' (3), if f (x) = (2x-3)5.
681. Find yV of the function y::= In (1 + x).
682. Find yVI of the function y = sin 2x.
68:3. Show that the function y = e- x cos x satisfies the differ
ential equation yIV +4y==0.
y
684. Find {(O), f' (0), f"(O) and f'" (0:
if f (x) = eX sin x.
685. The equation of motion of a paill
along the x-axis is

x = 100 + 5t-0.00lt 3
1.40 X

Find the velocity and the acceleration (


the point for tinles to = 0, t 1 = 1, an

= 10.

686. A point M is in motion around


circle x2 y2 = at with constant angul2
Fig. 18
velocity 00. Find the law of motion of i1
projection M, on the x-axis if at time t =
the point is at M o (a, 0) (Fig. 18). Find the velocity and the a<
celeration of motion of MI'
What is the velocity and the acceleration of M 1 at the in
tial time and when it passes through the origin?
What are the maximum values of the absolute velocity and tt
absolute acceleration of M J ?

'ec. 8J

69

l),rlvatlves of 811her Orders

687. Find the nth derivative of the function U= (ax + b)lI,


where n is e Il.atural number.
688. Find the nth derivatives of the functions:
1

1/-

b) y= v x.

and

8) 1/=-1-x;

689. Find the nth derivative of the functions:


1

a)lJ=sinx;

e)Y=I+x;

b) y = cos 2x;

f) y =

c) y = e-' ;
d) g=ln(l+x);

g) y = sin l x;

Jt

+: ;

h) y=ln(ax+b).

690. Using the Lei bniz rule, find ycn), if:


l+x

a) y=x.e";

d) y=

b) y=x l e- 2X ;

c) y == x 3 In x.

Yx;

y=(l-xl)cosx;

c)

691. Find fln) (0), if

f (x) = In I

Ix

B. Iltgher-Order Derivatives of Functions Represented


Parametrically and of Implicit Functions
In the following problems find

:~ .

692. a) {x=ln t, . b) {x=arctant,


y=t l ;
y=ln(1-t-t 2 );

x=acos t,

693. a) {

y = a sin t;
X=

a cos' t,

b) { y=asin't;

x=cos2t,

694. a) { y = sinl t;
b) {
.

d1x

If

y=l-t l

{x=a(t-sint),
.
y = a (1 - cos t),
{ x = a (sin t - t cos t),
d)
y=a(cost+t sint).
{ x=arctant,
695. a)
y = ~ tl ;

c)

x=e-at,
at
y=e

696. Find d---a,

{x=arcsint

c)

{ x=lnt,
b)

Y=l-t'

{X =

e' cos t,

t . t
Y ==e sin

Differentiation of Functions

70

d2y

. {x=ln(l+t

[Ch.2
2
),

697. Pmd dx 2 for / =0, If Y= /2.


698. Show that y (as a function of x) defined by the equations x = sin t, Y = aet Vz + be- t Vi-for any constants a and b
satisfies the differential equation
2

d2 y

(l-x)dx l

dy

x dx=2y.

:1;1 .

In the following examples find y'" =


X = sec i,
{ x = e- t ,
699. {
t
701.
a
y= an t.
y=t .
x=e-t cost,
.
dny
{ X= lo t,
700. { y=e- t 'sin.
. t
702. Find -d
n' if
m
x
y=t.
703. Knowing the function y = / (x), find the derivatives x",
x'" of the inverse function x = /-1 (y).
704. Find y", if x2 .-t- y2 = 1.
Solution. By the rule for differentiating a composite function we have
x an d !:J,1' = - -x )' =----2-.
y -xy'
2x-f- 2yy ' = 0 ; wh ence y ' = - Y
y x
y
Substituting the value of y', we finally get:
,
y2+ X 2
1
y' = III '

l'

---r=-

In the following examples it IS required to determine the


derivative y" of the function y = / (x) represented implicitly.
705. y2 --; 2px.
x2

706. Q2

y2

+ b2 =

1.

707. y=x-tarctan y.

708. Having the equation y = x + In y, find ~2;t and ::~.


709. Find y" at the point (1,1) if
x 2 +5xy+y2_2x+y-6=0.
710. Find y" at (0,1) if

x-xy+y4= 1.
711. a) The function y is defined implicitly by the equation
Xl
2xy + yl-4x-t- 2y-2 = O.

Find :~ at the point (1,1).


b)

dly

Find dx"

if x 2t y2 = al.

Sec. 6]

DiOerentials of First and Higher Orders

11

Sec. 6. Differentials of First and Higher Orders


1. First-order differential. The diOerential (first-order) of a function,
Y = f (x) is the principal part of it~ increment, which part is linear relative
to the increment I1x=dx of the independent variable x. The differential of a

o
Fig. 19
function is equal to the product of its derivatIve by the differential of the
independent variable
dy== y'dx,

whence
1/
1

:::::.dy .

dx

If M N is an arc of the graph oi the function y = f (x) (Fig. 19), MT is the


tangent at M (x, y) and
PQ

=~x==dx,

then the increment in the ordinate of the tangent


AT=dy

and the segment AN = ~y.


Example t. Find the increment and the differential of the function
y=3x 2 -x.
Solu tion. first method:
~y =

3 (x+

~X)I_(X+~x) -3x 2

or
~y =

(6x - I) ~x

+x

+ 3 (~X)I.

Hence,
dy= (6x-l) Ax= (6x-l) dx.

Second method:

y' =6x-l; dy=y' dx= (6x-l) dx.


Example 2. Calculate ~y and dy of the function y=3x 2 -x for x=l

and

~x=O.Ol.

Solution.
and

~y= (6x-I). ~x

+ 3 (~X)2= 50.01 + 3 (0.01)1=0.0503

dy=(6x-l) ~x==50.01 =0.0500.

72

DiOerentiation of Functions

[Ch.2

2. Principal properties of differentials.


1) de = 0, where c = canst.
2) dx == /ix, where x is an independent variable.
3) d (cu) = c du.
4) d(u v)=du dv.
5) d (uv)=u dv -f- v duo

6) d ( ~ )
V

= v du -

v2

u do

(0

#= 0).

7) df (u) =- f' (u) du.

3. Applying the differential to approximate calculations. If the increment


Ax of the argument x is small in absolute value, then the differential dy of the
function y=f(x) and the increment fiy of the function are approxilnately
equal;

that is,
whence

~y=:::;dy,

f (x + fix) - f (x) ~ f'

(x) fix,

f (x + fix)

(x) dx.

f (x) + l'

Example 3. By how much (approximately) does the side of a square change


if its a rea increases from 9 m2 to 9.1 m2 ?
Solu lion. If x is the area of the sq uare and y is its si de, then

y=yx.
It is gi ven that x = 9 and ~x = o. 1.
The increment fiy in the side of the square may be calculated approximatply as follows:
Ay===dy=y' Ax=

~r- O.1=O.016m.

2 f 9

4. Higher-order differentials. A second-order dtOerential is the differential


of a first-order differential:

We similarly define the dtOerentials of the third and higher orders.


If Y == f (x) and x is an independent variable, then
d 2 y = y" (dX)2,
d 3 y = y'" (dx)',

dny

= y(n) (dx)n.

But if y = f (u), where u = cp (x), then

d 2y = y" (dU)1 y' diu,


day == y'" (du)a 3y" du. d 2 u y' diU

and so forth. (Here the primes denote derivatives with respect to u).

712. Find the increment ~y and the differential dy of the function U=5x-t Xl for x=2 and L\x=O.OOl.

Sec. 6]

DiOerentlals of First and Higher Orders

73

713. Without calculating the derivative, find


S
d (I-x )
1

for x = 1 and ~x = - -;;.


v
714. The area of a square S with side x is given by S=x l
Find the increment and the differential of this function and explain the geonlctric significance of the latter.
715. Give a geometric interpretation of the increment and
differential of the follo\ving functions:
a) the area of a circle, S = nx 2 ;
b) the volume of a cube, v=x s
716. Show that \vhen ~x -+0, the increnlent in the function
y = 2x , corresponding to an increment ~x in x, is, for any x,
equivalent to the expression 2-'- In 2 ~x.
717. For \vhat value of x is the diITerenti al of the function
y == x 2 not equivalent to the incretnent in this function as L\x ---.. O?
718. Has the function Y==lxl a differential for x=O?
719. Using the derivative, find the differential of the fUllction

y = cos x for x = ~ and /),X --= 3~


720. Find the diITerential of the function
2

Y==

Yx

for x==9 and L\x==-O.OI.


721. Calculate the diITerential of the function
Y == tan x
Jl

for x::=. 3 and ~x -== 180


In the following problenls find the differentials of the given
functions for arbitrary values of the argunlent and its increluent.
1
722. Y=--=x tn
727. y=xlnx-x.
723

728.

724. !J= arc sin : .

729. r = cot <p

x
725. Y = arc tan -a .

730. s = arc tan e

x
'Y==T-x

I I-x

y=nl-f-x

+ cosec <p.
t

726. Y = e- x2
731 Find dy if x 2 -}-2xy- y l = a l
Solution. Taking advantage of the invariancy of the fOrln of a differential,
we obtain 2xdx-{-2(ydx+xdy)-2ydy=O
Whence
X+Yd x.
dy=--x-y

74

DiOerentiat ion of Funct ions

In the following examples find the differentials of the functiollS


defined implicitly.
732. (X+y)2. (2x+y)' = 1.
x

733. Y = e-Ii .
734. In V Xl

+ y. = arc tan JLx

735. Find dy at the point (1,2), if y'-y=6xl


736. Find the approximate value of sin 31.
Solution. Putting x=arc300=i and Ax=arc 1= 1~0' from formula (1)
(see 3) we have sin 31:::::::: sin 30

+ 1;0 cos 30=0.500+0.017. ~3 =0.515,

737. Replacing the increment of the function by the differential, calculate approximately:
a) cos 61;
d) In 0.9;
b) tan 44;
e) arc tan 1.05.
c) eO. 2 ;

738. What will be the approximate Increase in the volume of


a sphere if its radius R = 15 cm increases by 2 mm?
739. Derive the approximate formula (for I ~x \ that are small
compared to x)

vx + ~x ~ VX +

~x

lJsing it, approximate Y-5, V17, V70, V640.


740. Derive the approximate formula

- V-x + V=
ax
Vx+~x~
3
Xl
and find approximate values for VI0, V70, V200.
741. Approximate the functions:
8) y=x'-4x l +5x+3 for x=I.03;
b) f(x)=Vl +x
for x=O.2;

f (x) =

for X= 0 1 ;
d)y =e f or x= 105
..
742. Approximate tan 453'20".
743. Find the approximate value of arc sin 0.54.
744. Approximate VI7.

c)

xl

1
l+xx

Mean-Value Theorems

Sec. 7)

745. Using Ohm's law, I

15

=~,

show that a small change in


the current, due to a small change in the resistance, may be
found approximately by the formula

8/=-

I
R~R.

746. Show that, in determining the length of the radius, a


relative error of 1% results in a relative error of approximately
2 loin calculating the area of a circle and the surface of a sphere.
747. Compute diy, if Y = cos 5x.
Solut ion. d 2 y = y" (dx 2) = - 25 cos 5x (dX)2.

11

748. u = i-x2 , find d 2 u.


749. y = arc cos x, find d 2 y.
750. y = sin x In x, find d 2 y.
In x f d d 2
751. z=-X'
In
z.
752. z = x2 e- X , find d'z.
753.

xo&

Z = (-)- ,

... -x

find d

01

z.

754. II = 3 sin (2x -t- 5), find dnu.


755. y = eX cos a sin (x sin u), find dray.
Sec. 7. Mean-Value Theorems
1. Rolle's theorem. If a function f (x) is continuous on the interval
f' (x) at every interior point of tillS interval, and

a ~ x ~ b, has a derivative

f (a) -= f (b),
then the argunlent x has at least one value

I'

~,

where a <

,<

b, such that

(~) =0.

2. Lagrange's theorem. If a function f (x) is continuous on the interval

a ~ x ~ b and has a derivative at every interior point of this interval, then


f (b)-f (a) = (b-a) I'

(6),

where a < ~ < b.


3. Cauchy's theorem. If the functions f (x) and F (x) are continuous on the
interval a ~ x ~ b and for a < x < b have derivatives that do not vanish
simultaneously, and F (b) :/:- F (a), then

f (b) -f (a)

F (b)-F (a)

f'

(~)

= F' (~)'

where a <

<

b.

756. Show that the function f (x) = x-x' on the intervals


-1 ~ x ~ 0 and 0 ~ x ~ 1 satisfies the Rolle theorem. Find the
appropriate values of 6e

DiUerenttation of Functions

76

[Ch.2

Solution. The function f (x) is continuous and differentiable for all values

of x, and f (-1) =f (0) = f (1) =0. Hence, the Rolle theorem is applicable on
the intervals -1 ~ x <: 0 and 0 ~ x < 1. To find we fornl the equation

f' (x) = 1-3.r=O. Whence


and 0 < ~I < l.

~1 = -

-V ~ ;~2= -V ~.

where

-1

<

~1 < 0

757. The function /(x)=V(x-2)'

takes on equal values


1(0)=/(4)= V4 at the end-points of the interval [0.4). Does
the Rolle theorem hold for this function on [O.4]?
758. Does the Rolle theorem hold for the function
f (x) = tan x
on the interval [0, n]?
759. Let

f (x) =

x (x .,- 1) (x

t- 2) (x + 3).

Show that the equation


has three rea i roots.
760. l~he equation

f' (x) =0
eX = 1 t- x

obviously has a root x=O. Show that this equation cannot have
any other real root.
761. Test whether the Lagrange theorem holds for the function
{(x)

= x-x

on the interval (-2,1] and find the appropriate internlediate


value of
Solution. The function I (x) =x-x' is continuous and differentiable for
nil values of x, and I' (x)= 1-3x2 Whence, by the Lagrange fornlula, we
h3ve f(1)-f(-2j=O-6==(I-(-2)]f'(~),
that is, f' (s)~~2
Hence,

s.

1-3~2=-2 and ~=l; the only suitable value is


inequality -2 < ~ < I holds

~=-l,

for which the

762. Test the validity of the Lagrange theorem and find the
clppropriate intermediate point ~ for the function f (x) = X"/. on
the interval [-1,1].
763. Given a segment of the parabola y = x! lying between
two points A (1,1) and B (3,9), find a point the tangent to which
is parallel to the chord AB.
764. Using the Lagrange theorem, prove the formula
sin (x

where

x<;<x+h.

+h) -

sin x = h cos S,

Taylor's Formula

Sec. 8]

77

765. a) For the functions f (x) = r -t- 2 and F (x) = x' -1 test
whether the Cauchy theorem holds on the interval [1,2] and
find ~;
b) do the same with respect to f(x)=sinx and F(x)=cosx
on the interval [0, :].

Sec. 8. Taylor's Formula


If a function f (x) is continuous and has continuous derivatives up to the

(n-I)th order inclusive on the interval a ~ x ~ b (or b ~ oX ~ a), and there


IS a finite derivative fen) (x) at each interior point of the interval, then Taylor's fornzula
(x - a)2 t" (a l +
(x - - a)
- I f'" (a)+ ...
a + (x-a ) t ' (a) + -2-1f (x) == f ()
3t

... + (~;~~~l-I rn-I'(aH (x:~)~ fIn) (;),


where ~==a+8(x-a) and 0<8<1, holds true on the interval.
In particular, when a==O we have (Maclaurin's formula)
f (x) =f (0) +xf' (0)

where

~=Ox,

+fu f"

(0)

+ ... + (~n-:)I

fIn_I) (0)

+ :~ fin) m.

0<8<1.

[~xpand

the polynomial f(x)=x ' -2x l +3x+5 in positive integral powers of the binonlial x-2.
Sol ution. f' (x) == 3x 2 - 4x + 3; til (x) == 6x - 4~ f'" (x) = 6; fen) (x) = 0
766.

for n ~ 4. Whence

f (2) = 11; f' (2) = 7; {" (2) == 8; f"' (2)

= 6.

rherefor~,

x - 2x 2 + 3x+ 5 = II

(x - 2)2 8 (x - 2)1 6
+ (.r - 2). 7 +---.
+--.
2!
31

or
x l -2x 2 +3x+5= II

+ 7 (x -

767. Expand the function


term containing (x+ 1)1.
Solution. 1(") (x) =e X for all n,
eX

2)

+ 4 (X--2)2 -l- (x-2)'.

f (x) = eX in powers of
ten) (-1) =..!... Hence,

=..!- +(x+ I)..!.. + (x+ 1)2 ..!.. + (x + I).! + (x+ 1)


e

21

x -r-l to the

31

41

e~

'

where ~=-1+8(x+l); 0<8<1.

768. Expand the function


the term with (x-I)I.

f (x) = In x in powers of x-I up to

78

[Ch. 2

DiOerent iat ion of Functions

f (x) = sin x in powers of x up to the term conand to the term containing Xl.
770. Expand f (x) = eX in powers of x up to the term contain769. Expand

taining

Xl

ing x n - 1
771. Show that sin (a +h) differs from

sin a -+h cos a


2

by not more than 1/2 h


772. Determine the origin of the approximate formulas:
1
1
V-1 +x~ 1 +2"x-S-x
-1
1
b) V 1 + x~ 1 + "3 x- 9 x

a)

Ixl

<

1,

I x I< 1

2
,

and evaluate their errors.


773. Evaluate the error in the formula

e ~ 2+

1
2!

1, 1

+ 3! T

4i

774. Due to its own weight, a heavy suspended thread lies


in a catenary line y = a cosh -=-.
Show that for small Ix I the
a

shape of the thread is approximately expressed by the parabola


x2

y=a+ 2a

775*. Show that for Ix I~ a, to within

approximate equality
x
ea ~

(~

r,

we

have

the

y-.
a+x
a-x

Sec. 9. The L'Hospital-Bernoulli Rule for Evaluating Indeterminate Forms


1. Evaluating the indeterminate forms

%and :

. Let the single-valued

functions f (x) and cp (x) be differentiable for 0</ x-a I <h; the derivative
of one of them does not vanish.
If f (x) and cp (x) are both infinitesimals or both inflnites as x ~ a; that
is, if the quotient

~, then
co

~(~:)'

at x = a, is one of the indeterminate forms

lim

f (x) _lim l' (x)

x~a <p

(x) - %-+-a <p' (x)

provided that the limit of the ratio of derivatives exists.

or

Sec. 9]

L' Hospital-Bernoulli Rule for Indeterminate Forms

79

The rule is also applicable when a= 00.


If the quotient

~,(~1)

again yields an indeterminate form, at the point

x=a, of one of the two above-mentioned types and I' (x) and cp' (x) satisfy
all the requirements that have been stated for' (x) and p (x). we can then
pass to the ratio of second derivatives, etc.
However, it should be borne in mind that the limit of the ratio f (x)

may exist, whereas the ratios of the derivatives do not tend to any

Ir~~~

(see Example 809).


2. Other indeterminate forms. To evaluate an indeterminate form like
0-00, transform the appropriate product
(x).,. (x). where lim (x)=O and

'1

lim!2 (x) =~, into the quehent /1 (IX) (the

~a

'I

-wJ

(x)

form -0 (or, I( -) (the


1 x

12 (x)

In the case of the indeterminate form 00 -

',IX;
x

lim

if

%~a

duce the expression to the form

1_'t.

'I

(x)
(x)

form 00).
00

one should transform the

00,

[I-~: ~:n
',1 (x = 1, then

appropriate difference fdx)-f 2 (x) into the product f'<x)

first evaluate the indeterminate form

'1

and

we re-

1 X

(the form 0-.


0)

(x)

The indeterminate forms I 0<, 00 0 are evaluated by first taking loga e


rithms and then finding the limi t of the logarithm of the power ['I (x 12 ('
(which requires evalua ting a form like O 00).
I n certain cases it is useful to combine the L'Hospital rule with the
finding of limits by elementary techniques.
Example 1. Compute
In x
00
l1m - (form -).
X'~o cot x
00
(X) ,

Solution. Applying the L'Hospital rule we have


lim ~ = Urn (In x)'
%-+0

cot x

We get the indeterminafe form

x~o

(cot x)'

~;

== _ Urn
x-+o

sin

however, we do nof need fo use the

L'Hospital rule, since


l

lim

~-+o

sin x I sin x
--=
1m -sln x= 1-0=0.
x
%-+0 X

We thus finally get


lim In x =0.
cot x

x~o

80

DiOerent iat ion

of Funct ions

[Ch. 2

Example 2. COlTIf'ute

lim

x ~o

(_l__
..!-) (form 00-00).
sin x x
2

Reducing to a common denomi nator, we get

(1
I) =
----

ltm

sin 2 x

x-+o

x2

2
x (form -)
0
ltm x -sin!
x 2 sin! x
0
0

x~o

Before applying the L'Hospital rule, we replace the denominator of the latter fraction by an equivalent infinitesinlal (eh. I, Seco 4) x 2 sin! x_x4 We
obtain
x 2 - sin 2 x
11m - . - - -2 = Iun
4
(form -0).

. (1

1)

S111 2 X

x-+o

X-+O

The L 'Hospital rule gives


1- - 1) - 11m 2x-sin 2x - 11 m 2-2cos 2x
1Jm ( sin 2 x x 2 - x-+o
4x 8
x-+o
12x2
0

x-.+o

Then. in elementary fashion, we find

lim
x-+o

(_I__
.-'!')=liIn l-cos2x=lim 2sin2x=~.
sin x x
6x
6x
3
2

x-+o

x-+o

Example 3. Compute
a

lim (cos 2x) Xi (fonn 100 )


x-+o

Taking logarithms and applying the L'Hospital rule, we get


a

lim ln (tos 2x) X2 = HIn 3ln cos 2x = _ 6 lim tan 2x = _ 6


x-+o

X-i-O

x2

X -+0

2x

lienee, lim (cos 2x)Xi:y:e- s


x-+o

F"ind the indicated limits of functions in the following exatTIpleso


XS_2XI_X+2
776 1lin 3-7 +6
X-+l
x
X

Iimx'-2xl-x+2_Ilm3x2-4x-I_1

_
__
So IU t Ion.
X-'l
x'-7x+ 6
X-+J
3x 2 -7
2

777.

rlin x cos x-sin


x
a

x -+Q

778. lim
.r--+J

ot

779. lim cosh x-I


1 -cos x
x~o

I-x
1- sin ~
2

780.

tan x--sin x

lim x-sinx
.t~o

L' Hospital-Bernoulli

Sec. 9]

781. hmn

Rule for Indeterminate Forms

sec 2 x-2tanx
I -1- cos 4x

785. lim _x_ .

x?4

x-+cotj(x

tan x
782 1Im
nt- an Sx

2
. In (sin nlx)
1
786. 1m 1

x---+o
n sin x

X-i--

eX

783. hm --;.

787. lim (1- cos x) cot x.

t'---+oo X

.
784. 11m

81

x -+0

In x
Vi -

X-~'7J

Solution.lil11 (l-cosx)cotx=lim (I-cosx)cosx== litn (I-cosx).t=z


x-?o

x-?o
c:: lilll
X

-?>O

sin

x -+0

Sill X

SlIl .\.

= 0

cos \"
nx

792. linl x n sin!:., n>O.

788. liln (I-x) tan "2 .


.\"-...,.1

X-x'

789. lilll arc sin x cot x.

793. hnllnxln(x-I).

x~o

1)

\- - (794. I lIn

790. Hln (x ll e- X ), n>O.

X+l

t"~0

.\-1

Inx

791. Hln x sin!!:.. .


x

. Itnl
.
Solutaon.
X~1

= Htll

1) =Itm. xlnx-x+1
==
In \

(X
----In x

.\-}

I
..\.--+Inx-I
x
I
=linl

x-~11I1.\-I-x(\-1)

1
Ill'

x-+1}nx-

X---+l

79~

= ltnl-1- -I-=2
~71~+X2

5)

[ I JI
2 (1 -

x -+-l

( -:--3- . 2 _ -6 .
795. I lIn
x -+3 X
X.\
796. I lIn

(x-I)

X-i-l

'x)

1t)

. (X
hIll
---n cotx
2cosx

1]
V

d.

3 (1 -

x)

t~-

798. liln xx.


x~o

We

Solution.

have

xX=y;

Iny=-xlnx:

x
!

Inx
==lim-=lim--==O, whence limy=l, that
%-+0

-.!X

x-+o _

x2

linllny=limxlnx=x-+o

%-+0

IS, ltnl.\..\.=
%-+0

l.

.r-+o

82

DifJerent iat ion

of

799. linl xx.

804. lim Xl-X.

x-+>+ tI)

800.

[Ch.2

Funcf ions

C-+J

a
limx4 + 1n

tan~

805. lim(tan

%.

%-+0

%-+J

7) ".
1

801. lim x,'n x.

806. lim (cot x)ii1i.

%-+0

%-+0
rEx

802.

lim (l_x)cos a ,

807.

Iim( .!-.)tan
%-+0

~-+1

803. lim (I
%-+0

+ x fi .

808. lim (cot x)~in

%.

%-+0

809. Prove that the limits of


Xl

stnJ.

a) 1Im_.-=o;
x~o

sin x

b) lim x-s~n x = 1
%-+ClDx+slnx

cannot be found by the L'Hospital-Bernoulli rule. Find these


limits directly.

Fig. 20

810*. Show that the area of a circular segment with minor


central angle a, which has a chord AB=b and CD=h (Fig. 20), is
approximately
2
S~-bh

wi th an arbitrari Iy small reI ati ve error when a --+ O.

Chapter 11/

THE EXTREMA OF A FUNCTION AND THE GEOMETRIC


APPLICATIONS OF A DERIVATIVE

Sec. 1. The Extrema of a Function of One Argument


1. Increa~e dna decrease of 'unctions. Thl 1u11ctlon y= f (x) is called
on some Interval if, fo. any points Xl and X 2 which
belong to this Interval, frolll the inequality X I <X2 we ~et the Inequality I (x I )<
<I (x 2 ) (Fig 21a) [{ (XI! (.x 2 ) (Fig. 21b). i i f (x) is conttnuous on the
interval la, b) and f' (xO [t' (x)<OJ for a<. <b, then f (.x) increases (decreases) un the Interval [a. b).
increasing (decreaslItg)

o
(b)

(a)

o
Fig. 22

Fig. 21

111 the sinlplest cases, the don13ln of definition of f (x) may be subdivided Into t.l finite nUlllber of intervals of incrense and decrease of the function (intervals of 11l0notontcity). These Intervals are bounded by Cl itic
poi nts x [where I' (x) =~ 0 or f' (x) does not exist].
Example 1. Test .the following function for increase and decrease:
r

'

y=x 2 -2x+5.
Solution. We find the derivative

y'=2x-2=2(x-l).
Whence y'=O for x= 1. On a nUlnber scale we get two intervals of monotonicity: (-00, I) and (I,
co). Frolll (1) we have: I) if -oo<x<l, then
y'<O, and, hence, the functioll t (x) decreases in the interval (-00. I); 2)
if 1<x< -1- 00. then y'>O. and. hence. t he function f (x) increases in the interval (1,
00) (Fig. 22).

84

Extrema and the GeometrIc Applications of a Derivative

[Ch.3

Example 2. Determine the intervals of increase and decrease of the rune


tion
1
Y=x+2
Solution. Here. x= - 2 is a discontinuity

= -~22<0

of the function and y' =

for x:j:.-2. Hence, the function y decreases in the intervals

(x+ )
- oo<x<-2 and -2<x< + 00.

Example 3. Test the following function for increase or decrease:

1
y == 5
Solution

xS-a1 Xl.

Here,
(2)

Sol ving the equation X4 _X 2 =:.0, we find the points Xl = -1, x 2 =0, xs == 1
at which the derivative y' vanishes. Since y' can change sign onl y when
passing through points at which it vanishes or becomes discontinuous (in the
given case, y' has no discontinuities), the derivative in each of the intervals
(-00, -1), (-1,0), (0,1) and (1, +00) retains its sign; for this reason, the
function under investigation is monotonic in each of these intervals. To
determine in wh ich of the indicated intervals the function increases and In
which it decreases, one has to determine the sign of the derivative In each
of the intervals, To determine what the sign of y' is in the interval (- 00,
-1), it is sufficient to determine the sign of y' at sorne point of the interval; for example, taking X= -2, we get from (2) y' = 12>0, hence, y'>O 111
the interval (-00, -1) and the function in this interval increases Sirnilarly, we find that y'<O in the interval (-1, 0) (as a check, we can take
X= -

~).

y'<O

in the

interval

(0,1)

(here, we can use x = 1/2) and y'>O in the


interval (1, -t- 00).
Thus, the function being tested increases in the interval (- 00, -I), decreases
in the Interval (-1, 1) and again increases
in the interval (I, + 00).
2. Extremum of a function. If there
exists a two-sided neighbourhood of a point
Xo such that for every pC'int x:j:.x o of this
neighbourhood we have the inequality
X,
f (x! (x o), then the point Xo is called the
minimum potnt of the fu nct ion y =-= f (x),
Fig 23
whtle the nUlnber I (x o) is called the mInimum of the function y= f (x). Similarly, if
for any point x=x l of some neighbourhood of the point Xl' the inequality
f (x)<t (Xl) is fulfilled, then X J is called the maximum point of the function
I (x), and f (Xl) is the maximum of the function (F ig. 23). The rTiI nimum
point or maximum point of a function is its ext remal point (bending point),
and the minimum or nlaximum of a function is called the extremUtn of the'
funct ion. If Xo is an extremal point of the function I (x), then I' (x o) = 0, or
f' (xo) does not exist (necessary condition for the existence of an extremum).
The converse is not true: points at which f' (x) =0, or I' (x), does not exist
(critical polnts) are not necessarily extremal points of the function I (x).

Sec. 1]

Tlte Extrelna of a Function

of

One Argument

85

The sufficient conditions for the existence and absence of an extremum of a


continuous function f (x) are given by the following rules:
I. If there exists a neighbourhood (x o-6, xo+6) of a critical point Xo
such that f' (xo for X o- 6<x<x o and I' (x)<O for xo<x<xo +~, then Xo is
the maxinlum point of the function I (x); and if I' (x)<O for xo-6<x<xo
and f' (xO for xo<x<x() + 6, then Xo is the minimum point of the function

f (x).

Finally, if there is SOlne positive number () such that f' (x) retains its
sign unchanged for 0<1 x-xo /<6, then Xo is not an extremal point of the
functIon f (x).
2. Iff' (x o) =.
and t" (xo)<O,
then Xo is the Inaximum point;
iff' (x 0) == 0 and
(x 0) > 0, th en Xo is the 1T1 i n imum po in t; but if f' (xo) = 0,
1" (xo) -= 0, and f'" (xo):#:O, then the point Xo is not an extremal point.
~I\ore generally: let the first of the derivatives (not equal to zero at the
point x o) of the function f (x) be of the order k. Then, if k is even, the
point Xo IS an extrenlal point, nalnely, the l11axinlUl11 point, if t<k) (xo)<O;
Clnd it is the 111ininlum point, if t<k) (xoO But if k IS odd, then X o is not
(n ext re111 a I poi nt .
Example 4. Find the extrenla of the function

y:;= 2x

i +3 V
x

Solution. Find the derivative


y'=2-t-

Equating the

2
V-x+I).
-=V-(

Vx

derivativ~ y'

(3)

to zero, we get:

V x+ 1 ==0.

\VhenCL', we find the critical point Xl =-= -1. From fonnula (3) \ve have: if
x==- - : - h, where h IS a sufficiently sl11all positive nUl1lUer, then y'>O; but
if x=---= -1 h, then y' <0*). lienee, Xl::::: - I IS the maXlmU111 point of the
fu nctton !I, and ymax ::-:-- 1.

Eqllatln~

thr dcnolninator of the expression of y' in (3) to zero, \ve get

V-

x~

o,

whence \\e find the second critical point of the functIon xt=O, \\There there
IS no derIvatIve !I' For x== -h, \ve obviously have y'<O; for x== Iz \ve have
U'>O. Con"cquently, xt=-=O is the Il1inl111UI11 point of the functIon y, and
!/rnin==O (FIg. 24). It is also possible to test the behaviour of the function
at the point x= -1 by means of the second derivative

!I=-, ;/ '
3x V x
Here, y" <0 for Xl = - I and, hence, Xl = - I is the 111aXlInUm point of the
function.
3. Greatest and least val ues. The least (greatest) val ue of a cont inuous
function f (x) on a given interval [a, b] is attained either at the critical
points of the function or at the end-points of the interval {a, b].
*) If It is difficult to deternline the sign of the derivative y', one can
calculate arithmetically by taking for Iz a sufl1cillntly snlall positive number.

86

Extrema and the Geometric Applications of a Derivative

[CIl.8

Example 5. Find the greatest and least values of the function


y=x 8 -3x+3
on the interval -P/2~X~21/1.
Solution. Since

y'=3xl -3,

it follows tha t the critical points of the function yare

Xl

= -I

and

XI =

I.

x
Fig.

Fig. 24

~\J

Comparing the values of the function at these points and the values of the
function at the end-points of the given interval
Y(-I)=5;f(I)=I;Y( - I

~)=4 ~;

Y(2 ~)=ll ~.

we conclude (Fig. 25) that the function attains its least value, m = I, at
the point x= I (at the minimum point), and the greatest value M = II
at the point x=2 1 /. (at the right-hand end-point of the interval).

i-

Determine the intervals of decrease and increase of the fune .


iions:
811.
812.
813.
814.

Y= 1-4x-xl
Y=(X-2)1.
Y = (x -t- 4)1,
y = Xl (x- 3).

815. Y=--!-2.
x-

816.

Y=(x_I)2.

817. Y = xZ 818.

x- Hi
)

y=(x-3)Vx.

Sec. 1]

The Extrema

ot

a Function of One Argument

V-x.

x
819. Y==3-

+sinx.

820. y=x

87

821. Y= X In x.
822. y = arc sin (1

+ x).

824. Y = 2X::-a.
eX
825. y= x'

Test the following functions for extrema:


826. y = x" + 4x -t- 6.
Solution. We find the derivative of the given function, y'=2x+4.
Equating y' to zero, we get the critical value of the argument X= -2.
Since y'<O when x<-2, and y'>O when x>-2, it follows that X= -2 is
the minimum point of the function, and Ymln = 2. We get the same result
by utilizing the sign of the second derivative at the crit ical point y" = 2>0.

827. y=2+x-x l
828. y=x'-3x' +3x+2.
829. y=2x'+3xz -12x+5.

Solution" We find the derivative


y'=6x l +6x-12=6 (x l +x-2).
Equating the derivative y' to zero, we get the critical points Xl =-2
and XI = 1. To determine the nature of the extremum, we calculate the
second derivative y"==6(2x+l). Sinee y"(-2)<O, it follows thatxl =-2
is the maximum point of the function y, and ymax=25. Similarly, we havey" (10; therefore, Xz = 1 is the minimum point of the function y and
Ymln= -2.

830. y=x (x-12)2.


831. y = X (x-I)1 (X-2)'.
x
832. Y=x l +3'

840. y=2 cos

-i +3 cosi-.

841. y=x-ln(l+x).

833. y=

x -2x+2
x-I

834. Y = (X-2~~8-X)
16

835. y-= X (4-x 2 )


836. Y =
837.

842. y=x Inx.

843. Y =

In x,

844. y=coshx.

yXi"+8
x + '
l

x
Y=V xl - 4 '

838. Y= V(.r-I),

845 . y=xe x .

846. Y=XIe-~.

847. y=~ ,

839. Y = 2 sin 2x + sin 4x.


848. y = x - arc tan x.
Determine the least and greatest values of the functions on the
indicated intervals (if the interval is not given, determine the

88

Extrema and the Qeometric Applications of a DerivativI

[Ch. 8

greatest and least values of the function throughout the domain


definition).

~f

;xi'

853. y = Xl on the interval [-1,3].

849. Y= I
850. y = V-x(-10---x-).
851. Y= sin" x + cos" x.

12x + 1
a) on the interval r-1,5];
b) on the interval [-10,12].

854. y = 2x' -1- 3x l

852. Y = arc cos x.


855. Show that for positive values of x we have the inequality
1

x+-~2.
x

856. Determine the coefficients p and q of the quadratic trl'nomial y=xl+px+q so that this trinomial should have a minimum y=3 when X= 1. Explain the result in ~eometrlcal terms.
857. Prove the inequal ity

,It> 1+x

when x + o.

Solution. Consider the function


f (x) :a::,X -(1 +x).

In the usual
Hence,

wa~

we find that this function has

single minimum

f (0) =0.

f (x) > f (0) when x = O.


> 1 + x when x#: 0,

and so eX
8S

we set out to prove.

Prove the inequa1ities:


Xl
858. x- 6 < sinx<x
Xl

859. cos x > 1- '2


x

when x>O.
when x+O.

860. x-"2<ln(l+x)<x

when x>O.
861. Separate a given positive number a into two summands
such that their product is the greatest possible.
862. Bend a piece of wire of length 1 into a rectangle 80 that
the area of the latter is greatest.
863. What right triangle of given perimeter 2p has the greatest area?
864. It is required to build a rectangular playground so that
it should have a wire net on three sides and a long stone wall
on the fourth. What is the optimum (in the sense of area) shape
.of the playground if I metres of wire netting are available?

Sec. 1]

The Extrema of a Function of One ArgulfLCnt

89

865. It is required to make an open rectangular box of greatest


capacity out of a square sheet of cardboard with side a by cutting
squares at each of the angles and bending up the ends of the
resulting cross-like figure.
866. An open tank with a square base must have a capacity
of u litres. What size will it be if the least amount of tin is used?
867. Which cylinder of a given volume has the least overall
surface?
868. In a given sphere inscribe a cylinder with the greatest volume.
869. In a given sphere inscribe a cylinder having the greatest
lateral surface.
870. In a given sphere inscribe a cone \vith the greatest voluIne.
871. Inscribe in a given sphere a right circular cone \vith the
greatest lateral surface.
872. About a given cylinder CirCUITIScribe a right cone of least
volunle (the planes and centres of their circular bases coincide).
873. Which of the cones circumscribed about a given sphere
has the least volume?
874. A sheet of tin of width a has to be bent into an open
cylindrical channel (Fig. 26). What should the central angle cp be
so that the channel will have maxinlunl capacity?

IJ

~,

'f

A~8
a

Fig. 26

875. Out of a circular sheet cut a sector such that when madeinto a funnel it will have the greatest possible capacity.
876. An open vessel consists of a cylinder with a hemisphere
at the bottom; the walls are of constant thickness. What will the
dimensions of the vessel be if a mininlUlTI of rnaterial is used for
a given capacity?
877. Determine the least height h=OB of the door of aver
tical tower ABeD so that this door can pass a rigid rod MN of
length i, the end of which, M, slides along a horizontal straight
line AB. The width of the tower is d < 1 (Fig. 27).

Extrema and the Geometric Applications of a Derivative

[Ch.3

878. A point Mo (x o' Yo) lies in the first quadrant of a coordinate plane. Draw a straight line through this point so tha t the
triangle which it forms with the positive semi-axes is of least area.
879. Inscribe in a given ellipse a rectangle of largest area with
sides parallel to the axes of the ell ipse.
880. Inscribe a rectangle of maximum area in a segment of
the parabola y" = 2px cut off by the straight line x = 2a.
881. On the curve y = 1,~XI find a point at which the tangent
forms with the x-axis the greatest (in absolute value) angle.
882. A messenger leaving A on one side of a river has to get
to B on the other side. Knowing that the velocity along the bank
is k times that on the water, determine the angle at which the
messenger has to cross the river so as to reach B in the shortest
possible time. The width of the river is h and the distance between A and B along the bank is d.
883. On a straight line AB=a connecting two sources of light A
(of intensity p) and B (of intensity q), find the point M that
receives least light (the intensity of illumination is inversely proportional to the square of the distance from the light source).
884. A lamp is suspended above the centre of a round table
of radius r. At what distance should the lamp be above the table
~o that an object on the edge of the table will get the greatest
!1lunlination? (The intensity of illumination is directly proportional to the cosine of the angle of incidence of the light rays and
is inversely proportional to the square of the distance from the
\ig ht source.)
885. It is required to cut a beam of rectangular cross-section
01lt of a round log of diameter d. What should the width x and
the height y be of th is cross-sect io n
so that the beam will offer maximum
resistance a) to compression and b) to
p bending?
,~~--:t
:A---~----.vB

Fig. ~

Note. The resistance of a beam to compression is proportional to the area of its crosssection, to bending-to the product of the
width of the cross-section by the square of
its height.

886. A homogeneous rod AB, which


can rotate about a point A (Fig. 28),
is carrying a load Q kilograms at a distance of a em from A
and is held in equilibrium by a vertical force P applied to the
free end B of the rod. A Iinear cent imetre of the rod weighs
q kilograms. Determine the length of the rod x so that the force P
should be least, and find P min.

Sec. 2]

The Direction of Concavity. Points of Inflection

91

887*. The centres of three elastic spheres A, B, C are situated


on a single straight line. Sphere A of mass M moving with velocity v strikes B, which, having acquired a certain velocity,
strikes C of mass m. What mass should B have so that C will
have the greatest possible velocity?
888. N identical electric cells can be formed into a battery
in different ways by combining n cells in series and then combining the resulting groups (the number of groups is ~) in parallel. The current supplied by this battery is given by the formula
1= Nne
N R+n 2 ,

where <fj is the electromotive force of one cell, , is its internal


resistance, and R is its external resistance.
For what value of n will the battery produce the greatest
current?
889. Determine the diameter y of a circular opening in the
body of a dam for which the discharge of water per second Q
will be greatest, if Q = cy Vh-y, where h is the depth of the
lowest point of the opening (Iz and the empirical coefficient care
constant).
890. If Xl' XI' .. , x n are the results of Illeasurernents of equal
precision of a quantity x, then its Inost probable value will be
that for wh ich the sunl of the squares of the errors
n

(}=

~ (x-xi)Z
i=l

is of least value (the principle of least squares).


Prove that the nl0st probable value of x is the arithmetic mean
of the measurements.
Sec. 2. The Direction of Concavity. Points of Inflection
to. Thr concavity of the graph of a function. We say that the graph of a
differentiable function y = f (x) is concave down in the interval (a,b) [concave
up in the interval (al,b l )] if for a < x < b the arc of the curve is below (or
for a < x < bl above) the tangent drawn at any point of the interval (a,b)
or ot the interval (a1,b/)] (Fig. 29). A sufficient condition for the concavity
downwards (upwards) 0 a graph Y=f (x) is that the following inequality befulfilled in the appropriate interval:
f" (x) < 0 If" (x) > 0].
2. Points of Inftectlon. A point (x o, f (x o)) at which the direction of concavity of the graph of some function changes is called a point 0/ inflectIon
(Fig. 29).

Extrema and the Geonletrtc Appllcatlons of a Derivative

'92

For the abscissa of the point of inflection

Xo

[Ch. 3

of the graph of a function

y = {(x) there is no second derivat ive {" (x o) -= 0 or t" (x o). Points at wh ich
I" (x) == 0 or f" (x) does not exist are called critical points of the second kind.
The critical point of the second kind Xo is the abscissa of the point of inflection if {" (x) retains constant signs in the intervals x o-6 < x < X o and
X o < x < xo +6, where 6 is some posi-

tive number; provided these signs are


opposite. And it is not a point of
inflection if the signs of fIt (x) are the
same in the above-indicated intervals.
Example t. Determine the intervals of concavity and convexity and
also the points of inflection of the
Gaussian curve

y=e- x2
Solution. We have

y' = -2xe- x

and

y" = (4x 2 - 2) e- xz .

Equating the second derivative y" to zero, we find the critical points of tHe
second kind
1
X 1 = - Y2
and x t = Y2
These points divide the number scale - 00 < x < + 00 into three intervals:
1 (-00, Xl)' II (Xl' XI)' and III (x z' +(0). The signs of y" will be, re~pec-

I f
I}'

x
Fig. 31

Fig. 30

1ively, +, -, + (this is obvious if, for example, we take one ~oint in each
of the intervals and substitute the corresponding values of x into y) Therefore:
II the curve is concave up when curve

IS

00< x < -

;2 ;2

1 < x < Y2'


1

concave down when- Y2

and

< x < + 00;

The poInts

2) the

( Y2'
\ Ye
1) are

points of inflection (Fig. 30).


It will be noted that due to the symmetry of the Gaussian curve about
the y-axis, it would be sufficient to investigate the sign of the concavity of
this curve on the semiaxis 0 < x <
00 alone.

Asymptotes

Sec. 3]

93

Example 2. Find the points of inflection of the graph of the function


y=
Solution. We have:

V -x+2.
s

if =

-2

2 (x --t- 2) - .
"9

(1 )

It is obvious that y" does not vanish anywhere.


Equating to zero the denominator of the fraction on the right of (I), we
find that y" does not exist for x==-2. Since y">O for x<-2 and y"<Ofor
x> -2, it follows that (-2,0) is the point of inflection (Fig. 31). The tangent at this point is parallel to the axis of ordinates, since the first derivative y'
is infinite at X= -2.

Find the intervals of concavity and the points of Inflection


of the graphs of the following functions:
891. y=x' -6x2 +12x+4.
896. y=cosx.
892. y=(x+l)~.
897. y=-x-sinx.

893.

Y=X~3'

898. y=x1lnx.

Xl

894. Y =

X2+12.

899. y = arc tan x-x.

895. y =

V 4x'-12x.

900. y = (1

+x

2
) eX.

Sec. 3. Asymptotes
to. Definition. If a point (x,Y) is in continuolIs Inotion along a curve

{I::-=

f (x) in such a \\'uy that at least one of its coordinates approaches infinity

(aud at the salllC tinlc the distance of the point fronl sonle straight line tends

to zero), then this straight 1ine is called an asynzptote of the curve.


2. Vertical asymptotes. If there is a nunlber a stich that
Ihn

f ( t) =-= 00 ,

x~a

then the straight line x=a is an aSYlllptote (vertical asymptote).


3 Inclined asymptotes. If there are limits
lim f(x) =k
x
I

X~+CX)

and

rf (x)-ktx] = bt ,

lim
X-++

00

then the straight line y= ktx+ b1 will be an asymptote (a right inclined


asymptote or, when k 1 = 0, a right horizontal asymptote).
If there are limits

E_x_t_'e_rn_a_Q_n_d_t_h_e_G_e_o_m_et_'_ic_A..;..p..;...p_li_ca_t_to_n_s_o-=-'_a
__D_e_'_lv_a_t_tv _e__ [ell. ,1

9_4

and
Hrn [f (x) -krl = b2 ,

x-+-oo

then the straight line y = k1x + b2 is an asymptote (a left inclined asymptote


or, when k 2 = 0, a left horizontal asymptote). The graph of the function y = f (x)
(we assume the function is single-valued) cannot have more than one ri~ht
(inclined or horizontal) and more than one left (inclined or horizontal) asytnptote.
Example t. Find the asymptotes of the curve
x2
Y= xl-l
Solution. Equating the denominator to zero, we get two vertical aSYlnp-

1ote"
and x=1.

x=-l

We seek the InclIned asymptotes. For x


k 1 = lim
~ -~ +

IX)

- = lim
x
x-+-+ 00

-+ 00

we obtain

x
_ _ --1,
1

Y x2 -

. x -x
V-I)Yx -1 ==0,
t'~+oo
x"-l
2

b,=--=lJm(y-x)==ltm
X~+-:r..

-1

Fig. 32
h~nce,

the straight line y=x is the right asymptote. Similarly, whenx---..-oo,


we have
k a= lim

1..= -1;

x-+- CJ) x

ba=

lim (y+x)=O.
x-+-CJ)

Thus, the left asymptote is y= -x (Fig. 32). Testing a curve for asymptotes is simplified if we take into consideration the symmetry of the curve.
Example 2. Find the asymptotes of the curve

y=x+lnx.

95

Asymptute~

Sec. 3]

Solution. Since

lim y=

-00,

-++o

the straight line x=Q is a vertical asymptote (lower). Let us now test the
curve onl y for the inclined right asymptote (since x> 0).
We have:
k=

Urn
X-++(J)

b= lim (y-x)=
X--"'+

1L=1,
x

Urn In X=
X -++

00

00.

00

Hence, there is no inclined aSYlllptote.


If a curve is represented by the paratnetric equations x=cp (t), Y=1J' (t),
then we first test to find out whether there are any val ues of the parameter t
for which one of the functions cp (t) or 1J' (t) becomes infinite, while the other
remai ns finite. When cp (to) = 00 and 1J' (to) = c, the curve has a horizontal
aSYlllptote y=c. When 1J'(t o)=oo and cp(to)=c, the curve has a vertical
aSYlnptote x == c.
If cp (t 0) = 1J' (t 0) = 00 and

lim ' ~) === k; lim ['I' (t) - k({1 (t) 1== b,

I-+t o

cP

t -+ t n

then the curve has an incltned aSYlllptote y==kx+b.


If the curve is represented by a polar eq uation , === f (cp), then we can
find its asynlptotes by the preceding rule after transfortlling the equation of
the curve to the parailletric form by the fortnulas x =, cos cp = f (cp) cos q>;
y
sin cp ==- f (q sin (p.

==,

Find the asynlptotes of the following curves:


901. Y =

Xl

908. y=x-2+V~'
-\1+

(X-2)2

902. Y=x 2 -4x-1-3.


x2

909. y=e- x2 +2.


1

903. Y = x 2 -4

910. Y=-l
-ex

x'

904. Y=Xi+9
905. y =

V xl--l.

906. Y=

Y xl-t
x

907. y=

3
x2 + 1
xl-l

911. y=e x
_sinx
912

y- x

913. y = 1n (1 -t- x).

914. x=t; y=t+2arctant.


a

915. Find the asymptote of the hyperbol ic spiral r=-.


q>

Extrema and the Qeo/netrle Applications of a Derivative

96

[Ch. 3)

Sec. 4. Graphing Functions by Characteristic Points


In constructing the graph of a function, first find its domain of definition
and then determine the behaviour of the function on the boundary of this
domain. It is also useful to note any peculiarities of the function (if there
are any), such as symnletry, periodicity, constancy of sign, Inonotonicity, etc.
Then find any points of discontinuity, bending points, points of inflection,
asymptotes, etc. These elelnents hel p to determine the general nature of the
graph of the function and to obtain a mathematically correct outline of it.
Example t. Construct the gra ph of the function
x
Y=
x2 _ 1 .

Solution.
The function
0(0, 0). This
b) The

lim y=
X~-l

to

a) The function exists everywhere except at the points x== 1.


is odd, and therefore the graph is sytTlmetric about the point
simplifies construction of the graph
discontinuities are x = - 1 and x = 1; and lim y = 00 and

00;

hence, the straight lines x=

~-H+O

are vertical asymptotes of the

graph.
c) We seck inclined asymptotes, and find
kl =

lint
X---++

bl

!I =0,
00

lim y=

00,

x-.+oo

thus, there is no right asymptote. From the symmetry of the curve it follows
that there is no left-hand asymptote either.
d) We find the critical pOints of the first and second kinds, tha tis,
pOints at wh1ch the first (or, respectively, the second) derivative of the giveu
function vanishes or does not exist.
We have: .
x 2 -3
y'
(I)
3
(x 2 -1)'"
2x (9-x 2 )
y"
(2)
9V(x 2 -1)7

The derivatives y' and y" are nonexistent only at x= 1, that is, only at
pOints where the function y itself does not exist; and so the critical points
are only those at which y' and y" vanish.
From (1) and (2) it follows that
y'=o
y"=O

when x= V3;
when x=O and x= 3.

Thus, y' retains a constant sign in each of the intervals ( - 00, - V3)t
-1), (-1,1), (1, 3) and (V~ +00), and y"-in each of the
intervals (-00, -3), (-3, -1), (-1,0), (0, 1), (1,3) and (3, +00).
To determine the signs of y' (or, respectively, y") tn each of the indicated
intervals, it is sufficient to determine the sign of y' (or y") at some one point
of each of these intervals.

(-3:

Sec 4]

91

Graphinl! Functions by Characteristic Points

It is convenient to tabulate the results of such an investigation (Table I),


calculating also the ordinates of the characteristic points of the graph of the
function. It will be noted that due to the oddness of the function y, it is
enough to calculate only for x ~ 0; the left-hand half of the graph is constructed by the principle of odd symmetry.
Table I

II"

(0, 1)

IJ

SlOlh

1(1,

oo

--

IeXIst
non-

\lIon
l~X I~t

----Conelu-

Point
of

mllcctlon

Function
I t::a-;c~,
grdph h

del

lOl1l.1ve
do\\n

V3) I V3~1. 73 1( V3. 3) I

V3

----=~1.37

-1-

I
I

V'2

Fund Ion
Ol,-"on- dcer ea~c'5,
ttnui t I graph IS
conCAve
up

1(3. +00)

1.5

-t-

I I

Min.
point

FunctlOn
Inrrca-,e'l;
graph
IS lOlll3ve
up

+
0

Point Function
il1r:-rcn"cc;;
of
graph
illf1ec15
con.::ave
han
down

e) Using the results of the investIgation, \ve construct the graph of the
function (FIg 33).

,
----.-r---r----1~~r--~-~--~I---__

Fig. 33
4-1900

98

Extrema and the Geometric Applications

of a Derivative

Example 2. Graph the function


In x
y=-.
x

Solution. a) The domain of definition of the function is 0 < x < + 00.


b) There are no discontinuities in the domain of definition, but ae; we
approach the boundary point (x = 0) of the domain of definition we have
In =
x - 00.
I Imy= I'1m x-+-o

x-+-o

~Ience,

the straight line x=O (ordinate axis) is a vertical asymptote.


c) We seek the right asymptote (there is no left asymptote, since x cannot tend to - 00):
k=

b=

lim lL-o
X -

X--++(SJ

y=O.

lim
X~+(SJ

The right a~ymptote is the axis of abscissas: y =0.


d) We find the critical points; and have

I-Inx

Y =-x-2- '

,/' = 2 In x-3 .

x3

'

'I' and yIP exist at all points of the domain of definition of the f unction and
y'=O when Inx=l, that is, when x=e;

if =0

when In x =

that is, when x =e*,

We form a tabl~, including the characteristic points (Table II). In addition


to the characteristic points it is useful to Hnd the points of intersection of

Fig. 34
the curve with the coordinate axes. Putting y=O, we find x= 1 (the point
of intersection of the curve with the axis of abscissas); the curve does not
;ntersect the axis of ordinates
e) Util izing the results of investigation, we construct the graph of the
.unction (Fig. 34).

Table II

I
u

i
I
I

I
I

(1, e)

I;

I
-cc

y'

noneXlSt.

I
I

nonexist

I
1

I
-;-

I
I

(3-.+00)

4.49

I
3

JI--~O
33
3

concave.:

gTdph

do\'"n

I
I

I
I

Funct

II
I

,
IIICrCtl"cl::.

-r

-+-

ctlOO

Vert1C.al
asymptote

(e. e+) I e-

I
Boundary
pOint of
domain or
dt..f. 0' :un-

e:::::: 2.72

-::::::0.37
e

I
Conclu
Slons

I
I

i
:
I

II

y"

(0. 1)

II

F ,In' t

11" 1

RI .lr It

I"
~

conca\ c

do\\. n

Funct

1"1(r

IS.

J:!f,lph
lone-ave
do\\n

I
I
I

J\\ax
pOInt
Of

funl1.

Funct
decr .
graph
l~ c.oncave
dov.n

POlnt or
inflec t lon

Function
decreases;
graph
l~ concave
up

Extrema and the Geo/netrlc Applications of a Derivative

100

[Ch.

Graph the following functions and determine for each function


Its domain of definition, discontinuities, extremal points, intervals of increase and decrease, points of inflection of its graph,
the direction of concavity, and also the asymptotes.
-- d - a
l
916. y=x -3x
939. Y= V x+ I-V x-I.
6x 2 _X 4
940. Y= (X+4)2- (X-4)1.
917. Y=-g-'
941. y=V(X--2)2+V(x-4)~
4
918. Y = (x-I)I (x+ 2).
942. Y= Y
.

V- -

919.

y=(X-2):(X+4).

(x 2 _ 5)a
125 .

920. Y=
2
921
= x -2x+2
Y
x-I'
x4-3
922. Y=-x-'
923. Y=

945. Y=V~'
946. Y = xe- x .

y=(a+x;)eci-.

947.
948. y=e8X-x2-14.
949. y = (2+ x 2 ) e-x~.

925. Y= x 2 -t- 3'


8

950. Y =

926. Y = xZ--4
- .

953. Y = fiiX .
954. Y= (x -t 1) In:! (X-i- 1).

16

930. Y == x 2 (x-':4j
931 Y

3x

x
929. Y=~4'
x-

951. Y= V-x'
x2
x
952. Y=2"ln(j.

4x-12

21 x 1- x
In x

4~

928. Y = (x 2)2

V_x_.
x 2 -1
x

+~x .

927. Y= 4+x 2

943. Y= x Y~2_4'
944. Y=

+3
x

924. Y = x 2

4-x 2
8

955. Y = In (x

+1

x:J'

V-x 4- V4- x.
Y~ VS -i-x- V~8-x.

956 Y= In

2
-

1)

+- :t2~ I .

VX2 +)-1
x

932. Y =

957. y=ln(1

933.

958. y = In ( e + ~ ) .
959. y = sin x + cos x.

934. y=-xVx+ 3.

935. Y=

Vx

).

+sin 2x
.
960 Y= sin
x -2-.
961. y=cosx-cos l x.
962. Y = sin a x + cos' x.

-~.

936. Y= Vl-i2.
937. Y= VI-x',
938. y == 2x -1- 2-3

+e-

'1/' (x +1)1.

963. Y= 8111 X +COSX .

S_e_c_._fi~l

D--::ifJ:.-e_'e_n_tt_Oa_1_0:-'_a_n_A_'_c_,_C_u_,v_a_tu_'_e

964. Y=

sin x
sin (

965.
966.
967.
968.

976. Y = arc cosh ( x

x+ :)

y = arc sin (1-

969. Y=

dIe

~lTl

+ ~ ).

977. Y == e,lnx.

y = sin x sin 2x.


y = cos x co:; 2x.

y=x+ sinx.

l01

978. y = earc sin Vx.


979. y=e arc tanx.

V x::).

98~.

y._-'
I-x!

y = In sin x.

981. y = In tan (

T-i)

970. y = 2x-tan x.
971. y=xarctanxo

982. Y= Inx-arc tanx.


983. y = cos x-In cos x.
I
972. Y = x arc tan -x \vhen x =1=0 981. y = arc tan (In x).
and y=O when x=O.
985. !J = arc sin In (Xl + 1).
973. y = x-2 arc cot x.
986. Y = xx.
x
I
974. y = 2+ arc tan x.
987. y=x x
975. y =--= In sin x.
A good exercise is to graph the functions indicated in Examples ~~6-84Ho
Construct the graphs of the following functions represented
paralnetrica 11y.
988. x=(1.-2t, y==II_t-2t.
989. x=-::al'o~3t, y-==asint (a>O).
990. x=te',
y=te- t
t
991. x=l +e- , y=2t+-e- 21
992. x=a (sinh/-I), y===a(cosh/-l) (a>O).
Sec. 5. Differential of an Arc. Curv&ture
1. Differential of an arc. The dIfferential of an arc s of a plane curve
repre"ented by an equation in Cartesian coordinates x and y is expressed by
the fornl ula
ds === V(dX)2

+ (dy)2;

here, if the equation of the curve IS of the form


a) y={(x), then ds=

b) x={.(y).

vl+(tf

then ds=

dx ;

vl+(~friy;

c) x=q> (I). y= (t), then ds=

V-(-~-)-2-+-(-~-)-2dt;

V F'2 + F'l
d) F(x, y)=O, then ds=

I~~I

It

II F'l + F'l
dx=

I~~I

"dy.

Extrema and the Geometric Applications of a Derzvatlve

102

[Ch. 8

Denoting by a the angle fornled by the tangent (I n the direction of


increasing arc of the curve s) with the positive x-direction, \ve get
dx
cos a::=; ds '
.

dy

Sill

u== ds.

In polar coordinates,
ds=

y' (dr)t+(r drp)t =

rt-l-

(~)). dIp

Denot1n~ by ~ the angle between the radius vector of the point of the
curve and the tangent to the curve at this point, \ve have
dr
cos A.==_

ds'

t-'

sin

~ =-= r~;

2. Curvature of a curve. Thr curvature K of a curve at one of It,;;


points AI is the 11n1lt of the ratIo of the angle between the POSltIVl~ directinns of the tangents at the points M and N of the curve (angle of COl1ftll6ence) to tlH. length of the arc 1\1 N -.=: As when ~V -~ M (FIt::. 35), that I~,
~

K = Ii rn ~\~ = da
~I\ ~

()

/\')

ds '

U IS the angle oetwe(:'!J the posItIve directIons of the tangent


pOInt 1\-1 and the x-aXIS.

\\hC're

dt

the

o
FIg.

35

The radius of curvature R is the reciprocal of the absolute value of the


curvature, i. e.,
1

R=iKl

The circle

(K =~.

where a is the radius of the circle) and the straIght

line (K=~ are lines of constant curvature.

Sec 5]

Differential of an Arc. Curvature

103

We have the fo))owing formulas for computing the curvature in rectangular coordinates (accurate to within the sign):
1) if the curve is given by an equation explicitly, Y=f (x), then

K-

y"

- (1 .,- y'2)'/J '

2) If the curve IS gIven by an equation implIcitly, Ftx,

y)

=-=0, then

F;x F: u F~

F~x F~y F~
F~ F11 0

K= (F't + F'2)3/
x
. y

3) if the curve is represented by equation.; in paralnetric form, x=<p (t),

y = (t), then

K=

x'
x"

I
(x't

y' \

.Il'

+ y'2)J/

,
2

\vhere
,

dx
rtt '

In polar coordInates, \vhen the curve is given by the equation r=f

(~),

\vr have

\vhcre
,

dr

r =-

dcp

an d

3. Circle of curvature. The clrcle of curvature (or osculati1t~ ctrcle) of a


curve at the pOInt AI IS the lill1itln~ position of a circle dra\vn throu~h M
~nci t\VO other pOlllts of the curve, P and Q, as P ---+ M and Q- .. 1\-1.
The radius of the circle of curvature is equal to the radius of curvature,
and the centre of the circle of cur\'ature (the centre of curvature) ties on the
nOrInClI to the curvr dra\\'n at the point M in the dIrectIon of concavity of
the curve.
The coordInates X and Y of the centre of curvature of th~ curve are
conl puted fr0J11 the fornllJl 3'i
.

X=x-

y' ( 1

+" [1'2) ,Y=y+--,,-.


1 + y'l

The eLlo[ute of a curve is the locus of the centres of curvature of the


curve.
If in the fonnulas for deternlining the coordinates of the centre of curvature we regard X and Y as the current coordinates of a point of the evolute, then these fOrtl1Ulas yield pnralnetric equations of the evolute \\ ith
parameter x or Y (or f, if the curvE' itself is represented bV equations in
paralTIctric f orin)
Example 1. FInd the equation of th~ evolute of the parabola y =x~.

Extrema and the Geometr,c Applications of a Derivative

104

(Ch. 3'

6x
Solution. X = - 4x', Y = I +2 '. Eliminating the parameter x, we find

the equation of the evolute in explicit form,

1 3 (XT
Y=2+

)2/.

The lnvolute of a curve is a curve for which the given curve i~ an


evolute.
The normal Me of the involute r 2 is a tangent to the evolute r,; the
"""--"
length of the arc CC l of the evolute is equal to the corresponding increnlent
in the radius of curvature CC.:::-::M.C1-MC;
that is why the involute r 2 is also called the
evolvent of the curve r 1 obtained by unwinding
a taut thread wound onto r, (Fig. 36). To each
evolute there corresponds an infinitude of involutes. which are related to dIfferent initial
lengths of thread.
40 Vertices of a curve. The vertex of a curve
is a point of the curve at which the curvature
has a maximum or a minimurn. To dpternline
the vertices of a curve, we fornl the expression
of the curvature K and find its extrelnal points.
I n place of the curvature K we can take the
radius of curvature R= I ~I and seek its extremal
points if the computations are simpler in this case.
Example 2. Find the vertex of the catenary
x
y=a cosh - (a > 0).

Fig. 36

Solution.

Since

Y'=sinh!-andy,,=J..cosh~,
a
a
a
x

u cosh 2 !..
a

it follows that K=

dR

and, hence, R=acosh 2 - . We have -d =slnh2a


x
a

the derivative dd

to zero, we get sinh

2~=O,
a

whence we

critical point x=Q Computing the second derivative


it the value x=O,

~2X~

Equating

find the <;ole

and putting IOto

wegetdd2~1
o=~cosh2!..-1
o=!>o.
x x=
a
a x==
a

Therefore,

x=O is the minimuln point of the radius of curvature (or of thfl nlaximum

of curvature) of the catenary. The vprtex of the catenary y = a cosh.!.. is,


(l
thus, the point A (0, a).

Find the differential of the arc, and also the cosine and sine

of the angle forIned, with the positive x-direction~ by the tangent

to each of the following curves:


993. Xl + y2 = a 2 (circle).
x2
y2
.
994. a2 +b2 =1 (ellipse).
995 y" = 2px (parabola).

Sec. 5]
996.

DiUerenttal of an Arc. Curvature


X 2fs

997. y =

105

a 2 / S (astroid).
a cosh.!(catenary).
a
y2{a

998. x=a(t-sint); y===a(l-cost) (cycloid).

999. x=acosSt, y=asin1t (astroid).

find the differential of the arc, and also the cosine or sine
of the angle formed by the radius vector and the tangent to each
of the following curves:
1000. r = acp (spiral of Archimedes).
1001. r=!!.. (hyperbolic spiral).
q>

1002. r=a sec

2
;

(parabola).

r = acos 2 ; (cardioid).
1004. r=aCP (Iogarithlllic spiral).
1005. r'l.=a 2 cos2q> (lemniscate).
COIllpute the curvature of the given curves at the indicated
points:
1006. y=x4.-4x ' -18x z at the coordinate origin.
1007. Xl + x!I + y2 = 3 at the poi 11 t (I, 1).
x2
,l1008. al +;;2 = 1 at the vertices A (a, 0) and 8(0, b).
1009. x=f\ 1I=t 3 at the point (1, I).
1010. r 2 = 2a 2 cos 2ep at the vertices {p = 0 and <p == 1t.
lOll. At \vhat point of the parabola 1/2=8x is the curvature
equal to O.12H?
1012. Find the vertex of the curve y=-e"'.
Find the radii of curvature (at any point) of the given hnes:
1013. y=x s (cubic parabola).
x2 ~~

1014. a 2 + b 2 = I (ellIpse).
1003.

1015. x=-=

~~_In:/ .

1016. x=acos1t; y=asinSt (astroid).


1017. x===a(cost-+ tsint); y=a(sint-t~ost) involute of a

circle).
1018. , = aekq> (Iogarithlllic spi ral).
1019. r=-a(l +COSfp) (cardioid).
1020. Find the least value of the radius of curvature of the

para bola y2 = 2px.


1021. Prove that the radi us of curvature of the catenary
y = a cosh ~ is eq II aI to a segnlent of the nornla 1.
a
<:ofilpute the coordinates of the centre of curvature of the
given curves at the indicated points:

]06

Extrema and the Geometric Applications of a DerIvative

(Ch.3

1022. xy = 1 at the point (1, 1).


1023. ayl = x' at the point (a, a).

Write the equations of the circles of curvature of the given


curves at the indicated points:
1024. y=x z -6x+ 10 at the point (3, 1).
1025. y = eX at the point (0, I).
Find the evolutes of the curves:
1026. y2 = 2px (parabola).
x2
y2
1027. a2 b2 = 1 (ellipse).
1028. Prove that the evolute of the cycloid

x=a(t-sint),

y=a(l-cost)

is a displaced cycloid.
1029. Prove that the evolute of the logarithmic spiral

r = aekcp
is also a logarithmic spiral with the same pole.
1030. Show that the curve (the involute of a circle)
x = a (~os t + t sin t), Y =- a (sin t - tens t)
is the Involute of the circle x = a co~

t~ y =

a sin t.

Chapter IV

INDEFiNITE INTEGRALS

Sec. 1. Direct Integration


to. Basic rules of integration.
1) IfF' (x)

=- -;

f (.\), t h P 11

~f
where C is

~onstant.

nrbttrary

all

~x) t- C

(x) d x -= F

2) ~ Af (x) dx ~- A ~ f (x) dx, when' A is a constant quantity.


3)

~ [f1{X)::l:f,{"')ld\-'~fl(X)dx~ f 2 (x)dx.

Sf (x) d x --- F ( \") -t- C

4) 1f

~ f
In particular,

Sf

(Ll.\

nd

(f' (t), the n

(ll) du --:: F (u)

b) d \:

II --:

-!r F (a.\.

-f- (;.

j b)

{(/. ~ 0) .

1- C

2. Table of standard integrals.


.\"+1

1.
t

\ "d \" - - -- -

IZ-:

II.

JCd~x ~-= In I x I -;

I I l.

5---=:--

IV

dx

x 2 -t-a 2

11

! - (;
I

=1= -

C.

tlrc tan -

f-C==- -

(l

Jr X~:-_:.!\nlx-al-t_C
2a
x -1- a
Z

_(12

"t

Ia-x

dx .2=-==2 In a+ ",]

a"-- x

I.

+c

arc cot -

\:

+C

(a

=F 0).

(a#O).

(a

0).

V. (' y~_==lnJx+yx2-I:aJ+C (a#O).


J x +1
. x +C =-arccos-+
x
C
VI.
..r dx
arc~ln2

to.

VII.

y a 2 -x 2

5aXdx~:

aX

In a

+C

(a

>

0);

eXdx =e X

+ c.

(a> 0).

Indefinite Integrals

108

(Ch.4

VIII. ~ slnxdx=-cosx+C.
lX. ~ cosxdx=sinx+C.

5 d~

cos x

=tan x-l-C.

5.d~ x cot + C.
Xli 5s~:x=ln Itan ~ I+C=ln Iczosl'C x-cotxl+C.
X II I. 5c~:x = In Itan ( i + : ) 1+ C = In I tan .. + sec x 1+ C.
X1.

= -

Sll1 Z

XIV.

S sinhxdx=coshx+C.

XV.

Scosh x dx= sinh x+ C.

Jr cosdXh2 x =tanhx-{-C.

XVI.

5 .dh

XVII.

sin

X2

=-cothx+C.

EX81npie 1.

(ax 2 +bx+c) dx=

Sax dx+ Sb.l.dx+ Scdx=


= a 5x dx + b 5x dx -f- e
2

5 =-

dx

x -f- b -i
x~ -1- ex -t- C.
a "3
3

Applying the basic rules 1, 2, 3 and the formulas of integration, find the following integrals:
1031. ~ 5d 2 x 6dx.
2

1040.
1041

1033.

S (6x +Rx+3)dx.
S x (x -+ a)(x -t b) ix.

1034.

S(a + nx3)2 dx.

1042.

1035.

1043.

1036.

SV2pxdx.
S;~.

1037.

5(nx)-n- dx.

1032.

l-n

1038.5

1039.

(aT -xTrdx.

('J(Vx-l-l)(x-VX-J--l)dx.
-

1044.
1045.

r (X2+.:j~-2L dx.

.J

,\'~

S(x:-=-\Il~ I
JI"l lX.
jl (V-a- V-;)4
- V a~-- dx.

5X2~
5X2~
JV +-

1046..

7'
10'

dx
4 x2

~ V:~X2'

__

JY2-t-x V2-x
1047. --Y4_;4-- dx .
2

Direct Integration

Sec 1]

109

1049. a) )cot'lxdx;

1048*. a) ) tan 2 xdx;


b) ) tanh l x dx.

b) ) cothl X dx.
1050. ) 3 x ex dx,

3. Integration under the sign of the differential. Rule 4 consid(lrably


expands the table of standard integrals: by virtue of this rule the table of
integrals holds true irrespective of whether the variable of Integration is an
independent variable or a differentiable function.
Exanlple 2.

Y5x-2
S~~

~S
(5x-2) -+ d (5x-2) =
5
1

= -5

S -- du==-
1 u
2 (5x-2)
-+C=1

1
2

1
2

2-+C=- Y5x-2 f-C


5

'

2"

where we put u == 5x-2. \Ve took advantage of Rule 4 and tabular integral l.
~
x dx
1 ,.
d (x 2 )
1
-Example 3.
V---=-2
I
2 =-2 In (xl + Yl+x~)+C.
1 +.r
.J
1 + (x )2

y-

We iJllplied u =x2 , and use \vas made of Rule 4 and tabular Integral V.

Example 4.
x1e X" dx,~ ~
eX I d (x") = ~ ex1 + C by virtue of Rille 4 and
tabular intt.\grat VII.
III exatllples 2, 3, and 4 \\'(1 r(\duced the given integral to the following
fornl before ITlaklng u,;e of a tabular integral:

) f (q> (x)

q" (x)

dx =

) f (u) dll,

where u = 'P (x),

This type of transfornlatioll is called IntegratIon under the differential sign.


SOlne COlnnlon transforrnattoll~ of d Iff~\rcntials, which were lIsed in EX31npi es 2 and 3, are:
a) dx ==
LJ~ing

a1 d (ax -I

b)

(a

# 0); b) x dx = 2" d (x 2 ) and so on.

the basic rules and forIuulas of integration, find the following in-

tegrals:

1051**.5 ~dx .
a-x
1052** S2X-I-~d

1053.

1054.

2.\ -1-1

5~~t ix.

5aX::x'

1055.

ax+b

5x ++
a.x
2

x.

1056.

Sx-I

(1 dx.

dx.

IO Qr.:7sx2-t-fiX+7d

x +:~
)C.
1058.

SX4~':~ I dx.

110

Indefinite Integrals

S(a-l x b aYdX.
1060*. S(x ~ l)2 dx.
1059.

1061.

bdy
..r-.
y I-y

1062. ~ Va-bxdx.
1063*.
1064.

r .r

x2 + 1

1078.

S2:'~3'

1079.

1080.

ax -1- b

a2x2

-t- b2 dx.

r ;~.
a -x4
4

1081.

SI ;2xe dx.

1082.

..r-====

Y x 6 -1

1083.

dx

X2
\

t.

dx.

SYxt In x dx.

lCh.4

r . . / arci-x
sin x dx.

J JI

tan !-.
4 +X22 dx.

llfC

S3X~~-5'
1066. S7X~~-8'
1067. S(a-t b)-~:-b) x'
1065.

(O<b<a).

1084.
1085

rx-

Yafltan2x d

1086.

1 +4x 2

fV(l+x~)ln(x+ V-I
dx

1087. ~ ae- mx dx.

1068.

SX'~:2dx.

1088. ~ 42 -ax dx.

1069.

Sa~2dx.
-x

1089. ~ (et_e- t ) dt.

1070 S~X-t-6d
2
x -f-4

1071.

Jr Y~'
7 +8~2

1072.

r\ Y __
dx
.

",'

7 -5x 2

2x-5

1073.

S3x2 __ 2dx.

4
107.

3-2x
5x2 -t- 7 dx.

1075.

x.

S 3x-t-1
dx.
-t-l
J Y 5x__
f'

x+3

1076.

- dx.
S yx -4

1077.

Sx~.

xdx

-0

x.

S(e-~ +- e-~ Ydx.


(aX -- bX)2
1091. S
dx.
a -l
1092. J Ya" dx.

1090.

u>"b X

2X

1093. ~ e-ex'+J) X dx.


1094. ~ x 7 dx.
X2

x2

1095. J" eX dx.


1096.

J5

yx

dJl

Yx'

eX

1097.

eX - l dx.

+x 2 )

Sec. 1]

III

Direct I ntegrat ion

1098. ~ eX

1119.

1099.

H20. Scot xdx.

Va- be x dx.
S(e-~ + l)~ e~dx.

1100
1101

1122.

5 +adx
I

aX

2X

e-bx

1102.

1121.

S~
2x +3

1123.

l __ e-:.b."dx.

1103.

e'dt

..I -==-==:t

Sr

Stan xdx.

5cota~bd"x.
r~.

J tans

Stan Vx

1124. ~ x cot (Xl

;-x.
+ 1) dx.

1104. ~ sin (a + bx) dx.

1125.

j' sin xdxcos x .

1105. JCcos Vx 2 dx.

1126.

5cos -

l-e'

11 07.

+ sin ax)! dx.


dx
.1r l'OS V--x Y-.t

1108.

5sin (lg x) df

1106. ~ (cos ax

1127. ~sin6xcos6xdx.
cos ax d

5sin' ax:3x .x.d


II 29
x.
53 -t-

1128.

sin

1130.

j" ..rsin.\
x
- -C0S
-- d ,X.
y cos X-S1I1 x
2

1131. ~

1110*. ~ cos! xdx.

t 132

5t an

1112. ~coeaxdx.

1133.

J cos

1134.

1113.)'~.
. x
Sttl-

1115

1116.

dx

ytanx I
2

x lX.

cot

x f
x
.

- -2 l X
Stn

si t1 3x
2

(cos ax

dx

ax)2

dx.

sin ax

xdx

-2-2.

1137.

Sb-a
cosec 3x d
cot 3x x.

1138.

S (2 sinh 5x-3cosh 5x)dx.

1117. ~ x sin (1 - r) dx.

1118.

X
2 X d
"3 sec "3 x.

51cos4- 3x tx.
1136. 5
t- sin
1135.

sIn (ax+b)

5cos

V 1 -1- 3 cos x sin 2x dx.

III I. ~ sec (ax I b) dx.

J 3 cos ( 51:- :

cos 3x

1109*. ~Sitl!xdx.

1114.

sin - dx.

SCinx\'2- 1

dX

1139. ~ sinh! xdx.

Jndefinite Jntegrals

112

1140.

1141.

dx

5 dx

1143. ) tanh x dx.

-=--h.
X

Sill

5-h-.xdx

1144. ) cothxdx.

cos

5sInh

1142..

x cosh x

Find the indefinite integrals:


1145. ~ x tl5-xl dx.
1146.

5X.~';:-~

1165.

dX
StanVx-l .J'r x-I

3- Y2t3x2
2-1- 3x 2
dx.

t 166.

5!

1167.

5:' /dx.

1156.

11 59.

eare t::ln x -1-X

...\

V tan

x-2

,
v

X'

-1- I

V2Y dx.

dx.

5X,x'

1171.

y'-2

2 dx.

(1 -1- ~f
X(I+x 2 )dx.

1172. ) e,ln' x sin 2x dx.


1173. \

dx.
1174.

xdx
~r--=.
y I-x"

sin

1170.

.\ 3

t..

S(2 +2XI~-I) 2X~~ I'

SV

+ COS X

1169.

sin x

X
na x
c;;ec2 x

(I-SilJ

tan 3x. -3cot 3x dx.


d
1

1n( 1+x 2)-+_1

I -1"'\ 2
SUI

1157. ) a1nxcosxdx.
1158.

I-sin x

5
t 154. 5
x

dx

Slll X Z

1168. 5SinX-C('SXdx.

5x + cos x dx.

1153.

1155.

dx.

xl

'" dx
1151.JYex

1152.

dX '
x

cos-

SV Ix+ In x dx.

1148. ~ xe-

t 150.

1164.

5 dx.

1149.

1163.

1 dx.

5X8~

1147.

(GIL 4

1175.

dx.

5-3x

V4-3x

SeX~I'

dx

J (l -i- b) + (a -

1160. ) tan ax dx.


1161.

5sin' ; dx.

1176.

S.r:....e x _2 dx.

1162.

SY4-tan
s~_x
x

I t 77.

Jr.Sill axdxcos ax

b) Xl

(0< b <a).

2X

dx.

Integration by Substitution

Sec. 2]

1178.
1179.
1180.

Ssin C;t + fj>o) dt.


Sx (4 -ln
dx
x)
) arc cos i
2

4-x 2

dx.

1181. ~ e- tan x secl X dx.


t 182. , sin x cns x dx.

1184.

S
f

SI11 2

'-

dx
x CCJs 2

1185.

SYsecseclx tanx+x I dx.

1186.

S4+C05
cos 2x d
2x x.

1187.

S1 +cos
dx

x
2

1189. ~ Xl cos (x 3 -+ 3) dx.

1190.

X+x dx.
l-x 2

afC SIll

Vx + I)
1188. ) Vln (x+1+,\2
dx.

" Y2-su1" x

1183.

113

atanllA

--Jx

'" cr.sh 2 x

Sec. 2. Integration by Substitution


to. Change of variable in an indefinite integral. Putting
x~qJ(t).

t is a new variable and (p is a continuously differentiable function, \ve


will have:

wIH'r~

~ f (x)dx= ~ f [ljl (t ljl' (I) dt.

(1)

ThE' attelnpt is made to choose the function ({1 ill such a way that the right
side of (I) beCOlnl'~ 1110rc convcl1i~nt for int~gratlon.
EX31npie 1. Find

) x Y-t-I d.t.
Solution. It is natural to put t ==
Hence,

V x-I, \vhcnce x = t + 1 dnd


2

dx = 2t dt.

SOlnetilnes substitutions of the fornl


u = q> (x)

are llsed.
Suppose we succeeded in transforming the integrand

f (x) dx = g (u) du, where u = q> (x).

f (x) dx

to the foraD

(Ch. 4

Indefinite Integrals

llt

If ~ g (u) du is known, that is,

~ g (u) du = F (u) +0,


then

~ f (x) dx= F [q> (x)] +C.


Actually, we have already made use of this method in Sec. i ,3.
Exam pIes 2, 3, 4 (Sec. 1) nlay be sol ved as follows:
Example 2. u = 5x- 2;
-=-d__...x......--..

SV 5x-2

dx = 5' duo

du = Sdx;

J.. _du- = ~ ~ + C = ~
5

V "it

5 ~

Y-Sx---2 + C.

du
Example 3. u =x 2 ; du = 2x dx; x dX="2 .

=-21SVdU
1+ x
1+
S.;tdt
4

Exalnple 4. u = x 3 ;

Sx eXJ
2

dx

=-21tn(u+V1+U2J+c=-21tn(x2+Vl+x4)+C.
du

du = 3x2 dx; x2 dx = 3" .

=~

SeU

du

= ~ en +c = -} eX' + C.

2. Trigonometric su bstitutions.
1) If an integral contains the radical
x= a sin t; whence

2) If

Va -x
2

an integral contains the radical

whence

3)

the usual thin~

IS

to put

Vx 2 -a 2 , we put X~ll

"-CC

t.

If an integral contains the radical V.\2+ a2, we put x=a tan t; whence

V x2 + a2 =

a sec t.

It should be noted that trigononletric substitutions do not al ways turn


out to be advantageous.
It is sometimes Illore convenient to make use of hyperbolic substit lltlOI1S,
which are ~iInilar to trigonometric substitutions (see Exarnple 1209).
For more details about trigonometric and hyperbolic substitutions, see
Sec. 9.
Example 5. Find

JY?+1d
X

x.

115

Integration by Substitution

Sec. 2)

Solution. Put x= tan t. Therefore, dx=

5yX2+T S f
dx==-

x2

Ytan t -1--1 ~=S sec t cos t -.-!!:!.-=


ta n2 t
cos 2 t
SIn 2 t cos 2 t
dt
= sin! 2t i- cos! t dt = ~
cos t dt =
Sin! t cos t
sin t . cos t
cos t + sin 2 t
2

dt t

cos 2

=--=\11 I tan t-{-sec t 1--.I_-t-C=ln Itan t-I- 1-1-tan 2 t 1Sin

- YTtTafiit
+C=lnl x+ Y x + 11_ yx +1 +C.
tan t
x
2

1191. J\pplying the indicated substitutions, find the follo\vlng

integrals:
a

) I

d.\

. x V~ 2-~-2

x == 7 ;

'

b)

SeX~I'

c)

~x(5x' --3)'dx,

(1)

x=-In(,

"'
.I V

~.t__

l\) ,
fJ

r __

1==1/x ~ 1;

x -t- f '
cos x dx

(.

t :- - sin x.

"Vl~- ~1I12~~ ,

App) ying

5x 2 ---:3=I;

Stl i ta hIe

su bst itut ions, find

the f 0\ 1o\\'i ng integrals:

1192. ~ x (2x -\- 5)10 dx.

1W3.
t

1194.
1195.
1196.

\
I l-ry_.dx.

t -l-

Jx
\

/~_.
2~+ 1

dx
eX-l

y._.

SIn2xdx.
In 4x x

1199.

"' sin' x
..r-dx.
\ Y cos \'

e"

1200*.

dt

J x Vl-t-.\2

Applying trigonolnetric substitutions, find the follo\ving integrals:

.
j .r-
X2

1201.

1202.

dx

1203.

y I-x

Xl

lr
y

dx

2-x2

1204*.

j"'

~F.~

a! dx.

I y_.
x

dx

.. ,\

x 2 -1

116

'"defi/ute /ntet!ral.s

rCll. 4

1206*.

1205.

1207.

1208. Evaluate the integral


,..

dx

j YX(l-x)
by means of the substitution x = sin 2 t.
1209. Find

~ Vl1 +x 2 dx,
by applying the hyperbolic substitution x=asinh t.
Solution. We have: Va 2 + x 2 = Va 2 + a 2 Sl11h 2 t=a cosh t and dx=a cosh t dt.

Whence

~ Va2 +x2 dx= )


= a2

a cosh ta cosh t dt =

Scosh t dt = a 5cosh ;t + 1 dt = a; ( ~ sinh 2t + t ) + C =


2

a2

(sinh t cosh t

+ t) +C.

Since

sinh t =

a '

cosh t =

Ya-+x
a
2

and

x+ Va _+x
2

et = cosh t -1- sinh t = - - - a

we finally get
x
a
5V q2+ X2 dx =2 a 2+.\2+ 2 I n(x+
2

--

Ya 2 +x 2 )+Cl'

where C. ~ C- a In a is a new arbitrary constant.


2

1210. Find

putting x==acosht.

Sec. 3.

Integr~tion

by Parts

A formula fer integration by parts. If u = <p (x) and v = '" (x) are differentiable functions, then

~ II dv = uv - ~ v duo

IntegratIon by Parts

Sec. 31

111

Example 1. Find

~ x In x dx.
dx

Putting u=lnx, dv=xdx, we have dU=-X'

X2

xlnxdx=2 Inx -

x2

v=2 Whence

52x=2 Int -4+ C.


t

dx

x~

x2

Sometimes, to reduce a ~iven integral to tabular foro), one has to apply the
furmula of Integration by parts several tirnes. In certain ca~es, integration
by parts yields an equation froIll which the desired integral is determined.
Example 2. Find

~ eX cos xdx.
We have

~ eX cos x dx= ~ eXd (sin x) = eX sin x- ~ e:' sin x dx= eX sin x-l-

+ ~ eX d (cos x) ~= eX 5111 x + e:c cos x- ~ eX cos x dx.


lienee,

~ eXcosxdx=eXslllx+excosx-~eXcosxdx,
whence

eot; cus

x
t

dx =--= e (sin x
2

+ cos x) + c.

Applying the forrTIula of integration by parts, filld the follo\ving


integrals:
1211. ~ In xdx.

1221. \ x sin x co~ x d.\

121 2. ~ arctan x dx.

1222* ~ (x 3 +5x+6) I.:OS 2xdx.

1213. ~ arc

1223. ~

sill

xlix.

..I

Xl

In x dx.

1214. ~ x sill xrlx.

1224. ~ In 3 xdx.

1215. ~ xcos3xdx.

1225.

1216.

5eX dx.

1226.

5':.=- dx.
S ~.;dX.

1217. ~ x2- X dx.

1227. ~ x arc tan x dx.

1218**. ~

1228. ~ x arc

Xl

1219*. ~ (x l
1220*.

3X

e dx.
_

2x+5) e- x dx.

5xae-~ dx.

5111

x dx.

1229. ~ In (x + V 1 + x 2 )dx.

1230. j

xdx
SUI1X.

Jndefinite Jntegrals

118

1231.

x cos x d

Ssin! x

[Ch.4

1234. ~ eax sin bxdx.

x.

1235. ~ sin (In x) dx.

1232. ~ e" sin xdx.


I 233. ~ 3" cos x dx.

Applying various methods, find the following integrals:


1236. ~ x 3 r

x2

dx.

1246.

r"rt:~x dx.

l--x

1237. ~ eV~ dx.

1247. ~ xtan 2 2xdx.

1238. ~ (x 2-2x+3) In xdx.

1248.

I 239.

Sx In : +; dx.

1240.

S7dx.

r sin;e x dx.

I 249. ~ cos 2 (In x) dx.


1250**.

5(X2~

(~n x) dx.

1251 *.

Jr(X -1~xa

arc tan 3x dx.

1252*. ~Va2-x~dx.

I 243. ~ x (arc tan xY dx.

1253*. ~ V A -1- x 2 dx.

1244. ~ (arc sin X)I dx.

1254*. C x:!~

1241.

In2x

I 242. ~

1245.

Xl

1)2 dx .
2)2'

, ..r
_;(2
r 9 .

Sarc :~11 x lx.

Sec. 4. Standard Integrals Containing a Quadratic Trinomial


to. Integrals of the form

5 mx+n+
ax 2 -l- bx

c x.

The principal calculation procedure is to reduce the quadratic trinomial to


the for III
ax! + bx+c=a (x+ k)2 + 1.
(1)
where k and 1 are constant~. To perforrn the transformations in (1), it is
best to take the perfect square out of the quadrattc trinomial. The follo-

wing substitution may also be used:

2ax+b=t.
If m=O, then, reducing the quadratic trinomial to the form (1), we get
the tabular integrals II I or IV {see Table).

Sec 4]

Standard Integrals Containlng a Quadrattc Trtnomtal

119

Example t.

5
I
1
x- 4
----arc tan --+C=
2
V31
-4--4-

y3T

4x-5

y 31

y 31

lr- arc tan 1r--+ C.

If In : 0, then from the nUlnerator we can take the derivative 2ax + b


out of the quadrattc trtnolnial

mx -1- n dx ==
ax 2 -t- bx -1- c

(n- nlb) dx =-=

m (2ax -l- b) -12a


2a
ax 2 bx + c

n1
2.
(mb )
-2a ln / ax +b.\:+c/+ n- 2a
-

5axl+b,+c'
dx

and thus we arrive at the Integral discussed above.


Example 2.
' x--l
) - - - - dx-=
\2- x - l

ii(2X--l)-;
,\2-

- dl.:.=- In I x 2 _ x-l/-

x _1

}V + n dx. The luethods of calculation


J 1l.\2+bx+c
are sinlilar to thosE' analyzed above. The integral is finally reduced to tabuIII X

2. Integrals of the form

lar integral V, if a > 0, and VI, if


Example 3.

dx

J Y2\-3x-2x2=

Y 2

<~

o.

J V~_ ('t- ~
dx

r y1
=

1 . 4x-3
arc Sill -5- + c.

Example 4.

dX=..!..S
2x+2
dx+2f
dx
j Yx x+3
+2x+2
2
Y x +2x+2
J Y (x+ 1)2+ 1
"

= Y x + 2x +2 +21n (x + 1 + Y x----+-2x-+-2) +O.


2

I ndefi,nite Jntegrals

J20

3. Integrals of the form

verse substitution

S(mx+n) Vax +bx+c


dx

(Ch.4

. By means of the in-

1
--=t

mx+n

these integrals are reduced to integrals of the form 2.


Example 5. Find

s+
(x

Solution. We put

1)

~ x +1
2

x+l=T'
whence
dx:~

We have:

dx

dt

f2.

--~
f 2
+V(+-lr+
1 =-, Y-I-2t+2t =
rJ y ( d\ r ;:1 nit - ~ + -V t2- t + ~ I+

(x+l) Vx 2+1
= - ; 2-

1'1

dt

1 =-

t- 2

+4

+c=- ;Cjlnll-X+x?t2+1)I+c.
4. Integral~ of the form ~ Y ax2 + bx + c dr. By taking the perfect square
out of the quadratic trinOlninl, the given integral is reduced to one of the
following two basic integrals (sec examples 1252 and 1253):
1)

a -x dx= - Va -x +-- arc


j-'1r-'2
2

2)

2 -2

a2

-=-ax -1- C;

Sin

(a > 0);

V x2+A dx= ; VA 2+A + ~

In I x+

Vx 2+A l+c.

Example 6.

5YI-2x-x 2dx= ~ Y2-(l+X)2d(l+x)=


l-1-x .. r

=-2-

Pind the following inte6rals:


1255.

SX2+~:+5

1256.

Sx ~2X
2

. l+x
y 1-2x-x2 +arcslO
Y2 +C.

Integral ion of Rational Functions

Sec. 5)

1259.

1269.

1260.

1270.

S'" x

121

dx
yxz--t-x-I .
dA-

V.\2 -2

"\ (x-I)

d-<

(l

J(x+l) Yx

1261.

t 271.

1262.

1272. ~ x +2x+5dx.

1263.

1273. ~ Yx=?dx

1-2,'

1274. ~ 2-x-x 2 dy,


1264.

1275.
1265.

1276.

1266.

xdx

Sx -4x cos+3x
4

f}

Sl11 2

1277.

X-6s1I1X+12

eXdx

" \+fx

1267.

1278.
1279.

..

~2Xd.\

Vc~ X + 4-~~~s~~- 1

t'

1268.

1_

SlIl \

dx

In x d\

Jx YI-lInx-li1

t'

Sec. 5. Integration of Rational Functions


to. The nlethod of undeiernlined coefficients. IntegratIon of a rational
function, after takIng out thf whole part, reduces to lIltehratlon of the proper
ratlollal traetloll
p (x)
l

Q (x)'

(1)

where P (x) and Q (x) are Integral polynollllals, and the degree of the numerator P (x) is lower than that of the denolninator Q (x).
If
Q (x) == (x-a)'1 . . . (x-l),\

where a, l are real distinct roots of the polyllotniiJl Q (x), and a,


A are natural nUlllhcrs (root lllultiplicities), then decomposition of (1) i~t~
partial fractIons is justified:
P (x)
At
A2
(-----)2
x-a +.
Q (x) =:=:i x-a

a
+ (--)'X
x-a +

- L1
Lz
L).
+x-l+(x-l)2++(x l)1\.

(2)

To calculate the undetertllined coefficients At, A 2 , , both sides of the


identity (2) are reduced to an integral fornl, and then the coefficients of
like powers of the variable x are equated (tlrst nlethod). These coefficients may like\vise be detennined by putting {in equation (2) or In an equi.
valent equation] x equal to suitably chosen nUlnbers (second Inethod).

Indefinite Integrals

122

[Ch. 4

Example 1. Find

xdx
(x-I) (x+ 1)2=1.

Solution. We have:

Whence
x==A (x-t-I)~-+-81 (x-I) (x+ l)+8 2 (x-I).

(3)

a) First "Iethod of deternunlnR the coefficients. We rewrite identity (3) in


the (onn x = (A 8 1 ) x 2 (2A + B 2) x + (A - 8. - 8 2 ) Eq uati n~ the coefficients of identical powers of x, we get:
O-==A+B.; 1==2A,-B 2 ; O==A-8.-B 2
Whence
I
A=-
4 '

b) Second method of determtning the coefficlents. PuttIng x= 1 in identity


(3), we will have:

J=A.A,
Putting x= -1, we get:
-1=-8 2 -2,

Further, puttIng x=O,

\ve

Le.,

8 2 ==1/ 2

will have:
O==A--B.-8 2 ,

Example 2. Find

Solution. We have:
1

x3 -2x 2+x

1
A
8
x (x-I)2=X-+ x-I

+ (x-I)2

-and
I=A(x-I)2+Bx(x-l)-f-Cx.

(4)

When solving this example it is advisable to conlbine th e two methods


of determining coefficients. Applying the second method, we put x=O in
identity (4). We get 1= A. Then, putting x= I, we get I =C. Further, applying the first nlethod, we equate the coefficients of x 2 in identity (4), and
~et

Hence,

O=A+8,

11=-1,

Le.,

8.=.-1,

8=-1.

and

C=l.

Sec. 5]

Integration of Rational Functions

123'

Consequently,
1=

dx 5dx
5 dx
I
x-l+ (X_l)2=lnlxl-ln/x-11-x_lTC.
5-XI

If the polynonllal Q (x) has cOlnplex roots a


partial fractions of the fornl
A1x+B,
.\2+ px -f-q

+.

of mu It1phcity k, then

Akx -t-R k

(x2+PX-~_(/)k

(5)

will enter Into the expansion (2). lIere,


x2+px+q=[x-(a-r-lb)] [x-(a--lb)J

and AI' B I , , A k , Bk are undetennined coefJiclents \vhlch are detennlned


by the nlcthods given above For k =-= 1, the fraction (5) IS integrated din:ctIy; for k > 1) USl' is llHlde of the redLLctzoll Inethod; here, It is first advisable to represent the qua drahc trinonllal x'

+( q_~2)

+ px + q in the forIII ( x + f

-r

x+%=z.

and make the substitution

Example 3. Find

Solution. Sillce
theil,

PUltlfl~ .\'-1-2---:2, Wl' gl\t

2-

1 === J {2 2 t-

02 az:=:;

~ dz

(1:;- t 1) -

II (1 -t- z:!) -

z'1.

J (Z2 +112

dz-==

ell

1
C dl
I
~- --~ lz~t") .--- J i~-n l- J Id . -2 (z:!+ I) --- -2 (2 2 + 1)z
1
z -1- 1
--drr tall z- - -- --- -1- - -life tan z= - - - - - ~ (2' 2

2. The

-1-

1)

~ aI ( t a 11 z --t- C :::.:: 2

Ostrograd~ky

2 (Z2 . ~- 1)

\ -t- :3

2 (x 2 -t- 4 \'

+ 5) -

...!.- alet a 1J (x -.J - C)) _L C


2

fllcthod. If Q (.\) has Illultiple roots, then

\P(X)d\,:-=-:.~~t-1_rY(X) dx,
Q (.\)
(J 1 (.\) J Q2 (x)

(6)

tJ

where Q I (x) is the greatest


dcri vativ e Q' (x);

COlll1110n

Q! (x)

diVisor of the polynoillial Q (x)

and Its

== Q (x): Ql (x);

x (x)

and Y (x) are polynonriClls \vtth undetcrrnincd coefficients, whose degrees


arc, respectively, less by Ullity than those of Ql (x) and Q2 (x).
The undetermined coefficients of the polynonllals X (x) and Y (X) are
cOlnputed by differentiating the identity (6).
Example 4. Find
dx

J (x 3 -1)2

Indefinite Integrals

124
Solution.

~=AX2+Bx+C +5Dx2+Ex+F dx
3
Xl - I
,
x3 - 1
5 (x - I )2

Different i at i ng th is identi ty, we get

'Or

(2Ax+B) (x 3 -1)-3x 2 (Ax 2 +Bx+C)

{x -1)2
'

(x 3 _1)2

+ Dx 2+ Ex+F
x3 -1

1= (2Ax+ B) (x 3 -1)-3x l (Ax 2 + Bx+C) + (Dx 2 +Ex+F) (x3 _1).

Equating the coefficients of the respective degrees of x, we will have:


D=O; E-A=O; F-2B:.=O; D+3C=0; E+2A=0; B+F=-l;
whence
A=O;

B=

-"3; C=O; D=O; =0;

and, consequently,

dx

S(x --1)2=-'3 x - I - a
3

F=-3"

Sx dx-1

(7)

To compute the integral on the right of (7), we decompose the fraction


-.!-I into partial fractions:

x-

lhat is,
1=L{x 2 +x+I)+Mx{x-I)+N{x-I).

(8)

L=}.

Putting x= I, we get

Equating the coefficients of identical degrees of x on the right and left


lof (8), we find
L+M=O;

L-N=I,

-or
I

M--_o
3'

N=--a.

Therefore,

dx

Xl -

I=

Sx -dx I -

Sx +x+2+ I d x =
2

=3 1n I x-ll-filn (x
and

+x+ I) - Y3 arc

+ +

dx

(x'-1)2

SX

tan

2x 4- I

V3

+C

+I
Y-3 +C.

x
J
x2 X 1
2
2x
3(x3 _1)+gln (x-I)! +3 Y3"arctan

FInd the fol1owlng integrals:


dx
1280.
(x+a) (x+ b)

1281

-5t+9
.\,2-5x+6 dx

5ex +
1283. 5

1282.

dx
I) ex

2X2

+ 2) (x + ~)

-f- 41 x - 91

(x-I) {x+3)lx-4)dx

See. 6]

/ntegratinf;! Certain I,rational Functions

5X'+2
x3_5A2+4xdx.

1293.

1294.

5
1285. 5 (Xd~_
1284.

1)2'

r-I

1286.

1295.

1287.

1296.

-x dx.
x4-6x3+ 12~2+6

4-3X

x3 -6-\2-t-12x-8 dx.
5r2 -t- 6x+ 9

(X_t-1)2 dx
5 x2-8x+7
1289. 5
3x_IO)2 dt .
2x-3
1290. 5(x -3x+2)3 dx.
-t-x+ I
1291. 5 (x2 + I) dx.
1292. 5JI./ 1 dx.

1288.

125

1297.

(X-3)2

1298.

(-\2_

1299.

X3

1300.

Applying Ostrogradsky's method, find the following integrals:


1301

1302.

{x

1303.

dx

-1- 1)2 (x 2 + 1)2

dx
(t 4 _1)2'

r (.t d'(+ 1)4 .

~)

1304.

xl - 2 \ 2 ~- 2

J(.l2_2x+2 2dx .
n

Applying different procedures, find the integrals:


1305.

5(.1. + 1)'"~X3 13

8)

1310*5 Xlx~Xt I)'

dx.

X2_ \ -t- 14
(x-tl):' (x-2) dx.

5
1308. 5
1309. 5
1307.

-t- 1)2

5(x2-1-2~+-2){X2+2x+5).

1313.

Int~grating

1314.

5lt~I\'.
5~
Xl

AG

Certain Irrational Functions

Integra Is of the f .. rm
PI

R [.x. ( aX+b)Q;
cx+d

5
where

1312.

1)2'

dx

dx
x3 - 4x 2 -t- 5 ( - 2

5xlS~

d'(

.t4 {x3

Sec. 6.

1311.

\'3

1306. J x I2 -2\,4 1- 1 dx.

R is

a rattonal functIon and

P2

ax+b\q~
ex +d) , ... dx,

PI' ql' Pa' qa are whole l1ul1lbers.

(1)

Jnde{iniie Jniegra[s

12J

(Ch.4

Integrals of form (1) are found by the substitution

ax -l- b
n
--=2
cx+d

where n is the least cornmon lTIulti pie of the lllllnbcrs q., q2'
Example 1. Fi nd

J\----~--V 2x - 1- V 2x - 1

~ubstitution

Solution. The

C2z dz
3

d.\

J V2x-I- V2t-1 J

2.\ -1
= 2

Z2_ Z

=2S(Z+I+z

= zo1 leads to
z dz =

II)dZ=(Z+ll2+21nlz--ll+C=

(1-1- V2x=f)~ -I-ln (V2x-1-1)2 +c.

Find the integrals:

r ./3
_dx.
r x-I
"

-V
j

t.\

1117.

dx

t 320. ('

_d_t_
~r-1----\

" J/.t--=-r
- - - ([

132:3.

1)3'

dx

V-=-l
'-1- V-
X

.\

1324. C

'Ir
"Y t- --1
laJ9. , ~-- -..==- dx.
t.

I ("2-- .\)

1322.

lb.
1.\ --

'1

13~5.

V\,+l

dx.

x+2

t.

JV~+l ~XV(~

,..
1318. \)

I'VX"

1321.

1316.

an Integral of the fOrIn

J 2-1

==

1315.

\+ 1

'V\r

i'

.\..

~__1=1 dx.

\-

1
x -1- 3
-.r-'==-- dx.
x 2 r 2x ~- J

_tt+! +-=-_ dx.

J(x-l-l)2- YX-1-1

20 I ntegraJ~ of the form

Pn (x)

SV~a).,-2~+-b-x-+-c dx
where Pn (x) is a polynol1llal of degree
Put
Pn(x)

lrax2

~\ f

+ bx + c dx~ Qn-l (x)

(2)

tt

lr
r axl+bx+c+",

S.r----.:.='
dx
r

a.\2

-t- bx -t- c

(3)

where Qn_l(X) is a polynolnial of degree (n-l) with undeternlined coefflcients and A is a num b~r.
The coefficients of the polynomial Qn-l (x) and the nUlllber A are found
by differentiating identity (3).

Sec. 6}

Integrating Certaln Irrational Functions

127

Whence

.\4~_4x2 =(3Ax2 +2Bx+C)


Y x2 + 4
Multiplying by
x, \ve obtain

Yx +4
2

Yx

+4+(Ax +Bx +Cx+D)x+


.r.\ 2 -f- 4

Y +4

.l2

and equating the coefficients of identical degrees of

1
A =4 '

8==0,

1
C=-;
2

D=-=O;

A=-2

Hence,

3. Integrals of the form

(x-a)n

;:X

+bX+C'

They are reduced to lntrgra\s of the form (2) by the substitution:


_I_==t .

..\-a

Find the integrals:


1326.

1:J27.
1328.

r2dt

1329.

"If - - '- - - .

J'

,\2_ ~-1-1
r~

1r ~-==-"l dx.

1330.

J' )-"\"
X6

1/--=====

,\ r 1-1- x 2

1331.

dx.

4. Integrals of the binonlial differentials

x ln (a

+ bt")P dx,

(5)

where tn, nand p are rational Ilunzbers.


Cheby~hev's conditions. The integral (5) can be expressed in tenns of a
finite COlnbll1ation of e1rl11entary functions only in the following three cases:
I) If P i" a whole nUlnber;
2) if 111-t-l is a whole llulllber. I-Iere, we Inake the substitution a+ bxn =
n

= zS, where s is the denonlinator of the fraction p;

3) if

111-1-

n
ax-n+b=zs.

I + P is a whole nUll1ber. Here, use is made of the substitution

Indefinite Integrals

128

(Ch. 4

-------------~---=------------=----

Example 3. Find

1
1
Solution. Here, nt= -2 ; n=T;

--!"+l
2
m+
1
1
n

P=3" ;

=2. Hence,

4
we have here Case 2 integrability.

The substitution
1 +x 4

=2 3

yields x= (z3-1)4; dx= 12z (zl-I)3 dz Therefore,


I

where

x-

Z=

~(I +x+)+ dx= 12 5z~;:~~:r dz=

I-I-

= 12

-x.

S(z'-ZS) dz = z1-3z +C,


4

Find the integrals:


3

1332. ~
1333.

1334.

Xl

(l

+ 2x") -2 dx.

1335.

dx

, i / - 5.

it'

1336. "

SV~,
1 +~ .

~ x"

dx

(2

133 7. \

-3

J Y.t

1 +- x

dx

"

'y 1+ x .
~4

"

, x

~.

+ x")-;-

V + Vx--
dx

Sec. 7. Integrating Trigonometric Functions


1. Integrals of the form

sinmxcosnxdx=I m n'

where nz and n are integers.


1) If m=2k+ 1 is an odd positive nUlllber, then we pUf
1m n = -

~ sin"k x cos n xd (cos x) = - ~ (I-cos" x)k cos n xd (cos x).

We do the sanle if

n is an odd positive number.

Exall1ple 1.

~ s1n1o x cos'x dx = ~

51.1

10

x (I-sin" x) d (sin x) =

sin x sin 13 x
;:::::-11--13+ C.
11

(1 )

Integrating Trigonometric Functions

Sec. 7]

129

2) If m and n are even positive numbers, then the integrand (1) is


formed by tneans of the formulas
1
sln2 x= 2 (I-cos 2x),

1
cos! x="2 (I

tran~

+ cos 2x).

sin x cos x="2 sin 2x.


Example 2. ~ cos 2 3x sin' 3x dx = ~ (cos 3x sin 3X)2 sln 2 3x dx =
=

sin2 6x 1- cos 6x
1
-42
dX=8

Jr (sin 2 6x-sin 2 6x cos 6x) dx=

="815(I-COS12x.
2
stn 2 6x cos 6x ) dx=

= ~ (~_ sin 12x _ .-!- sins 6x ) + C


8

3) If m =
parity, then
1 m," =

5
5(

24

~t

and n = - v are integral negative nunlbers

1 -t- t - 2 -

identical

cosec'" x sec' -2 xd (tan x) =

1)

of

S1I11'- ::os' x

18

=5 +

V-2

(1

an x

+ tan

x)

(1

d (tan x)

~+v

--1

tan 2 x)
t lJ.
an' x

d (tan x).

In particular, the fotlo\ving integrals reduce to this case:

5 X=21'-~Jj~ -i)
54 = 5

5co~'~ x=s

((+ ~\

dx (
x and
d
sintJ. - cosP- sin" x + 2 2 2

SI::

Example 3.

cos x

sec 2 xd (tan x) =

5+
(I

tan 2 x) d (tan x) =

1
=lan x +"3 tan 3 x+ C.
Example 4.

5-; =ds ~
d:

Sill

1~(1+tan2 ;)2

=8

+tan

5-1900

tan

sX

dx

sina

!.- cos' .!.


2

2)[

sec 2 .!-dx=-

~] d( tan i )= ~

=i 5

tan -3

seeS

1- dx-=

tan-a

2x
!-.+-+
2
tan

_I-x +21n Itan ~ 1+ ta:

[-

2tan 22"

;]

-I- C.

Indefinite Integrals

130

[Ch.4

4) Integrals of the form ~ tan m x dx (or ~ cot m x dx). where m is an integral positive number, are evaluated by the formula
tan 2 x=secx-l
(or, respectively, cot l x = cosec 2 x-i).
Example 5.

t an' x dx =

tan' x (sec' x-I) dx = ta~' x -

tan' x dx =

tanS x
tanS x
-=-3-- (sec2x-l)dx=-3--tanx+x+C.
5) In the general case, integrals Jm n of the form (1) are evaluated by
means of reduction formulas that are usually derived by integration by parts.
2
2
E
16
dx
SSln x+cos x d
xamp e .
cos' x =
cos' x
x=

==

sin x
S--=slnx
dx.
cos' x
cos x
5sinsinx--dx+
x

1
2 cos 2 X

1
2

e ----

5cos
cos x
5-cosdx-x=
--dx+
2

C : :2-

2 cos x

+-2Inltanx+secxl+C.

Find the integrals:


1338.
1339.

~ cos' xdx.

1341.
1342.

dx

sin - cos a -

~ sins xdx.

~ sin' x cos' x dx.


SsIn
"x
2" cos sX
2 d x.

1340.

1352. )

1353.

) sin (x+ ~ )
.
sin x cos x

1354.

Ssin'
cos x d
x x.

1355.

~ sec s 4x dx.

1343.

~ ~in' xdx.

1356.

~ tan' 5x dx.

1344.

~ sin~xcos'xdx.

1357.

~ cot' xdx.

1345.

~ sin! x cos' x dx.

1358.

~ cote xdx.

1346.

~ cos' 3xdx.

1359.

1347.
1348.
1349.

Ssindx x
Scos'dx x
Ssin'
cos' x d
x x.
4

1361.
1362.

~ sinS x Vcosxdx.

1360.

dx

1363.

1351.

Ssin' dxcos' x

1364.

dx

sinS x

5'l tan "3 + tan 4'X) dx.


~ x sin' x'dx.
5 x.

J350.

sin l x cos 4 x

dx.

cos' x d
sin. x

S dxx
SYtan
dx
x
YSin

cos' x

1ntegrating T rigonometrlo Functions

Sec. 7]

131

2. Integrals of the form ~ sin mx cos nx dx, ~ sin mx sin nx dx and

~ cos mx cos nx dx. (n these cases the following formulas are usedl
I)

slnmxcosnx=~

[sin (m+n)x+sln(m-n) x];

1
2) sin mx sin nX="2 [cos (m-n) x-cos (m + n) xl;
I

3) cos mxcos nX="2 [cos (m-n) x +cos (m + n)

Example 7.

xl.

Ssin9xsin xdx= S~ [cos ax-cos lOx) dx=>

=~ sin 8x- 2~ sin lOx + C.


Find the integrals:
1365.

~ sin 3x cos 5x dx.

1369. ~ ros(ax t- b)ros(ax-b)dx.

1366.

~ sin lOx sin 15 xdx.

1370. ~ sin rot sin (rot +cp) dt.

1371. ~ cos x cos 3x dx.

x d

Scos. 2"x cos. 32x dx.

1367
1368

1372. ~ sin x sin 2x strl 3x dx.

x.

Ssin 3" sln"3

30 Integrals of the form

~ R (sin x, cos x) dx,


where

(2)

R is a rational {unct ion.

1) By means of substitution
x

tan'j=t,
whence
.

I - t2

2t

slnx=l+t l

'

COSt=l+tl'

2dt

X=I+t l

'

integrals of form (2) are reduced to Integrals of rational functions by the


variable t.

new

Example 8. Find

5+
1

dx
sin x + cos x

Solution. Putting tan ~ =t, we will have


2dt

1=

1+t 2
2t
I-t!

1+ l+ta+l+t l

[Chi 4

Indefinite Integrals

132

2) If we have the identity


R (- sin x, -cos x) e i R (sin x, cos x),
Iben we can use the substitution tan x = t to reduce the integral (2) to a
rational form.
Here,
t
1
sinx= '1r
' cosX= '1r-2
2
f 1 t
f I
t

and

x = arc tan

t,

dx = I

+dtt

Example 9. Find

51+ sin x
dx

Solut ion. Putting


tan x= t,

we will have

(3)

t2

sln x = 1 + tl

dt

= Y12

=/.

dt

dx=

'

1+ t2

51+2t
dt
1 S d (t y'2)
t)
=Y2
1+(tY-2)2
( 1+ l+t

J= ~

(l+t

2
)

lr-

arc tan (t ,. 2) + C =

Y1 2 arc tan (,.lr-2 tan x) + C.

We note that the integral (3) is evaluated faster if the nurnerator and
denominator of the fraction are first divided by cos x.
In individual cases, it is useful to apply artificial procedures (see, for
example, 1379).

Find the integrals:

53+::0SX

1373.

1382*

5
1375. 5

1374.

58-4sinx+7cosx'

1386.

1378.

Scos x + dx2 sin x + 3

1387.

dx

1379**. 53s~nx+2cosx d

51+

1381 *

+ 3 cos x x.

tanx dx
I-tan x
I

S(I-cos
sin x
5

X)I

dx
I

sin 2x
1+ sin2 x dXI
cos 2x

Scos4 x + sin4 x

d
XI

1388.

1389*.

5 .

1390*,

Sl+s1nx-cosx
I-s~nx+cosx d
X

S1+:x
. cos x

dx

1385.

1377.

1380.

dx

3 sin 2 x + 5 cos 2 X

SI-sin
sin x d
x XI

2 sin x

5sin!..( + 3 sin xcos x-cos x


1384*. 5.
:X.
.
sin x- sin x cos x

1383*

dx
sin x+ cos x '
cos x d
1+ cos x XI

1376.

cos x
d
sin 2 x-6sillx+5 X.
dx
(2-s1n x) (3-sin x)

Sec. 8]

Integration of Hyperbolic Functions

133

Sec. 8. Integration of Hyperbolic Functions


Integration of hyperbolic functions is completely analogous to the integration of trigonometric functions.
The following basic formulas should be remembered:
1) cosh! x-sinh! x = 1;
2) sinhl x =

3) cosh 2 x =

"2 (cosh

(cosh 2x-I);

4) sinh x cosh x =

2x + 1);

sinh 2x.

Example t. Find
Solution. We have

Scosh

x dx=

5; (cosh 2x+ I) dx= ~ sinh 2x+ ; x+C.

Example 2. Find

~ cosh' x dx.
Solution. We have

S cosh a x dx= Scosh l xd (sinh x) = S (I +sinh! x) d (sinh x) =


.
==slnh

sinha x
x+- +c.
3

Find the integrals:


1391.

S sinh' x dx.

1397.

S tanh' xdx.

1392.

S cosh

1398.

Scoth& x dx.

1393.

Ssinh' x cosh xdx.

&

x dx.

1394. Ssinhl xcosh l xdx.

5smh xd;OSh x
1396. 5sinh" :~OShl x

1395.

1399.

1400.

52 sinh x + 3 cosh x

1401*.
1402.

dx

sinh! x + cosh!

dx

5tan::-I

Ssinh x dx

Vcosh 2x

Sec. 9. Using Trigonometric and Hyperbolio Substitutions for Finding


Integrals of the Form
SR(x, Yaxl+bx+c)dx.

where R is a rational function.

(1)

Indefinite Integrals

134

[Ch.4

+ +

Transforming the quadratic trinomial ax! bx c into a sum or difference


of squares, the integral (I) becomes reducible to one of the following types
of integrals:

I) ~ R (z,

V ml -Zl) dz;

2)

~ R (z.

V ml+zl ) dz;

3)

~ R (z,

Y Zl

ml) dz.

The latter integrals are, respectively, taken by means of substitutions:


1) Z= m sin t or z= m tanh t,
2) z=mtan tor z=msinh t,
3) z m sec t or z = m cosh t.

Example 1. Find

Solution. We have

x!+2x+2=(x+ I)!+ 1.
Putting x+ l=tanz, we then have dx=sec 2 zdz and

_rJ (x+ I)' Y(x+


dx
S
1)'+ 1

sec! z dz =
tan! z sec z

J-

Scos
z dz =
sin' z

V x2 + 2x + 2 +
X+I

= __
.1_ + C =
SUI Z

C.

Example 2. Find
Solution. We have

x! + X + I =

(x + 2"1)2 + 4"3

Putting

Y3.

x+'~=-2- slnht

we get

J=

I) Y3

S( -Y3.

and

Y3

dX=-2- cosh tdt,

Y3

sInh t- - - cosh t -2- cosh t dt=


222
3

}ras .

=-8-

sInh t cosh! t dt-

cosh! t dt =

3 Y3 cosh' t
3 (I
.
=
- -3- - -sinh t
8
8 2

Since

sinh t = .,} 3

(x +

~).

cosh t =

V Xl +

and

t=ln (x+

;+YXI+x+I)+ln

1 t)

cosh t-l--

+C

;3'

+ 1

V sing Reduction Formulas

Sec. 111
we finally have

1 ( X + -1 )
J = -31 (X 2 + X + 1) -I -, 4
2

135

Y Xl + X + 1-~ In (x+ ~ + Yx l +x + 1) +

Find the integrals:


1403. ~ V3-2x-x l dx.

1409.

SV x

1404. ~ V2

1410.

S(Xl +x+ 1)1 dx.

-6x-7 dx.
I

1405.

-+- Xl dx.

J V 9+x dx.
"

Xl

1406. ~ V x l -2x

+2 dx.

S(x-I)Vxdx-3x+2
1412. r
1411.

dx

1413.

SVx +x dx.

1414.

"

J (xI-2x+5)1

1407. ~ V Xl -4 dx.
1408.

S(I +

dx

VI-x'

Xl)

(l-x 2 )

dx

Yl+x

Sec. 10. Integration of Various Transcendental Function s

Find the integrals:

5 :;x

1415. ~

(Xl

+ 1)1 elx dx.

1421.

1416. ~

Xl

cosl 3x dx.

1422.

1417. ~ x sin x cos 2xdx.

1423.

5 In ~ +; dx.

1418. ~ e2X sinl x dx.

1424.

SIn' (x + VI + x') dx.

1419. ~ eX sin x sin 3x dx.

1425.

Sx arc cos (5x -

1420. ~ xe x cos x dx.

1426.

Ssin x sinh x dx.

elX

-2

dx

Ye2~ +e~ + 1

Xl

2) dx.

Sec. 11. Using Reduction Formulas

Derive the reduction formulas for the following integrals:


1427.

In=5(xl~xal)n; find II and I .

1428. In =

~ sinn xdx; find I. and II'

C.

Indefinite Integrals

136

1429.

In=S cosd~ x ;

[Ch.4

find I. and I".

1430. In = ) xne- x dx;

find 110"

Sec. 12. Miscellaneous Examples on Integration

S + x xdxV
1449. S
Yl- 2x

1448.

(1

I-

XC

xdx

2 - X4

r x+

1450.

1 ..!. dx.

J (x + 1)
2

1451 *.

dx

(x 2 +4x) Y4-x 2

1452. )

lR=9 dx.

1453. ~

V x- 4x

1454.

dx.

S Yx dx+x + 1
2

1455. ) xVx 2 +2x+2dx.


1456.

Sx

1457. \
v x
1458.
1459.

/x

xZ-1

Y dx1 -x' .

'"' dx
j VI+X"

Sy

5x
dx.
1+xt

1460. ~ cos x dx.


1461.

dx.

cos x sInS x

5 + yCOtX
dx
x

1463. S
x dx.
Vcos
1462.

Sln 2

sinS

ax

1464. ) cosec s 5x dx.


14655sin2xd
COSO 1&

X.

Miscellaneous Examples on Integration

Sec. 12]

1466.

Ssin ( : -x )Sin(: + x) dx.

1484.

+ ~ ) dx.

1485.

1467. Stan' ( ;
1468.
1469.

S2sinx+3cosx-5
dx

S2 + 3dxcos!

1486.

Scos x+2 sin xcosx+2


dx
sin x
1471. S dx
sin x sin 2x
dx
1472. S
(2 + cos x) (3 + cos x)
1473. S
seclx
dx.
Ytan x+4 tan x+ 1
1470.

1475.

SYu +slnax ax dx.


cos

Scos!
xdx
3x

Yl-x dx.
Yl-x
S sinh x cosh x d
sinh x+ cosh x x.
sinh

1488.
1489.
1490.

S
S

1491.

1495.

2X

1481.

S'IX
3x
sIn 2 cos 2dx.

1482.

5(sin x +dxcos x)!

1483.

S(tan x+dx1) sin

x arc tan x dx.


YI +x 2

+ 13 dx.

lX
e

..!.. dx.

+ 1)

S1_4xdx.
2

1492. ~ (x' -1) 1O-2X dx.

1477. ~ x'-e~' dx.

1480.

eX

1494.

1479. ~ Xl In Vl-xdx.

dx

e!X _ 6e x

(eX

1476. ~ x sin x dx.

~ xe dx.

e2X -2e x

1493. ~

1478.

~ sinh x cosh x dx.

1487. \ y d x .
~ sIn
x

1474.

137

l/

eX

t 1 dx.

Sarc::" x dx.
Sx arc sin ~ dx.

1496. ~ cos (In x) dx.


1497. ~ (Xl - 3x) sin 5x dx.
1498. ~ x arctan (2x+3)dx.
1499.
1500.

Sarc sin y"X dx.


SI x I dx.

Chapter V

DEFINITE INTEGRALS

Sec. 1. The Definite Integral as the Limit of a Sum


1. Integral sum. Let a function f (x) be defined on an interval a~x~b,
and a=xo < XI < ... < xn=b is an arbitrary partition of this interval into
n subintervals (Fig. 37). A sum of the form
n-l

Sn = ~

f (~i) AXi'

(I)

[=0

where
Xi~6ie=;;;;;Xi+l; &xi=xi+l-xi;

i=O, 1, 2, ... (n-l),

is called the integral sum of the function !(x) on [a, b]. Geometrical1y, S"
is the algebraic area of a step-like figure (see Fig. 37).

o1
Fig. 37

to

Fig. 38

2. The definite integral. The limit of the sum Sn' provided that the
number of subdivisions n tends to infinity, and the largest of them, dX',
to zero, is called the definite integral of the function f (x) within the limits
from x=a to x=b; that is,
n-l

11m

~ f (M tui = ~ f (x) dx.

max Ax, -. 0 I =0

(2)

The Definite Integral as the Limit

Sec. 1]

of a Sum

139

If the function f (x) Is continuous on la, b), it is integrable on [a, b); Le.,
the limit of (2) exists and is independent of the mode of partition of the
interval of integration [a, b) into subintervals and is independent of the
choice of points ~i in these subintervals. Geometrically, the definite integral
(2) is the algebraic sum of the areas of the figures that make up the curvilinear trapezoid aABb, in which the areas of the parts located above the x-axis
are plus, those below the x-axis, minus (Fig. 37).
The definitions of integral sum and definite integral are naturally generalized to the case of an interval [a, b), where a> b.
Example 1. Form the integral sum Sn for the function
f (x) = 1 +x
on the interval [1,10] by dividing the interval into n equal parts and choo!ing points
that coincide with the left end-points of the subintervals
[xi, xi+tl. What is the lim Sn equal to?

ai

-+

CJ)

So I
'
H ere, I:! xr=-n-=--;
10-1
9 an d tfoi=xI=xo+t'A xi= 1
utlon.
(~i)=l+l

9i

9i

9i
+n.

WIlence

+-=2+-. Hence (Fig. 38),


n
n

n-l

n-l

S'n = ~f (;i) dXi

'\.~ (
9i) 9 18 81
=~
2+--; n=n n + n 2 (0+ 1 + ... +n-l)=
1=0

1=0

=18+8In(n-l)=18+81
nZ

(1_-!.)=58-.!.._81
n

2n '

lim Sn =-= 58 -.!..

2'

n-+CJ)

Example 2. Find the area bounded by an arc of the parabola y =x2 the
x-axis, and the ordinates x = 0, and x = a (a > 0).
Solution. Partition the base a into n equal y
parts dX=~. Choosing the value of the funcn
tion at the beginning of each subinterval, we will
have
Yl = 0; Y. =

(:

y, = [ 2 ( :

YJ .,. ,

Yn=[<n-l):r
The areas of the rectangles are obtained by multiplying each Yk by the base Ax= ~ (Fig. 39).
n
Summing, we get the area of the step-like figure

Sn=: (:Y l l+2 +3 +,..+<n-l)').


1

Using the formula for the sunl of the squares of integers,


n

n (n

+ 1) (2n + 1)
6

Fig. 39

[Ch.5

Definite Integrals

140

we find
aSn (n-l) (2n-l)
6n l
and. passing to the limit. we obtain
S:'C lim Sn
n -+

lim a l (n-l) n (2n-l) _ al


6n I
3

n -+

(J)

a;)

Evaluate the following definite integrals, regarding them as the


limits of appropriate integral sums:
b

1501. ~ dx.

1502.

1503.

~ x'dx.

-!

10

1504. ~ 2 x dx.

S(v. +gt) dt,

V.

1505*. ~ x' dx.

and g are constant.

1506*. Find the area of a curvilinear trapezoid bounded by

the hyperbola
by two ordinates: x = a and x = b (0 < a < b), and the x-axis.
1507*. Find
x

f (x) = ~ sin t dt.


o

Sec. 2. Evaluating Definite Integrals by Means of Indefinite Integrals


1. A definite integral with variable upper limit. If a function
continuous on an interval [a. bl. then the function
x

F(x) =

~ f (t) dt
a

is the an\iderlvative of the function f (x); that is,


F' (x) =

f (x) for

a ~ x <; b.

2. The Newton-Leibniz formula. If F' (xl = f (x). then


tJ

~ f (x) dx=F (b)-F (a).


a

t (t)

Is

Evaluating Definite Integrals by Indefinite Integrals

Sec. 2]

141

The antiderivative F (x) is computed by finding the indefinite integral

Sf (x) dx = F (x) + C.
Example 1. Find the integral
s

S y;4 dx.
-I

1508. Let
b

I=

Find

5I::

(b

> a>

1).

dJ

1) da;

dJ

2) db

Find the derivatives of the following functions:


x'

1509. F (x) = ~ 1n t dt

(x> 0).

1511. F (x)

vX'

1510. F(x)= ~ Vltrdt.

= Sr /2 dt.

1512. I =

S cos (r) dt

(x

1513. Find the points of the extremum of the function


x

Y=

sin

t dtIII th e region
.
x> 0 .

S-to

Applying the Newton-Leibniz formula, find the integraisJ

Sl+x

1514.

dx

1516.

-x

0
-

1515.

S dx

X'

S et dl.

1517. S:cos t dl.


0

-I

Using definite integrals, find the limits of the sums:


1518**. lim

n-'a;)

1519**. lim
'l-+a;)

1520.

21 + ... +n 11).
(';+n
n
n
( +1 1+ +1 2+ ... +_+1 ).
n
n
n
fl

P
lim IP+2 +;;.+n (p>O).
p

n-+CIO

nil

> 0).

Definite Integrals

142

[Ch.6

Evaluate the integrals:


2

S(x l -2x+3) dx.

1521.

1534.

1522. ~

(V 2x + Vi") dx.

1523.

51+yrii dye
1

y2dy

1535.

SYx-2dx.
-8

4.

1536.

1537. ~ sin 8 q> dq>.


o

dx

e2

1538.

S
5
1

1539.
xdx

I
4.

~ ~':~.

X!+d:x +5'

lMl.

5X!-~:+2

5Z8~

1542.

5 :elX dx.
I

I dz.

1543.

Scosh xdx.
o

n
4.

1632. ~ sec l ada.

1544.

trco::!x'
In

VI'

11

P-

1533.

Scot' q> dq>.

1531.

S tanxdx.
.
:t

-1

1530.

~n x) dx.

:t

1540.

1529.

sin

x!+3x+2'

1528.

dx

x In x

e
e

1dX
X
I

-2

1527.

Scos! ada.
:t

y 25+ 3x.

1526.

1525.

5Y +4
n

1524.

dx

J Y 5+4x-xl

dx

I VI-x

154.1).

S sinh! x dx.
o

Improper Integrals

Sec. 3]

14~

Sec. 3. Improper Integrals


1. Integrals of unbounded functions. If a function I (x) is not bounded
in any neighbourhood of a point c of an interval [a, b] and is continuous
for a ~ x < c and c < x <; b, then by definition we put
b

c-e

c+e

r f (x) dx = elim~o Jr f (x) dx + e-+o


lim r f (x) dx.
J
J

(1)

If the limits" on the right side of (1) exist and are finite, the improper integral is catted convergent, otherwise it is divergent. When c = a or c == b, th..:
definition is correspondingly simplified.
If there is a continuous function F (x) on [a, b] such that F' (x) = f (x)
when x rI: c (generalized antiderivative), then
b

~ f (x) dx=F(b)-F (a).

(2)

a
b

If /I (x) I <;; F (x) when a <;; x <;b and


tegral (1) also converges (comparison test).

If f (x)

~0

r(x)

and 11m
x~c

SF (x) dx

converges, then the in-

I c-x 1m ~-" A t=

00,

f (x)- I

A '1= 0, i. e.,

c-x

liIJ

when x -+ c, then 1) for m < 1 the integral (1) conver~es, 2) for In ~ 1 the
integral (1) diverges.
2. Integrals with infinite limits. If the function f (x) is continuous when
a ~ x < 00, then we assunle
b

t:rJ

r f (x) dx =
J

lim
b-+~

rJ f (x) dx

(3)

and depending on whether there is a finite limit or not on the


the respective integral is called convergent or divergent.
Similarly,
b

lim

(x) dx=

a-+-rD

-QO

r f (x) dx

ri~ht

of (3),

~ f (x) dx= a~~ Sf (x) dx.

and

OIl

b-+-+rD a

-QO

rD

If

II (x) 1<; F (x)

and the integral

gral (3) converges as well.

If

f (x) ~ 0 and

SF (x) dx

11m f (x) x m = A #r.

%-+rD

converges, then the Infe

a
00,

A #:

o.

i. e.,

f (x)-Ii when
x

x -+ co. then 1) for m > 1 the integral (3) converges, 2) for m ~ 1 the integral (3) diverges.

Definite Integrals

144

(Ch.5

Example t.

5ax-=
1

x2

lim

2 -+ 0

-52 2+
dx

lim

2 -+ 0

-1

5' dx
. (1
)=00
2= 11m
--1 ) + l'1m (1
--1
e

2 -+ 0

e~0

and the integral diverges.


Example 2.
b

GO

51+x
dx

= lim

b-+GO

5I +x = lim
dx

b-+(I)

(arc tan b-arc tan O)=~

Example 3. Test the convergence of the probabllity integral


(4)

Solution. We put
1

~ e- xl dx= ~ e- x2 dl:+ Se- x2 dx.


0 0 1

The first of the two integrals on the right is not an improper integral, while
the second one converges, since e- ~ ~ e- x when x ~ 1 and
(I)

e-Xdx= Urn \ e-xdx= lim (-e- b +e- J )=e- 1 ;

b-+~

b-+GO

hence, the integral (4) converges.


Example 4. Test the following integral for convergence:

(5)
Solution. When' x-+-+ 00, we have
1
1

VxI+1 yxa(l+;a)
Since the integral

converges, our integral (5) likewise converges.


Example 5. Test for convergence the eltiptic integral
1

dx

S
VI-x
o

(6)

I mproper Integrals

Sec. 3)

145

Solution. The point of discontinuity of the integrand is x= 1. Applying


the Lagrange formula we get
I
I
VI-x. = .. /(I-x).4x a =

where x <

!.. . - . '
2x:

(I-x)"

< 1. Hence, for x -. 1 we have

Xl

Since the integral

converges, the given integral (6) converges as well.

Evaluate the improper integrals (or establish their divergence):


1546. 5 dx
tx

5
5

(X)

1554.

-e

dx

I +x l

(X)

d
:

5
1548. 5dx
1547.

1555.

-1

dx

x!+4x+9

-00

(X)

xP

1556. ~ sin xdx.


0

1549. 5

1557. 5 dx
In
2

dx

(x -1)2

0
1

1550.

5
5

dx

VI-xl

1558.

GO

1551.

1552.

d:.

Sx In
dx
x
QI)

1559.

S x dxInx

GO

dx
1560. Sx Inlx

SdxXl

GO

n
GO

1553.

S7.
dx
1

(a> I).

1561. ~ cot X dx.


0

(a> 1).

[Cia. ,

Definite Integrals

J46

CIO

CIO

1562. ~ e1563.

kx

(k

dx

> 0).

dx

5x'+ 1

1565.

CIO

Sa~t~nt dx.

dx

S-'-----5xa
o

1566.

X -

o
CIO

1564.

5(XI~

1)1

integrals~

Test the convergence of the following


100

dx

1567. )

Vx+2 V x+x'

dx
V=::e.
S
l-x4
o

1571.

5l:XX
1573. 5~ d x.

1572.

ex>

Xl

n
I

co

1570.

S
o

xdx

Yx + 1
5

1574*. Proye that the Euler integral of the fil st kind (betafunction)
1

B (p, q) = ~ xp - t (l-x)q-t dx
o

converges when p > 0 and q > O.


1575*. Prove that the Euler integral of the second kind (gamma-function)
QI)

r (P) = ~ xP-1e-

dx

converges for p >

o.

Sec. 4. Change of Variable in a Definite Integral


If a function f(x) is continuous over a<;x<;b and x-q>(t) II. function
continuous together with its derivative <p'(t) over a..;t";Pl where a==-cp(a)
and b= cp (~). and f [q> (t)) is defined and continuous on the Interval a<t<p.

Sec. 4)

0/ Variable in a Definite Integral

Change

then

147

fJ

~ f (x) dx = ~ f [q> (t)) q>' (t) dt.


a

Example 1. Find

(a> 0).
Solution. We put

x=asint;
dx=a cos t dt.
Then t=arcsin~
a

P=

and,

consequently,

we can

take a=arcsinO=O,

arc sin 1= ; . Therefore, we shall have


n

Sx

2
2

x2 dx=

Va 2

a2 sin 2 t Ya 2 -a 2 sin 2 tacostdt=

=a&

Ssln

t cos t dt =

~&

n
I

Ssln 2t dt = ~. S(I-cos 4!) dt =


2

a4

n
(

=8 t -4 sln4t

2
1

nat

=16

1576. Can the substitution x = cos t be made in the integral


2

~ Vl-x1dx?
o

Transform the following definite integrals by means of the


indicated substitutions:
n

1577.

~ Vx+ Idx, x=2t-1.


I

S f (x) dx, x = arc tan t.

t 580.

1578.

dx

S YI-x '

X=

sin t.

Sf (x) dx (b > a)

's"

1579.

SYxdx+l
-.

1581. For the integral


b

x= sinh t.
t

Defil1ite Integrals

148

[Ch. 5

indicate an integral linear substitution

x=at + p,
as a result of which the limits of integration would be 0 and 1t
respectively.
Applying the indicated substitutions, evaluate the following
integrals:
4

1582.

dx

S1+ V-'
x

18

1583.

2/
(x-2) ,
2/
3
(x-2) I

dx,

x-2=z'.

1nl

1584.

SVex-l dx,
o

t 585

dt

S3 + 2 cos t

n
I

dx

1586.

1 +a sin1x
S
o
l

tan x = t.

Evaluate the following integrals by means of appropriate


substitutions:
1

1587.

VI

15S8.

Ins

S ~I dx.

5y~

S
o

1589.

lr--

eX reX-l
eX
dx.

+3

1590.

dx.

S
o

2x+

dx

Y3x+ 1

Evaluate the integrals:


s

1591.

1592.

Sx Y x dx+5x+l
l

1593. ~ Vax-x dx.

In

S(l ~XX.)I

1594.

S,,5 ~Xcos

-I

1595. Prove that if

f (x) is an even function, then


a

-a

~ f(x) dx = 2 ~ f(x) dx.

Sec. 5]

But if

Integration by Parts

f (x)

149

is an odd function, then


a

~ f (x)dx =

o.

-0

1596. Show that


~

5e- X'dx=2 5e-X'dx= se-;dX.

-~

1597. Show that


n
1

-dx- - Ssin
-x xx
d
arc cos x -

S
o

1598. Show that


n

2!.

~ f(sinx)dx= ~ f(cosx)dx.
o

Sec. 6. Integration by Parts


If the functions u (x) and v (x) are continuously differentiable on the
Interval [a, bl, then
b

~ U (x) v' (x) dx=u (x) v (x)

1- ~

v (x) u' (x) dx.

(I)

Applying the formula for integration by parts, evaluate the


following integrals:
n
co

1599.

S x cos xdx.

1603.

~ xe-xdx.

1600.

1601.

~lnxdx.

1604.

~ e- ax cos bxdx

co

Sx'elXdx.
0

11.

1602.

co

~ eX sin xdx.
0

1605. ~ e -axSIn bxdx


0

(a> 0).
(a> 0).

Definite Integrals

150

[Ch.5

1606*. Show that for the gamma-function (see Example 1575)


the following reduction formula holds true:

r (p + 1) = pr (p)

(p

> 0).

From this derive that r (n + 1) = nl, if n is a natural number.


1607. Show that for the integral
I

I n= ~sinnxdx= ~ cosnxdx
o

the reduction formula


1

n-l

l n-I

n=-n-

holds true.
Find In' if n is a natural number. Using the formula obtained,
evaluate I, and 110
1608. Applying repeated integration by parts, evaluate the
integral (see Example 1574)
1

B(p, q)=~xP-J(l-x)q-Jdx,
o

where p and q are positive integers.


1609*. Express the following integral in terms of B (betafunction):
2

I n.m = ~ sin m x cos n x dx,


o

if m and n are nonnegative integers.


Sec. 6. Mean-Value Theorem
1. Evaluation of integrals. If ,(x)
b

~F(x) for a~x~b,

~ f (x) dx ,e;;;; ~
a

F (x) dx.

~
a

b
Ip (x)

dx,e;;;;

(I)

If ,(x) and q> (x) are continuous for a ~ x


m

then

b and, besides, q> (x) ~ 0, then


b

~ f (x) Ip (x) dx 0;;;; M ~


a

Ip

(x) dx.

(2)

where m is the smallest and M is the largest value of the function f (x) on
the in terval [a, lll.

Mean-Value Theorem

Sec. 6)

151

In particu Jar, if cp (x) == 1, then


b

m (b-a) EO;; ~ f (x) dx..;;; M (b-a).

(3)

The inequalities (2) and (3) may be replaced, respectively, by their equiva
lent equalities:
b

~ f (x) <p (x) dx = f (c) ~


a

<p (x) dx

a
b

~ f (x) dx = f (~) (b-a),


a

where c and ~ are certain numbers lying between a and b.


Example l. Evaluate the integral
!!.

/= ~

1+

sin' x dx.

Solution. Since O.c;;; sln l x E:; I, we have

that is,
1.57 < I

<

1.91.

2. The mean value of a function. The number


b

.... = b 1 a

Sf (x) dx
a

Is ca lied the mean value of the function

f (x) on the interval a <; x <; b.

1610*. Determine the signs of the integrals without evaluating


them:
I

a) ~ x' dx;
-1

b) ~ x cos x dx;
o

In

C) SSi~X dx.
o

Definite Integrals

152

[Ch. b

1611. Determine (without eval uating) which of the following


integrals is greater:
1

a)

SVI +x

or

dx

0
J

b)

Xl

sin I x dx

~ x sin l xdx;

or

c)

dx;

Se%'dx

Se"dx.

or

Find the mean values of the functions on the indicated intervals:


1612. f(x)=x

o~x~

1613. f(x)=a+b cos x,


1614. f(x)= sin! x,
1615. f (x) = sin 4 x,

1.

-n~x~3t.

O~x~rc.
O~x~rc.

1616. Prove that

5V
o

dx
2+x-~

bt
1 ~
lies
e ween -32 ~ 0.67 and .. r-

y2

0.70. Find the exact value of this integral.


Evaluate .the integrals:
11.

1617.

SV 4 t

Xl

dx.

SxVtanx.

1620*.

n
+1

1618.

58~XI.

5sin x dx.
I

1621.

)(,

-I

&

In

1619.

5lO+~cosx
o

1622. Integrating by parts, prove that

5~d <_I_

200n

0<

lOon

lOOn'

T he Areas of Plane Figures

Sec. 7]

153

Sec. 7. The Areas of Plane Figures


1. Area in rectangu lar coordinates. If a conti nuous curve is defined in
rectangular coordinates by the equation Y=f (x) [I (x) ~O], the area of the
curvilinear trapezoid bounded by this curve, by two vertical lines at the

1(

Fig. 40

Fig. 41

points x=a and x=b and by a segment of the x-axis


is given by the formula
b
S=

ae=;;;;;x~b

(Fig. 40).

Sf (x) dx.

(1)

a
2

x
Example 1. Compute the area bounded by the parabola y = -2 ' the
straight I ines x = 1 and x -= 3, and the x-axis (Fig. 41).

x)

I
I
I

g::f, (z)

-2
Fig. 42

Fig. 43

Solution. The sought-for area is expressed by the integral

154

Definite Integrals

[Ch. 5

Example 2. Evaluate the area bounded by the curve x=2-y_ y l and


1he y-a xis (Fig. 42).
Solution. Here, the roles of the coordinate axes are changed and so the
sought-for area is expressed by the integral

s=

S(2-y_ y2)dy=4 ~,
-I

where the limits of Integration Yl = -2 and Y. = 1 are found as the ordinates


of the points of intersection of the curve with the y-axis.

r
y

Fig. 44

Fig. 45

In the more general case, if the area S is bounded by two continuo us


curves y ='1 (x) and y = f 2(x) and by two vertical lines x =a and x = b, where
11 (x) <af. (x) when a<x~b (Fig. 43), we wilt then have:
b

S=

(2)

[f2 (x)-f.(x) dx.

Example 3. Evaluate the area S contained between the curves


y=2-x2 and yl =x.

(3)

(Fig. 44).
Solution. Solving the set of equations (3) simultaneously, we find the
Jinlits of integration: x.=-1 and x 2 = 1. By virtue of formula (2), we obtain
1

s=

x' - 3 xa) 1 =22 .


(2-x -x2/ 8) dx= ( 2x-3
5
-I
15

-1

If the curve Is defined by equations in parametric form x = q> (t), Y = "I' (t)
then the area of the curvilinear trapezoid bounded by this curve, by tw ~

The Areas

Sec. 7]

of Plane Figures

vertical lines (x =a and x= b), and by a segment of the x-axis is expressed


by the integral
t3

S = ~ (t) cp' (t) dt.


t.

where t 1 and t. are determined from the equations


a=cp(tt) and b=cp(t 2 ) ['P(t)~O on the interval [tt, t z ]].
Example 4. Find the area of the ellipse (Fig. 45) by using its parametric
equations
x=- a cos t,
{ y == b sin t.
Solution. Due to the synlnletry, it is sufficient to compute the area of a
quadrant and then multiply the result by four. If in the equation x=a cos t
we first put x:=: 0 and then x =a, we get the lilnits of integration i 1 = ~ and
t 2 = O. Therefore,
n

= ~ b sin a (-

sin t) dt = ab

Ssin! t dt = ~b
0

and, hence, S=Mb.


2. The area in polar coordinates. If a curve is defined in polar coordinates b~' the equation r =-= f (cp), then the area of the sector AOB (Fig. 46),
bounded by an arc of the curve, and by t\\'o radiUS vectors OA and OB.

o~-...

....._---X
Fig. 47

Fig. 46

which correspond to the val ues <PI = a and <PI =~, is expressed by the
integral
f}

1
S = 2"

Jr [f (cp)]2 dq>.
a

Example 5.

,2

= a Z cos 2q>

Find
(Fig. 47).

the

area

contained inside

Bernoulli's

lemniscate

Definite Integrals

156

[Ch. 5

Solution. By virtue of the symmetry of the curve we deter111ine first one


,quadrant of the sought-for area:

Whence S::: al.

1623. Compute the area bounded by the parabola y=4x-x l


and the x-axis.
1624. Compute the area bounded by the curve y = In x, the
x-axis and the straight I ine x = e.
1625*. Find the area bounded by the curve y=x (x-i) (x-2)
and the x-axis.
1626. Find the area bounded by the curve y3 = x, the straight
line Y= 1 and the vertical line x=8.
1627. Compute the area bounded by a single half-wave of the
sinusoidal curve y= sinx and the x-axis.
1628. Conlpute the area contained between the curve y=tanx,
the x-axis and the straight line x=1- .
1629. Find the area contained between the hyperbola xy=m'l.,
the vertical lines x = a and x = 3a (a> 0) and the x-axis.
t 630. Find the area contained between the witch of Agnesi

Y=

03

Xl

+a and the x-aXIS.


2

1631. Compute the area of the figure bounded by the curve


y=-=x 3 , the straight line y=8 and the y-axis.
1632. Find the area bounded by the parabolas y"c:2px and
x" = 2py.
,.
1633. Evaluate the area bounded by the parabola Y= 2x-x
and the straight line y=-x.
1634. Compute the area of a segment cut off by the straight
line y=3-2x from the parabola y=x".
1635. Compute the area contained between the parabolas yrc:r.x,
Y=i and the straight line y=2x.
1636. Compute the area contained between thQ parabolas
x2

Y="3 and y=4-3"x


1637. Compute the area contained between the witch of
1

Xl

Agnesi y= 1+x2 and the parabola y= 2.


1638. Compute the area bounded by the curves ya:ae".
and the straight line x == 1.

II-a;s-~

Sec. 7)

x2

al

T he Areas of Plane Figures

157

1639. Fi nd the area of the figure bounded by t"he hyperbola


y2
b2 = 1 and the straight line x = 2a.
1640*. Find the entire area bounded by the astroid

x' +y' =a'.


1641. Find the area between the catenary

Y= a cosh!.,
a

:e

the y-axis and the straight line Y= (e t + 1).


1642. Find the area bounded by the curve a2y2=x2(a2_xl).
1643. Compute the area contained within the curve

1644. Find the area between the equilateral hyperbola X 2 _ y 2 =


= 9, the x-ax is and the diameter passi ng through the point (5,4).
the x-axis,
1645. Find the area between the curve y=~,
x
and the ordinate x=l (x>l).
X
1646*. Find the area bounded by the cissoid y2=2a '
-x
and its asymptote x = 2a (a> 0).
x (x-a)2
1647*. Find the area between the strophoid y2 = 2a-x and
its asymptote (a> 0).
1648. COlllpute the area of the two parts into which the
circle X 2 _t- y 2=:8 is divided by the parabola y2=2x.
1649. Compute the area contained between the circle Xl + y2 = 16
and the parabola x 2 = 12 (y-l).
1650. Find the area contained within the astroid
X= a cos' t;
y= b sin' t.
1651. Find the area bounded by the x-axis and one arc of
the cycloid
X = a (I - sin t),
{ y = a (1 - cos t).
1652. Find the area bounded by one branch of the trochoid
{ x=at-bsint,
y=a-b cos t

(O<b:s;;;;;a)

and a tangent to it at its lower points.

Definite Integrals

158

{Ch. 6

1653. Find the area bounded by the cardioid


{

X = a (2 cos t-cos 2/),


y = a (2 sin t - sin 2/).

1654*. Find the area of the loop of the foli urn of Descartes
3at 2

3ut

x=l+t S ;

Y=l+t"

1655*. Find the entire area of the cardioid f=a (l+cosq.


1656*. Find the area contained between the first and second
turns of Archimedes' spiral, r = aq>

(Fig. 48).
1657. Find the area of one of the
leaves of the curve r = a cos 2fp.
1658. Find the entire area bounded by the curve = a l sin 4(p.
1659*. Find the area bounded by
the curve , = a sin 3(p.
1660. Find the area bounded by
Pascal's limac;on

,2

Fig. 48

= 2 + cos cp.

1661. Find the area bounded by the parabola r = a secl ~

and the two half-lines cp

=:

and cp

=;.

1662. Find the area of the ellipse f = 1+ ePcos q> (8< 1).
1663. Find the area bounded by the curve , = 2a cos 3(p and

lying outside the circle r = a.


1664*. Find the area bounded by the curve x4 + y4 = Xl + yl.
Sec. 8. The Arc Length of a Curve
1. The arc length in rectangular coordinates. The arc length s of a curve
Y=f (x) contained between two points with abscissas x=a and x=b is
b

s= ~

VI +U'2dx.

Example 1. Find the length of the astroid x 2/1 y2 /1 :=a 2/a (Fig. 49).
Solution. Differentiating the equation of the astroid, we get
II,

y,=_L,.
x/

The Arc Length of a Curve

Sec. 8]

159

For this reason, we have for th~ arc length of a quarter of the astroi d:

{s=

SVI+ ~:;:
o

dx=

5:::: dX=~ ~

Whence s = 6a.
2. The arc length of a curve represented parametrically. If a curve Is
represented by equations in parametric form, x = cp (t) and y = "I> (t), then the
arc length s of the curve is
tJ
S

~ YX'I+y'l

dt,

tI

where t l and t 2 are values of the parameter that correspond to the extremities
of the arc.

x
---1~--------_----I"-'-."-'-'-

27Ca

Fig 49

Fig. 50

Example 2. Find the length of one arc of the cycloid (Fig. 50)
X = a (t - sin t),
{ y=a (I-cos t).
dx

dy

Solution. We have dt =a (1- cos t) and dt = a sin t. Therefore,


In

= S Val (I-cos

2n

t)l+ a l

sln l t dt =

2a

Ssin ~ dt =8a.
0

The limits of integration t 1 =0 and t l =2Jt correspond to the extreme poinfs


of the arc of the cycloid.
If a curve is defined by the equation r=f (cp) in polar coordinates, then
the arc length s is
P
s= ~ V,I +,,1 d<p.
a

where a and ~ are the values of the polar angle at the extreme points of
the arc.

Definite Integrals

160

(Ch. 6

Example 3. Find the length of the entire curve , =a sln l ~ (Fig. 51).

The entire curve is described by a point as cp ranges from 0 to 3n.

Fig. 51
Solution. We have

,'= a sin t

cos : ' therefore the entire arc length of

Ihe curve is
an
S

= S'Vi

a~

a2 sin' ..!.
3

+a

sin' ..!. cos 2 ..!. dlp = a


3
3

J. sln ..!.3 dlp = 3na2


2

1665. Compute the arc length of the semicubical parabola


yl = x from the coordinate origin to the point x = 4.
1666*. .Find the length of the catenary y = a cosh ~
from the
a
vertex A (O,a) to the point B (b,h).
1667. Compute the arc length of the parabola y = 2
from
x =0 to X= I.
1668. Find the arc length of the curve y = eX lying between
the points (0, I) and (I ,e).
1669. Find the arc length of the curve y = In x from .t = Va
to x= VB.
1670. Find the arc length of the curve y = arc sin (e-') from
x=O to X= 1.
1671. Conlpute the arc length of the curve x=lnsecy, lying

Vx

between y=O and y= ~ .


1672. Find the arc length of the curve x= ~ y2_ ~ Iny from
Y= 1 to y=e.

Sec,

Volumes of Solids

9)

161

1673. Find the length of the right branch of the tractrix


z

X= Va -

yZ+aln

la+ Y~I

from y=a to y=b (0< b<a).

1674. Find the length of the closed part of the curve 9ay l =

= x (x-3a)l.

1675. Find the length of the curve y= In (coth ~) from x=a

to x==b (O<a<b).

1676*. Find the arc length of the involute of the circle

x=a(c?st+tsint),} from t=O to t=T


y=a(slnt-tc05t)

1677. Find the length of the evolute of the ellipse

1678. Find the length of the curve


x ==- a (2 cos t- cos 21), }
y=a(2 sint-sin2t).

1679. Find the length of the first turn of Archimedes' spiral


r = atp.
1680. Find the entire length of the cardioid r=a(l+cos<p).
1681. Find the arc length of that part of the parabola
, = a sec!; which is cut off by a vertical line passing through
the pole.
1682. Find the length of the hyperbolic spiral r<p = I from the
point (2"/2) to the point C/2,2).
1683. Find the arc length of the logarithmic spiral r = aemcp ,
lying inside the circle r = a.
t 684. Find the arc length of the curve cp = ~
+ ~) from
r = 1 to , = 3.

(r

Sec. 9. Volumes of Solids


to. The volume of a solid of revolution. The volumes of soB4s formed by
the revolution of a curvilinear trapezoid (bounded by the curve !I ~f (x), tbe
x-axis and two vertical lines x = a and x = b J about the x- and 'y-axes are
6 - 1900

[Ch.6

Definite Integrals

162

expressed. respectively, by the formulas:


b

~ yldx;

I) Vx=n

2) V y =2n

~ xydx*).
a

Example I. Compute the volumes of solids formed by the revolution of a


figure bounded by a single lobe of the sinusoidal curve y = sin x and by the
segment 0 < x ~ n of the x-axis about: a) the x-axis and b) the y-axis.
Solution.
n

a) VX=-n

Ssin

n2

xdx=T;

:t

b) Vy=2n

~ xsinxdx=2n(-xcosx+sinx)~=2nl.
o

The volume of a solid formed by revolution about the y-axis of a figure


bounded by the curve x=g(y), the y-axis and by two parallel1ines y=c and
y=d, may be determined from the formula
d

Vy=n ~ xldy,
c

obtained fronl formula (I), given above, by interchanging the coordinates


x and y.
If the curve is defined in a different form (parametrically, in polar coordinates, etc.), then in the foregoing formulas we must change the variable of
Integration in appropriate fashion.
In the Inore general case, the volumes of solids formed by the revolution
about the x- and y-axes of a figure bounded by the curves Yl ='1 (x) and Y2 = f 2(x)
[where fl(X)~f2(X)], and the straight lines X=Q and x=b are, respectively,
equal to
b

Vx=n

~ (y:-y:) dx
a

and
b

Vy=2n

~ x (YI-YI) dx.
a

Example 2. Find the volume of a torus formed by the rotation of the


circle x 2 + (y-b)1 =a2 (b ~ a) about the x-axis (Fig. 52).
*) The solid is formed by the revolution, about the y-axis, of a curvilinear
trapezoid bounded by the curve y=f (x) and the straight lines x=a, x=b,
and y=O. For a volume element we take the volume of that part of the solid
formed by revolving about the y-axis a rectangle with sides y and dx at a
distance x from the y-axis. Then the volume element dV y =2nxydx, whence
b

Vy=2n

~ xydx.
CI

Volumes of Solids

Sec. 9]

Solution. We have
Yl ==b- Val-xl and Y2=b+

Therefore,

Va -x2.
2

Vx=n ~ [(b+ Val-xl)I-(b- Va 2-xl )2J dx =


-a

~ Val-xl dx =

= 4nb

211,1

alb

-a

(the latter integral is taken by the substitution x =a sin t).

-Q

Fig 52

Fig. 53

The volunle of a solid obtained by the rotation, about the polar axis, of a
sector formed by an arc of the curve r == F (rp) and by two radius vectors
~ =-~ a, cp == ~ may be computed froln the formula
p

Vp = ; n

5,1

sin Ipd Ip.

Thls same formula is conveniently used when see{<ing the volume obtained
by the rotation, about the polar axis, of some closed curve defined in polar
coordinates.
Example 3. Determine the volume formed by the rotation of the curve
, == a sin 2cp about the polar axis.
Solution.
n

Vp= 2.~ n

5
I

,1

sin Ip dip =

na'5

siu l

5
I

= ~ na S
3

sin 4 ep cos' (p dq>

== ~ .na'.
105

21p

sin Ip dip

Definite lntegrals

164

[Ch. 5

2. Computing the volumes of solids from known cross-sections. If S = S (x)


Is the cross~ctional area cut ofT by a plane pprpendicular to some strai~ht
line (which we take to be the x-axis) at a point with abscissa x, then the
volume of the solid is
XI

V=

~ S (x)dx,
Xl

K 1 and x! are the abscissas of the extreme cross-sections of th~ solid.


Example 4. D~termine the volume of a wedge cut oft a circular cylinder
by a plane passing through the diampter of the base and inclinpd to the base
at an angle u. The radius of the base is R (Fig. 53).
Solution. For the x-axis w? take th~ diam~ter of the base along which
the cutting plane intersects the base, and for the y-axis \ve take the dialneter
of the base perpendicular to it. The equation of the circumference of the base
is Xl y2 = R2.
The area of the spction ABC at a distance x from the origin 0 is
I
1
2
S (x) = area ~ ABC = 2" AB. Be = 2"' yy tan a = Y2 tan Q. Therefore, the soughtfor vol urne of the wedge is

where

V=2

SyZtanadx=tana S(RZ-xZ)dx= ~ tanaR z


o

1685. Find the volume of a solid formed by rotation, about


the x-ax is, of an area bounded by the x-axis and the parabola
y= ax-r (a >0).
1686. Find the volume of an ellipsoid formed by the rotation
Xl
y2
of the ellipse aa-+/;1-=1 about the x-axis.
1687. Find the volume of a solid formed by the rotation, about
the x-axis, of an area bounded by the catenary y = a cosh!...,
the
a
x-axis, and the straight lines x == a.
1688. Find the volume of a solid formed by the rotation, about
the x-axis, of the curve y = sin 2 x in the interval between x = 0
and X=1t.
1689. Find the volume of a solid formed by the rotation, about
the x-axis, of an area bounded by the semicubical para bola y2 = x=s,
the x-axis, and the straight line x == ].
1690. Find the vol ume of a sol id formed by the rotation of
the same area (as in Problem 1689) about the y-axis.
1691. Find,t,he volumes of the solids formed by the rotation
of an area bounded by the lines y=e x , x=O, y=O about: a) the
x-axis and b) the y-axis.
1692. Find the volume of a solid formed by the rotation, about
the y-axis, of that part of the parabola yl = 4ax which is cut otT
by the straight line x = a.

Sec. 9]

Volumes of Solids

165

1693. Find the volume of a solid formed by the rotation, about


the straight line x = a, of that part of the parabola yJ. = 4ax which
is cut 011 by this line.
1694. Find the volume of a solid formed by the rotation, about
the straight line y = - p, of a figure bounded by the parabola
yl = 2px and the straight line x = ~

1695. Find the volume of a solid formed by the rotation, about


the x-axis, of the area contained between the parabolas y = x2
and y= V"t.
1696. Find the volume of a solid formed by the rotation.
about the x-axis, of a loop of the curve (x-4a) y2 = ax (x- 3.2)
(a >0).
1697. Find the volume of a solid generated by the rotation

of the c)'ssoid y2=-2


AI
about its asymptote x=2a.
a -x
1698. Find the volume of a paraboloid of revolution whose
base has radius R and whose altitude is H.
1699. A right parabolic ~egment \vhose base is 2a and altitude h
is in rotation about the base. De~ermine the volume of the resulting solid of revolution (C:lvalieri's "lemon").
1700. Show that the volume of a part cut by the plane x = 2a
off a solid formed by the rotation of the equilateral hyperbola
x z _ y2 = a" about the x-axis is equal to the volulne of a sphere
of radius a.
1701. Find the volume of a solid formed by the rotation of a
figure bounded by one arc of the cycloid x=-a (t- sin t).
y=a(l--cost) and the x-axis, about~ a) the x-axis, b) the y-axis,
and c) the axis of symmetry of the figure.
1702. Find the volume of a solid formed by the rotation of
the astroid x=acos't, y=b sin' t about the y-axis.
1703. Find the volume of a solid obtained by rotating the
cardioid r=a(l +coscp) about the polar axis.
1704. Find the volume of a solid formed by rotation of the
curve r=acos 2 cp about the polar axis.
1705. Find the volume of an obeli<;k whose p~rallel bases are
rectangles with sides A, B and a, h, and the altitude is h.
1706. Find the volume of a right elliptic cone whose base is
an ellipse with semi-axes a and b, and altitude h.
1707. On the chords of the astroid x2/a+y"J/3.=a2/J, which ar,e
parallel to the x-axis, are constructed squares whose sides are
equal to the lengths of the chords and whose planes are perpendicular to the xy-plane. Find tHe volume of the solid formed by
these squares.

D_e...:..fi_n_ite_In_t-:;eg~ra_l_s

_16_6

[Ch. 5

1708. A circle undergoing deformation is moving so that one


of the points of its circumference lies on the y-axis, the centre
describes an ellipse ;: +~= I, and the plane of the circle is
perpendicular to the xy-plane. Find the volume of the solid
generated by the circle.
1709. The plane of a moving triangle remains perpendicular
to the stationary diameter of a circle of radius a. The base of
the triangle is a chord of the circle, whi Ie its vertex slides along
a straight line parallel to the stationary diameter at a distance h
from the plane of the circle. Find the volume of the solid (called
a conoid) formed by the motion of this triangle from one end of
the diameter to the other.
1710. Find the volume of the solid bounded by the cylinders
x" -t- Zl = at and y2 + Zl == al.
1711. Find the volunle of the segment cut off froIll the elli py2
22
tic paraboloid 2p-l-2q=x by the plane X=Q.
1712. Find the volume of the solid bounded by the hyperbox2
y"
Z2
loid of one sheet a2+b2 -2"=1
and the planes z=O and z=h.
C
2

22

x
2+
b! +2"= 1.
c

1713. Find the volume of the ellipsoid a

y2

Sec. 10. The Area of a Surface of Revolution


The area of a surface formed by the rotation, about the x-axis, of an
arc of the curve y == f (x) between thE' poi nts x = a and x = b, is expresse-d by
the form uta
b
b
Sx=2n

Syd;x dx =2:t SY VI+y'ldx


a

(I)

(ds is the differen tial 01 the arc of the curve).

,y

2a

x
Fig. 54

-....&.---__

--L

no

21Ca

___

Fig. 55

I f the eq ua lion of the curve is represented differen t1 y, the area of the


surface Sx is cbtained from formula (1) by an appropriate change of variables.

Sec. 10]

167

The Area of a Surface of Revolution

Example 1. Find the area of a surface formed by rotation, about the


x-axis, of a loop of the curve 9y 2 =x (3-X)1 (Fig. 54).
Solution. For the upper part of the curve, when 0 e=;;;;; x <; 3, we have
y= ; (3- x)

Whence the differential of the arc ds= ;~ dx. From for-

yx.

mula (I) the area of the surface

Jr31
a

S = 2n

..r- x+ 1
(3 -x) r x 2 Yx dx = 33t.

Example 2. Find the area of a surface formed by the rotation of one arc
of the cycloid x==a (t-sln t); y==a (I-cos t) about its axis of symmetry
(Fig. 55).
Solution. The desired surface is formed by rotation of the arc OA about
the straight line AB, the equation of which is x=na. Taking y as the independent variable and notIng that the axis of rotation
AB is displaced relative to the y-axIs a distance na, we
y
will have
2a

= 2n

ds
(Tea-x) dy dYe

5
o

Passing to the variable t we obtain


J

S =2n

j" (no-ot+a sin t)

y (~~r + (~~r

dt

=-

5
5(n

= 2n

(no-at

+ 0 sin t) 2a sin ~

dt

= 4no

sin

~-

t sin

~ + sin t sin ~)

B
FIg.

dt =

4.

56

tJ'" =8Jt (n- 4) at.

t
t
t
=4na 2 [ -2ncos2+2tcos2-4sin2+3s1n'2

1714. The dimensions of a parabolic mirror AOB are indicated


in Fig. 56. It is required to find the area of its surface.
1715. Find the area of the surface of a spindle obtained by
rotation of a lobe of the sinusoidal curve y= sinx about the
x-axis.
1716. Find the area of the surface formed by the rotation of
a part of the tangential curve y = tan x from x = 0 to x
about the x-axis.
1717. Find the area of the surface formed by rotation, about
the x-axis, of an arc of the curve y::::z e-~, from x = 0 to x ==

=: '
+00.

168

Defin ite Integrals

lCh.6

1718. Find the area of the surface (called a catenoid) formed


by the rotation of a catenary y = a cosh!...
about the x-axis from
a
X = 0 to x === a.
1719. Fi nd the area of the surface of rotation of the astroid

a2/3 about the y-axis.


1720. Find the area of the surface of rotation of the curve
x={ y2_ ~ loy about the x-axis from y= 1 to y=e.
1721 *. Find the surface of a torus formed by rotation of the
circle x 2 +(y_ b)2-==a 2 about the x-axis (b>a).
1722. Find the area of the surface forlned by rotation of the
ellipse ~:+~2=1 about: I) the x-axis, 2) the y-axis (a>b).
1723. Find the area of the surface formed by rotation of one
arc of the cycloid x = a (t - sin t) and !J = a (1- cos t) about: a) the
x-axis, b) the y-axis, c) the tangent to the cycloid at its highest
point.
1724. Fincl the area of the surface forlned by rotation, about
the x-axis, of the cardioid

%2/8 _~ y2/3 ==

x=a(2cost-cos2t), }
y = a (2 sin t- sin 2/).
1725. Determine the area of the surface formed by the rotation
of the lemniscate r 2 == a l cos 2cp about the polar axis.
1726. Determine the area of the surface formed by the rotation
of the cardioid, = 2a (1 + co~ cp) about the polar axis.

Sec. 11. Moments. Centres of Grav:ty. Guldin's Theorems


to. Static moment. The static moment relative to the i-axis of a material
point A having mass m and at a distance d fronl thz I-axis is the quantity

M,=md.

The static moment relative to the I-axis of a system of n material ('oints


with Inasses ml , m2 , , mn 1ying in the plane of the axis and at distances
d1 , d z , , dn is the sum
n

M l = ~midi'

(1)

I=J

where the distanct's of points lying on one side of the l-axis have the plus
sign, thost' on th~ other side have the minus sign. In a similar manner we
define the statIc moment of a system of potnts rt'lative to a rlane.
If the massps continuously fill the line or figure of the xy-plane, then the
Rtatic moments Mx and My about 1he x- and y-axes are exoressed ,respectively) a~ integrals and not as the sums (1). For the cases of geometric figures,
the density is considered equal to unity.

Moments. CentrtS of Gravity. Guldin's Theorems

Sec. 11]

In particul ar: 1) for the curve x = x (s); y = y (5),

Is the arc length, we have

wh~re

the parameter s

Mx=~y(s)dS; My=~X(S)dS
o

Y (dx) 2 + (dy)2

(ds =

169

(2)

is the differential of the arc);

!---~
T---~

v
b

t--- b

Fig. 58

Fig. 57

2) for a plane fi~ure bounded by th~ curve y = y (x),


vertIcal ltnes x= a and y.==.b, we obtain
b

MX =

5Y I y I dx;

Find the statIc

th~

x-axis and two

My =

Example 1.

I
--I

Sx r y I dx.

(3)

monl~nts

about the

x- and

y-axe3 of a triangle

+f = 1, x::=O, y=O (FIg. 57)

bounded by the straight lines: ~

Solution. Here. y=b ( 1- : ) . Applyir.g formula (3), we obtain

and

My=b

{.,

X )

J xli-a-

dx=

a2 b

ti-

2. Moment of inertia. The nl0mellt of Inertia, about an l-a'Ci5, or a rn1fcrial point of rna~s m at a dIstance d fronl th~ l-aXl'i, I') the llulnber II-=- fnd 2
Th~ moment of cnertia, about an I-axis, o[ a systel1l of n nlalenal pOints
with masses mJ , m:. __ , mil IS the surn
Il

J,=-=-~ nllj~,
i ... l

Definite Integrals

170

[Ch.5

where d1 , d l , dn are the distances of the points from the I-axis. In the
case of a continuous mass, we get an appropriate integral in place of a sum.
Example 2. Find the moment of inertia of a triangle with base band
altitude h about its base.
Solution. For the base of the triangle we take the x-axis, for its altitude,
the y-axis (Fig 58).
Divide the triangle into intlnitely narrow horizontal strips of width dy,
which play the role of elementary masses dm. Utilizing the similarity of
triangles, we obtain
h-y
dm=b --dy
h

and
b

dl x =y"dm=7i y2 (II-g) dy.

Whence
h

r
1
I X--"= Ii J y2 (h-y) dy = 12 biz'.
b

3. Centre of gravity. The coordinates of the centre of gravity of a plane


figure (arc or area) of mass M are computed from the formulas
-

My

Mx

x=M' y=,

where M x and My are the


ric figures, the mass M is
For the coordinates of
curve y=f (x) (a ~x ~b),
we have
B

~
-

static moments of the mass. In the case of geolnetnumerically equal to the corresponding arc or area.
the centre of g;ravity (~ Y) of an arc of the plane
connecting the points A la, f (a)l and B [b, f (b)],

b
X

d!>

x= -.-5- =

B
X

Y 1 + (y')Zdx

yds

y=--s

~b------

~ Vl+(y')2dx
a

y 1 + (y')2 dx

II

V I + (y')' dx

The coordinates of the centre of gravity (~ Y) of the curvilinear trapezoid


a ~ x ~ b, 0 <;; y ~f (x) Inay be computed from the formulas
b

Sxy dx
-

x=-S- ,

Sy2 dx
a

y = --S-

where S =

~ Y dx is the area of the figure.


a

There are similar formulas for the coordinates of the centre of gravity of
a volume.
Example 3. Find the centre of gravity of an arc of the semicircle
x 2+ y2 = a 2; (y ~ 0) (Fig. 59).

Sec J11

Moments. Centres of Gravity Guldin's Theorems

171

Solution. We have
Y -==

Y-a-_-x-

y' == _-_x_

2;

Ya z

x!

and
ds

~r
y

--=.

_adx .
Yal-x!

+ (y')! dx.:.=

Whence
a

' f ax
J ds =-= Ya -x d

M y::-:=

t ---=

0,

OJ

-a
a

-a

(l

Ya -x

Mx== I'"' yds--- \'

'-a

'"
-a

_udx -

Va -x
Z

_: 2a 2 ,

M-

adx

J -V-a--x-'

na.

-a

l-1ence,

x~-O;

Y=-=

Fa.

4. Guldin's theorems.
Theorem t. The area of a surface obtaIned by the rotation of an arc of
a plane curve about SOIne axis lying in the same plane as the curve and not
intersecting it is equal to the product of the length of the curve by the
circunlference of the circle described by the centre of gravity of the arc of
the curve.
Theorem 2. The volulne of a solid obtained by rotation of a plane figure
about sonle axis lying in the plane of the figure and not intersecting it is
equal to the product of the area of this fi~ure by the circumference of the
circle described by the centre of gravity of the figure.

-a

Fig. 59

1727. Find the static moments about the coordinate axes of


a segment of the straight line
~+1[=1
a
b

lying between the axes.

Definite Ifltegrals

172

(Ch. I

1728. Find the static moments of a rectangle, with sides a and b,


about its sides.
1729. Fi nd the static momen~s, about the x- and y-axes, and
the coordinates of the centre of gravity of a triangle bounded by
the straight lines x+y=a, x=O, and y=O.
1730. Find the static moments, about the x- and y-axes, and
the coordinates of the centre of gravity of an arc of the astroid
I

x'-ty'=a',
lying in the first quadrant.
1731. Find the static moment of the circle
r = 2asin q>
about the polar axis.
1732. Find the coordinates of the centre of gravity of an arc
of the catenary
x
y=acosh -a
from x= -a to x=a.
1733. Find the centre of gravity of an arc of a circle of radius tJ
subtending an angle 2a.
1734. Find the coord inates of the centre of gravity of the arc
of on~ arch of the cyeloid
X= a (t- sin t); y = a (I-cos t).
1735. Find the coordinates of the centre of gravity of an area
x2
LI I
bounded by the ell ipse Q2 -1- b2 = 1 and the coord i na te axes (x ~ 0,
y~O).

1736. Find the coordinates of the centre of gravity of an area

bounded by the curves


2

r-

y=x, Y=~ x.
1737. Find the coordinates of the centre of gravity of an area
bounded by the first arch of the cycloid
x=a(t-sinl), y=a(l-cost)

and the x-axis.


1738**. Find the centre of gravity of a hemisphere of radius a
lying ahove the xy-plane with centre at the origin.
1739**. Find the centre of gravity of a hOlno~eneous right
circular cone with base radius r and altitude h.
1740**. Find the centre of gravity of a honl0~eneous hemisphere of radius a lying above the xy-plane with centre at the
origin.

Sec. 121

ApplginR Definite lnieRrals to Solution of Phl/sical Probltms

173

1741. Find the moment of inertia of a circle of radius a about


its d iarne~er.
1742. Find the mOITlents of inertia of a rectangle with sides
a and b about its sides.
1743. Find the mOlllcnt of inertia of a right parabolic segment
with base 2b nnd altitude h about its axis of sYlnrlletry.
1744. Find the moments of inertia of the area of the ellipse
~: + ~: = 1 about its principal axes.
1745**. Flnd the polar monlent of inertia of a circular rinq
with radii R 1 and R, (R,<R 2 ) that is, the nlonlent of inertia
about the axis passing through the centre of the ring and perpendicular to its plane.
1746**. Find the moment of inertia of a homogeneous right
circular cone \\lith base radlus R and altitude H about its axis.
1747**. Flnd the Inonlent of inertia of a hotnogeneous sphere
of radius a and of rnass M about its diaillcter.
1748. Find the surlace and volullie of a torus obtained by
rotating a cIrcle of radIus a about 8n aXIs lying in its plane
and at a distance b (l> >a) frOll1 its centre.
1749. a) Dcterrnine the po~ition of the centre of gravity of
2

an arc of the astroid x 3 -~ {/3 =a7 lying in the first quadrant.


b) Find the Cl'n~rc of gravity of an area bounded by the curves
2
y2 = '2px and x == 2py.
17fiO**. a) Find the centre of gravity of a sen~icircle usin~~
Guldin's theorerTI.
b) Prove by Guldin's theorem that the centre of gravity of
a triangle is distant from its base by one thlrd of its altitude
Sec. 12. Applying Definite Integrals to the Solution of Physical Problems
1. The path lraverfied by a point. If a point IS In motion ;lloll~ some
curve and the Clhsolute value of th~ velocity u =-1 (t) is :J known fUI1(uon of
the time t, then the path traversed by the pOint in an Interval of time
(t l , tIl is

Example 1. The velocity of a point is


(J=O.l, m/~ec.
Find the path s covpred by the point In the interval of time T = 10 ~ec followIng the conlOlc.celnent 01 motion. What is the Iuean velucity ~f mollon
durlng lhis interval?

[Ch.S

Definite Integrals

174
Solution. We have:

s=

10

t4110
O.lt'dt=O.lT 0 =250 metres

5
o

and
V mean

S
= y=
25

mjsec.

2. The work of a force. If a variable force X = f (x) acts in the direction


of the x-axis, then the work of this force over an interval [XI' x 2 ] is
X2

A= Sf (x) dx.
Xl

Example 2. What work has to be performed to strC'tch a spring 6 Cln, if


a force of 1 kgf stretches it by 1 cm?
Solution. According to Hook's la\v the force X kgf stretching the spring
by X m is equal to X == kx, where k is a proportionali ty constant.
Putting x=O.OI m and X=l kgf, we get k=lOO and, hence, X==IOOt.
Whence the sought-for work is
0.06

A=

0.06

=0.18 kgm

100xdx=50x21

3. Kinetic energy. The kinetic energy of a material point of mass nz and


velocity v is defined as

The kInetIC energy of a system of n material points with masses


ml , ma, , mn having respective velocities VI' VI' , v n ' is equal to
n

K=

L m~Vi.

(I)

I. =1

To compute the kinetic energy of a solid, the latter is appropriately partitioned into elementary parttcles (which play the part of material points); then
by summing the kinetic energies of these particles we get, in the limit, an
integral in place of the sum (I).
Example 3. Find the kinetic energy of a homogeneous circular cylinder
of density tJ with base radius R and altitude h rotating about its axis with
angular velocity roo
Solution. For the elementary mass dm we take the mass of a hollow
cylinder of altitude h with inner radi us r and wall thickness dr (Fig. 60).
We have:
dm = 211:' .h6 dr.
Since the linear velocity of the mass dm Is equal to
ki net Ic energy is
v 2dm

dK =- -2- = n,'ro 2 h6 dr,

= fro,

the elemenfary

Sec. 12]

Applying Definite Integrals to Solution of Physical Problems

175

Whence
R

K = 3tCl)2hfJ

r d, ~
S
o
3

noo2fJR4h
4
.

4. Pressure of a liquid. To compute the force of liquid pressure we use


Pascal's law, which states that the force of pressure of a liquid on an area S
at a depth of immersion h is
p=yhS,
where V is the specific weight of the liquid.

Fig. 60

FIt! 61

Example 4. Fi nd the force of pressure experienced by a selniclrcle of


radiu5 r submerged vertically in water so that it5 diameter is flush with the
\vater surface (Fig 61).
Solution. We partition the area of the semicircle into elements-strips
parallel to the surface of the water. The area of one such element (ignoring
higher-order infinitesimals) located at a distance h frotn the surface is
ds =-= 2xdh = 2

V,2 -

h2 dlz.

The pressure experienced by this elernent is


dP =yh ds = 2yh

V ,2-h" dh,

where y is the speciflc weight of the water equal to unity.


Whence the entire pressure is

1751. The velocity of a body thrown vertically upwards with


initial velocity Vo (air resistance neglected), is given by the

17i

Ie.":. 6

Definite Integrals

formula

v= vo-gt,
where t is the time that elapses and g is the acceleration of gravity. At what distance from the initial position will the body
be in t seconds from the time it is thrown?
1752. l"he velocity of a body thrown vertically upwards with
initial velocity Vo (air resistance allowed for) is given by the
fornlula

v=c.tan( - ~

t+arctan

~O),

\vhere t is the time, g is the acceleration of gravity, and c is


a constant. Find the altitude reached by the body.
1753. A point on the x-axis performs harmonic oscillations
about the coordinate origin; its velocity is given by the fornlula
v = V o cos (J}t,

where t is the time and [Jo, CJ) are constants.


Find the law of oscillation of a point if when t = 0 it had
an abscissa x=O. What is the nlean value of the absolute magnitude of the velocity of the point during one cycle?
1754. The velocity of motion of a point is v = ie- o. o't m/sec.
Find the path covered by the point {roIn the commencenlcnt of
motion to full stop.
1755.. A rocket rises vertically upwards. Considering that when
the rocket thrust is constant, the acceleration due to decreasing
weight of the rocket increases by the law J = a A bt (a- bt > 0),
find the velocity at any instant of time t, if the initial velocity
is zero. Find the altitude reached at tim~ t = t 1
1756*. Calculate the work that has to be done to pump the
\vater out of a vertical cylindrical barrel with base radi us Rand
altitude H.
1757. Calculate the work that has to be done in order to pump
the water out of a conical vessel with ver1ex do\vnwards, the
radius of the base of which is R and the altitude H.
1758. Calculate the work to be done in order to pump wateJ
out of a semispherical boiler of radius R =]0 m.
1759. Calculate the w.Jrk ne~dcd to pum~ oil out of a tank
through an UP4Jcr opening (lhe tank has the shape of a cylinder
with horizontal axis) if the specific weight of the oil is y, the
length of the tank H and the radius of the base R.
1760**. What work has to be done to raise a body of massm
from the earth's surface (radius R) to an altitude h? What is
the work if the body is removed to infinity?
..

Sec. 121

Appl"int! Definite lntellrals to Solution of Ph,,-;lcal Problems

171

1761**. T\\'o electric charges eo = 100 CaSEand e1 =200 CGSB


lie on the x-axis at points X o = and x, -= J em, respectively.
What work will be done if the second charge is moved to point
XI = 10 em?
1762**. A cylinder with a movable piston of diameter D=20 em
and length l = 80 em is filled with steam at a pressure
l
p = 10 kgf em What work must be done to halve the volume of
the s1eam with temperature kept constant (isotherlnic process)?
1763**. De~ermine the work perforrned in the adiabatic expansion of air (having ini1 ial volume Vo = 1 rna and pressure
P. -= I I<gfjcm l ) to volume VI = 10 In'?
1764**. A vertical shaft of weight P and
i
radius a rests on a bearing AB (Fig. 62).
The frictional force between a small part a
of the base of the shaft and the surface of
the su pport in contact with it is F = Jlpa,
\\'here P = canst is the pressure of the shaft
on the surface of the su pport referred to
unit area of the support, \vhile Jl is the coefficient of friction. Find the work done by the
frictional force during one revolution of the
shaft.
1765**. Calculate the kinetic energy of a
Fig. 62
disk of Inass M and radius R rotating with
angular velocity (1) about an axis that passes through its centre
perpendicu lar to its plane.
1766. Calculate tile kinetic energy of a ri~ht circular cone of
mass M rotating \\lith angular velocity (1) about its axis, if the
rad j us of the base of the cone is R and the aItitude is H.
1767*. What work has to be don~ to stop an iron sphere of
radius R = 2 mc~r~s rotating \vith angular velocity 0) = 1,000 rplD
about its dianleter? (Specific \veight of iron, y = 7.8 g,tcnl J . )
1768. A vertical trian~lc wIth base b and altitude h is submerged vertex dJwn\\'ards in wa1er sa that its base is on the
surface of the water. Find the preSSUie of the \vater.
1769. A vertical dam has the sh lP~ of a trapezoid. Calculate
the water pressure on the dam if w~ kn)w that the upper ba~e
a=70 In. the lower bU5e b=:5Q m, and the height h=~O ffi.
1770. Find the pressure of a liquid, whose sp~cific weight is V,
on a vertical ellipse (wl1h axes 2a and 2b) \vhose centre is subnlerged in the liquid to a distance h, \vhile the lTIaJur axis 2a
of the ellipse is parallel to the level of the liquid (h;::: b,.
1771. Fl nd the water pressure on a vrrllcal circular cone
with radius of base R and altitude H subln~rged in \va~el verlex
downwards so that its base is on the surface of the walcr.

,P

178

Definite 1ntegrals

[Ch. 5

Miscellaneous Problems
1772. Find the mass of a rod of length 1 = 100 cm if the linear
density of the rod at a distance x cm from one of its ends is
6=2+0.001

Xl

gjcm.

1773. According to empirical data the specific thermal capacity


of water at a temperature to C (O:s;; t :s;; 100) is

c= 0.9983-5. 184 x 10- 5 t + 6.912 X 10- 7 t 2


What quantity of heat has to be expended to heat I g of water
from 0 C to 100 C?
1774. The wind exerts a uniform pressure p gjcm l on a door
of width b cm and height h cm. Find the moment of the pressure
of the wind striving to turn the door on its hinges.
1775. What is the force of attraction of a material rod of
length 1 and mass M on a material point of lTIaSS m lying on
a straight line with the rod at a distance a from one of its ends?
1776**. In the case of steady-state laminar now of a liquid
through a pipe of circular cross-section of radius a, the velocity
of flow v at a point distant r from the axis of the pipe is given
by the formula
P (2
2)
t1 = 4fll a - r ,
where p is the pressure difference at the ends of the pipe, f.1 is
the coefficient of viscosity, and 1 is the length of the pipe.
Determine the discharge of Liquid Q (that is, the quantity of
liquid flow.ing through a cross-section of the pipe in unit tinle).
1777*. The conditions are the same as in Problem 1776, but
the pipe has a rectangular cross-section, and the base a is great
compared with the altitude 2b. Here the rate of flow t1 at a point
M (x,Y) is defined by the formula

v = 2~l [b 2-(b- y)2J,


Determine the discharge of liquid Q.
1778**. In studies of the dynamic qualities of an automobile,
use is frequently made of special types of diagrams: the velocities v are laid off on the x-axis, and the reciprocals of corresponding accelerations a, on the y-axis. Show that the area S bounded
by an arc of this graph, by two ordinates V=OI and V=OI' and
by the x-axis is numerically equal to the time needed to increase
the velocity of motion of the automobile from vJ to VI (acceleration time).

Sec. 12) Applying Definite Integrals to Solution

of Physical Problems

179

1779. A horizontal beam of length 1 is in equilibrium due to


a downward vertical load uniformly distributed over the length
of the beam, and of support reactions A and B (A =B = ~ ) t
directed vertically upwards. Find the bending moment Mx in
a cross-section x, that is, the moment about the point P with
abscissa x of all forces acting on the portion of the beam AP.
t 780. A horizontal beam of length I is in equilibrium due to
support reactions A and B and a load distributed along the
length of the beam with intensity q=kx, where x is the distance
from the left support and k is a constant factor. Find the bending moment M x in cross-section x.
Note. The intensi ty of load distribution is the load (force) referred to
unit length.

1781*. Find the quantity of heat released by an alternating


si nusoidal current

1=loSin(~t-q
during a cycle T in a conductor with resistance R.

Ch'lpter VI

FUNCTIONS OF SEVERAL VARIABLES

Sec. 1. Fa1ic Notions


1. The concept of a function of stveral variables. Functional notation.

A variable quantity

1 is called a single-valued function of two variables x,


y, if to ec)ch set of their values (x, y) in a givpn ron~t' there corre-;ponds a
unique vatul' of z The variables x and yare called arguments or independent
variubles. The functional relation is denoted by

2=f(x, y).

Similarly, we define functions of three or more arguments.


fxample 1. Express the volume of a cone V as a function of its gcneratrix x ann of its base radIus y
. Solution. Frorn geometry we l<now that the volume of a cone is

where h is the altitude of the cone. But h=

Vx 2-

y2. Hence,

This is the desired functional relation.


The value oi the function z=t(x.y) at a
pOI nf P (a.b). that is, when x = a and y =: b,
is drnoted by I (alb) or f (P) Generall y speakin~. the ~~eornetnc rerresntation of a runct ion like z -=-! (x,y) in a r'ctangular coordi
nate system X, Y. Z is a surface (Fig. 63).

Example 2. Find I (2, -3) and t ( I,


x2 + ,)2

f (X,y) = -,-,-

Fig. 63

~xy

Solut;on. Substitutinq r= 2 and g= -3, we find


:l2

Il2,

+ (_3)2

-J)= :l.2.(-3)

13

-12

~) if

Sec.

Basic Notions

I}

Putting x= 1 and replacing y by

Y)
f ( I, x =
thaI is, f ( 1.

1Lx)\ = f (x,

!L.,
x

181

we will have
y

)1

1+ ( X
x 2 y2
( 1I)' = '2xy ,
21 x

y).

2. Domain of definition of a function. By the domain of definition of a


function 'I == f (x. y) WP undcr~tand a set of point~ (x, y) in an xy-plane in
which the given function is d~fin~d (that is to say, in w:,ich It takes on defInite real value~) I n the slnlplest cases, the rlotnain of definition of a function i~ a finite or infinite part of the xy-plane bounded by one or several
Curves (the boundan, of the domain).
Slnlll(lrly. for a funrtion of three variables u=f (x, y, z) the domain of
definit ion of the functlon IS a vol ulne in .\yz-space.
Example 3. Find the domain of definition o[ the function
1

z= Y4_.\2 __ 2 .
y
Solution. The run~tion haC\ real values if 4_X 2 _
The I at ter I nrq ua I it Y is sat i ~fi('rl hy thr coord inates of
circle of radil1\ 2 with centre ~t the coordinate orll!ln.
nition 01 the [unction is the InterIor of the circle (FIg

y2 > 0 or x 2 + y2 < 4.
poi nts I yi ng inside a
The domain of defi64).

FIg 65

FIg. 64

Example 4. Find the domain of definition of the function

z = arc sin 2'

+ V--).y

Solution. The first tenn of the funct ion


-2,s=;x~2.

when {

~~

defined for - I E:;

IS

The second term has rt'al values if

g. or

when {

~:z

g.. The dom a

In

xy~O,

of

i ~ 1 or

Le., In two C35es:

definition of the entire

function is shown in fig. 65 and includes the boundaries of the domain.

F_u_n_c_ti_on_s_o~f_S_e_ve_r_al_Va_r_ia_b_le_s

1_8_2

,[Ch. 6

30 Level lines and level surfaces of a function. The level line of a function z = f (x, y) is a line f (x, y) -== C (in an xy-plane) at the points of wh ich
the function takes on one and the same value z=C (usually labelled in
drawings).
The level surface of a function of three arguments u = f (x, y, z) is a surlace f (x, y, z) = C, at the points of which the function takes on a constant
value u=C.
Example 5. Construct the level Ii nes of
the function z=x2 y.
Solution. The equation of the level lines
c
has the form x 2 y == C or y =.: I .
x

Putting C=O, I, 2, ... , we get a fanlily


of level lines (Fig. 66).

1782. Express the volume V of a


regular tetragonal pyramid as a function of its altitude x and lateral edge y.
1783. Express the lateral surface S
of a regular hexagonal truncated pyramid as a function of the sides x and y
of the bases and the altitude z.

Fig. 66

1784. Find 1(1/2,3), 1(1, -1), if


f(x, y)===xy+-=-.
y

1785

Find
X2 _

f (Y,x), f (-x,

-y),

f(

f, ~ )

1
I

f (x,

y) ,

if

/(x, Y)=-2-.
xy
1786. Find the values assumed by the function

f (x,

+ x-y

y) = 1

at points of the parabola y = x , and construct the graph of the


function
F (x) = f (x, x 2 ).
2

1787. Find the value of the function


xC + 2X 2y2+ y4

Z==--~2

l-x

at points of the circle x 2 y2


1788*. Determine !(x), if

_yl

f(~)= ~(y>O).
1789*. Find

f (x,

y) if

f(x+y, x--y)=xy+yl.

Sec. 1]

Basic Notions

183

1790*. Let z = VY + f (Vi-i). Determine ihe functions


z if z==x when y= i.

f and

z = xf ( ~ ). Determine the functions f and z if

1791". Let

Yi +yl when x= i.
1792. Find and sketch the domains of definition of the following functions:

Z==

a) z=Yi-X 2 _ y2;
b) z = i-t_(X_y)2;

i) z=Yysinx;
j) z = In (x 2 + y);

c) z=ln(x+y);

k)

z = arc tang -+1


2 2 ,
xy

I)

1
Z=-_
x +!J

d) z=x-f-arccosy;
e) z=Yi-x 2 -!- Yi_ y 2;
f) z=arcsin lL
;
x
g)

z == y7=4 + Y4=iI;

h) z==Y(xl+y2-a2)(2a2_x2_yl)

(a> 0);

x-y

2 ,

m)

I
Z=V y-y
.;
x

n)

Z=x=r+y ;

0) zY sin (x 2 -+_ y2).

1793. Find the domains of the following functions of three


arguments:
a) u = ~rx + yy + V"z;
c) u = arc sin x + arc sin y + arc sin z;
b) u = In (xyz):
d) II = Y l-x 2 _yl_ZI.
1794. Construct the level lines of the given functions and determine the character of the surfaces depicted by these functions:
a) z==x+y;

d) z=Yxy;

g) z= ~ ;
x

11)

c)

Z=X

2
_

y 2;

f) z=

I-lxI-luI;

z=

i) z=

h;
2X
x

+ y I.

1795. Find the level lines of the following functions:


a) z=ln(x2 +y);
d) z=f(y-ax);
b) z=arcsinxu;
e)
c) z = f <YXl + y2);
x

z=f(JL).

1796. Find the level surfaces of the functions of three independent variables:
a) u=x+y-t-z;
b) u = Xl yl + Zl;

c) u=xl-t

gl_ZI.

Funct ions of Several Variables

184

(Ch.6

Sec. 2. Continuity
1. The limit of a function. A number A is called the limit of a function
2=1 (x, y) as the point P' (x. y) approaches the point P (a, h). if for any
B > 0 there is a tJ > 0 such that when 0 < Q < tJ, wherE:' Q -== Ylx-a)1
(y-b)'

is the distance between P and p', we have the inequality

I!(x, y)-AI<e.

Jn th is case we write

lim

f (x,

y) = A.

x-+a

y-+b

2. Continuity and points of discontinuity. A function


continuous at a pOint P (a, b) if

= f (x,

y) is called

linl f (x, y) = f (a, b).


x-.Q
y-+b
ran~e

A function that is continllous at all points of a given

is called

continllO:.lS over this range

A function f (x, y) may cease to be continuous either at separate points


(isolated pOlnt of discontinuity) or at points that form one or several lines
(lines of dlscontinuity) or (at times) more complex geometric objects.
Example 1. find the discontinuities of the function
xy+ 1

z=A 2 _y
Solution. Tile function will be m~aningless if the denomin3tor becomes
zero. But )(2_ y -==O or y=x 2 i~ the equation of a parabola. Hence, the given
function has [or its discontinuity the parabola y==.x 2

1797*. Find the following limits of functions:


. sit] xy
e) Ii n
a) Ii itl (x~ T yl) sin! C) I (Ln _ _ ;
xy ,

x-+o

"-~2

IJ -+ J

, x+y.
b) ILn
--z-+2 t
X ...

oo.x

l/

00

1/-+0

d) Ii.n (1

_X_ ;

x-~ox+y

X-+O

+ 1!.. ) x;

X ~o..\.
IJ ..... 0

XJ

X-+OO

X 2 _!J2

f) lIn ~+2-

y-.k

1798. Test the fotlowi ng III nction for conti nuity:

+ y2 ~ It
when x"l. +y"l. > 1.

f(x, y)= { Vl-x"l.-y2 when x

1799. Find points of discontinuity of the functions:


a) z = tn Vx~
b)

+ yl,.

c) Z --

d)

= l,x,-y
- - )2

l - x2 _y2

z =cos -.!-
>..u

Sec.

31

Partial DeritJatives

185

1800. Show that the function


2xy
h
I
I
0
z= { Z--+
x y 2 W en x + y =F ,
o when x=y=O

is continuous with respect to each of the variables x and y separately, but is not continuous at the point (0, 0) with respect to
these variables together.

Sec. 3. Partial Derivatives


1. DeOnltion of a partial derivative. If z =f (x, y), then assunling, for
example, y constant, we get the derivative
OZ _ 10
~

uX

1m

f (x + A",

~t'~O

y) A

f (x, y) - f'% (X, Y),


-

uX

which is called the part ial derivat ive of the function z with respect to thevarIable x. I n similar fac;;hion we define and denote the partial derivative of
the function z w!th respect to the variable y It is obvious that to find partial
derivativ~~, one can use the ordinary formulas of differentiation.
Example 1. Find the partial derivatives of the function

z=lntan~.
y

Solution. Regarding y as constant, we get


iJz

ax = t---X
an - cos

-2X

Ii = ---:2X.
YSlng

Similarly, holding x constant, we will have


iJz
Y

1
tan y

1 (
x)
2x
-2 =. 2x.
cos! Y
yl sin y
u

(11 = - - x - _ - x

Example 2. Find the partial derivatives of the follOWing fUJlction of three


arguments:
1
U =X y 2 z + 2x-3y + z + 5.
Solution.

ax

iJu =.3. X I y 2 z +2 ,
iJu
iJy=2x 'y z-3,

au
-=x y2+1.
iJz

'

2. Euler's theorem. A function f (x, y) Is called a homoleneous function of


degree n if for every real factor k we have the equality

f lkx,

ky) ==: kill (x. y)

Functions of Several Variables

186

(Ch. 6

A rational integral function will be homogeneous if all its terms are of one
and the same degree.
The following relationship holds for a homogeneous differentiable function
.of degree n (Euler's theorem):
xf~ (x, y) + Yf~ (x, y) = nf (x, y).

Find the partial derivatives of the following functions:


1801. z=x'+y'-3axy.
1808. z=xy
x-y

1802. Z=x+y
1803

1809. z=e

= 1LX

Z=

.!L
%

1 8 1 0 . .. /
Z=

1804. z =: VX 2 _ y l.
1805.

sin

1811.

Yx 2x+y2

arc SI n JI

z = In sin x+a
yy

X _

x2

2
y2

1812. u=(xy)z.
1813. u=zxy.

1806. z==ln{x+Vr+y').
1807. 2=arctan.[.
x
"

1814. Find f1&(2, 1) and f y (2, 1) if f(x, y)=

181 5. Fin d f~ ( 1, 2, 0),

';J (1,

f:

x
Vr---xY+ y .

2, 0),
(1, 2, 0) if
f(x, y, z)=ln(xy+z).

Verify Euler's theorem on homogeneous functions in Examples 1816 to 1819:


x+y
1816. f (x, y) = Ax 2 + 2Bxy-Cy 2. 1818. f (x, y) = V"j""'f'"';j

x!+y

1817.

Z=

x2+ y 2.

1820. Find

~(~~,

1819. f(x,y)=lnJ!..
x
where r=VxZ+yZ+zl.

'x g;ax
/ar +

1821. Calculate ::

,if x=rcos<p and y=rsin<p.

acp

az
1822. S how t h at x

ax

dz

Y ay=2, If z= In (xl+xy+yl).

OZ

OZ

1823. Sh OVl t h at x(jX+U ay=xy+z,

h
1824. Sh ow tat

If

JL

z=xy+xe%

au
au au 0, If u=(x-y)(y-z)(z-x).
ox+ay+az:Z:

1825. Show that ~+aau+~u=lt if u=x+x- Y


ox

uZ

J826. Find z = z (x, y), it ~


= !+x I .
uy X y

y-z

Sec. 4]

Total DiUerential of a Function

187

1827. Find z==z(x, y) knowing that

x 2 y2
~=-x

02

uX

and z(x,y)=siny when x=l.

1828. Through the point M(1,2,6) of a surface z=2x l +y.


are drawn planes parallel to the coordinate surfaces XOZ and
YOlo Determine the angles formed with the coordinate axes by
the tangent Ii nes (to the resulting cross-sections) drawn at their

common point M.
1829. The area of a trapezoid with bases a and b and altitude h is equal to S=I/ 2 (a-l-b)h. Find

:~, :~, :~

and, using

the drawing, determine their geonletrical meaning.


1830. Show that the function
I
2xy
. f 2 , 2 =1= 0
J ~+2 ' 1 X -1- Y
,
f (x, y) --=--' x y
,
0 , if X=IJ==O
~,

has partial derivatives f~ (x, y) and f~ (x, y) at the point (0, 0).
although it is discontinuous at this point. Construct the geometric image of this function near the point (0,0).
Sec. 4. Total Differential of a Function
to. Total increment of a function. The total incretnent of a function
z = f (x, y) is the difference
~z-= ~f (x, Y)-=f (x-t- Ax, y+ ~y)-f (x, y).
2. The total dift'erential of a function. The total (or exact) differential of
a function z==f (x, y) is the principal part of the total increlnent ~z, \vhich
is linear with respect to the increments in the argulnents f1x and Ay.
The difference between the total increnlent and the total differential of
the function is an infinitesimal of higher order compared with Q= VAx 2 + li y 2.
A function definitely has a total differential if its partial derivatives are
continuous. If a function has a total differential, then it is called differentiable. The differentials of independent variables coincide with their incrernents, that is, dx= ~x and dy= Ay. The total differential of the function
z = f (x, y) is computed by the fonnula

oz

dz

dz = 0)(
- dx -t- oy"
- d'l.
Similarly, the total differential of a function of three arguluents u =/ t x, y, z)
is computed fronl the formula

du=

ou dx au
ou
ax + ay dy+ iJz dz.

Example 1. For the function

f (x,

y)=x l t-xy_ y 2

find the total increlnent and the total differential.

(Ch. ,

Functions of Several Variables

J88

Sol ution.

~f {x, y)

f (J( + f!x,

=.: {( ~

y + ~y) = (x + AX)I + (x + Ax) (y + ~y) - (y + Ay)2;


+ l\X)2 + (x + Ax) (y + Ay) - (y + Ay)21- (x 2 + xy- y2) =

==- 2x.Ax + A.\2 +x Ay + y. Ax + ~x.Ay-2y. Ay-fl y 3 =

= [(2x+ y) Ax + (x-2y) Ay) + (Ax! + Ax Ay- ~~l').

Here, the expression df=(2x+y)Ax+(x-2y)Ay is the total differential of


the function. while (Ax 2 + tuAy-Ay 2) is an Infinitesimal of higher order
compared with V ~.t2 + Ay2.
Example 2. Find the total differential of the function

z=

az

dz

az

ax .r l x +

Solution.

Yx

;
y2

+y2.

a-=
. 1--'::'
Y r.\.2 + yl
xdr+ydy
.. r - -
r l! -t- yl

..r - - dx+
r AI + y!

11"-'-dy
2
r

.\

+ y2

3. Applying the total differential of a function to approximate calculations.


suffirifn~ly small I Ax I and I Ay I and, hence. for ~ufficiently small
Q= Y A\I+ Ayl, we have for a differentiable function z=flx. y) the approximate equality Az:::::: dz or
For

Az ~

az Ay.
axaz Ax+ay

Example 3. The altitude of a' cone is H = 30 em, the radius of thE' base
R = 10 cm. How will the volume of the cone change. if we increase H by
3 mm and ditninish R by 1 mm?
Solutlol\. The volume of the cone is V =
nR 2H. The change in volume
we replace approximately by the differential

AV::.;:dV=3' n(2RH dR+R 2 dH)=

= 3"1 n (-210300.1 + 1000.3) =

-IOn =:::-31.4 em'.

Example 4. Compute 1.02'01 approximately.


Solution. We consider the function z =-= xY . The desired number may be
considered the increased value of this function when x= 1, y=3, Ax=0.02,
dy=O.01. The initial value of the function z = 1'= 1,
f!z::::::: dz= yxY - 1 Ax+xY In x Ay= 3.1.0.02+ 11n 10.01 =0.06.
Hence, 1.02'01::::::: 1 + 0.06= 1.06.

1831. For the function f (x, y) = x 2 y find the total increment


and the total diflerential at the point (I, 2); compare them if
a)

~x=l, ~y=-=2;

b)

~x=O.I,

~y=O.2.

1832. Show that for the functions u and v of several (for


example, two) variables the ordinary rules of differentiation holdJ
a) d (u

+ u) = du + dv;

c) d ( ~ )

b) d (uu) = u dv

= v dU;u dv

+ v du;

Sec. 4]

189

Total Differential of a FunctIon

Find the total differentials of the following functions:


1833. z==x'+y'-3xy.
1841. z=lntan1!....
1834. z=x 2y'.
x
1842. Find

x 2 _l/!

1835. Z= 1+"1
.\

df (1, 1), if
x

f(x.y)=z.

1836. z=sin x+cos y.


1837 . z=yx.
y
1838. z == In (x 2 -~- yl).

1843. u = xyz .
_
1844. U = Vx 2 -t- y2 + zj.

1839.

f(x,y)=ln(l+i).

1840.

1=

arc tan yx

+aretan~.
Y

-t..

1845.

u=(xy+i-r

1846.

u =

arc tan x~

1847. Find df(3, 4,5) if


2
j(x,y,z)= .. r_ _2.

+y
the other b = 24 cm.
r

AI

1848. One side of a rectangle is a = 10 em,


How will a diagonal 1 of the rectangle change if the side a is
increased by 4 rum and b is shortened by 1 mIn? Approximate

the change and compare it with the exact value.


1849. A closed box with outer dimensions 10 em. 8 em,
and 6 em is made of 2-mm-thiek plywood. Approximate the
volume of nlaterial used in making the box.
1850*. The central angle of a circular se~tor is 80 it is desired
to reduce it by 1. By how much should the radius of the sector
be increased so that the area will remain unchanged, if theoriginal leng:h of t he radius is 20 cm?
1851. Approximate:
0

a) (1.02)1. (O.97)2~ b) V(4.05)2 + (2.93)1;


c) sin 32cos 59 (when converting degrees into radius and
calculating sin 60 take three significant figures; round otT the
last digit).
0

1852. Show that the relative error of a product is approximately equal to the sum of the relative errors of the factors.
1853. Measurements of a triangle ABC yielded the following
da1a:
side a = lOOm2m.
side
b = 200m3 m,
angle
C :== 60 10. To what degree of accuracy can we conlpute the
side c?

1854. The oscillation period T of a pendulum is computed


from the fOflnula

T= 2nyr _

I
g ,

Funct ions of Several Variables

190

[Ch. 6

where l is the length of the pendulum and g is the acceleration


of gravity. Find the error, when determining T, obtained as a
result of small errors ~l = a and ~g = p in measuring land g.
1855. The distance between the points Po (x ot Yo' and P (x, y)
is equal to Q, while the angle formed by the vector PoP with
the x-axis is a. By how much will the angle a change if the
point P(P o is fixed) moves to PI (x+-dx, y+dy)?
Sec. 5. Differentiation of Composite Functions
to. The case of one independent variable. If z = f (x, y) is a differentiab Ie
function of the arguments x and y, which in turn are differentiable function~
of an independent variable t,
x = cp (t), Y = 'I> (i),
then the derivative of the composite function z=f (cp (i), lJ' (t)] may be
puted from the form ula
dz
dt

= ~ dx + az dy

ax dt ay d t

C0I11-

(I)

In particular, if t coincides with one of the arguments, for instance x,


then the "total" derivative of the function z with respect to x will be:
dz _~ + QZ eJJ!

dx- ax

ay dx

(2)

Example I. Find ::. if


z=eIX + 2Y, where x=cos t, y=t 2

Solution. From formula (I) we have:


dz
~
IcOSt+2t 3
dt =e,x+2Y3 (-sin t) eIX + 2Y 22t =e'X+2Y (4t-3 sin t) =e
(4t -:3')int).

Example 2. Find the partial derivative iJiJZ and the total derivative dz Ii
x
dx'
z=e"Y, where g=cp(x).

Solution. ~=yeXY.
From formula (2) we obtain

dz

dx = yexy

+ xexy cp' (x).

2. The case of several Independent variables. If z is a composite function of


several independent vari~bles, for insta~ce, z = f (x,y), where x= cp (u,v),
Y=1I' (u, tI) (u and v are Independent variables). then the partial derivatives z
with respect to u and v are expressed as
iJz = ~ ~ + ~ iJy
iJu ax au ay iJu

(3)

Sec. 5]

DiOerentiation of Composite Functions

191

and
az

az ax

az ay

au = ax au + au au .

(4)

In all the cases consi dered the following formul a holds:


az
az
dz= axdx+aydY

(tile invariance property of a total dzgerential).


Example 3. Find : : and

z = t (x,

:~. if

u
x=uv, y=--V.

where

y),

Solution. Applying fonnulas (3) and (4), we get:


az,
,
1
== f x (x, y). u
fy (x, y)

au

and

oz
_ ' (x,
av -fx

F'

y) u-, y (x, y) fj2 .

Example 4. Show that the function z == cp (Xl

az

+yZ)

satisfies the equation

az

yax -x ay=O.
Solution. The function cp depends on x and y via the intennediate argutHent x2 + y2 = t, therefore,

az dz at
ax=dl
ax=CP , (x I + y 2) 2x

and

~= ~ aat =q>' (xl y2) 2y.


uy dt y
Substituting the partial derivatives into the left-hand sid{) of th{) equation, we get
OZ
az
Y -x ay = ycp'(xl + y2) 2x-xcp' (Xl y!) 2y = 2xyq>' (Xl + y2)-2xy q>'(x2 y2)ii5iO,

ax

that is, the function z satisfies the given equation.

1856. Find :: if
z=~,
where
y

1857. Find

x=ef , y=lnt.

:~ if

u = In sin

..(y, where x = 3t

l
,

Y=

vt'+l.

1858. Find ~: if

u=xyz, where x=tl+l. y=lnt, z==tant.

Functions

192

(Ch.6

:~ if

1859. Find

U=

of Several Variables

X 2+yl

1860. Find :

,where x=R cost, y=R sin t, %=H.


if

Z = ltv,

where u == sin x, v = cos x.

1861. Find :; and :; if

z=arc tan!.
and y=x".
x
1862. Find :; and

~ if

z =xY , where y = cp (x).


1863. Find :; and :; if
z=f(u, v), where u=xl_y', v=e xy

1864. Find ~ and ~ if

z = arc tan...!.,
y where
1865. Find :: and
Z

X=

u sin 0, Y= u cos v.

~ if

= f (u), where

u = xy

+ yx

1866. Show that if

then

u = cD (Xl + y' + 2 1 ), where x = R cos q> cos '1'.


y = R cos cp sin '1', z = R sin cp,
~-o
oq>-

au
and 0lJ'=0.

1867. Find ~ if
U = f (x, y, z),
1868. Show that if

where

y = cp (x), z = 1J' (x, U).

z=f (x+ay),
where f is a differentiable function, then

oz

iJy =a

az

ax

Sec. 61

Derivative in a Given Direction

193

1869. Show that the function


w=f(u, v),

where u=x+at, v=y+bt satisfy the equation

aw _ aW+ b ow
-a ox
oy

iJt

1870. Show tl-)at the function

z = y cp (X 2_ y 2)
fles th e equa tIon x1 a;+-y
oz I iJy=!j2
az z
sa t IS
1871. Show that the function

z = xy + xcp (
f
th e equa tIon x-a
az -I-Y~=xy
az
sa t ISles
x
uy
1872. Show that the function

z = eY cr
satisfies the equation
1873. The side of
of 5 mjsec, the other
is the rate of change

(X 1_ y 2)

~)

+ z.

(ye::' )

~+xy~=xyz.

rectangle x -:~ 20 m increases at the rate


side y == 30 m decreases at 4 m/sec. What
of the perilneter and the area of the rect-

angle?
1874. The equations of Illotion of a material point are
x = t, y

~= t

z:= t l

What is the rate of recession of this point from the coordinate


origin?
1875. T\vo boats start out from A at one time; one moves
northwards, the other in a northeasterly direction. Their velocities are respectively 20 kmfhr and 40 knljhr. At what rate does
the distance between them increase?
Sec. 6. Derivative in a Given Direction and the Gradient of a Functioll
10 The derivative of a function in a given direction. The derivative of a
function z=f(x, y) ift a given direction I=PP, is

az =
iJl
7_ 1900

lhn

f (PI )-f (P)

P1P ~ 0

PIP

194

Funct ions of Several Variables

[eh. 6

where f (P) and f (PI) are values of the function at the points P and PI
If the function z is differentiable, then the following formula holds:
iJz iJz
iJz .
m=axcosa +aySlna,

(1)

where a is the angle formed by the vector I with the x-axis (Fig. 67).

x
Fig. 67

In similar fashion we define the derivative in a given direction I for a


function of three argunlents u = f (x, y, z). In this case

all
at = ax cos u

du

+au
au cos ~ + au
az cos y,

(2)

where a, ~, yare the angles between the direction I and the corresponding
coordinate axes. The directional derivative characterises the rate of change
of the function in the given direction.
Example 1. Find the derivative of the function z=2x 2 -3 y 2 at the point
P (1, 0) in a direction that makes a 1200 angle with the x-axis.
Solution. Find the partial derivatives of the given function and their
values at th~ point P:

axaz = 4x; (az)


Ox p = 4;

oz
= -6y.' a(~)
iJy
y p

=0.

Here,
1

cosu=cos 1200 = - 2
sin a = sin 1200 = -2-3
Applying formula (1), we get

1)

OZ
( - - +0. lIr3"
-=4
f
2
a1
2
-2-= -

The minus sign indicates that the function diminishes at the given point and
in the given direction.
2. The gradient of a function. The gradient of a function z == f (x, y) IS
.a vector whose projections on the coordinate axes are the corresponding par-

Sec. 6]

Derit.'ative in a Given Direction

195

tial derivatives of the given function:


02

02

grad z == ox I + oyJ

(3)

The derivative of the given function in the direction I is connected with


the gradient of the function by the following formula:

02 = pro)., gra d z.
at
That is, the derivative in a given direction is equal to the projection of the
gradient of the function on the direction of differentiation.
The gradient of a function at each point is directed along the normal to
the corresponding level line of the function. The direction of the gradient of
the function at a given point is the direction of the maximum rate of increase
of the function at this point, tlUtt is, when l=grad z the derivative
on its greatest value, equal to

~~

takes

In sitnilar fashion we define the gradient of a function of three variables,

u -= f (x, y, z):

au

iJu

iJu

grad u=iJ-x 1-+- uy


~ J + a- k.
Z

(4)

The gradient of a function of three variables at each point is directed along


the nonnal to the level ~urface passing through this point.
Example 2. Find and construct the gradient of the function z.=;x 2 y at
the point P (I, I).

y
2
1

-~
- - --~
PI
I

3 X

Fig. 68
Solution. Compute the partial derivatives and their values at the point P.

~=2xY; (~)p=2;

(~)p=l.
Hence, grad
7*

z=21+J

(Fig. 68).

Funct ions of Several Variables

196

[Ch. 6

1876. Find the derivative of the function z = x!-xy-2y J


2) in the direction that produces an angle
'()f 60 with the x-ax is.
1877. Find the derivative of the function z=x 8 -2x 1 y-+ xy2+ 1
.at the point M (1, 2) in the direction from this point to the
point N (4, 6).
1878. Find the derivative of the function z == InVx! -~ y! at
the point P (1, 1) in the direction of the bisector of the first

at the point P (1,

quadrantal angle.
1879. Find the derivative of the function u=x!-3yz+5 at
the point i'A (1, 2, -1) in the direction that iorms identical
angles with all the coordinate axes.
880. Find the derivative of the function u = xy + yz ,- zx at
the point M (2, 1, 3) in the direction from this point to the
point N (5, 5, 15).
t881. Find the derivative of the function u = In (eX + eY -t-~ e1)
at the origin in the direction which forms with the coordinate
axes x, y, z the angles a, ~, V, respectively.
1882. The point at which the derivative of a function in any
direction is zero is called the stationary point of this function.
Find the stationary points of the following functions:
a) z==x!+xy+y2-4x-2y;
b) z === Xl yl -3xy;
c) u == 2y 2 -t- z!-xy-yz 2x.

1883. Show that the derivative of the function z=L taken


x
at any point of the ellipse 2x2 -1- y2 = C! along the normal to the

ellipse is equal to zero.


1884. Find grad z at the point (2, 1) if

z = Xl + yl- 3xy.
1885. Find grad z at the point (5, 3) if
Z==

Vx

_y!.

1886. Find grad u at the point (1, 2, 3), if u =xyz.


1887. Find the tnagnitude and direction of grad u at the
point (2, -2, 1) if

1888. Find the angle between the gradients of the function


Z= In ~ at the points A (1J2, 1/4) and B (1, 1).

Sec. 7]

Higher-Order Derivatives and DiUerentialb

197

1889. Find the steepest slope of the surface


z=x! + 4y
at the poi llt (2, 1, 8).
1890. Construct a vector field of the gradient of the following
functions:
a) z=x+y:
b) z=xy;
Sec. 7. Higher-Order Derivatives and Differentials
1. Higher-order partial derivatives. The second partIal derivatives of a
function Z=== f (x, y) are the partial derivatives of its first partial derivatives.
For second derivatives we use the notations

:x (a:)

t:

::~ =

x (x, y);

a z ::::: f"Xli (x,


ayo (DZ)
ax = axay
2

y) and ~o forth.

Derivatives of order higher than second are similarly defined and denoted.
If the partial derivatives to be evaluated are continuous, then the result
of repeated dtUerentlailon is independent of the order in which the differentzatlon IS perfornled.
Example 1. FInd the second partIal derIvatives of the functIon
Z ==-

Flr~t

Solution.

arc tan !...


y .

find the first partial derIvatives:


az
1
I
y
ax=~ Y=.\2+ y 2 '

l-t-y'l

az

ay=~

+ y2

x )

!/t = -

x2+y".

Now differentiate a second time:


iJ'lz _ ~

ax

az

ay2 =

ax

(_y_
)__
+ y! -

0 (

ay -

2xy

(x 2

x!

x2

X)

+ y2

cJ2 Z
0 ( Y )
ox ay = oy x! + y2 =

+ y2)1

2xy
+ y2)1
l
1.(x + y2)_2y.y

= (xt

(xl

+ y2)1

xl_gl

+ y2)2

(x 2

We note that the so-called "mixed" partial derivative Inay be found in a


<titTerent way, namely:
0 22
cJ2Z
a(
X)
1. (x 2+ y2)-2x.x
x 2 _ y:!
2
2
ox ay ay ax =
x + yl = (Xl + y2)1
= (x + y2)2

ax -

Functions of Several Variables

198

2.

=/ (x,

[ell. 6

Higher-order differentials. The second diUerential of a function


y) is the differential of the differential (first-order) of this function:
d 2 z = d (dz)

We similarly define the differentials of a function z of order higher than


two, for instance:
and, generally,

dnz = d (dn-1z).

If z = t (x, y), where x and yare independent variables, then the second
differential of the function z is computed from the fonnula
d

a~

a~

z= ax2 dx 2+ 2 ax ay

dx dy

a~

+ ay 2 dyl.

(1)

Gener all y, the followi ng symbolic fornl ula holds true:

dnz=( dX~ +dy a~rZi


it is fornlally expanded by the binomial law.
If z = f (x, y), where the arguments x and yare functions of one or sev-

eral independent variables, then


02Z d 2
d! z~ax2
x

. y
02
+ 2 oxoydXd

az

02 Z 2 az 2
2
+ay 2 d y +axdx+ayd Y

(2)

If x and yare independent variables, then d 2 x=O, d 2 y=O, and formula (2)
becomes identical with formula (1).
Example 2. Find the total differentials of the first and second orders or
the function
z = 2x 2 -

3xy _ y2.

Solution. First method. We have


OZ
02
(jX=4x-3y, oy= -3x-2y.

Therefore,
dz
iJz
oz= ax dx+ ay dy= (4x-3y) dx-(3x+2y) dYe

Further \ve have

whence it follows that


2

a2 z

dZ==ox2dx

a z dx dY+a02
+ 2 axoy
y

Z
2

dY =4dx -6dxdy-2dy.

Second method. Differentiating we find


dz=4x dx-3 (ydx+xdy)-2y dy= (4x-3y) dx-(3x+2y) dYe

Differentiating again and remelnbering that dx and dy are not dependent on


x and y, we get
d 2z= (4dx-3dy) dX-(3dx+2dy) dy=4dx 2 -6dxdy-2dy 2.

Seco 7]

Higher-Order Derivatives and DiOerentlals

1893. Find

O~2~y if

1894. Find

iJ~2;y if
z=arctan

1895. Find

t+y
l-xy

~~ if

V x 2+y2 +zlo

r =

1896. Find all second partial derivatives of the function

u = xy + yz -+ zxo
97 F In
18.
o

OIU

ox dy
2

1898.

a
r-'1 n d iJxayZ
o

az

JOf

If

z = sin (xy)

1899. Find

f:

f:

(0, 0),

r/

(0, 0), f~'l (0, 0) if

f(x, y)=(l-+ x)m(l +y)n.


1900. Show that

02 Z

iJxiJy

z if
= iJLjiJx
iJ2

z=arc sin
1901. Show that

2
2

iJxay

02

ayox

Y-

x- y
x

if

z=xY
1902*. Show that for the function

f (x,

y)

x 2 _y.

= xy x--a-+y

199

Functions of Several Variables

200

[provided that f (0, 0) = 0] we have


f ~y (0, 0) = - 1, f~x (0, 0) =
1903. Find

[Ch. 6

+ 1.

if

OIZ
02
ox
axo!zoy , oy
2 ,

z = f (u, v),
where u =

Xl

+ y., v = xy.

1904. Find ::~ if u=f(x, y, z).


where z = cp (x, y).
.
o!z
02Z O!Z.
1905. Find
oxay , ay2 If

ax.'

z = f (u, v), where u = cp (x, Y), v = 'I' (x, y).


1906. Show that the function

u -= arc tan JLx

satisfies the Laplace equation


alll

a2u

-+--0
ax! ay
2 -

1907. Show that the function


1

u=ln, ,
where r=V(x-a)2+-(y-b)2, satisfies the Laplace equation
02 U

au
2

ox. -1- oy2 = o.

1908. Show that the function


u (x, t) = A sin (a'At + cp) sin 'Ax

satisfies the equation of oscillations of a string


02 U
iJt2 =

0 !U

ax2

1909. Show that the function


u(x, y, z, t)=

1
y_
e
(2a 1T t)3

(X-XO)2+(y- YO)2 +(% -Zo)l


4a 2 t

(where x o' Yo, zo' a are constants) satisfies the equation of heat
conduction

Sec. 7]

Higher.Order Derivatives and DiOerentials

~Ol

1910. Show that the function

+ 'i' (x + at),

u = cp (x-at)

\vhere cp and ' are arbitrary twice differentiable functions, satisfies the eq uation of asci llations of a string
2
a2 u
2 au .
at 2 =a ox l
1911. Sho\v that the function

z = xcp ( ~

)f- '!' ( ~ )

satisfies the eq uation


Xl

a2z + 2x

iJ2 z

Y iJxay

ax 2

+ Yay!
iJlz = 0
.
2

J 912. Sho\v t hat the function

u = cp (xy) + l/ xy ( ~ )

satisfies the equation


1913. Sho\v that the function z = f [x -i- cp (y)] satisfies the equation
az a2z
az a2z
ax ax ay =

1914. Fi 11 d u

--=:;

II

(x. y)

ay ax

if

a
a-a
x y =0.
211

1915. Determine the fornl of the function u=u(x, y), \\'hich


satisfies the equation
1916. Find d!z if
1917. Find d 2 u if

u = xyz.
1918. Find d1z if
z=cp(t), where t=X 2_'t-yl.

1919. Find dz and d!z if


Z

= uf)

where u = -y , v = xy.

Functions of Several Variables

202

[Ch.6

1920. Find d 2 z if
z=f(u, v), where

U=QX,

v=by.

1921. Find d1z if


Z

= f (u,

1922. Find d'z if

v), where u = xeY , v = ye".


z = eX cosy.

1923. Find the third differential of the function


z=xcosy-~y sinx.

Determine all third partial derivatives.


1924. Find df(l, 2) and d2f(1, 2) if
f(x, y)=x l +xy+y2-4Inx-IOlny.
1925. Find d 2f (0, 0,0) if
f (x, y, z) = Xl + 2y 2 -1- 3z 2 -2xy -t- 4xz + 2yz.
Sec. 8. Integration of Total Differentials
to. The condition for a total differential. For an expression P (x, y) dx +
y) dy, where the functions P (x, y) and Q (x, y) are continuous in a
simply connected region D together with their first partial derivatives, to be
(in D) the total differential of some function u (x, y), it is necessary and sufficient that

+ Q(x,

iJQ

ox

iJP
====

oy .

Example 1. Make sure that the expression


(2x -t- y) dx -f- (x + 2y) dy
is a total differentIal of some function, and find that function.
Solution. In the given case, P=2x+y, Q=x+2y. Therefore,

= I,

and, hence,
(2x+ y) dx

~=:: =

au
+ (x+ 2y) dy = du = ou
ax dx+ iJy dy,

where u is the desired function.

ou

It is given that ax == 2x + y; therefore,


u= ~ (2x+y) dx=xl+xy+q> (y).
Buf on the other hand

and

Finally we have

iJu

oy=x+Q>' (y)=x+2y, whence <p' (y)=2y, cp(y)=y2+C


u == x 2+xy

+ y2 + c.

(2x+ y) dx+(x+ 2y) dy=d (xl+xy+ y2+C).

Sec. 8J

'ntegration of Total DIfferentials

203

2. The case of three variables. Silnilarly, the expression


P (x, y, z) dx+ Q (x, y, z) dy+ R (x, y, z) dz,
where P (x, y, z), Q(x, y, z), R (x, y, z) are, together with their first p~rtial
derivatives, continuous functions of the variables x, y and z, is the total
differential of some function u (x, y, z) if and only if the following conditions
are fu Ifi lied:

Example 2. Be sure that the ex pression


(3x 2

+ 3y-l) dx + (Z2 -t- 3x) dy + (2yz + 1) d2

is the total differential of some function, and find that function.


Solution. Here, P=3x!+3y-1, Q =z2+3x, R=2I1z+ 1. We establish
the fact that

and, hence,

(3x 2 -f-3y-l)dx+(z2+3x) dy+{2yz+ I) dz=du

au

uX

au
du
dX+ - dY+- dz,
ay
az

\vhere it is the sought-for function.


We have
au
ax =3x2 + 3/j-l,
hence,

u= ~ (3x 2 +3y-l) dx=x'+3xy-x+q> (y. z).


On the other hand,

au

a(j)

~=3x+a-=z +3x,
uy
y
au aq>
-=2yz--1-1,
az iJz

- =

\vhence dq> =Z2 and aacp =2yz+ 1. The problem reduces fo finding fhe function
y
z
of hvo variables cp (y, z) whose partial derivatives are known and the condition for total differential is fulfilled.
We Hnd CI':
q> (Y. z) = ~ zldy = yz2 + Ijl (z).

:: =2yz + Ijl' (z)=2yz+ I,


'1"(z)=I, 'I'(z)=z+C,
that is, cp (y, 2)= yz 2+ Z+C. And finally,

u=x' +3xy-x+ yz2+ z+C.

204

Functions of Several Variables

I-laving convinced yourself that the expressions given below


are total differentials of certain functions, find these functions.
1926. y dx+x dYe
1927. (cosx+3x1y)dx+(X 3 _y2)dy.
1928

(x+2y)dx+ydy
(x -1- y)2

x+2y
2x-y
dx -,----11929. Z--+
x
y2
x - y I dy.
1
x
1930. -II dx-dye
y2
II

1931.

x x-t- y
2

dx+

X Y+ y
2

dy.

1932. Determine the constants a and b in such a manner that


the expression

+ 2x Y -1- y2) dx - (x 2+ 2xy + by 2) dy


(Xl + yl)2

(ax 2

should be a total differential of some function z, and find that


function.
Convince yourself that the expressions given below are total
differentials of some functions and find these functions.
1933. (2x -t- y + z) dx + (x + 2y -r- z) dy + (x -r y -l- 2z) dz.

-r

1934. (3x
2y 2 3z) dx (4xy 2y -z) dy + (3x - y - 2) dz.
1935. (2xyz-3yz 8xy 2 2) dx +
(x 2 z- 6xyz 8x2 Y -t- 1) dy -1- (xly- 3xy 2 -l- 3) liz.
2

1936.

(~'-;2)dx f-(~-:2)dY+(~-:')dz.

1937. x dx + y dy + z dz .

+y2+ Z2

1938*. Given the projections of a force on the coordinate axes


X=-y(x+ y)2 ,

y=~
(x+ y)2 ,

where A, is a constant. What must the coefficient A be for the force


to have a potential?
1939. What condition must the function f (x, y) satisfy for the
expression

f (x,

y) (dx

+ dy)

to be a total differential?
1940. Find the function u if
du

= f (xy) (y dx -J- x dy).

Sec. 9]

DiUerentiation of / nlplicit Functions

205

Sec. 9. Differentiation of Implicit Functions


1. The case of one independent variable. If the equation f(x, y)=O, where
x and y, defines y as a
function of x, then the derivative of this irnplicitly defined function, provided
that f~ (x, y) i= 0, may be found fronl the formula

I (x, y) is a differentiable function of the variables


dy
dx = -

Higher-order derivatives are

f~ (x, y)
f~ (x, y)

(1)

found by successive differentiation of formula

(I)
2

dy
d y.
Example 1. Flnd d- and -d2 If
x
x
(x 2
th~

+ y~)' -

3 (x 2

+ y2) + 1 == O.

Solution. Denoting the left-hand side of this equation by f (x, y), we find
partial denvatlves

== 3 (x 2 + y2)2. 2x- 32x-= 6x [(x 2 + y2)2 -1],


y) -= 3 (x 2 + y2)2. 2y-3 2y == 6y [(x 2 + y2)2_1].

I~ (x, y)

1;/ (x,

\\lhenCl\ applyIng forll1ula (1), \ve get


dy
f~ (x, y)
6x [(x 2
dx == --f~ (x, y) = - 6y [(x 2

+ !/)2_1]
x
+ y2)2_1] = -y-

To find the second derivative, differentiate \vith respect to x the first derivatIve WhICh \ve have found, taking into consideration the fact that y is a functlun of x'
dy
l.y_x
y_x{_'yt)
d:!11
d (
x )
dx
\
= _Y~ + x2
dx2=dx

--y

= -

y2

y2

y3

2. The case of several independent variables. Sirndarly, if the equation


F (x, y, z) =-=0, where F (x, y, z) is a differentIable function of the variables

x, y and z, defines z as a function of the independent variables x and y and


F~ (x, !I, z) :p 0, then the partial derivatives of this irnplicitly represented
function can, generally speaking, be found fro III the fonnulas

oz
at =

F~ (x, !It z)

F ~ (x, y, z)'

az

F;/ (x,

oy =

F ~ (x, y, z)

y, z)

(2)

Here is another way of finding the derivatives of the function z: differentiating the equation F (x, y, z) =0, \ve find

aF
of
of
-dx+-dy+-dz=O.
ox
cJy
iJz
Whence it is possible to detennine dz, and, therefore,
iJz

ax

QZ

and ay

Funct ions of Several Variable')

206

Example 2. Find

:~

and

:~

[Ch. 6

if

x 2 _2 y 2

+ 3z -yz + y== o.
2

Solution. First method. Denoting the left side of this equation by F


we find the partial derivatives

(x, y, z),

F;(x, y, z)=2x, F;(x, y, z)=-4y-z+l, F;(x, y, z)==6z-y.

Applying formulas (2), we get


iJz
ox =

F~ (x, y, z)
2x
F z, ( x, y, z) = - 6z-y;

dz
ay =

F~ (x, !I, z)
p'z (x, y, z)

1-4y-z
6z-y

Second method. Dlfferentiating the given equation, \ve obtain


2x dx-4y dy+6z dz-y dz-z dy+dy=O.
Whence we detennine dz, that is, the total differential of the inlplicit function:
dz =2xdx+(1-4Y-2)dy .
y-6z
Comparing with the formula

dl=:;dX+~dY.

we see that

az
2x
az 1-4y-z
ax = y-6z ' ay = y-6z

3. A system of Implicit functions. If a systelTI of two equations


F(x, y, u, v)=O,
{ o (x, y, u, v)=o

defines u and. v as functions of the variables x and y and the Jacobian

aF of

D (F, 0)
D (u, v)

au av :1= 0
ao ao
'
au iJu

then the differentials of these functions (and hence their partial derivatives
as well) may be found from the following set of equations
dF
aF
iJF
aF
ax
dx + ay dy + au du + av dv = 0,

ao
ao
ao
{ iJO dx +-a
du-+- v dv =0.
x
y dy +-au

-a

a-

Example 3. The equations


u+v=x+y, xu+yv=l

define u and

t1

.
iJu au do
av
as functtons of x and y; find -iJ 'iJ--' ~ and x
y ux
ay .

(3)

DiOerentlation of Implicit Functions

Sec. 91

207

Solution. First method. Differentiating both equations with respect to x.


we obtain

\\'hence

Sim ilarl y

\ve

find
au

v+y

av

v+x

dy=-x-y' ay=x-y
Second nlethod. By differentiation we find two equations that connect the
differentials of all four variables:
du+dv==dx+dy,
+ v dy~.

x du +u dx+y dv
Solvin~

this system for the dIfferentials du and dv, we obtain

du= _ (u

+ y) dx-t- (v -t- y) dy ,

dv ~ (u

x-y

+ x) dx + (v +x) dy

x-y

Whence

4. Parametric representation of a function. If a function z of the variables x and y is represented paralnetricatly by the equations
x=x(u, v), y=-=-y(u, v), z=z(u, v)

and
D (x, y)
D (u, v)

then the differential of this function nlay be found froln the following system
of equations

ax
ax
dX-a-dll-f--a dv,
II

ay

ay
dy = au da + av dv,

(4)

az
az
dz -== a-u du -1- iJ-v dv.
az

Knowi ng the differential dz ~ p

az

ox=p and dy-=.q

d~

+q dy,

we find the partial derivatives

Functions of Several Variables

208

[Ch.6

Example 4. The function z of the arguments x and y is defined by the


(lquations
x=u+v, y=U 2+V 2 , z=ul+V S (u;6v).
OZ
.
az
Find ox and ay
Solution. First method. By differentiation we find three equations that
connect the differentials of all five variables:
dx=du+dv,
{ dy= 2u2du + 2v 2dv,
dz -::::: 3u du + 3v dv.
From the first two equations we determine du and dv:
du=- 2vdx-dy
2 (v-u) ,

dv==.dy-2udx.
2 (v-u)

Substituting into the third equation the val ues of du and dv just found, we
bave:
<lz = 3u 2 2v dx-dy + 3v 2 dy- 2u dx =
2 (v-u)
2 (v-u)
= 6uv (u-v) dx+3 (V 2_U 2 ) dy:=.: -3uv dx -t-~ (u + v) dYe
2(v-u)
2
Whence
oz
az
3
-3uv, ay="2 (u+V).

ax=

Second method. From the third given equation we can find


az =3u 2 au +3v 2 av;
ax
ax
ox

aZ=3u 2 au +3v2aV .
ay
ay
ay

(5)

Differentiate the first two equations first with respect to x and then \vith
res pee t to y:
l=au + av

ax ax'

au
av
{ O=2Ll+2v ox
ax'

From the first system we find


au
v
av
u
ax=v-u' ax=u-v
Fronl the second system we find
au
1
av
oy 2 (u- v)' ay

2 (v-u) .

'Substituting the expressions :: and :; into formula (5), we obtain


OZ 3 2 V
2
U
iJx= U v_u+ 3v u-=v=-3uv,
OZ _ 3

iJy- U 2(U-v)+ v '2(v-u)

3 (
2 u-i-v).

209

Dtfferentiation of Impltcit Functions

Sec. 9]

1941. Let y be a function of x defined by the equation


x2 y2
~+1J2=1.

FInd

dy d 2y
dx' dx2

day

and dxS
1942. Y is a function defined by the equation
x!-t- y 2+2axy==O (a>l).

Show that ~~ = 0 and explain the result obtained.


1943. Find ~ if y = 1 -\- yX,
d2 y .

dy

1944. Find dx and dx 2 If y = x -1- In y.

(ddx2y ) if
x 2-2xy -l- y2 + x + y- 2 === O.

(lfJ!)
dx

1945. Find

and

t=l

X=l

Taking advantage of the results obtained, sho\v approximately


the portions of the given curve in the neighbourhood of the point
x == 1.
1946. The function y is defined by the equation
In 'Vx 2 -1- y2 =-= a arc tan.11- (a=i=O).
x

dy

Find dX and

d 2y
dx 2

d dy
d y If
an d -1947 F 111
dx
dx 2

1 -+- xy-ln (e xy -t- e -xY) = o.


1948. The function z of the variables x and y is defined by
the equation
xa -1- 2y' -f-- Z3 - 3xyz-- 2y + 3 = O.

FInd

axiJz and azay

1949. Find ~ and

ax

az
ay if

x cos y -1- y cos z + z cos x = 1.

l!)SO. The function z is defined by the equation


Xl + y2_ zl-xy = o.

Find

iJz

ax

and

az
au for the systenl of values x = - 1,

y = 0, z = 1.

210

Functions of Several Variables

[Ch. 6

ax ayaz

y, z)=O. Show that a-a-a-=-I.


y z x
1953. z = cp (x y), where y is a function of x defined by the
equation 'I\J(x, y)=O. Find ::.
1954. Find dz and d2 z, if
x 2 + y 2 + Z2 == al.
1952. f(x,

1955. z is a function of the variables x and y defined by the

equation
2x 2 + 2y2 -+- Z2 -- 8xz - z -t- 8 .= 0

Find dz and d2 z for the values x = 2, y ==- 0, z.:.:= I.


1956. Find dz and d2 z, if In z=x+U+z-1. What are the
first- and second-order derivatives of the funct ion z?
1937. Let the function z be defined by the equation
x 2+ y2 -1- Z2 = cp (ax + by +- cz),
where (p is an arbitrary differentiable function and a, b, care
constants. Show that

az + (az-cx) dY
az == bx-ay.

(cy- bz) ax

1958. Show that the function z defined by the equation

F(x-az, y-bz)=O,

where F is an arbitrary differentiable function of t\VO argU111entS t


satisfies the equation

az

az
ay= 1.
az az
Show that x-ax+Yay=z.
a ax + b

X zy )

1959. F ( z'

==0.
1960. Show that the function z defined by the equation
y = xcp (z) + 'P (z) satisfies the equation
a2z(aZ)I_2~az

JX2 ay

a z +a Z(aZ)I_ O
2

axayaxay

ay2 ax

1961. The functions y and z of the independent variable x are


defined by a system of equations Xl +yl-zl =~O, Xl + 2y l + 3z1 =4.
.
dy dz d 2y d2z f
Find ax, dx' dxl' dx l or x = 1, y = 0, Z = 1.

Change of Variables

Sec. 10]

211

1962. The functions y and z of the independent variable x are


defined by the following system of equations:
xyz = a, x

+ y + z == b.

Find dy, dz, d y, d z.


1963. The functions u and v of the independent variables x and y
are defined implicitly by the system of equations
2

u=x+y,

Calculate

uv=y.

iJu a2u a2a a2 u iJu au 02U a2 u a2u


~'aY'~2'~~'~2'~'~'a~2'axaY'~2

au

for x==O, y= 1.
1964. The functions u and v of the independent variables x
and yare defined implicitly by the system of equations
u-f-v=x,

u-yv=O.

Find du, dv, d2 u, d 2 v.


1965. The functions u and

tJ of the variables x and yare


defined implicitly by the system of equations

x = rp (a, v), y =- 'P (u, v).


V

rln

au au au av

a-'
x -aY ' a-'
x aY

1966. a) Find
b) Find

:~

and

~,if X=llCOSV,

y=usinv,

~z
and adz,
if x=u+v, y=u-v,
vX
y

z=cv.

Z=llV.

c) Find dz, if x=eu+t', y=eu-t', z=uv.


1967. z = F (r, q where rand q> are functions of the variables

x and y defined by the systenl of equations


x = , cos cp, y = , sin cp.

az d az
Find ax an ay
1968. Regarding z as a function of x and y, find

~ and ~, if

x=acoscpcos,!" y=b sin cp cos, z=csin'1'.


Sec. 10. Change of Variables
When changing variables in differential expressions, the derivatives in
them should be expressed in terms of other derivatives by the rules of differentiation of a composite function.

Functions of Several Variables

212

[Ch.6

1. Change of variables in expressions containing ordinary derivatives.


Example 1. Transform the equation
2
2 d y
dy a2 _
x -d
2+2xd-+2Y-O
x
x x

x=+.

putting
Solu tion. Express the derivatives of y with respect to x in tenTIS of the
denvatives of y with respect to t. We have
dy

dy

en

dy = dt =

dx

d y
dt 2

=!!.(t!J!.)
dx ,dx

dx
dt

d(dY)
en dxdX =

= -t 2 dy

dt '

-72

2
2y
_ (2t dy + t 2 d 2 ) (_t 2 ) = 2t a dy + (-I d y
dt
dt
dt
dt 2

dt

+.

Substituting the expressions of the derivatives iust found into the given
eiua tion and replacing x by

we get

2y

Y td ) 2 1 (
Y
1 t s (2ddt+
12
dt 2 + "T _ tf).. ddt ) +a 2ft~y~ 0

or

Example 2. Transfonn the equation


2

d y (d Y )' dy
x dx 2 + .dx =0,

ax

taking y for the argument and x for the function.


Solution. Express the derivatives of Y with respect to x in terrns of the
derivatives of x with respect to yl
dy
I
{lX= dx ;

dy
2

dy d (
2
dx = dx

1)

1) dy

d (
= dy :;

dx =

d 2x
dy 2

- (:;

r:;1

d 2x
dy 2

= - (:;

Substituting these expressions of the derivatives into the given equation, we


will have

::~] +(dX)3- ~=O,


rL-(dX)3
dy

dy

dy

Sec. 10]

Change of Variables

213

or, fina 11 y,
X

(dX)2 == o.

d:!x
d y 2 - 1 + dy

Example 3. Transfonll the equation


dy

x+- y

dx= x-y'

by passing to the polar coorciinates


rcos q>,

X==

y== r sin cpo

(1)

Solution. Considering r as a function of ep, frorn formula (1) we have


dx == cos <p dr - r sin ep dep,

dy == sin ep dr -t- r cos (V d(j),

\vhence
dy

ax

.
dr
.
sin (P - -I r cos ep
sin <p dr r cos ep dq>
d(p
= cos q> dr - r sin cp d(p =
dr ._-.cos q> dq> - r sin cp

Putting into the ~iven equatIon the expressIOns lor x, y, and ~. we will have
. q> iiq>
d, + r cos q>
sIn

cos <p
or, after

dr

dcp -

.
r sin <p

r cos q> ~- r sin q>


r COS cp - r SI n q> ,

51 nlpliHca Hons,

dr
dq> =-: r.

2 Change of variables in expressions containing partial derivatives.


Exaillple 4. Take the equation of oscillations of a string
(a i= 0)

and change it to the new independent variables a and

~=x-t-at.

~,

where a=x-at,

Solution. Let us express the partial derivatives of II with respect to x and t


in tenns of the partial derivatives of Ll with respect to a and ~. Applying
the fonnulas for differentiating a conlpos1te function

au all aa

all

at = act at + a~

\Vl~

get

a~

at'

au

au aa all

a~

ax = oa ax + a~ ax'

Functions of Several Variables

214

lCh.6

a2 u
aaa~ =0.
Example 5. Transfonn the equation x 2 aaZ

== ~_-.!.-

+ y2 aaYZ = Z2, takin~ II = X,

t'

==

for the new independent variables, and w=l._J.. for the ne\v
x
Z
x
funetton.
Gz.In t eftllS 0 f th e
1 denva
. tIves az
So Iution. Lc t us express tl Ie par t la
ax an d ay
fI

partial deri,vatives

~~

and

:~.

To do this, differentiate the given relation-

ships between the old and new variables:

dII = d x,
On the other hand,

dv = dx2 _ dy , dw -_ ~2 _ dz .
x
y~
x
Z2

ow
Ow
dw=-du+-dv.

au

Therefore,

av

ow

iJw

dx

dz

-du+-dv=--au
av
x2
Zl

or

Whence

2( -1 -dw
I i)W)
z aw
--- dx+- -dy
2

dz=z

and, consequently,

Xl

au

av

y2

cJz
1 ow
1 OW)
ax = z Xi - au - Xi iJv
I (

au

Change of Variables

Sec. 10]

and

21a

oz Zl ow
ay= y2 OV

Substituting these expressions into the given equation, we get

X2Z2

(.!_ owall _.!x Ow)


ou

+Z2

Xl

or

aw=o
OU

oW =

au

Zl

1969. Transform the equation


2

x 2 d y2
dx

+ 2x ~dx -f- y = 0 '

putti ng x === et
1970. Transform the equation
2

d2 y

dy

(l-x )d>..2
- - x dx
-=0 '

putti ng x == cos t.

x
Fl~

69

Transform the following equations, taking y as the are

1971

gUlllent:
2

d y -t- 2y
a) -dx
2

(dY)1
=0
dx

'

)! = O.
dx

b) '!Jt dy3 _ 3 (d 2y2


dx dx

1972. The tangent of the angle J! formed by the tangent line


MT and the radius vector OM of the point of tangency (Fig. 69)
is expressed as follows:
,

y-X-

tan

~==--
1 lL y'

+x

216

Funct ions of Several Variables

[Ch.6

Transform this expression by passing to polar coordinates:


x=rcoscp, y=rsincp.
1973. Express, in the polar coordi nates x = r cos cp, y = r si n cp,
the formula of the curvature of the curve
y"

K=

[1

+ (y')2]S/2

J974. Transform the following equation to new independent


variables u and v:
az
az
y--x-=O
ax
ay ,

if u=x, v=x!+y~.
1975. Transform the following equation to new independent
variables u and v:
az az
x-+y--z=O
ax
ay
,
Or
t

y
tl=x, v=xo
1976. Transform the Laplace equation

a u 02 a
ox -to ay =
2

to the polar coordinates


x= r

cos cp,

y = r sin cp.

1977. Transform the equat ion


2
2 az
2 02Z
X dx z - Y ay 2 =0,
putting u~xy and v=~.
y
1978. Transform the equat ion

az

az

Y ax -Xay = (y-x) z,

by introduci ng new independent variables


u =xz + y2,

=~x +J..
y

and the new function w=lnz-(x+y)o


1979. Transform the equat ion

az
2

OX 2 -

az
2

2ox ay

az
2

+ iJy

= 0,

taking u = x + y, v = JL for the new independent variables and


z
x
w= -x for the new function.

The Tangent Plane and the Normal to a Surface

Sec. 11]

217

1980. Transform the equation


02 Z

ax 2

+ 2 ax02ay

Z _L

02Z

oy 2

putting u=x+U, v=x-y, W=xy-z, where w=w(u, v).


Sec. 11. The Tangent Plane and the Normal to a Surface
1n. The equations of a tangent plane and a normal for the case of exp 1fcIt representation of a surface. The tangent plane to a surface at a point 1\1
(point of tangency) is a plane in wh ich lie all the tangents at the point M to
various curves drawn on the surface through this point.
The normal to the surface is the perpendicular to the tangent plane at the
point of tangency
If the equation of a surface, in a rectangular coordinate system, IS gi\ten
in explicit form, Z== f (x, y), where' (x, y) is a differentiable function, then
th e eq ua t ion of the ta ngent pIa ne a t the poi nt M (x o, Yo, 2 0) 0 f the surf ace is
Z-

2 0 =-=-

f.~ (x o , Yo) (~X - xo) +,;/ (x o, Yo) (Y - Yo).

(1)

Here, 1 0 == f (x o, Yo) and X, Y, Z are the current coordinates of the point of


the tangent plane.
The equa bons of the nOrInal are of the fOrIn

X-x o

f.\ (xu, Yo)

Y -Yo

== - , - - - =

'11 (xo, Yo)

Z-zo
--I'

(2)

where 4Y, Y, Z are the current coordinates of the point of the nornlal.
Example t. Write the equations of the tangent plane and the nortnal to

x
2"
2

the sur face

Z=

Y2 at t he po i 11 t ,\1 (2, - 1, I).

Solution. Let us lind the partial derivatives of the gIven function and
their values at the point M

oz

ax=x,

OZ) At --2 ,
( ax

dz

(ayi!!)

-=-'>'/
cy
-~ ,

AI

=2

Whence, applying fornlulas (I) and (2), we will have z-1=2(x-2)+2(Y--1-I)


or 2x-j-2y-z-I=O which is the equation of the tangent plane and x 2 2=
= y ~ 1= z

II, which is the equation of the normal.

2. Equations of the tangent plane and the normal for the case of implicit representation of a surface. When the equation of a surface is represented
iIn pI icit Iy,
F (x, y, z)=o.

and F (x o, Yo, zo) =0, the corresponding equations will have the fOrIn

F~(xo' Yo' zo) (X-xo)+F;(x o, Yo,

20 )

(Y-Yo)+F~ (X o, Yo' zo) (Z-lo)=O (3)

Functions

218

of Several Variables

which is the equation of the tangent plane, and


X-xo
V-Yo
Z-zo
F~ (xo, Yo, zo)
F ~ (x o, Yo' 2 0)
F ~ (x o, Yo'

[Ch.6

(4)
20)

which are the equations of the normal.


Example 2. Write the equations of the tangent plane and the normal to
the surface 3xyz-z'=al at a point for which x=O, y=a.
Solution. Find the z-coordinate of the point of tangency, putting x=O,
y=a into the equation of the surface: -z'=al , whence z= -a. Thus, the
point of tangency is M (0, a, -a).
Denoting by F lX, y, z) the Jeft-hand side of the equation, we find the
partial derivatives and their values at the point M:
F~=3yz,

(F~)M= -3al ,

F;=3xz,

(F~)M=O,
(F~)M= -3aI

F~=3xy-3zl,

Applying formulas (3) and (4), we get


-3a1 (x-O) +0 (y-a)-3a 2 (z +a) =0
or x 1-z+a=0, which is the equation of thp tangent plane,
x-O y-a z+a
-3a 2 =-O-= -3a 2
or

y=y 0 a=zt a,which are the equations of the normal.

1981. Write the equation of the tangent plane and the equations of the normal to the following surfaces at the indicated
points:
a) to the paraboloid of revolution Z=X 2+y2 at the point
(1. -2,5);

x2

y2

Z2

b) to the cone 16+9-8=0 at the point (4,3,4);

to the sphere X 2+y2+ Z 2=2Rz at the point (Rcosu


R sin a, R).
1982. At what point of the ellipsoid
c)

x2

u2

Z2

QI+/jZ+Ci=1
does the normal to it form equal angles with the coordinate axes?
1983. Planes perpendicular to the x- and y-axes are drawn
through the point M (3, 4, 12) of the sphere r +!I + Zl = 169.
Write the equation of the plane passing through the tan~ents to
the obtained sections at their common point M.
1984. Show that the equation of the tangent plane to the
central surface (of order two)

ax" + by" +czl=k

Sec. 11]

The Tangent Plane and the Normal to a Surface

219

at the point M (x o' Yo, zo) has the form


axox + byoY -~- cZoz = k.
1985. Draw to the surface x2+2!1-~-3z2=21 tangent planes
parallel to the plane x -t- 4y -to 6z = o.
x2
y2
Z2
1986. Dra\v to the ell ipsoid a2 -~ lJ2 + CZ = 1 a tangent plane
\vhich cuts off equal segments on the coordinate axes.
1987. On the surface X 2+y2'_ Z2 -2x=-=0 find points at \vhich
the tangent planes are parallel to the coordinate planes.
1988. Prove that the tangent planes to the surface xyz = m3
form a tetrahedron of constant volume \vith the planes of the
coord inates.
1989. Sho\v that the tangent planes to the surface Vx + ~/y +
-1 V Z-== Va cut off, on the coordinate axes, segJnents \vhose Slun
is constant.
x2
y2
Z2
1990. Show that the cone 2-1--b
2 =-"i and the sphere
a
c
Xl -~ yl

-t- ( z -

b2

+ C2)2 = C2b'l. (b I -1- c)

- c-

are tangent at the points (0, + b, c).


1991. The angle bet\veen the tangent planes dra\vn to given
surfaces at a point under consideration is called the angle bet'lt'een
1u,10 surfaces at the point of their intersection.
1\t \vhat angle does the cyl inder Xl -t- y2 = R2 and the sphere
(X-R)2

Hl + z ~ R 2 intersect at the point M (~,

r:r, o)?

1992. Surfaces are called orthogonal if they intersect at right

angles at each point of the I ine of their intersection.


Sho\v that the surfaces x 2 + !l 2 + Zl = rl (sphere), y = x tan (IJ
(plane), and Z2 == (Xl +!I) tan 2 '1' (cone), \vhich are the coordinate
surfaces of the spherical coordinates r, {fl, '1', are I11utually orthogonal.
1993. Show that all the planes tangent to the conical surface
z =~ xt ( ~) at the point M (xo' Yo, zo)' where X o 7=- 0, pass through
the coordinate origin.
1994*. Find the projections of the ellipsoid

x 2 -1-- y2 -I- zl-xy-l

on the coordinate planes.


1995. Prove that the nornlal at any point of the surface of
revolution z = f <V x 2 -l- yl) (/' =F 0) intersect the axis of rotation.

220

[Ch.6

F unct ions of Several Var iables

Sec. 12. Taylor's Formula for a Function of Several Variables


Let a function f (x, y) have continuous partial derivatives of all orders
up to the (n + 1) th inclusive in the neighbourhood of a point (a, b). Then
Taylor's formula will hold in the neighbourhood under consideration:
1 ,
,
j(x, Y)=f(a, b)+Tj [fx(a, b) (x-a)+fy(a, b) (y-b)]-t

"
2"
" 2
+2!1 lfxx(a,
b) (x-a) +2fx l/(a, b) (x-a) (y-b)+fY/J(a, b) (y-b) ]-t- ...

+ niI
where
R n (x, y)= (n

[ (x - a)

[(x-a) :x +

~ I)!

axa+ (y -

aJn f (a, b) + R

b) iJ y

(y- b) ~r+J

(a

+ e (x-a),
(0

I n other nota tion,


!(x+h, y+k)=f(x,

1,

Y)+TI

<

(x, !/),

(l )

b+ 0 (y-b)]

<

I).

2 "

[hfx<x, y)+kfl/(x, y)]-t 2! [h fX;l (x, y)-f-

,. (x, y) + k2tyy
" (x, y)] + ... + ti1t [a
0JIl f (x, Y) -1+ 2hktXY
h ax + k ay

a +k!ln+l f(x-I-ell;

+_1__), [h
1
a.x
(n +.

or
M(x, y)= :, df(x,

Y-1-0k).

(2)

... + ~! dnf (x, y) + (n ~ 1) I d n Hf (x+ 011; !I -+- Ok)

(:~)

uy

Y)+~d2f(X, y)+ ...

The particular case of formula (1), when a=b=O, is called Maclaurin's


formula.

Similar formulas hold for functions of three and a lar~er nUlllber of


variables.
Example: Find the increment obtained by the function f (x, y) = , \ 3 _
-2y'+3xy when passing from the values x= I, y=2 to the values x1--=-I-l-h,
YJ=2+k.

Solution. The desired increment may be found by applying fOrn1l11a (2).


First calculate the successive partial derivattves and their values at the
given point (1, 2):

f~ (x, y) = 3x 2 + 3y,

1:(1,2)=3.1+3.2=9,

f~ (x, y)= -6 y 2+3x,

f;/{I,2)=-6.4-!-3.!=-21,

f:

t:

x (x, y) =- 6x,
y (x,

fuy (x

t:.\ (1, 2) = 6 1 == 6,

y) c= 3,

f:y(l, 2) = 3,

y) = -12y,

f~y(l, 2)=-12.2=-24,

I;;~ (x, y) = 6,

f:~x( 1, 2) = 6,

f~~y (x, y) == 0,

f~:~{l, 2) = 0,
f~;J~( 1, 2) = 0,
f~~y(l, 2) = -12.

f~~~ (x, y) = 0,
f~~1J (x, y) = - 12,

Sec. 12]

221

Taylor's Formula tor a Function of Several VarIables

All subsequent derivatives are identically zero. Putt}ng these results


into formula (2), we obtain:
1
Af(x, Y)=f(l+h, 2+k)-f(1, 2)=TI [h.9+k{-21)]+

+~ W6 + 2hk3 -I- k" (-24)] + ~!

(h"G + 3h"kO+ 3hk"O+ k l (-12)] =

=9h-21k -~ 31z 2

+ 3hk-12k

+h l -'2k l

1996. Expand f(x-+-h,y+k) in a series of positive integral


0 f han d kif

po\ve rs

f (x, y)

= ax2 -1- 2bxy -t- cy2.

the function f(x, y)=-x 2+2xy+3y 2_6x- 2y-4 by Taylor's fornlula in the neighbourhood of the point
(--- 2, 1).
1998. Fi nd the increnlent received by the function f (x, y) =
=-x 2 y \vhen passing frorTI the values x=l, y=l to
1997. Expand

x I =--~ 1 -t Iz, y 1 =-= 1 -t - k.

t 999. Expand the function f (x, y, z) == Xl --t- yl + z! + 2xy-yz--4x-3y-z-j-4 by Taylor's formula in the neighbourhood of
the point (1,1,1).
2000. Expand f(x i h, y~ Il, z-l I) in a series of positive integral po\\'ers of h, k. and I, if
f(X,!I,Z)~--=X2

!/2-t-z 2 -2xy-2xz-2yz.

2001. Expand the follo\ving function in a i\\aclaurin's series


up to terms of the thIrd order inclusive:

f (x,

y) ~ e"t; sin y.

2002. Expand the follo\\"ing function in a Maclaurin's series


up tot e rIllS 0 for de r f0 uri 11 c Iusive :

f (x,

!I)

== cos x cos y.

2063. Expand the follo\ving function in a Taylor's series in


the ne i gh bou rhood 0 f the poi nt (1, 1) 11 P toternlS of ordcrtwo
i I1C IllSi \'e:

f (x,

y)

= y.'(.

2004. Expand the follo\ving function in a Taylor's series in


the neighbourhood of the point (1, -1) up to ternls of order
three inclusive:

222

Functions of Several Variables

(Ch. 6

2005. Derive approximate formulas (accurate to second-order


terms in a and ~) for the expressions
1+(1. b) ... /"'(l+a)m+(l+~Y'
JI
2
'

a ) arc t anl_~'
if

lal and

I~I

are small compared with unity.

2006*. Using Taylor's formulas up to second-order terms,

approximate
a) vr03; VO.98; b) (0.95)2.01.
2007. Z is an implicit function of x and y defined by the
equation z'-2xz+y=O, which takes on the value z= 1 for X= 1
and Y== I. Write several terms of the expansion of the function
z in increasing powers of the differences x--l and y-l.
Sec. 13. The Extremum of a Function of Several Variables
to. Deftnition of an extremum of a function. We say that a function
has a maximum (minImum) f (a, b) a t the point P (a, b), !f for a 11
points P' (x, y) different from P in a suffici{'ntly sITlall neighbourhood of P
the inequality f (a, b) > f (x, y) [or, accordingl y, f (a, b) < f (x, y is fulfilled.
The generic term for maximum and minilnum of a function is ext retnu f11.
In similar fashion we define the extremum of a function of three or t110re
vanables.
2. Necessary conditions for an extremum. The points at which a differentiable function f (x, y) may attain an extremum (so-called stat,onary points)
are found by solving the following system of equations:

t (x, y)

t~ (x, y) -==0,

1;/ (x,

y) -- 0

(1)

(necessary cClnditions for an extremutn). Systenl (I) is equivalent to a single


equation, df (x, y) =0. In the general case, at the point of the extrel11tll11
P (a, b). the function f (x, y), or df (a, b) = 0, or df (a, b) does not exist.
3. Sufficient conditions for an extremum. Let P (a, b) be a stationary
point of the function f (x, y), that is, df (a, b) _-.: o. Then: a) if d 2t (a, b) < 0
for dx 2 + dy 2 > 0, then f (a, b) is the nzaximunl of the function f (x, y); b) if
d 2 f (a, b) > 0 for dx 2 + dy 2 > 0, then f (a, b) is the Inintmum of the function
t (x, y); c) if d 2f (a, b) changes sign, then t (a, b) IS not an ex tremum of f (\", y).
The foregoing conditions are equivalent to the folloWing: let f~ (a, b) :-.:
= f~ (a, b) == 0 and A = f: x (a, b), B =--= y (a, b), C ~~ f~y (a, b). We fonn the
dtscrimlnallt

t:

Then: 1) if ~ > 0, then the function has an extremunl at the point


P (a, b), namely a maximuln, if A < (or C < 0), and a mininlunl, If A > 0
(or C > 0); 2) if Ii < 0, then there is no extremum at P (a, b); 3) if l\ == 0,
then the question of an extremum of the function at P (a, b) rema ins open
(which is to say, it requires further investigation).
4. The case of a function of many variables. For a function of three or
nlore variables, the necessary conditions for the existence of an extrenlurn

Sec. 13]

The Extremum of a Function of Several Variables

223

are similar to conditions (I), while the sufficient conditions are analogous to
the conditions a), b), and c) 3.

Example 1. Test the following function for an ex tremum:


z=x+ 3xy l-15x-12y.
Solution. find the partial derivatives and fonn a system of equations (1):

axoz ==3x + 3y2-15== 0;


2

oz =6xy-12 =0
dY

or

x!+y2-5=O,
{ xy-2=O.
Sol ving the systenl we get four sta tionary points:
P J (I,2); P2 (2, I); P.(-I,-2); Pt(-2,-I).
Let us find

t~le

second derivatives

az
az
02 Z =6x
a-a
x
x y =6y, ay
a-2=6x,
2

and fortn the discrirninant t:. = AC - 8 2 for each stationary point.


1) For the pOInt PI:

A=(Oa2~)
\

p.

=6,

Z
B=(oO!a
=12, C=(002~) =
)
x Y PI
Y PI

=6, L\.=AC-B 2 =36-144 < O. Thus, there is no extremUI11 at the pOint PJ.
2) For the point P 2 : A -= 12, B ==6, C= 12; ~ -= 144-36> 0, A > O. At PI
thp function has a mininlUnl. ThIs tniniJTIUnl IS equal to the valu' of the
fune t ion for .\ - - 2, y =-= l'
zmin -~8+G-30-12=-28.
3) For the point P,: A=--=-6, 8==-12, C=-::-6; A=36-144 <0. There
extrelllUl1l.
4) For the point P4: A--=- -12, B = -6, C = -12; A = 144-36 > 0, A < n.
At the point P t the function has a nlaxhnuln equal to zmdx = -8-6 -t- 30-t
-t- 12 - ~ 28
5<t. Conditional extrenlum. In the shnplest case, the conditional extrenllJ/ll
of a function f (.\, y) IS 8 111axinlUl11 or nlininlunl of this function which IS
attained on the condition that its 3r~unlents are related by the equatioll
(P (x, =() (coupling equation). To find the conditlonaJ extrE:'murn of a function (x, y), given the relationship cr (x, y) = 0 \\'C fonn the so-called Lagrarlge
1"6

110

y)

fUllction

F (x,

y)

-= f (x, y) -t- A.<p (x, y),

where A is an undeternllned t1lllltipticr, and we seek the ordinary extremUl11


of this auxiliary function. The neces~ary conditions for the extrel11UI11 red uce
to a systelll of tluC'e equations:

aF

_~ of

aF

of

ax -- ax

+ iv ocp
_0
ox - ,
otp

iv -a=O'
d y ===a-+
y
y

(2)

q> (x, y) -:: 0

with th ree unknowns x, y, A, fronl which it is, generall y speaking, possible


to dete nnine these unknowns.

224

Functions of Several Variables

[Ch.6

The question of the existence and character of a conditional extremUln is


solved on the basis of a study of the sign of the second differential of the
Lagrange function:
o2F
a2 F
olF
2
d F(x, y)= ox! dx2+2axoydxdY+oy2 d y 2
for the given systeln of values of x, y, A obtained from (2) or the condition
that dx and dy are related by the equation

~ dx+ :: dy=O

(dx 2 +dy 2 =F 0).

Namely, the funchon f (x, y) has a conditional maxitnum, if d' F < 0, and a
conditional minimum, if d!P > o. As a particular case, if the discriminant A
of the function F (x, y) at a stationary point is positive, then at this point
there is a conditional maximurn of the function f (x, y), if A < 0 (or C < 0),
and a conditiona I minimunl, if A > 0 (or C > 0)
In similar fashion we find the conditional extrelnum of a function of
three or Inore variables provided there is one or several coupling equations
(the number of which, however, nlust be less than the number of the variables)
Here, we have to introduce into the Lagrange function as many undetenll1ned
multipliers factors as there are coupling equations.
Example 2. Find the extrelnum of the function
z==6-4x-3y

provided the variables x and y satisfy the equation


x2 -1- y2::= 1

Solution. Geolnetrically, the problem reduces to finding the greatest and


least values of the z-coordinate of the plane z=6-4x-3y for points of its
intersection with the cylinder ~2+y2= 1
We fonn the Lagrange function
F (x, y) ~ 6 - 4x - 3y -1- A (x 2+ y! -1).

We have :: =-4+2J.x. C::;=-3+2'J..Y. The necessary conditions yield the


following systenl of equations:
-4+2AX==0,
3+2Ay-=O,
{
x 2 + y2= 1.
Solving this system we find
5
1v 1 =2 '
and
5
'A
- - 2'
2Since

It follows

tha~

The Extremum of a Function of Ser.'eral Variables

Sec. 13]
5

If A-==2'

x"=5

225

and Y=r;' then d2 F > 0, and, consequenHy, the function

has a conditional minimum at this point. If A=- {. x=- : and y=- ~


then d2 F < 0, and, consequently, the function at this point has a conditional
Inaxilnunl.
Thus 1
16

zmax=6+ 5
Zmin =6-

+ 59 =11,

16

5- 5

= 1.

6". Greatest and smallest values of a function. A function thaf is differentiablc in a limited closed region attains
its greatest (smallest) value either at a stationary point or at a point of the boundary
of the region.
Example 3. Determ inc the grea test and
sll1allest values of the function
z-=x 2+ y2_ xy +x+ y
in the region
x~o, y~O, x-t-y~-3

Solution. Thc indicated re~ion is a triangle (Fig. 70).


1) Let us find the stationary pOints:

I
'\\

z't == 2x-y
l'y

-to 1 =0,

Fig. 70

~ 2lJ-x+
1.=.0;
.

\\'hence x-=-l, 11--==-1; and \\'e get thc point M (-I, -1)
1\t "1 the valuc of the function zl'1=-1 It is not absolutely necessary
to test for an extrenlunl
2) Let us investt~ate the function on the boundaries of the region.
When x-o \ve have Z=y2+ y , and the problenl reduces to seeking the
greatest and sll1allec;t valucs of thiS function of one argulIlent on thE' interval
-3 ~ y ~ O. Investigatin~, \\'e find that (Zgr)x=o= 6 at the point (0, -3);
(Zsm)",=o=--{- at the POl\1t (0.

When y=:...Q \ve get

_1/ 2)

Z==X2~-X.

point (-3.0); (Zsm)y=o=-

Shnilarly, we find that (Zgr)v=o=6 at the

at the point (_1/ 2, 0)

Whl'1l x-ry=-3 or y-==-3-x


\VC

\VC

will have z=3x 2 +9x+6. Similarly

. (32" - 2:~ ) ; (z~r)x+v=-a=6

find that (zsm)x+v=-a=- -:[ at the pOint \ -

nlctres coincides with (Zgr)x=o and (lgr)..v.=oo On the straight line x+ y=- 3
we could test the function for a conait ional extremUlll without reducing to
a function of one argunlcnt.
3) Correlating all the values obtained of the function z, we conclude
that zJlr= 6 at the points (0, -3) and (-3, 0); z5m = -1 at the stationary
point M.
8-1900

Functions of Several Variables

226

[Ch. 6

Test for maximum and minimum the follo\\ring functions of


two variables:
2008. z = (x-I)2 + 2y2.
2009. Z = (X-I)2 -2y'.
2010. 2 =x2 +xy+ y2 -2x-y.
2011. 2=X 8 y 2(6-x-y)(x>O, y>O).
2012. z = x4+ y4-2x + 4xy-2y .
.. /

x2

y2

1 -7-1)2
2
2014. Z= 1_(X +y2)2/ a
2
2015. z=(x 2+y2)e-ex +y2 ).
2013. z=xy

2016. Z =

I +x-y

y 1 +X 2+y2

Find the extrema of the following functions of three variables:


2017. U=X 2+y2 +z2- xy + x -2z.
2018.

y2

02 2

u=x+y>O, z>O).
y
z
4x +-+-(x>O,

Find the extrema of the following implicitly represented functions:


2019*. x 2+y2+ zl-2x+4y-6z-ll =0.
2020. x'-y2--3x+ 4y + Z2 + z-8=0.
Determine the conditional extrema of the following functions:
for x+y= 1.
for X 2 +yl=5.

2021. z=xy
2022. z.=x+2y
2023. Z = x

+y

f or "2
x
l/
-I- "3
= I.

y-x = ~ .
for Xl + y2 + Z2 = 9.

2024. z = cos l x+ cos l y


2025. u = x- 2y

for

+ 22

x2

Z2

x -t- y2 -J- Z2
for -a2 +-b2 + -c2 = I (a >b > c > 0).
2027. U= xy 2 z 3
for x+y+z=12(x>O,y>0,z>O).
2028. u = xyz
provided x -f-y+z=5, xy+yz+zx=8.
2029. Prove the inequality

2026.

U=

x+y+z
3

if

y2

V-xyz,

x~o, y~O, z~O.


Hint: Seek the maximum of the function u=xyz provided x+y+z=S.

Finding the Greatest and Smallest Values of Functions

Sec. 141

227

2030. Determine the greatest value of the function z = 1 +x+ 2y


in the regions: a) x~O, y~O, x+y~ 1; b) X~O, y<:O,
x-y~ 1.

2031. Determine the greatest and smallest values of the functions a) z=x1y and b) Z=X I _ y 2 in the region x'+y2~1.
2032. Determine the greatest and smallest values of the function z=sinx-l-siny-t-sin(x+y) in the region O~x~ ~.
31

O~Y~2

2033. Determine the greatest and smallest values of the function z=x'+y'-3xy in the region O~x~2, -1~y~2.
Sec. 14. Finding the Greatest and Smallest Values of Functions
Example 1. It is required to break up a positive nunlber a into three
nonnegative nUIIlbers so that their product should be the greatest possible.
Solution. Let the desired ntllnbers be x, y, a-x-y. We seek the maximunl of the function f (x, y) ==-:xy (a-x-y).
According to the problell1, the function f (x, y) is considered inside a
closed triangle x~o, y~O, x+y~a (Fig. 71).

x
Fig. 71
Solving the systenl of equations
f~(x. y)=ay(a-2x-y)=O,
{

fy

(x, y)==x(a-x-2y)=O,

,i)

we will have the unique stationary point (;


for the tntel'lor af the
triangle. Let us test the sufficiency conditions. We have

8*

Functions of Several Variables

228
Consequent\ y,

[Ch. 6

"(a a)

2
A= 'xx 3' "'3 =-3"
a,

" (a3' 3"a) = - 3I a,

B=f xy

" (a3'

2 a and
3a ) =-3
L\=AC-B 2 > 0, A < o.

C=fuu

(~. ~)

the function reaches a maximum. Since f (or. y) = 0 on


the contour of the triangle, this maximunl will be the greatest vdlue, which

And so at

is to say that the product will be greatest, if X= Y =a-x-y = a ,and the


3
aB
greatest value is equal to 2f'
Note The ploblenl ran also be solved by the l11ethods of a conditional
extremum, by seeking the maximum of the function u ==xyz on the condition
that x+y+z==a.

2034. From among all rectangular parallelepipeds with a


given volume V, find the one whose total surface is the least.
2035. For what dimensions does an open rectangular bathtub
of a given capacity V have the smallest surface?
2036. Of all triangles of a given perimeter 2p, find the one
that has the greatest area.
2037. Find a rectangular parallelepiped of a given surface S
with greatest volume.
2038. Represent a positive number a in the form of a product of
four positive factors which have the least possible stun.
2039. Fil1d a point M (x, y), all an xy-plane, the sunl of
the squares of the distances of which from three straight lines
(x=O, y==O, x-y+ 1 =0) is the least possible.
2040. Find a triangle of a given perimeter 2p, \vhich, lipan
being revolved about one of its sides, generates a solid of
greatest volunle.
2041. Given in a plane are three nlaterial points PI (xl' !Jt)'
PI (XI' Y2)' P a (x a, Ya) with masses m m 2 , rna For \vhat position
"
of the point P (x, y) will the quadratic
moment (the moment ()f
inertia) of the given system of points relative to the point P
(Le., the SUITI m l P t P2 i-m2P2P2 +m aP ap2 ) be the least?
2042. Draw a plane through the point M (a, b, c) to [orIn
a tetrahedron of least volume with the planes of the coordinates.
2043. Inscribe in an ellipsoid a rectangular parallelepiped of
greatest volume.
2044. Determine the outer dimensions of an open box \vith a
given wall thickness ~ and capacity (internal) V so that the
smallest quantity of material is used to make it.

Sec. 141

Finding the Greatest and Smallest Values of Functions

229

2045. At what point of the elli pse


x2
y2

i2+V=l
does the tangent line to it form with the coordinate axes a triangle of sIl1allest area?
2046*. Find the axes of the ellipse
5xl + 8xy + 5y 2 = 9.
2047. Inscribe in a given sphere a cylinder having the
greatest total surface.
2048. The beds of two rivers (in a certain region) approxinlately represent a parabola y=x 2 and a straight line x-y-2=O.
I t is required to connect these ri vers by a straight canal of least
length. Through \vhat points will it pass?
2049. Find the shortest distance fronl the point 1\;1 (1, 2, 3)
to the straight line
x

T= -3=2
2050*. The poi nts A and B are situated in different optical
111e(lla separated by a straight line (Fig. Z2). The Yclocity of

13

I
Fig. i3

light ill the first 111CdiuI11 is vI' in the second, v 2 Applying the
Ferlllat principle, according to which a light ray is propagated
along a line A,\1B \vhich requires the least tillle to cover, derive
the la\v of refraction of light rays.
2051. Using the Ferlnat principle, derive the la\v of reflection
of a light ray frolll a plane in a homogeneous medium (Fig. 73).
2052*. If a current J Hows in an electric circuit containing a
resistance R, then the quantity of heat released in unit tinle is
proportional to J2 R. Deterilline how to divide the current I into

Functions of Several Variables

236

[Chi 6

currents I l' 12 , I a by means of three wires, whose resistances are


R1 , Ra, R a , so that the generation of heat would be the least
possible?
Sec. 15. Singular Points of Plane Curves

f (x,

to. Definition of a singular point. A point M (x(\' Yo) of a plane curve


y)=O is called a singular point if its coordinates satisfy three equations

at once:

f (xo,

Yo) = 0,

f~ (x o, Yo) =

o.

f~ (xo' Yo) = O.

2. Basic types of singular points. A t a singular point M (xo, Yo), let the
second derivatives

A ==
B=

':x

f:

(xo' Yo)'
(xo' Yo)'

C = f~y (xo, Yo)

be nof all equal to zero and


then:
a) if Ii> O. then M is an isolated point (Fig. 74);
b) if Ii < 0, then M is a node (double point) (FIg. 75);
c) if Ii = 0, then M is either a cusp of the first kind (Fig. 76) or of the
second kind (Fig. 77), or an isolated point, or a tacnode (Fig. 78).

Fig. 74

Fig. 75

When solving the problems of this section it is always necessary to draw


the curves.
Example 1. Show tha t the curve y2 = ax! + x has a node if a > 0; a 11
isolated point if a < 0; a cusp of the first kind if a = O.
Solution. Here, f (x, y) ax 2+x-y2. Let us find the partial derivatives and equate them to zero:

==

f~(x, y)=2ax+3x2 =0,

f~ (x, y)=- 2y=O.

Singular Points of Plane Curves

231

0):

This system has two solutions: 0 (0, 0) and N ( - : a,


but the
coordinates of the point N do not satisfy th~ equation of the given curve.
Hence, there is a unique singular point 0 (0, 0).

x
Fig. 78

Fig. 77

Fig. 76

Let us find the second deriva tives and their val ues at the pOint 0:

,:% (x,
':y(X,
f;,/(X,

y)

= 2a + 6x,

y)=O,
y)=-2,

= 2a,

8=0,
C=-2,

~=AC-B2=-4a.

y
v

o
Fig. 79

Fig. 80

Fig. 81

Hence,
if a > 0, then A < 0 and the point 0 is a node (Fig. 79);
if a < 0, then 1\ > 0 and 0 is an isolated point (Fig. 80);
if a::--::O, then ~-=O. The equation of the curve in this cas~ will be
y2=X' or Y= YX'; y=exists only when x~O; the curve is symmetric
about the x-ax is, which is a tangent. Hence, the point M is a cusp of the
first kind (Fig. 81).

Functions of Several Variables

232

[Ch.6

Determine the character of the singular points of the following curves:


2053. y" = - x" +x".
2054. (Y_X I)2 = Xl.

2055. a 4 y!

= a 2 x' -

Xl.

y 2 _x 2 _ y" = o.
3
2057. x +yl-3axy=O (folium of Descartes).
2056. X

2058. y2 (a-x) = Xl (cissoid).


2059. (x 2+y!)I=a 2 (x"_y2) (lemniscate).
2060. (a+x)yl=(a-x)x! (strophoid).
2061. (Xl y2) (x-a)1 = b2x 2 (a> 0, b > 0) (conchoid).

Consider three cases:


1) a>b, 2) a=b, 3) a<b.

2062. Determine the change in character of the singular point


of the curve y2=(x-a)(x-b) (x-c) depending on the values of
a, b, c(a~b~c are real).
Sec. 16. Envelope
10. Definition of an envelope. The envelope of a family of plane Cllrvefi
is a curve (or a set of several curves) which is tangent to all tinps of the
given family, and at each point is tangent to sonle line of the given faillily.
2. Equations of an envelope. If a family of curves

f (x,

y, CL):::::O

dependent on a single variable parameter a has an envelope, then till' Ilarametric equations of the latter are found fronl the system of equations
{

',(X. y. a) =~'
fa (x, y, a)-D.

(1 )

Eliminating the parameter a fronl the system (1), we get an equation of


the f onn
D (x, y) =0.
(2)
It should be pointed out that the formally obtained curve (2) (the ~o
called "discriminant curve") may contain, in addition to an envelope (if
there is one), a locus of singular points of the given farnily, which locus IS
not part of the envelope of this f am j} y.
Whfn solving the problems of this section it is advisable to Illake

drawings.

Exanlple. Find the envelope of the family of curves

x cos a+ y sin a-p =0 (p = const. p > 0).

Sec. 16]

233

Envelope

Solution. Th~ given family of curves depends on the parameter


the system of equations (1):
xcosa+ysina-p=O,
{ - x sin a 1- Y cos a -= O.

Q.

Form

Solving the system for x and y, we obtain parametric equations of the


envelope
x=pcosu, y==psina.
Squaring both equations and adding, we eliminate the parameter a:
x 2 -1- 1/ 2 == p2

Thus, the envelore of this family of strai~ht lines is a circle of radius p


with centre at the origin. This particular family of straight lines is a family
of tangent lines to thIs circle (Fig. 82).

2063. Find the envelope of the family of circles


(x-a)t

+ yt =

a; .

2064. Find the envelope of the family of straight lines

Y --kx+.P..
2k
(k is a variable parameter).
2065. Find the envelope of a family of circles of the same
radius R whose centres lie on the x-axis.
2066. Find a curve which forms an envelope of a section
of length 1 when its end-points slide along the coordinate axes.
2067. Find the envelope of a family of straight lines that
form with the coordinate axes a triangle of constant area S.
2068. Find the envelope of ellipses of constant area S whose
axes of synlnletry coi Heide.

Funct ions of Several Varl abies

234

[Ch. 6

2069. Investigate the character of the "discriminant curves"


of families of the following lines (C is a constant parameter):
a) cubic parabolas y=(x-C)';
b) semicubical parabolas 11 = (x-C)';
c) Neile parabolas yl = (X-C)I;
d) strophoids (a+x) (y_C)2 =r (a-x).

Fig. 83

2070. The equation of the trajectory of a shell fired from a


point 0 \vith initial velocity V o at an angle a to the horizon
(air resistance disregarded) is

Taking the angle a as the parameter, find the envelope of all


trajectories of the shell located in one and the sanlC vertical
plane ("safety parabola") (Fig. 83).
Sec. 17. Arc Length of a Space Curve
The diOerenttal of an arc of a space curve in rectangular Cartesian coordinates is equal to
ds = V dx 2 + d!l2 + dz 2 ,
\vhere x, y, z are the current coordinates of a roint of the curve.

If

x=x(t),

y=y(t),

z=z(t)

are parametric equations of the space curve, then the arc length of a section
of it from t=t l to t=t 2 is
S=

dx )
Jtr,VI" ( lIt
+
2

t1

dy ) 2

df

+ (dZ)
Cit

dt.

S~c.

235

The Vector Function of a Scalar Argument

18]

In ProbletTIs 2071-2076 find the arc length of the curve:

2071. x=t, y=t', Z=2~'

from
3

2072. x=2cost, y=2sint, z=-t


3t
2073. x=etcost) y=etsint,
x2

Xl

2074. Y=T' z=6


2075. Xl = 3y, 2xy = 9z

t=O to t=2.

from t==O to t='Jt.

z=et

from t=Otoarbitraryt.

from x=O to x=6.


from the point 0 (0,0,0) to M (3,3,2).

2076. y=aarcsin~, z= a In ~ from the point 0(0,0,0)


a

a-x

to the point M (x o, Yo, zo).


2077. The position of a point for any time t (t >0) is defined
by the equations
x=2i, y=lnt, z=t l
Find the mean velocity of motion between times t = 1 and t = 10.
Sec. 18. The Vector Function of a Scalar Argument
10 The derivative of the vector function of a scalar argument. The vector

tUtlct ion a == a (t) may be defined by specifying three scalar functions ax (t),
ay (t) and az (t), \\'hich are its projections on the coordinate axes:

a=a x (t) I +ay (t)J+a z

<t) k.

The derivative of the vector function a::::: a (t) with respect to the scalar
argument t is a new vector function defined by the equality
~ _ lim a (I -1- Llt) -a (t)
dt - At ~ 0

llt

da x (t)

i -1- day

dt

(t)

J + dar (t) k

dt

dt

The modulus of the derivative of the vector function is

x
I 1= .V. l(da )2 + (da,v) +
~
dt

dt

dt

dadtz)1

The end-point of the variable of the radius vector r=r(t) describes in space
the curve

r=x (t) 1+ y (t) J+ z (t) 11,


which is called the hodograp1J of the vector r.
The derivative :~ is a vector, tangent to the hodograph at the corre
sponding point; here,

Itit I=dt,
dr

ds

where s is the arc length of the hodograph reckoned from some Initial point.
For example,

I~~ 1= 1.

Functions of Several Variables

236

parametl~r t

If the

is the time, then

~ =(/

=tv

[Ch.6

is the velocity vector of the

extremity of the vector r, and :;~= ~~


is the acceleration vector of the
extremity of the vector r.
2. Basic rules for differentiating the vector function of a scalar argument.
1)

2)

da

db

de

dt (a+b-c)=dT+(jf-df ;

(ma)=m

dq>

~~

3) (if (cpa)=(jf

4) (j[

, where m is a constant scalar;

da
a+ cp dt'

da

where cp (i) is a scalar function of t;

db

(ab)=Fb+adt;

da

db

5) (if (aXb)=dtXb+aXdt;

6)

7)

da

a [ep (i)] =dfP

~~

dq>.
F'

=0, if lal=const.

Example t. The radius vector of a moving point is at any instant of


Hnle defined by the equation

r= i-4t 2j+3t 2k.

(I)

Detennine the trajectory of motion, the velocity and accelera tion.


Solution. From (1) we have:
x== 1, y==-4i 2 , 2=3i 2
Eliminating the time i, we find that the trajectory or nlotion is a straight
line:
x-I
y
z
-0-=-4=3"
Frolll equation (1), differentiatin~, we find the velocity

dr

([[=- 8ij+6tk

and the acceleration


d2r
di 2 =-Bj+6k.

The nlagnitude of the velocity is

I~~ 1= Y(-

81)2+ (61)2= 10 I t I

We note that the acceleration is constant and l~

I::~I= Y(-8)2+ 6 =10.


2

Sec. 18]

The Vector Function of a Scalar Argument

237

._~---------~----..::::....--------

2078. Show that the vector equation r..:.-r t = (rl-r 1) t,


where r 1 and r l are radius vectors of two given points, is the
equation of a straight line.
2079. Determine which lines are hodographs of the following
vector funct ions:
a) r=at+c;
c) r = a cos t + b sin t;
l
b) r = at -t- bl;
d) r=acosht-t-bsinht,

where a, b, and c are constant vectors; the vectors a and b


are perpend icular to each other.
2080. Find the derivative vector-function of the function
a (/)::= a (I) a O(I), where a (t) is a scalar function, \vhile aO (t)
is a unit vector, for cases \vhen the vector a (t) varies: 1) in
length only, 2) in direction only, 3) in length and in direction
(general case). Interpret geolnetrically the results obtained.
2081. Using the rules of differentiating a vector funct i::>n with
respect to a scalar argument, derive a formula for differentiating
a fnixed product of three vector functions a, b, and c.
2082. Find the derivative, with respect to the paranleter t,
of the volunlc of a parallelepiped constructed on three vectors:
a = i + tj+ t 2 k;
b=2ti-j+ t 3 k;
c = - tli -f- tSj+ k.
2083. The eq uat ion of nlot ion is
r =---= 3i cos t --~- 4j sin

t,

where t is the tinlC. Deterlnine the trajectory of ITIotion, the


velocity and the acceleration. Construct the trajectory of nlotion
and the vectors of velocity and acceleration for times, t = 0,

'-0

-T

and t = ~

2084. The equation of nlotion is

r = 21 cos t

+ 2j sin t -J- 3kl.

Dctcrll1ine the trajectory of nlotion, the velocity and the acceleration. \Vhat are the nlagnitudes of velocity and acceleration
and \vhat directions have they for time t = 0 and t = ~ ?
2085. The equation of nl0tion is
r = I cos a cos rot -f- j sin a cos rot + k sin rot,
where a and (t) are constants and t is the tilne. Determine the
trajectory of motion and the lTIagnitudes and directions of the
velocity and the acceleration.

Functions of Several Variables

238

(Ch.6

2086. The equation of motion of a shell (neglecting air resistance) is

where V o {vox, v oy ' v oz } is the initial velocity. Find the velocity


and the accelerat ion at any instant of time.
2087. Prove that if a point is in motion along the parabola
y =~,
z == 0 in such a manner that the projection of velocity
a
on the x-axis remains constant (:; = canst). then the acceleration remains constant as well.
2088. A point lying on the thread of a screw being screwed
into a beam describes the spiral
x=acos6,

y=asin6,

z=h6,

where 6 is the turning angle of the screw, a is the radius of the


screw, and h is the height of rise in a rotation of one radian.
Deternline the velocity of the point.
2089. Find the velocity of a point on the circumference of a
wheel of radius a rotating with constant angular velocity ro so
that its centre moves in a straight line \vith constant velocity v O
Sec. 19. The Natural Trihedron of a Space Curve
At any nonsingular point M (x, y, z) of a space curve r==r(t) it is possible to construct a natural trihedron consisting of three ITIutually perpendicular pI anes (Fig. 84):
1) osculating plane MM t M 2

containin~

the vectors

c:;;

and

~:~;

2) normal plane MM:M which is perpendicular to the vector '::;

and

3) rectifying plane MM l """ which is perpendicular to the first two J1lanes.


At the intersection we obtain three straight lines;
1) the tangent MM l ; 2) the principal normal MM z ; 3) the buzofltlul MM"
all of which are defined by the appropria te vectors:

1) T=

~~

(the vector of the tangent line);

d 2r
.
2) B=dfX dt2 (the vector of the blllormal);
3) N = Bx T (the vector of the principal normal);
dr

The corresponding unit vectors

-r=iTi;

~=iBi;

\'=iNi

Sec. 19]

The Natural rrihedron of a Space Curve

239

may be computed from the formulas


d'f

tiS

dr

~=dS ;

v=

I:: I;

1J=~Xv.

If X, Y, Z are the current coordinates of the point of the tangent, then


the equations of the tangent have the form

X-x
TJI:

Z-z

Y-y

(1)

=r;=r-;-'

Rectlfying
plane

dr

T=at
Fig.

~4

wh('rc T x -= :; ; T y= :~-. T z= :: ; from the condition of perpendicularity


of the hne and the plane \ve get an equation of the normal plane:
Tx(X-X)1-Ty(Y-y)+Tz(Z-z)=O.

(2)

If in C'quations (1) and (2), \\re replace T x ' Ty , T z by B.'(, B", Bz and N".
Ny, N z , we get the equations of the binornlal and the princij'al normal and,
respectively, the osculating plane and the rectifying plane.
Example 1. Find the basic unit vectors T, \' and Ii of th(l curve

x=t, U=t 2 , z=t'

at the point t=l.


Write the equations of the tangent, the principal nornlal and the binor
n1al at this point.
Solution. We have
r=tl+tIJ+tl/l
and

:~ =1+2tj+3t 1k.

d1r

dtl == 2J

+6tll.

Functions of Several Variables

240

[Ch.6

Whence, when t = 1, we get


dr

T=d[ =1+2J+3k;
B= dd~

d
Xdt~=

I'

1 j2 k3 =6i-6j+2ki

N=BXT=I~ JO ~1~_221-16j+18k.
123

Conseq uentl y,
l'

p __ 31-3j+k

1+2j+3k
Vl4

--

V19

v=

-111-81+911
V266

Since for t= 1 we have x= 1, y= 1, z= 1, it follows that


x-I y-l z-1
-1- = -2- = -3are the equations of the tangent,
x-I y-l z-1
-3-=-3 = - 1

are the equations of the binornlal and


x-I
-11

11-1 2-1
-8 =--9-

are the equations of the principal normal.


If a space curve is represented as an intersection of two surfaces
F (x, y, z) =0, a (x, y, z) ==0,
. .

d 2r

dr

then In place of the vectors (j[ and dt 2 we can take the vectors dr {dx, dy, dz}
and d2r {d 2x, d2y, d2z}; and one of the variables x, y, 2 nlay be considered
independent and we can put its second differential equal to zero.
Example 2. Write the equation of the osculatin~ plane of the circle
X2+y2+ Z2=::6, X+Y+2~-=0
(3)
at Its pOInt M (1, 1, -2).
Solution. Differentiating the systenl (3) and considering x an independent
variable, we will have
xdx+ydy+zdz=O,
dx+dy+dz=O
and
dx + dy + Y d 2y + dz 2 + Z d 2z -= 0,
d 2 y+d 2 z=O.
Putting x= 1, y== I, 2==-2, we get
dy==-dx;
2

d y == -

3' dx

dz=O;
d2z =

"3 dx 2

Sec. 19]

The Natural Trihedron of a Space Curve

241

Hence, the osculating plane is defined by the vectors

and {D. -

{dx. -dx, o}

~ dx, idX.}

or
{I, -I, O}

{a,

and

-1, I}.

Whence the normal vector of the osculating plane is

B=I~a -{
~I=-I-j-k
-1 I
and, therefore, its equation is
- I (x-l)-{y-I)-(2+2)==O,

that is,

x+y+z=o,
as it should be, since our curve is located in this plane.

2090. Find the basic unit vectors

T,

v,

x==:l-cost, y=sint,

of the curve

z=t

at the point t = ~ .
2091. Find the unit vectors of the tangent and the princi pal

norillal of the conic spiral


r = et (I cos t

+j

sin t i- k)

at an arbitrary point. Determine the angles that these lines Illake


\vith the z-axis.
2092. Find the basic unit vectors T, v, p of the curve
at the point x

== 2.

y=x 2 ,

z= 2x

2093. For the screw line

x = a cos t,

Y == a sin t,

= bi

write the equations of the straight lines that forln a natural


trihedron at an arbitrary point of the line. Deternline the direction cosines of the tangent line and the principal normal.
2094. Write the equations of the planes that form the natural
trihedron of the curve
x" -1- y2 -t- Z" = 6, x 2- y2 -}- Z2 == 4

at one of its points 1\1 (1, 1, 2).


2095. Fornl the equations ot the tangent line, the normal
plane and the osculating plane of the curve x=t, y=t', z=t'
at the point M (2, 4, 8).

242

Functions of Several Variables

[Ch.6

2096. Form the equations of the tangent, principal normal,


and binormal at an arbitrary point of the curve
tC
t'
t2

X=4' Y=3'

Z=2

Find the points at which the tangent to this curve is parallel


to the plane x+3y+ 2z-10=0.
2097. FOrlTI equations of the tangent, the osculating plane,
the principal normal and the binormal of the curve
t2

x=t, y=-t, z=2


at the point t = 2. Compute the direction cosines of the binormal
at this point.
2098. Write the equations of the tangent and the normal
plane to the following curves:
a) x=Rcos1t, y=R sintcost, z=R sint for t= ~ ;
b) Z=X 2 +y2, x=y at the point (I, 1,2);
c) X2 +y2+ Z2=25, x+z=5 at the point (2, 2V3, 3).
2099 Find the equation of the normal plane to the curve
Z=X 2 _ y 2, y=x at the coordinate origin.
2100. Find the equation of the osculating plane to the curve
x=et , y=e- f , z=tV2 at the point t=O.
2101. Find the equations of the osculating plane to the curves:
a) x2_l-Y~+Z2=9, X2_ y 2=3 at the point (2,1,2);
b) x 2 = 4y, x' = 24z at the point (6, 9, 9);
c) x 2 + Z2 = a 2 , y2 t Z2 = b 2 at any point of the curve (x o' Yo. zo).
2102. Form the equations of the osculating plane, the principal
normal and the binormal to the curve
y2=X, x 2 =z at the point (I, I, 1).
2103. Form the equations of the osculating plane, the
pal normal and the binormal to the conical scre\v-line x =
y=tsint, z=bt at the origin. Find the unit vectors
tangent, the principal normal, and the binormal at the

princit cos t,
of the
origin.

Sec. 20. Curvature and Torsion of a Space Curve


1. Curvature. By the curvature of a curve at a point M we mean the
number
1
1I mcp- .
K=-=
R ~s-+o bas

Sec. 20)

243

Curvature and Torsion of a Space Curve

where cp is the angle of turn of the tangent line (angle of contingence) on a


segment of the curve MN, f!s is the arc length of this segment of the curve.
R is called the radius of curvature. If a curve is defined by the equation
r=r (s), where s is the arc length, then

~=I~~I
For the case of a general parametric representation of the curve we have
dr

Icn

d2rl

([t2

(1)

I~~r

R=
2. Torsion. By
mean the number

of a curve at a point M we

torsion (second curvature)

1
1.
0
T=-= 1m ~S-t>O

~s '

where 0 is the angle of turn of the binormal (angle of contingence of the


second kind) on the segment of the curve AT.V. The quantity Q is called the
radIus of torsion or the radius of second cart'at ure. If r= r (5), then
drd 2rd ar

~= I ~1=dS(fS2(fSi
1
Q

ds

(ddsr)2'
2

where the minus sign is taken when the vectors ~~ and


direction, and the pi us sign, when not the same.
If r= r (t), \vhere t is an arbitrary parameter, then

'Y

have the same

dr d 2r dar

Q=

di (fl2 lfii

(dr

d2

r)2.

(2)

dt X dt2

Example 1. Find the curvature and the torsion or the screw-line

r= I a cos t + j a sin t + k

bt

(a

>

0).

Solution. We have
dr

di = - I a sin t +J a cos t +kb,


d 2r

dt 2 = - I a

dar

cos t - j a sin I,

= - I a sin t - j a cos t.

dt a
Whenc~
2

dr d r

dt X dt 2 =

I -a
I
sin t
-a cos t

aJcos t kb \ = I ab sin t -Jab cos t +a2i


-a sin t 0

Functions of Several Variables

244
and

I-a

[Ch.6

acos I
a cos t 0

d d 2 dB
sin t
t b
2
; dt~ dt~= -a c~s t -a sin t 0 = a b.

a sIn t -

Hence, on the basis of formulas (1) and (2), we get

yaz-tbi

(a2+b2)3/2 = a 2 +b2

and

a2b

Q=

a2

(a 2

+b = a
2)

b
2

-t- b2

Thus, for a screw-line, the curvature and torsion are constants.


30 Frenet formulas:
dv _
T+~
ds--7[ Q'

dT _"
([S-7['

dP
v
ds=-Q"

2104. Prove that if the curvature at all points of a line is


zero, then the line is a straight line.
2105. Prove that if the torsion at all points of a curve is zero,
then the curve is a plane curve.
2106. Prove that the curve
x=1+3t-t-2t 2 , y=2-2t-t-5t 2 , z=l-{1

is a plane curve; find the plane in which it lies.


2107. Conlpute the curvature of the following curves:
a) x=cosf, y=sint, 2=cosh t at the point t==O;
b) x 2- y2 -t- Z2 == 1, y2 - 2x -~- z = 0 at the poi nt (1, 1, 1).
2108. Compute the curvature and torsion at any point of the
curves:

a) x = et cos t, Y = e' sin t,

b) x=a cosh t, y=a sinh

t.

==

et ;

z=at (hyperbolic screw-line).

2109. Find the radii of curvature and torsion at an arbitrary


point (x, y, z) of the curves:
a) x 2

b)

= 2ay,

X =

Xl

= 6a

z;

3p2y , 2xz = p2.

2110. Prove that the tangential and normal components of


acceleration ware expressed by the formulas
dv

UJT = dtt',

'W y

v2

= R \',

where v is the velocity, R is the radius of curvature of the


trajectory, ~ and v are unit vectors of the tangent and principal
normal to the curve.

Sec. 20]

Curvature and Torsion of a Space Curve

245

2111. A point is in uniform motion along a screw-line , =

= ia cos t -1- ja sin t + btk with velocity v. Compute its accelera-

tion w.

2112. The equation of mot ion is

r= tl-t- t j+ tSk.
2

DetermiRe, at times t~O and t=l: 1) the curvature of thetrajectory and 2) the tangential and normal components of the.
acceleration.

Chapter VII

MULTIPLE AND LINE INTEGRALS

Sec. 1. The Double Integral in Rectangular Coordinates


1. Direct computation of double integrals. The double integral of a confunction f (x, y) over a bounded closed region S is the limit of the
corresponding two-dimensional integral sum

~inuous

~ f (x, y) dx
(S)

dy=

~ ~f (xi. Yk) !J.x,l!Yk.

lim
max dXI ~
max dYl' -?

(I)

where ~xi=xi+.-xi' ~Yk=Yk+l-Yk and the sum is extended over tho~e


values of i and k for which the points (xi, Yk) belong to S.
2. Setting up the limits of integration in a double integral. We distinguish two basic types of region of integration.

D
I

X,

Fig. 85

c
x

Fig. 86

1) The region of integration S (Fig. 85) is bounded on the left and right
by the straight lines x=x. and x=x (XI> x.), from below and from abovE
by the continuous curves Y= (J). (x) (AB) and y = (J)I (x) (CD) [<PI (x) ~ q>1 (x),
each of wh ich intersects the vertical x = X (x .-; X < XI) at only one point (seE
Fig. 85). In the region S, the variable x varies from Xl to XI' while the va
riable y (for X constant) varies from Yl = q>l (x) to Ya = q>1 (x). The integral (1) ma~

Sec. 1]

T he Double Integral in Rectangular Coordinates

247

be computed by reducing to an iterated integral by the formula

X2

(5)

Xl

CPI

(x)

~ dx ~

f(x, y)dxdy=

f(x, y)dy,

CPI

(X)
q>2 (x)

~ f (x,

where x is held constant when calculating

y) dy.

CPI (x)

2) The region of integration S is bounded from below and from above


by the straight linE's Y=YI and Y=Y2(Y. > YI)' and fronl the left and the
right by the cont inuous curves x = '1'1 (y) (AB) and x = '1'2 (y) (CD) ['1'2 (y) ~"" (y) l.
each of which intersects the parallel Y = Y (y, E;;; Y ~ Y2) at only one point
(Fig. 86).
As before, we have

Hf

(x, y) dx dy =

(5)

her~, in the integral

'1'3

y.,

1J'2

(Y)

'It

1J'1

(II)

~ dy ~

f (x, y) dx,

(Y)

~ f (x,

y) dx we consider y constant.
'1'1 (y)
If the region of integration doe~ not belong to any of the above-discussed
types, then an attempt is luade to break it up into parts, each of which does
belong to one of these two types.
Example 1. Evaluate the integral
1

1=

~ dx ~
o

(x+y)dy.

-2

Fig. 87
Solution.

Example 2. Deternlinc the lilnits of integration of the integral

f(x, y)dxdy

(S)

Multiple and Line Integrals

'248

[Ch.7

if the region of integration S (Fig. 87) is bounded by the hyperbola y2_ X 2= 1


and by two straight lines X= 2 and x=- 2 (we have in view the region containing the coo rdinate origin).
Solution. The region of integration ABeD (Fig. 87) is bounded by the
straight lines x-= -2 and x=2 and by two branches of the hyperbola

y=1+x2

y-==-Yl+x2

and

that is, it belongs to the first type. We have:

YI+X 2

f (x. y)dxdy=

(S)

~
- 2

dx
_

f(x, y)dy.

YI+X 2

Evaluate the follo\ving iterated integrals:


2

2113. ~ dy ~ (Xl
o

-I

55
dx

(x

dy

n
--

x2dy

55
1

a cos q>

~ dcp ~ r l sin l (p dr.

2119.

11' I-X:'

7.

dx

~ r dr.
a sin q>

2116.

2118. ~ dcp

+ Y/&

1 51 x dy
2115. 5dx 1 + y2
!

y2-4

In

2117. ~dY ~ (x-l-2y)dx.

4,

2114

+ 2y) dx.

2120. ) dx

Vi-Xl_yl dy.

Write the equations of curves bounding regions over which the


following dduble integrals are extended, and draw these regions:
2

2-y

2121. ~ dy ~ {(x, y) dx.


-0

!!~_ I

4
1

X+8

2122. ~ dx ~ {(x, y)dy.

2125. ~ dx

x2

I
4

V 2S-X"

~ {(x, y)dy.

IO-Y

2123. ~ dy ~ {(x, y) dx.


o

2X

2124. ~ dx ~ {(x, y) dy.

X+2

2126. ~ dx ~ {(x, y) dy.

-I

X2

Set up the limits of integration in one order and then in the


other in the double integral

H{(x, y) dxdy
(S)

for the indicated regions S.

The Double Integral in Rectangular Coordinates

Sec. I]

2127.
C(O, 1).
2128.
2129.
C(O, 1).
2130.
C(2, 7),
2131.
o (0, 0),

249'

S is a rectangle with vertices 0 (Ot 0), A .(2,0), B (2, 1),


S is a triangle with vertices 0(0,0), A(l, 0),8(1,1).

S is a trapezoid with vertices 0 (0, 0), A (2, 0), B (1, 1),


S is a parallelogram with vertices A (1, 2), B (2, 4),
D(I, 5).
S is a circular sector DAB with centre at the point
\vhose arc end-points are A (1, 1) and 8 (--1, 1) (Fig. 88).

B(-I,f)

A(~f)

Fig 89

21 a2. S is a rIght para boIic segnlent AOB bounded by the


parabola BOA and a segment of the straight line BA connecting
the poi nts B (--- 1, 2) and A (1, 2) (F~ig. 89).
2133. S is a circular ring bounded by circles \vith radii r == 1
and R == 2 and \\'ith cotTIITlOn centre 0 (0, 0).
2134. S is bounded by the hyperbola y2_x- = 1 and the circle
x 2 I 1/ 2 == 9 (the region containing the origin is meant).
2135. Set lip the linlits of integration in the double integral

~ ~ f (x, y) dx dy
(S)

if the region S is defined by the inequalities


a) x;~~O; y~O; x-{-y~l;
d) y;-..~x; x~-l;
b) x2-t-y2~a2;
e) y~x~y-+ 2a;
c)

x 2 -t- !l 2 ~ x;

y~l;

0 ~ y ~ a.

Change the order of integration in the follo\ving double integrals:


"

J2,X

3x2

2136. ~ dx ~

f (x,

y) dy.

2137. ~ dx ~ f(x. y)dy.


Q

2X

Multiple and Line Integrals

250
a

2138. ~

Y a2 -x2
dx S f(x, y)dy.

a -x

Y 2ax-x2

2141.

~ dy

2142. ~dy

f (x, y) dy.

f (x, y) dx.

Y a _y2

~ f(x, y)dx.
y2
:&

Ym

2a

2140.

l-y

-Yl-y2

2a

2139. ~ dx

[Ch. '1

~dx

~ f(x, y)dy.
Y 2ax-X 4

~ d.x Sf

2143.

YR2_X 2

(x,

y) dy+

dx

R V2

f (x, y) dy.

sin x

2144. ~ dx ~ f (x, y) dy.


o

Evaluate the following double integrals:


2145.
xdx dy, where S is a triangle with vertices 0 (0, 0),

H
(5)

A(I, 1), and 8(0. I).


y

y
(0,2)

B(O'1)a...............~A(f,l)

e(G, 1)

A(2,O)X
Fig. 90

2146.

x
Fig. 91

Hxdx dy, where the region of integration S is bounded


(5)

by the straight line passing through the points A (2, 0), B (0, 2)
and by the arc of a circle with centre at the point C (0, 1), and

radius 1 (Fig. 90).

The Double Integral in Rectangular Coordinates

Sec. 1]

2147.

SSV :xd~

I'

a -x-y

251

where S is a part of a circle of radius

(5)

a with centre at 0 (0, 0) lying in the first quadrant.

2148.

HV

Xl -

yl

dx dy, where S is a triangle with vertices

(5)

0(0,0), A(I, -1), and 8(1, 1).


2149. HVxy-yldxdy, where S is a triangle with vertices

o (0,

(5)

0), A (10, 1), and 8 (1, 1).


x

2150.

HeYdxdy,

where S is a curvilinear triangle OAB bound-

(5)

ed by the

parabola y"=x and the straight lines x=O, y= 1


(Fig. 91).
2151. SS:Id~~, where S is a parabolic segment bounded by
(5)

the parabola y= ~ and the straight line y=x.


2152. Compute the integrals and draw the regions over which they
extend:
11:

+cos x

a) ~ dx

11:

S if sin xdy;

b)

Xl

sin l ydx.

"

S dx S y
o

S dy S
11

11
2

a cos y

C)

dy;

cosx

When solving Problems 2153 to 2157 it is abvisable to make


the drawings first.
2153. Evaluate the double integral

~ Sxy dxdy,
(S)

if S is a region bounded by the parabola yl = 2px and the straight


line x=p.
2154*. Evaluate the double integral

~ ~ xydxdy,
(S)

extended over the region S, which is bounded by the x-axis


and an upper sem icircle (x - 2)2 y" = 1.

252

Multiple and Line Integ rals

[ChI 7

2155. Evaluate the double integral


dxdy

SS Y2a-x'
(S)

where S is the area of a circle of radius a, which circle is tangent to the coordinate axes and lies in the first quadrant.
2156*. Evaluate the double integral

Hydxdy,
(S)

-where the region S is bounded by the ax is of a bsc issas and an


"arc of the cycloid

x=R (t -sin t),


y=R(l-cost).
2157. Evaluate the double integral

~ ~ xydxdy,
(S)

-in which the region of integration S is bounded by the coordinate axes and an arc of the astroid

x=R cos s t, y=R sinS t (O~ t~ ;).


2158. Find the mean value of the function

region

S{O~x~

1,

O~y~

f (x,

y)

== xyl in the

I}.

Hint. The ,nlean value of a function I (x, y) in the

7 = ~ SSf (x,

rC~IOl1

S is the nUIuber

y) dx dy.

(~)

2159. Find the mean value of the square of the distance of


a point M (x, y) of the circle (x-a)2+yl~R2 from the coordinate origin.
Sec. 2. Change of Variables in a Double Integral
to. Double integral in polar coordinates. In a double integral, when passing
fronI rectangular coordinates (x, y) to polar coordInates (r, cp), which are
connected with rectangular coordinates by the relations
x = r cos cp,
y = , sin tp,
we have the formula

Hf
~)

(x, y) dx dy=

~~
~)

(r

cos '1', r sin Ip) r dr dip,

(I)

Change of Variables in a Double Integral

Sec. 2]

253

If the region of integration (S) is bounded by the half-tines r

==Q

and

r =- ~ (a <~) and the curves , == r) (q and r = r2 (q, where '. (q


and
'2 (q [r 1 ({p) ~ r 2 (cp)] are single-valued functions on the interval a ~ rp ~ p,

then the double integral nlay be evalualed by the forrnula


f3

rJ

(CM

~ ~ F (cp. r) r dr dcp = ~ dcp ~ F (cp. r) r dr,

'1 (q

(S)

'2 (cp)

~ F(cp. rlrdr

where F(cp. r)=f(rcoscp. r sin cp). In evaluating the integral

'\ (cP)

we hold the quantity cp constant.


If the region of integration does not belong to one of the kinds that has
been exalnined, it is broken up into parts, each of which is a region of a
given type.
2. Double integral in curvilinear coordinates. In the J110re gener al case,
if in the double integral

~ ~ f (x.

y) dx dy

(~)

it is required to pass froln the variables x, y to the variables ll, (I, which
are connected with x, y by the contInuous and differentiable relationshlOs
x:...=. {f' (a, tI),

Y-=(ll, u)

that establ1sh a one-to-one (and, in both directions, continuou~) correspondence


behveen the points of the region S of the -,"y-plane and the points of SOllIe
region S' of the UV-plane, and if the Jacoblan

ax
I-==D(x, y)== JU
D (ct, v)
iJx

ay

au

ay

avav

retains a constant sign in the region S, then the fonnula

~ ~ f (x.

y) d.t dy;=

(~)

~ ~ f [rr (u,

v). '" (u. v) I III

d Il d v

(~')

holds true
The lilnits of the new integral are detef111incd fronl general rules on the
basis of the typc of region S'
Exarnple 1. In pas~ll1g to polar coordinates, evaluate

) ~ Vl-x

-1J 2 dx dy,

(S)

where the region S is a circle of radIUS R == 1 \vith centre at the coordinate


origin (Fig 92).
Solution. Puttin~ x-=r~oscp, y::..::rsincp, \v~ obtain:

l'1" l-x 2 - y 2 -=

VI -

(r cos cp)2 - (r sin tp)2 =

Vi -

(I

254

Multiple and Line Integrals

[eh. 7

Since the coordinate r in the region S varies from 0 to 1 for any cp, and cp
varies from 0 to 231, it follows that
21t

55 Yl-xZ-yZdxdy= 5 Sr Yl-rZdr=i n.
dq>

(5)

Pass to polar coordinates rand cp and set up the limits of


integration with respect to the new variables in the followin~
integrals:
1

2160. ~ dx ~ f (x, y) dy.


o

2162.

2161. ~ dx ~ f eVxz+yZ)dy.
o

Hf (x,

y) dx dy,

(5)

where S is a triangle bounded by the straight lines Y=X, y=-x


y=l.
1

2163.

Sdx Sf ( ~) dy.
x2

-I

2164.

Hf (x, y) dx dy, where S is bounded by


(s)

+ y2)2 = a2 (Xl _

{x 2

the lemniscatE

y2).

y
y

x
Fig. 92

Fig. 93

2165. Passing to polar coordinates, calculate the double inte


gral

~ ~ ydxdy,

(5)

where S is a semicircle of diameter a with centre at the poin


C ( J 0) (Fig. 93).

i-

Sec. 2]

255

Change of Variables in a Double Integral

2166. Passing to polar coordinates, evaluate the double inte

gral

~ ~ (Xl + yl) dxdy,


(5)

extended over a region bounded by the circle Xl + y. = 2ax.


2167. Passing to polar coordinates, evaluate the double integral
~ ~ Val-xl_yl dxdy,
(5)

where the region of integration S is a semicircle of radius a with


centre at the coordinate origin and lying above the x-axis.
2168. Evaluate the double integral of a function f(r, qJ) = r
over a region bounded by the cardioid r = a (1 + cos cp) and the
circle r = a. (This is a region that does not contain a pole.)
2169. Passing to polar coordinates, evaluate
11 a 2 -x2

~ dx

VXI+y1dy.

2170. Passing to polar coordinates, evaluate

~ ~ Val-xl_yldxdy,
(S)

where the region S is a loop of the lemniscate


(x 2 + yl)1 = at. (xt. - yl)
(x ~ 0).
2171*. Evaluate the double integral

f f Vl-~-~dXdY,

'CS)

Xl

y2

extended over the region S bounded by the ellipse aZ+/ii= 1 by


passing to generalized polar coordinates:

ax =

y.
r cos CPt b = r sin qJ.

2172**. Transform
c

f3x

~ dx ~ f (x, y) dy

(O<a<p

uv=y.

ax

and c>O) by introducing new variables u=x+Y,

256

Multiple and Line Integrals

[Ch. 7

2173*. Change the variables u=x+y, v=x-y in the integral


t

~ dx ~ f (x, y) dy.

2174**. Evaluate the double integral

~ ~ dxdy,
(S)

where S is a region bounded by the curve


X2
a2

_ y2

Y2)2 _
2
-

+b

h2

k2

Hint. Make the substitution


Y = br sin (p.

x == ar cos CPt

Sec. 3. Computing Areas


1. Area in rectangular coordinates. The area of a plane region S is

S=~~dXdY.
(S)

If the region S is defined by the inequalities a ~ x ~ b, cP (x)


then
b

"I' (x),

'\I> (x)

~ dx ~

S=

~y~

dy.

q> (x)

2. Area in polar coordinates. If a region S in polar coordinates rand q>


~ cp ~~, f (cp) ~ r ~ F (cp), then

is defined by the inequalities a

=~~

r dcp dr =

(S)

F (cp)

dcp
f

r dr.

(q

2175. Construct regions whose areas are expressed by the in-

tegrals
a)

~ dx ~ dy;
-]

Va 2 _y2

X+2

b) ~ dy

x2

~ dx.
a-lJ

Evaluate these areas and change the order of integration.


2176. Construct regions whose areas are expressed dy the integrals
n

arc tan

a)

dcp

sec cp

, dr;

Compute these areas.

b)

~ drp
n

a (J+coc;

cp)

r dr.

257

Conzpu tintZ Areas

Sec 8]

2177. Compute the area bounded by the straight lines x = YI


x=2y, x+v=a, x+3y=a(a>O).
.
2178. Conlpute the area lying above the x-axis and bounded
by this axis, the parabola y2=4ax, and the straight line x-!-y=3a.
2179*. Compute the area bounded by the ellipse
(Y_X)2

-~r =

1.

2180. Find the area bounded by the parabolas

y2 = lOx -J- 25 and y2

= -6x + 9.

2181. Passing to polar coordinates, find the area bounded by


t he lines
x2 _t y 2==2x, x2 -i- y 2=4x, y=x, y=O.

2182. Find the area bounded by the stra ight 1ine r cos rp = 1
a nd the circle r == 2. (The area is 110t to conta ina pole.)
2183. r:ind the area bounded by the curves
r = a (1

-~

cos (r) and r =-= a cos fl (a> 0).


l

2184. Find the arpa bounded by the line


y2 )
-+-4
9
X2

2 _

x2
----a
y2

-- 4

2185*. Find the area bounded by the ell ipse


(x- 2y t- 3)2

+ (3x -1- 4y- 1)2 =-== 100.

2186. Find the area of a curvilinear quadrangle bounded by


the arcs of the parabolas x 2=alj, x 2=by, y2=ax, y2=~X(O<

<a<b,

O<a<~).

Hint. Introduce the new variables u. and tI, and put


x 2= uy, y2 =-= vx.

2187. Find the area of a curvilinear quadrangle bounded by


the arcs of the curves !J2 = ax, y2 === bx, xy === a, xy = ~ (0 < a < b,
O<a<~).
Hint. Introduce lhe nc\v variables u and

XY=U,
9 -1900

tI,

y2=VX.

and put

Multiple and Line Integrals

258

[Ch. 7

Sec. 4. Ccmputing Volumes


Thp volume V of a cylindroid bounded above by a continuous surface
z = f (x, y), b 10\\' by the pie' ne 2 =- 0, and on the sidrs by a righ1 cylindrical
surface, which cuts out of the xl/-plane a region S (Fig. 94), is equal to
V=

~ ~

t(x, y) dx dy.

tS)

2188. Use a double integral to express the volume of a pyramid wilhvertices 0(0,0 0), A(I,O,O), 8(1, 1,0) and C(O,O, 1)
(Fig. 95). Set up the limits ot integration.

B (~I,O)

x
Fig. 94

Fig. 95

In Problems 2189 to 2192 sketch the solid whose volume is


expressed by the given double integral:
1

I-X

2189. ~dX~(l-x-Y)dY,
o

0
2-X

2190.

~ dx ~
o

V 1- x- 2

2191.~dX

~ (l-x)dy.

0
2

(4 - x - y) dy.

2192.

~ dx ~
0

(4 - x - y) dy.

2-X

2193. Sketch the solid whose volume is expressed by the ina

tegral ~ dx
o

l/(l'2.:Ki

Va! -r-y

dy; reason geometrically to find the

value of this integral.


2194. Find the volume of a solid bounded by the elliptical
paraboloid z=2x 2+y2+t, the plane x+y=l, and the coordinate planes.
21i:S. A solid is bounded by a hyperbolic paraboloid

z=xl_Jt

and the planes y=O, z=O, x= 1. Compute its volume.

Com'Ju,tin,! th~ Afea~

Sec. 5)

of

S'1.f faces

259

2196. A solid is bounded by the cylinder x 2 +-z~ =a3 and the


planes y=O, z==O, y=x. C)mpute its volume.
Find the voluln~s bounded by the followiilJ surfaces:
2197. az = y2, x 2 +- y2 == ,..2, Z = O.
2198. Y=Vx, y==2 J,tlx, x+z=6, z=O.
2199. z = r -r- y2. Y = x2, y = 1, Z = o.
2200. x+y-j-z=a, 3x+y=a, {x+-y=a, y=O, z=O.
x2

2201. 2a
2202. x 2

+ c2

Z2

= 1, y = -a x, y = 0, z =0.
+- y2 = 2ax, Z = ax, Z = px (a > Pl.

In Problems 2203 to 2211 use polar and generalized polar


coordinates.
2203. Find the entire volume enclosed between the cyl inder
2
2
Z
2
X + y2 = a and the hyperboloid x + y3 - z! = _ a .
2204. Find the entire volume contained between the cone
2(x 2 tif)-Z2=O and the hyperboloid x 2 +_y2_ ZJ. =_a 2
2205. Find the volunle bounded by the suriaces 2az = x 2 + y'l. t
Xl

-t- y 2 _ z2 = a2 z=o.
2206. Deterlnine the volume of the ellipsoid
x2

y'.!.

Z2

l2 -1- IJ2 -t- C2 = 1.

2207. Find the volume of a solid bounded by the paraboloid


== x 2 -+ y'l and the sphere x 2 .,- yl -t- Z2 = 3a 2 (The volume lying
inside the parclboloid is nleanL)
2208. Cornpute the volurne of a solid bounded by the xy-plane,
the cylinder x 2 -t y2==2a~t and the cone Xl +y2=Z2.
2209. C0l11pute thp VOlUI11e of a solid bounded by the xy-plane,
the surface z =- ae- (x:l-f l/~), and the cyl i.nder x 2 + y2 = R J
2210. Compute the volunlc of a solid bounded by the xy-plane,

,,\ 2
!l 2
x2
y2
x
the paraboloid z=-2-1-2' and the cyltnder 2+-b2=2-.
a
b
a
a
2211. In what ratio does the hyperboloid x 2 +y2_ z2=al
divide the volUIlle of the sphere x 2 -+ yl + Z2 ~ 3a2 ?
2212*. Find the volunle of a sol id bounded by the surfaces
z=x+y, xy= 1, xy=2, y=x, y=2x, z=O{x>O, y>O).

2az

Sec. 5. Computing the Areas of Surfaces


The area

of a srnooth single-valued surface z = f (x. y). whose projection


th~ region S, is equal to

on the xy-plane is

9*

Multiple and Line Integrals

260

2213. Find the area of that part of the plane ~

[Ch. 7

+ f+ f

which lies between the coordinate planes.


2214. Find the area of that part of the surface of the cylinder x 2 + y2 = R 2 (z ~ 0) \\'h ich lies bet\veen the planes z = mx and
z = nx (Tn> n > 0).

2215*. COOlpute the area of that part of the surface


cone x 2- y2 = Z2 \vhich is situated in the first octant
bounded by the plane y -1- z = a.
2216. Compute the area of that part of the surface
cylinder x 2 + if = ax which is cut out of it by the
x 2 -t- y2 + Z 2 =_ a 2
2217. Compute the area of that part of the surface
x2
11 2
sphere x 2 -+ y2 + Z2 = a 2 cut out by the surface l2 + fj2 = 1.

of the
and is
of the
sphere
of the

2218. Compute the area of that part of the surface of the


paraboloid y2+ z2=2ax which lies between the cylinder y2=ax
and the plane x = a.
2219. Compute the area of that part of the surface of the
cylinder x 2+ y2 = 2ax which lies between the xy-plane and the
cone x 2-~ y2 = Z2.
2220*. Compute the area of that part of the surface ot the
cone X 2_!J2=Z2 which lies inside the cylinder x 2 +y2::::::2ax.
2221*. Prove that the areas of the parts of the surfaces of the
paraboloids x 2 +y2='2az and x 2_ y 2=2az cut out by the cylinder x2 -1- y2 == R2 are of equivalent size.
2222*. A sphere of radius a is cut by two circular cylinders
whose base diarlleters are equal to the radius of the sphere and
which are tangent to each other along one of the dialneters of the
sphere. Find the volurne and the area of the surface of the re111a in i ng part of the sphere.
2223* An opening with square base whose side is equal
to a IS cut out of a sphere of radi us a. The ax is of the open iug
coincides with the diameter of the sphere. Find the area of the
surface of the sphere cut out by the opening"
2224*. C0111pute the area of that part of the hel icoid
z=carctan JL
\vhich lies in the first octant between the cylinx
2
ders x ..,.y2=a 2 and X 2+y2=b 2,

Sec. 6. Applications of the Double Integral in Mechanics


to. The mass and static moments ot a lamina. If S IS a region in an
xy.plane occupied by a lanlina, and Q (x, y) IS the surface density of the
laIn ina at the point (x, Y), then the mass M of the lamina and its static

Sec. 6]

of the Double Integral in Mechanics

Applications

261

moments Mx and My relative to the x- and y-axes are "rxpressed by the

double integrals

M=

~ ~

Q(x, y)dxdy, M X =

~ ~ YQ(x,

y)dxdy,

(~)

(S)

My= ~ ~ xQ

(x, y) dx dy.

(1)

(5)

If the lamina is homogeneous, then Q (x, y) = const.


2. The coordinates of the centre of gravity of a lamina. If C

centre of gravIty of a lamina, then


My

<x: Y>

is the

Mx

x=M' Y=M'

where M is the tnass of the lamina and At x' My are its static moments relative to the coordinate axes (see to). If the lamina is hornogeneous, then in
fonnulas (1) we can put Q = 1.
3. The moments of inertia of a lamina. The ITIOments of inertia of a
laluina relative to the x- and y-axes arc, respectively, equal to

1x=

~~

y2 Q(x, y) dx dy,

(5)

/ y=

~ ~

x Q (x, y) dx dy.

(2)

(5)

The l110ment of inertia of a lanlina rl'lative to the ongin is


10 =

~ ~

(X 2 +y2)Q(X, y)dxdY=/x+1y.

(3)

(.."

Putting Q (x, !/) -== 1 in fornlulas (2) and (3), \ve get the geolnetric moments of
lncrtJa of a plane figure.

2225. Find the nlass of a circular lanlina of radius R if the


density is proportional to the distance of a point fronl the centre
and is equal to () at the edge of the lalnina.
2226. A laIllina has the shape of a right triangle \vith legs
08 === a and OA == b, and its density at any point is equal to the
distance of the point froln the leg 0 A. Find the sta t ic IIl0I11ents
of the lamina relative to the legs OA and DB.
2227. COlnpute the coordillat{)s of the centre of gravity of the
area GlnAnO (Fig. 96), which is bounded by the curve !J= 5inx
and the straight 1ine OA that passes through the coord inate origin
and the vertex A ( ~, 1) of a sine curve.
2228. Find the coordinates of the centre of gravity of an area
bounded by the cardIoid r=a(l-t-cos(p).
2229. Find the coordinates of the centre of gravity of a circular sector of radius a with angle at the vertex 2a (Fig. 97).
2230. Compute the coordinates of the centre of gravity of an
area bounded by the paraholas 1}2 = 4x 1- 4 and y2 = - 2x +4.
2231. C01l1pute the rnoment of inert ia of a triangle bounLied
by the straight linesx+y=2, x=2, y.=2 relative to the x-axi~~.

[Ch. 7

Multiple and Line lnfe/!ra!s

262

2232. Find the monlent of inertia of an annulus with diameters d and D (d <D): a) relative to its centre, and b) relative to
its diameter.
2233. Cumpute the moment of inertia of a square with side a
relative to the axis. passing through its vertex perpendicularly to
the plCine of the square.
2234i:. C )mpute the moment of inertia of a segment cut ofT
the parabola y2 ==ax by the straight line x=a relative to the
straight line y=-a.

x
Fig. 96

Fig. 97

2235*. Compute the moment of inertia of an area bounded by


the hyperbola xy=4 and the straight line x-t-y==5 relative to
the straight line x=y.
2236;;. In a square la~in1 with side a, the density is proportional to the distance from one of its vertices. C)mpute the nlOment of inertia of the lamina relative to the side that passes
through tftis vertex.
2237. Find the moment of in~rtia of the cardioid r=a(l +cos<p)
relat ive to the pole.
2238. Compute the moment of inertia of the area of the lernn iscate (2.=.-:. 2a 2 cos 2(J) rela tive to the axis perpendicular to its
plane In the pole.
2239*. C3mpute the moment of inertia of a homogeneous lamina
bounded by one arc of the cycloid x=a(t-sint), y=a(l-cost)
and the x-aXIS, relative to the x-axis.

Sec. 7. Triple Integrals


to. Triple integrals in rectangular coordinates. The triple inte~ral of the
function !(x, y, ~) extended over the region V is the limit of the corresponding threefold iterated sum:

~ ~ ~ ,(x, ~',
V

z)dxdy dz=lim
m;JX

'Xi -+

max h")
max ~zk

0
-+ 0

-+ 0

~~~f(Xtl YJI
i

z")AXtAYJAz,,.

!jec. 7]

Triple

'nte.~,als

Evaluation of a triple Integral reduces to th~ succeSC;;lve comput'.Jt1on of th~


three ordinary (on~fold iterated) integrals or to the COinputatlon of one
double and one sin~le integral.
Example I. COlllpute
J = ~ ~ ~ X 3y 2 z dx dy dz,

where the region V is defined by the inpqualities


O~x~ 1, O~Y~x,
O~z~xy.
Solution. We have
1

1=

XII

Jt.

XII

j dx 5dy Jx3y2z dz 5dx 5xV f Idy=


=

..

xI'

=JdX.
o

X'II"

Xl 1/' xI

2- dY =J 25

\1. x 10
1
dX-=tj lod~=lll)'

Example 2. Evaluate

~~~

x d x dy dz,

(V)

t~xtel1drd

x2

over the volulne of the ellipsoid Q2 -t-

11
Z2
b
2 +- C2 =

1.

Solution.
a

~ ~ ~x

dx dy dz -=

a
2

x dx

-a

H')

~~
(5

\Ve therefore ti lla 11 y

SSSx

-V l-~.C -V l-~=nbc

Syz=nb

~el
2

x2 S yz dx,

-a

1/
x
lJ2
+ (2Z~ -= 1- Q2
'x=
2

\\ here S JlZ IS the area of the el1i pse

dy dz =

u.>

const, and is equal lo

l-~).

dxdydz=,nbc

Sx l-~) dx=~na3bC,
2

-a

(V)

2. Change of variables In a triple integral. If in th~ triple il1h"\~rat

~ ) ) f (x, y.

z) d.t d y d z

( /)

it is required to pass from th~ variables x, II, 2 to the variables u, v, w,


which arl~ conncct~'d with X,!I, z bv the rl-'I at ion~ x = q> (u. v, w), y = 'I' (u, v, w),
z =X (u, v, w), whlr~ the functlon'\ q>. '1'. X ar~:
1) continuous together with their pal tial fir~t derivatives;
2) in one-to-one (lnd. In both direLtiolls. contil1uJu;;) correc;;pondence bet\veen the poi.,ts of the region of integratlon V 01 Xyz-spdce and the pOints of
sOlne region V' of UVW -space;

264

Multlple and Line Integrals

[ell. 7

3) the functional deternlinant (Jacobian) of these functions

ax

au
/=

D (x, y,!l.=
D (u, v, w)

iJx

ax

iJu ath

ay ay

iJy

au au

dw

au av iJw
az az az

retains a constnnt sign in the regIon V, then \\'e can lnake use of the formula

~ ~ ~ f (x, y,

z) dx dy dz =

(V)

~ ~ ~ f Irr (u,

v, w), (u, v, w), X(u, v, w))

III du dv dw.

(\I')

o~

__-+

h
_

x
Fig. 98

fIg. 99

I n part Icular,
1) for cylindrical coordinates r, cp, Iz (FIg. 98), where
x == r cos (P, y
sin (P, z ::::- II ,
we get I -= r;
2) for spherical coordinates cp, ,~, r (cp is the longitude,
r the radius vector) (FIg. 99), \vhere

=,

x = r cos cos q>,

y= r cos 'P sin q>,

'p

the latitude,

= r sin ~"

we have 1::-:: r cos 'i'.


Example 3. Passing to spherical coordinates, compute
2

~ ~ ~ yX 2 +y2+ Z2dxdydz,
(V)

\\there V is a sphere of radius R.


Solution. For a sphere, the ranges of the spherical coordinates (p (longItude), 'I' (latitude), and r (radius vector) will be

SPC

7J

Triple Integrals

265

We therefore have
n
2";

~~ ~

y x2-+!l -I- 22 dx dy dz =

H')

~ drp ~ d1Jl ~
n

r r 2 cos 1Jl dr=1tR",

3. Applications of triple integrals. The volU1ne of a region of three-dimensiona I xyz-space is

V=~~~dXdYdZ.
(\/)

The

1!laSS

of a solid occupying the region V is

Mc-~ ~ ~ ~

y(\'. Y. z) dxdydz.

(V)

where y (x, !/, z) is the density of the body at the pOint (x,!I, z).
The static l1Z011zenls of the body relative to the coordinate planes are

M.w ~=

~ ~ ~ ,. (x. Y. z) z dx dy dz;

Ml'z=

~~ ~

(V)

,,(x. Y. z):.dxdydz;

0')

Mz.\"=

~ ~ ~ y(x. y.

z) ydxdydz.

(V)

The coordinates of tire celltre of gravlty are


i\lyz A'zx -

x="M-'

A-1 XI'

!I:::-:::~, z=~.

If the solid is homogeneous, then \ve can put y (x, y, z) == 1 in the fornlulas for the coor(ltnates of the centre of grav1ty.
The l1Z011Zettts of inertia relative to the coordinate axes are
I x=

~ ~ ~ (y2+ Z2) Y (x. Y.

z) dx dy dz;

(V)

ly=

~ ~ ~ (Z2+ X2) Y(x. Y. z)dxdydz;

lz=

~ ~ ~ (xt+yt)y(x. y. z)dxdy dz.

(\')

(\')

Putting 'V (x. y, z)= 1 in these fornlulas. \\'e get the geoilletric Jnomen{s
of inertia of the body.

A. Evaluat ing triple integrals


Set up the linlits of integration in the triple integral

~ ~ ~ f (x.
(V)

for the indicated rl'giolls V.

y. z) dx dy d z

266

Multinle arrd Lire Integrals

(Ch. 7

2240. V is a tetrahedron bounded by the planes

x+y+z= 1,

z=O.
2241. V is a cylinder bounded by the surfaces
x 2 +y" =R 2 , z=O, z=H.
2242*. V is a cone bounded by the surfaces
x=O,

,,2

x2
-2

y=O,

Z2

+ b = 2"'
2

= c.

2243. V is a volume bounded by the surfaces


z = 1- x2- y2, Z = o.

Compute the following integrals:


I

2244.

r dx rdy r

dz

J0 0
J 0JVx+y+z+l
.. f

4X-I/2

V-22245. ~ dx ~ dy
xdz.
~
2

"V x

VQi":X2

2246.

Sdx

.\

V-U2-_-X'l-_~y3

o
I

2247.

I-X

dz

j'

dy

0
I-X-II

~ dx ~ dy ~

xyzdz.

2248. Evaluate

555

d"ldydz

(x

-t- II + Z + 1)3 t

(\I)

\\lhere V is the region ot Integration bounded by the coordinate


planes and the plane x+y+z=l.
2249. Evaluate

~ ~ ~ (x + Y + Z)I dx dy dz,
(V)

\\,here V (the region or integration) is the common part of the


palabolold 2a,~x2+y2 and the sphere x'+y2+z2~3a2.
2250. Evaluate

where V (region ~ integration) IS the common part of the


spheres x2+ y2 t- Z2 ~ R~ and x 2 + y2 + Z2 ~ 2Rz

Triple Integrals

Sec. 7]

267

2251. Evaluate

~H zdxdydz,
n')

where V is a volume bounded by the plane z = 0 and the upper


.
.
x2
11 2
Z2
half of the ellIpsoId -2-t':-b
= 1.
2 +-2
II
C
2252. Evaluate

rs (i2 +

SJ

Z2 )
-f- C2
dx dy dz,

y2

{j2

(V)

"There l! is the interior of the ell ipsoid

x: + b11: + 2:c =

1.

2253. Evaluate

~ ~ ~ z dx dy dz,
n')

\vhere V (the region of integration) is bounded by the cone


1z2

z! == R~ (x 2 t- y2) and the plane z == h.


2254. Passing to cylindrical coordinates, evaluate

~~~ dxdydz,
(\ ')

\vhcrc V is a region bounded by the surfaces x:! -\- y2 ix 2 -I y2 -=-= Z2 and contaIning the point (0,0, R).
2~55. Evaluate
1":! \. - x'

~ tlx

~ dy ~ z V x 2 + y2 tiz,

OliO

first transfortning it to cylindrical coordinates.


2256. Evaluate
lr 21x -

2r

dx

A .I

4f~J_y~

dy

-V~

dz,

first tra nsformi ng it to cy Ii ndrical coordi nates.


2257. Evaluate
R

~ dx
-R

V R2 -

V R.I- x 2

dy

x.l -

y2

(Xl -I- yl) dz.

-VR.I-x2

first transforming it to spher ical coordinates.

Z2

= 2Rz.

268

[Ch. ~

Multiple and Line Integrals

2258. Passing to spherical coordinates, evaluate the integral

~ ~ ~ V Xl +y2 + Zl dx dy dz,
(V)

where V is the interior of the sphere x2 -I- y2 -1- Z2

x.

B. Computing voluflles by means of triple integrals


2259. Use a triple integral to compute the volume of a solid
bounded by the surfaces
y2=4a2 -3ax, y" = ax, z==h.
2260**. Compute the volume of that part of the cylinder
x 2 + y2 = 2ax which is contained between the paraboloid r + y2 = 2az
and the xy-plane.
2261*. Compute the volume of a solid bounded by the sphere
x 2 + y2 + Z2 = a2 and the cone Z2:.-= x2 + y~ (external to the cone).
2262"'. Compute the volume of a solid bounded by the sphere
2
2
X +y2+ Z 2=4 and the paraboloid X +y2=3z (internal to the
paraboloid).
2263. Compute the volutne of a solid bounded by the xy-plane,
the cyli nder x 2 -~- y2 = ax a nd the sphere x 2 + y2 -t- Z2 == a2 (i nterna 1
to the cylinder).
2264. Compute the volume of a solid bounded by the paraboloid
y2

z~

b!+ CZ = 2 a and the plane x=a.

Z~
C

C. Applications of triple integrals

to mechanics and physics

x
Fig. 100

2265. Find the rnass M of a rectangular parallelepiped 0 ~ x ~ a,


O~y~b, O::=;;z~c, if the denv
sity at the point (x, y, z) is
(l(x, y, z)=x-~y-~-z.
2266. Out of an octant of the
sphere x 2 + y2 -I- Z2 ~ c2 , X ~~ 0,
y~O, z~O cut a solid OABC
bounded by the coordinate planes
1 (a~c, b ~ c) (Fig. 100). Find the mass

and the plane : + : =


of this body if the density at each point (x, y, z) is equal to
the zcoordi nate of the poi nt.
2267*. In a solid which has the shape of a hemisphere
2
Xl + y2
Z2 ~ a t Z ~ 0, the density varies in proportion to the

Sec. 8]

Improper Integrals Dependent on a Paranleler

269

distance of the point from the centre. Find the centre of gravit~'
of the soli d.
2268. Find the centre of gravity of a solid bounded by the
paraboloid y2 +2z 2 == 4x and the plane x= 2.
2269*. Find the nlonlent of inertia of a circular cylinder,
\vhosc altitude is h and the radius of the base is a, relative to
the axis which serves as the diameter of the base of the cylinder..
2270*. Find the mOI1lcnt of inertia of a circular con~
(altitude, Iz, radius of basc, a, and density Q) relative to
the diaIllcter of the base.
2271**. rind the forcc of attraction exerted by a homogeneous
cone of altitude It and vertex angle t (in axial cross-section) on
a l11aterlal point containing unit Inas~ and located at its vertex.
2272**. Sho\v that the force of attraction exerted by a honl0geneous sphere on an external nlaterial point does not change if
the entire I1laSS of the sphere is concentrated at its centre.
Sec. 8. Improper Integrals Dependent on a Parameter.
Improper Multiple Integrals
1. Difl'erentiation with respect to a parameter. In the case of certain
f('stnctlons iJllpOscd on thp functIons f (x. a), f~ (x, a) and on the correspondIn~ illlpro!Jcr Integrals \\TC have the Lelb111Z rule
00

r:r:

d
da

JCf (x,

et) dx ==

II

Jr fa' (X,

a) dx.

'I

Example 1. By diff<?rentiatinl! \vith respect to a paranletcr, evatuate


00
t

e-l}x2 _e-;,o'\:l
- - - - - dx

(a

> 0,

> 0).

Solution. Let
00

~ - - - - dx==F(a,
e-l'Jo~1 _e-~xl

~).

Then

00

aF (a,

aa

Whence F(a,

t\) = _

r xe-'lX I dx= 2a-!- eJ

lJx1

! :::z

_J.-.
2a

~)==- ~ Ina+C(~). To find C(~). we put a=~ in the latter

equation. We have 0= -

In

~ + C (~).

Whence C (P) ="2 tn~. Hence,

F {a, ~):.=

-2 111 a+2"ln p=2"ln

~
a.

270

[Ch. 7

Multiple and Line Integrals

20 Improper double and triple integrals.

f (x,

a) An infinite region. If a function


region S, then we put

Hf

(x, y) dx dy= 11m

(S)

(J

y) is continuous in an unbounded

Hf

(x, y) dx dy,

(1)

-'s(O)

where (J is a finite region lying entirely within S, where 0 -+ S signifies that


we expand the region 0 by an arbItrary law so that any roint of S should
enter it and rel11ain in it. If there is a limit on thp right and if it does not
depend on the choice of the rpl!ion 0, then the corresponding itnproper integral IS called cOllverR,eHl, othcr\vise it is dIvergent.
If the inh'gratHI !(x, y) i~ nonnegative l{(x, y)~Ol, then for the con
ver~ence of an t1l1rJop~r integral it IS nt'ce~sary and sufficient for the limit
on the right of (I) to exist at least for one system of regions a that exhaust
the region S.
b) A di\continuous function. If a function f (x, y) is every\\'here continuous in a bounded closed region S, excppt the point P (a, b), then we put

Hf

(x, y) dx dy= lim


e -+

(S)

Hf
0 (5

(x, y) dx dy,

(2)

)
E

where Se is a region obtained from S by elinl1nating a snlall region of dla


metfr E ihat contaIns thp roint P. If (2) has a limit that dops not depend
on the tYre of snlall r(3gions elimira t (3d from 5, the in1rroper integral under
cons\deration is called (onverr..enl, oth( rWlse it is divergent.
If f (x, y)~O, UHn the limit on the ri~ht of (2) is not dependent on the
type of regions elitninated from S; for instance, such reg:ons may be circles
of radius

with ccntrp at P.

The concept of improper double integrals is readtly extended to the case


of trirle integrals.
Example 2. Test for convergence
(3)

where S is the entire xy-plane.


SotuHon. Let a be a circle of radius Q with centre at the coordinate
origi n. Passing to ~olar coordi nates for p :1= 1, we have

1(0)=

55
(~

21t

(1

5 ~d;l)p
=5

+~~ yElP = Sd<p


0

(l

21t

If p < 1, then Urn 1 (0)= lim


o

then

-to S

lim I (0) = ~l

Q-+fD

p-

2 P
1
2 (1+r )I- IO
I-p
0

1(0)= 00

dcp=~ [(1+Q2)I-P_I].
I-p

and the integral diverges. But if p> 1,

~ e:t:

and

the

integral

converges.

For

p = 1 we have

Improper Integrals Dependent on a Parameter

Sec. 8]

1 (0)= ISn dlpSQ I'd' 2=rtln(1+Q2);


o

lim 1(0)=0),

+r

O~CI)

diverges.
Thus, the integral (3) converges for p

2273. Find

>

that

is,

271

the

integrat

1.

I' (x), if
Ql)

f (x) = ~ e- xy' dg

(x

> 0).

2274. Prove that the function


+~

LJ

C xf (z)
= J x 2 + (y-zr' dz
-00

satisfies the Laplace equation


iJ2 u
iJx2

iJ!ll

+ d!J2 = o.

2275. The La place transfornlatlon F (p) for the function


is defined by the fornlula

f (t)

c)CI

F (p) = ~ e-t.tf (t) dt.


u

Find

d)

F (p),

f (I) == cos ~t.

if:

f (I) =

a)

1:

b)

t (t) = eat;

2276. l"aking advantage of the rornlula


1

5xn-'dx

~ (n > 0),

compute the integral


1

~ xn -'1nxdx.
I)

2277*. Using the formu la


Ql)

5eo

eval uate the integral

pt

dt

(P > 0),

c)

f (t) =

sin ~t;

272

M uLUpie and Line Integrals

rCh. 7

Applying differentiation \vith respect to a parameter) evaluate

the following integrals:


00

2278.

Se-'

;e-;i dx (u

> O.

> 0).

o
eX)

2279.

Se-,x-;e-;;x sin mxdx (u > O.

>

0).

o
eX)

2280.

C arc tan ax d

+x

x (1

x.

2)

o
1

2281. Sln(I~)dX
2
x YI-x 2

(/u/< 1).

o
00

2282. ~e-ax Si:~X dx (u~ 0).


o

Evaluate the following improper integrals:


ex>

cr.,

2283. ~ dx ~
o

y2

2284.

dy.

~ dy ~ eu dx.

2285.

e-(HIJ)

Jrrj 4dxdy
_j- y2 , \vhere S is a region defined by the inequali(5)

x~

ties

Y~X2.

1,
00

00

Cdx S {x 2 +y2_I_a
dr 2 )2 (a>O).
2286*. J
2287. The

Euler-Poisson integral defined by the forl11ula


00

00

J= ~

e- x'

dx may also be written in the form I = ~ e- Y ' dy. Eval-

uate / by multiplying these formulas and then passing to polar


coordinates.
2288. Evaluate
00

00

00

Sdx ~ dy ~ (x2 + y2 ~ Z2 + 1)2


o

273

Line Integrals

SefJ. 9]

Test for convergence the improper double integrals:


2289**. ~ ~ In V Xl -+ y2 dxdy, where S is a circle x 2 + y2 ~ 1.
(S)

2290.

55 (X~~d:2)'1.'

where S is a region defined by the ine

(5)

quality x 2 -1 y2

1 ("exterior" of the circle).

dx dll

where S is a square Ix!:=;;;; 1, Iyl:=;;;;l.


5
2292. 555 (X2::'Y:~2)'1. , where V is a region defined by the
JV(X~y)21

2291*.

(S)

0')

inequality x 2 -t- y2 -1-

Z2 ~

1 ("exterior" of a sphere).

Sec. 9. Li ne Integrals
1. Line integrals of the first type. Let f (x, y) be a continuous function
CJnd y = cp (x) [a ~ \ ~ b) be the equation of some snlooth curve C.
Let us construct a system of points 1\1 , (xi, y,) (i == 0, 1, 2, ... , n) that
break up the curve C into elenlentary arcs A1 ,':-;M i == ~si and let us fornl the
Il

integral

SUln

Sn -== ~ f (xi, y,) fls ,. The ltmit of this sunl, \vhen n ~

00

and

l-I

lliax

f1s i ~ 0,

IS c~lled

a ILne I1ltegral of the fIrst type


Il

lim

{(xi. Yi) !'lsi =

(ds

IS

~ f (x,

y) ds

1l"\X;i=1

the arc differential) and is evaluated fronl the fonnula


b

~ f (x,

y) ds =

{(x,

er (x Y 1 + (cp' (xW dx.

In the case of paranletric representatIon of the curve C: x==<p (t),


Y --= 'l' (t) [a ~ t ~~], \ve have

~ f (x,
C

Ii

y) ds =

~ f (cp (t), ~, (t Y <p'l (t) + 1\J'1 (t) dt.


a

Also considered are line integrals of the first type of functions of three
variables f (x, y, z) taken along a space curve. These integrals are evaluated
in li'<e fashion A line integral of the first type does not depend on the direction of the path of il1terzratton; if the integrand f is interpreted as a linear
density of the curve of integration C, then this integral represents the mass
of the curve C.

274

Multiple and Line Integrals

(Ch. 1

Example 1. Evaluate the line integral

~ (x+ y) ds,

where C is the contour of the triangle ABO wi th vertices A (1, 0), B (0, 1),
and 0 (0, 0) (Fig 101).
Solution. Here, the equation AB is y= I-x, the equation DB is x=O,
and the equation OA IS Y = O. We therefore have

(x+y)ds=

~ (x+y)ds+ ~ (x+y)ds+ ~ (x+y)ds=


AB

oA

80
I

=) Y2dX+) ydY+) xdx= Y2 + I.


o

2. Line Integrals of the second type. If P (x, y) and Q (x y) are continuous functions and y = (f) (,\) 1S a sInoa th curve C that runs from a to b as
t

y
B

Fig. 101

x varies. then the corresponding ltne tntegral of the second type is expressed
as follows:

) P (x, If) dx
C

+ Q (x,

y) dy = ) (P (x, q> (x + q>' (x) Q (x, q> (x1 dx.


a

In the more genera) case when the curve C is represented parametrically:


%=<p(t), y=lj)(t), where t varies (roln a to p, we have
p
~ p (x, y) dx + Q (x, y) dy + ) (P (q> (t), (t)) q>' (t) + Q (q> (t), (t ' (t tIt.
C

Similar formulas hold for a line integral of the second type taken over a
space curve.
A line Integral of the second type clzanaes stgn when the dtreclton of the
path of tl1teg ratton lS reversed. This integral may he interpreted nlE.'chanically
8S the work of an appropriate variable force {P {x, y), Q (.x, y)} along the
curve of integrdtton C
Example 2. Evaluate the line integral

~ y 2 dx+x 2 dy,
C

275

Line III feR! als

Sec. 9)

where C is the upper half of the ellipse x=a cos t,


clockwise.
Solution. We have

y=b

sin t traversed

y2 dx

+ x 2dy =

{b 2 sin 2 t ( - a sin t)

+a

cos 2 t b cos t] dt =

= -abo

sill' t dt

+ a'b Scos' t dt = ~

ab".
n
n
3. The case of a total dift'erentfal. If the inte~rand of a line integral
of the second type is a total differential of some sIngle-valued function
U ~ U (x, y). that is, P (x, y) dx+ Q (x. y) dy-=dU (x. y). then this line integral
IS not dependent on the path of integration and we have the Newton-Leibn iz
formula
(xz. Y2)

+ Q (x,

P (x, y) dx

y) dy == U (x 2 , Yl) -U (XI' YI)'

(1)

(XI' YI)

\vhere (XI' YI) IS the initial and (x 2 y~) is the terminal point of the path
In particular, if the contour of inte~ratioll C IS closed, then

~p

(x. y) dx -\- Q (x. y) dy

=0

(2)

If ]) the contour of inteL!r:ltion C ic; cont:lined entirely within some


simnlv-connected re!iO,l ~ Clnd 2) the functions P (r. l/) and Q (x, y) totfether
\\'Ith th . .' ir partial derlv~ttivcs af the first areier are continuous in S, then a
neCCS"iarv and sufficiel1t co Hhtio 1 fa, tho? exi~tellce of the function U is the
i,lenticai fulJilrllent (In S) of the equality
iJQ uP
(3)
ax =dy

(see

jnte~ration

of total difrl"rentials)

If conditions one and hvo are not ruldocs not guarantee the existence of a
sll1l!le-vall1(,o 'unction U, and fOrlnulas ~ 1) and (2) may prove wrong (s('e
Problenl 2.t 2) v.,'e give a 111ethod of Bnding a function U (x, 1/) lronl its
total diflcrcn1ial based on the lise of line integrals (\vhich is yet ~1t-:other
method of inte~ratil1g a total differential). For the contour of Integration C
let us take a brok('11 11l1e P(,P I A1 (FIt! }(2). where p(\ (X(I' Yo) is a fixed f oint
and M (x, y) IS a variable pOI nt. Then a long PoP I \ve have y =. Yo and dy -= 0,
and along P 1"'1 \ve have dx:.::;: 0 We get:
JiUed, the." prcscllcc of conditton (3)

(x. y)

U (x. y)- U (x o' Yo) =-=

p (x, y) dx + Q (x, y) dy =

(X Ut Yo)

~ p (x.

Yo) dx+

Yo

Xo

SimIlarly, integrating with respect to PtP"M, we have


y

U (x. y)-u (.Ko Yo) = ~ Q (x o' y) dy + ~ p (x. y) dx.


Uo

~ Q (x.

%0

y) dg.

276

Multiple and Line Integrals

[CIl.7

Example 3. (4x+2y) dx+ (2x-6y) dy=dU. Find U.


Sol ution. Let Xo = 0, Yo = O. Then
1/

V (x, y) =

S4x dx+ S (2x-6y) dy+C=2 x2 +2xy-3y2+C


o

or

V (x, 1/)= S -6ydy+


o

S(4x+2y) dx+C=-3 y 2+2.t 2+3xy+C,


0

where C ==- U (0, 0) is an arbitrary constant.

M(x,y)

,Po (;xo, Yo) I P, (x, {Jo)

llo

Xo

Fig. 102
4. Green's formula for a plane. If C is the boundary of a region Sand
the functions P (x, y) and Q (x, y) are continuous together with their firstorder partial derivatives in the closed region S-t-C, then Green's fOrf1lula holds:

1P dx+Q dy= 55 (~~ -:) dxdy,


C

(5)

here tl'e circulation about the contour C is chosen so that the region S should
remain to the left.
5. Applications of line integrals. J ) An area bounded by the closed contour C is

S=

- ydx= xdy
c

(the direction of circulation of the contour is chosen counterclockwise).


The fo lowing formula for area is more convenient for application:

S=

~ (xdy-ydx)= ~
C

(~).

2) The work of a force, having projec!ionsX=X(x, y, 2), Y=Y(x, y, Z),


y, z) (or, accordingly, the work of a force field). along a path C is

z= Z (x,

Sec. 91

277

Line Integrals

expressed by the integral


A=

X dx

+Y dy +Z dz.

If the force has a potential, i.e., if there exists a function U ==U (x,
(a potential functIon or a force function) such that
iJU

-=)(,
ax

ij, z)

au

iJU
-=-=Y, -=--=z,
iJy
dz

then the wort" irrespective of the shape of the path C, is equal to


(X J II'!"

A=

(X,:. th.

Z,:)

Xdx-j-Ydy-!-Zdz=
zd

(\"1. Vi'

where (\"1' Yl'

~
(t 1 , 111'

Zl)

is the Initial ano (x 2, Y'2.'

z.d

dU=U(X 2'Y2' Z2)-U(X"y. ".),


zd
Z2)

is

th~

tefl111nal point of the path.

i\. Lille Integrals of the First Type


Evaluate the follo\ving line integrals:
2293. ~ xy ds, where C is the contour of the square I x I + I y I = a
c

(a

> 0).

2294. \ .r -2 ds ;;-~-, where C is a segment of the straight line

cr

i- y

t-

,t

connecting the points 0 (0, 0) and A (1, 2).


2 II
r
x
2295. J xy ds, \\'here C is a qua rter of the ellipse i2 + ~ = 1
c

1ying in the first quadrant.


2296. ~ y'ds, where C is the first arc of the cycloid x = a (t - sin t).
c

y = a (1 - C()~ f).
2297. ~ V-X2-~-'_-y-2 lis, \vhere C is an arc of the involute of the
c

circle x::-:-a(eost i-tsint), !J=a(slnl-tcost) \0~t~21t].


2298. ) (x 2 + y2)2 ds, where C is an arc of the logarithmic spi
c
ral r ~ ae l1lrp (Ill> 0) froln the point A (0, a) to the point 0 (- 00, 0).
2299. ) (x +y) ds, where C is the right-hand loop of the lemc

niscate r" = a" cos 2<p.


2300. ) (x 1- y) ds, where C is an arc of the curve x = t.
c
Y = ; ; , z = t ' (0 ~ t ~ 1].

Multiple and Line Integrals

~78

2301.
X

[eh. 7

~c xI +d~
+ zI ' where C is the first turn of the screw-line
y

= a cos t, Y = a s j nt,

bt .
2302. ~ -V 2g -1- Zl ds, where C is the circle Xl + gl +Zl = a"
Z=

x=y.
2303*. Find the area of the lateral surface of the para bolic
cylinder g=iX- bounded by the planes z=O, x=O, z=x, y=6.
2304. Find the arc length of the conic screw-line C x=ae t cost,
y = ae l sin t, z = ae t from the point 0 (0, 0, 0) to the point A (a, 0, a).
2305. Deterlnine the Blass of the contour of the ellipse
~~ = I, if the linear density 01 it at each point M (~, y) is

:: +

equal to

'y ,.

2306. find the mass of the first turn of the screw-line x = a cos I,

y=asint, z=bt) if the density at each point is equal to the


radi us vector of this point.
2307. Determine the coordinates of the centre of gravity of
a half-arc of the cycloid
x=a(t-sint), y=a(l-cost)

[O~t~jtl.

2308. Find the moment of inertia, about the z-axis, of the


first 1urn of the screw-l ine x ==- a cos t, Y =- a sin t, Z = bt.
2309. With what force will a mass M distributed with uniform density over the circle x 2 -+ y2 == a 2 , Z == 0, act on a Blass nl
located at the point A (0, 0, b)?

B. Line Integrals of the Second Type


Evaluate the follo\ving I ine integrals:
2310. ~ (x l -2xg)dx+ (2xy+y2)dy, where AB is an arc of the
AB

parabola Y= x 2 from the point A (1, 1) to the point B (2, 4).


2311. ~ (2a- y) dx \. x dy, where C is an arc of the first
c

arch of the cycloid


x=a(t-sint), y=a(l-cost)

which arc runs in the direction of increasing parameter t.


2312. ~ 2xgdx-x!dy taken along different paths emanating
VA

from the coordinate origin 0 (0, 0) and terminating at the point


A (2, 1) (Fig .l 03):
a) the straight line OmA;

Lin~

Sec. 9]

I nte~ralf;

2i9

b) the parabola OnA, the axis of symnletry of which is the


y-axis;
c) the parabola OpA, the axis of symmetry of which IS the
x-axis;
d) the broken line OBA;
e) the broken line OCA.

2313. ~ 2xydx +-x'dy as in Problem 2312.


OA

2314*. th (x+ u) dX-(X-L/)


dy
2

:Y
counterclocl{wise.

taken alonQ the circle x 2

y2

.....

y2

==al

cl~ 1) t-----II-~ A (2, 1)

B(2,O)
Fig. 103

2315. ~ y'dx+x'd!l, where C is the upper half of the ellipse


r:

x ~ a cos t, Y = b sin t traced clocl<wise.

~ cosy dx- sin xdy taken along the segment AB of the

2316.

AH

bi~ector

of the second quadrantal angle,

point A is 2 and the ordinate of B

#X!l(llt~~~:

IS

if the abscIssa at the

2.

where C is the right-hand loop 01 the


lenlnlscate ,z =- 0 COS 2(p traced counterclocl<wise.
2318. Evaluate the line integrals \vith respect to expressions.
which are total ditTerentials:
2317.

dIll,

(3, oi)

(2, 3)

a)

xdy+ ydx,

b)

)
(0, I)

( - l~ 2)

(I. I)

xdx+ydy, c) ) (x+y)(dx+dy),
(0, 0)

(2, 1)

d)

J,-

(I, 2)

x-ax is),

y dx- x dy

y2

(along a path that does not intersect the

Multiple and Line Integrals

~80

[CIL.7

(x, y)

e)

dx-t-dy (along a path that does not intersect the


x+y

(~.+)
straight line

x-~y=O),

(x 2 Y2)

f)

cp (x) dx -\- 'i' (y) dy.

(x", Yl)

2319. Find the antiderivative functions of the integrands and


evaluate the integrals:
(a, 0)

a)

(x t

+ 4xy 8) dx-+ (6X 8y l_5y t) dy,

(-2, -1)
(I, 0)

b)

x ~~=~)~x (the integration path does not intersect the

(0, - I )

straight line y = x),


(a, I)

c)

S (x+~;~;)tYdY (the integration

path does not intersect

(I, I)

the straight line y=-x),


(1,1)

d)

S (v x2x + y2 + Y) dx+ (-V x2Y+ y2 -I- x) dYe

(0, 0)

2320.

Co~pute

Xl

y2

taken clockwise along the quarter of the ellipse 02+ fi!= 1 that
1ies in the first quadrant.
2321. Show that if f (u) is a continuous function and C is a
'Closed piecewise-smooth contour, then

1c f (Xl + yl) (x dx+ y dy) = O.


2322. Find the antiderivative function U if:
a) du=(2x+3y)dx+(3x--4y)dy;
b) du = (3x l -2xy y2) dx _(Xl -2xy+ 3y2) dy;
c) du = eX - Y [(1 + x -t. y) dx (l-x-y) dy];

d) du= ~

!!JL.
x+y+x+y

Line Integrals

Sec 9)

281

Evaluate the line integrals taken along the following space


curves:
2323. ~ (y-z)dx+(z-x)dy+(x-y)dz, where C is a tum
c

of the screw-l ine

= a C?S i,
f xy=a
slnt,
l z bt,
=

corresponding to the variation of the parameter t froln 0 to 2n..


2324.
Y dx +- z dy +- x dz, where C is the ci rc1e

p
c

x = J? cos a cos t ,

J Y = R cos a sin i,

l z = R sin a. (a.

= const),

traced in the direction of increasing parameter.


2325. ~ xydx+yzdy+zxdz, where OA is an arc of the
0.4

circle

x 2 -1- y2 --1-

Z2 =

2Rx,

= x,

situated on the side of the xz-plane \vhere !I> O.


2326. Evaluate the line integrals of the total differentials:
(6, .1, 8)

a)

xdx-'r-ydy-zdz.

(1,0, -3)
(cr, b. c)

b)

yzdx+zxdy-i-xydz,

(1, 1, 1)

(:s, -I, s)

c)

x cl~ I_-_~~ ,
Y .\2 _1_ y2 -1- Z2

(0, 0, 0)

( x.

d)

1/,

2..)

Xli

liZ dx + Zt dr/-I- xy dz (the

xyz

integration pa th is si tun teo

(1. 1,1)

i 11 the fi rs t oc tan t).

C. Green's Fornlula
2327. Using Green's forlnula, transfOrlTI the line integral
l=p1lx2 -1 y 2dx+y[xy+ln(x-+ VX 2+y2)] dy,
C

where the contour C bounds the region S.

282

[Ch.7

Multiple and Line Infellfals

2328. Applying Green's formula, evaluate

J=

ffc 2 (r -t- if) dx + (x +y)Z dy,

\vhere C is the contour of a triangle (traced in the positive direction) with vert ices at the points A (1, 1), B (2, 2) and C (1, 3).
Verify the re~ult ob1ained by conlputin~ the inteeral directly.
2329. Applying Green's forlTIula, evaluate the inLcgral

ffe _xZy dx + xyz dy,


where C is the circle x 2 + y" = R 2 traced counterclockwise.
2330. A parabola AmB, whose aXIs is the y-axIs and whose
chord is AnB, is drawn through the points A (1, 0) and B (2,3).
Find
(x + y) dx-(x-y) dy directly and by applying Green's

ff

AmBnA

fornlula.
2331. Find

~ eXY[yZdx 1-(1 +xy)dy], if the points A and B


AmB

lie on the x-axis, while the area, bounded by the integration


path AnlB and the segrr~ent AB, is equal to S.
2332*. Evaluate tI; x dl~-!l2d!. Consider t\VO cases:
+b
a) when the origin is outside the contour C,
b) when the contour enrircl(\s the origin n times.
2333**. Show that If C is a closed curve, then

l ).

ffc cos (X,

n)ds=O,

-where s is the arc length and n is the outer norl11al.


2334. Applying Green's formula, find the value of the integral
I =

ffe [x cos (X, n) +- y sin (X,

n)] ds,

\vhere ds is the differential of the arc and n is the outer normal to


lhe contour C.
2335*. Evaluate the integral
~dx-dy

::f x+y ,
c

taken along the contour of a square with vertices at the points


A (1,0). B(O, I), C(-l, 0) and D(O, -1), provided the contour
is tra<.:ea countercl()~kwi~e.

28~

Line integrals

Sec. 9]

D. Applications of the Line Integral

Eva1 uate the areas of figures bounded by the following curves:


2336. l'he ellipse x=aLost, y=bsint.
2337. The astroid x=acos 3 t, y-==asin 3 t.
2338. The cardioid x=a (2cost--cos2t), y=a (2sintsin 2/).
2339*. A loop of the folium of Descartes x 3 +y:l-3.2xy=O

(a> 0).

2340. The curve (x -t- y)3 = axy.


2341*. A circle of radius r is rolling \vithout sliding along a
fixed circle of radius R and outside it. Assuming that ~ is an
integer, find the area bounded by the curve (epicycloid) described
by some point of the nlc>villg circle. Analyze the particular case
of r == R (cardioid).
2342*. A circle of radius r is rolling without s1idin~ along
a fixed circle of radius R and inside It. Assunling that !i, is an
integer, find the area bounded by the curve (hypocycloid) described by some point of the nloving circle. Analyze the particular
case when , = ~ (astroid).
2343. A field is generated by a force of constant nlagnitude F
in the positive x-direction Find th(."\ \vork that the fIeld does
when a Inaterial poi I1t traces clock\V1Se a quarter of the circle
x 2 -t- y2 == RJ. I yi ng i 11 the first quadra nt.
2344. Find the \vork done by the forLe of rravity when
a material point of nlass nl is Il10ved lrOll1 position ,1 (xl' !it' z,).
to position B (x 2 , Y2' Z2) (the z-axis is directed 'vertically upwards).
2345. Find the work done by an elastic force dIrected towards
the coordinate origin If the l11agnltudc of the force is propJrtional to the distance of the point flOIll the origlll and if the point
of application of the force traces counterclock\vise a quarter of

the ellipse ~+~~= 1 Iyin~ in the first quadrant.


2346. Find t:le p)tential rUl1~tion of a for~c R {X, Y, Z}
and deternline the work don~ by the force over a given path if:
a) X=O, Y=-=O. Z-=-I1Zg (force of gravity) and the material point is nl0ved fronl p0sitiol1 A (Xl' y" Zl) to position
B (x~p Y2' zs);
b) X ~ _~t:,
r

y = _Jl~.
,

tt:_,

Z=- ,
where J1 = canst and
, =-= V Xl + y" + z" (Newton attractive force) and the material point
moves from position A (a, b, c) to infinity;
~-~--=.-

Multiple and Line Integrals

~84

[Ch.7

X = - k 2 x, Y = _k 2 y, Z = - k 2 z, where k = canst (elastic


force), and the initial point of the path is located on the sphere
2
Xl + y2 + Z2 = R , while the terminal point is located on the sphere
c)

x2

+ + Z2 =
y2

r 2 (R

> r).

Sec. 10. Surface Integrals


to. Surface integral of the first type. Let f (x, lj, z) be a continuolls
function and z=-=q> (x, y) a smooth surface S.
The surface integral of the fIrst type is the liruit of the integral surn
n

SS
f (x, Y, z) dS = lim .~ f (Xi,
S
n-+oot=J

Yi' Z,) Mi.

where ~Si is the area of the Ith element of the surface S, the point (xi, YP
Zi) belongs to this elelnent, and the mctximum diameter of elements of partition tends to zero.
The value of this inte~ral is not dependent on the choice of side of the
surface S over which the integration is performed.
If a projection a of the surface S on the xy-plane i~ single-valued, that
is, every strdight line parallel to the z-axis intersects the surface S at only
one point, then the appropriate surface integral of the first type Inay be
calcula ted froln the formula

SSf (x,
S

y, z) dS

=- SSf [x,

y, q> (x, y)]

VI + cp: (x, y) + CP~' (x, y) dx dy.

(0)

Example t. COlnpute the surface integral

H
s

(x+y+z)dS,

where S is the surface of the cube O~x~l, O~y~l, O~z~l.


Let us comput(3 the sum of the surface integrals over the upper edge of
the cube (z = 1) and over the lower edge uf the cuue (z = 0):
J 1

J 1

11

SS (x+ y) dxdy=

00

00

00

(x+y+ 1) dx dy+

(2x+2y+ 1) dxdy=3.

The desired surface integral is obviously three tilnes greater and equal to

~~
s

(x

+ y + z) dS = 9,

2. Surface integral of the second type. If P = P (x, !I, z), Q = Q (x, lj, z),
z) are continuous functions and S+ is a side of the snJooth surface S characterized by the direchon of the normal n {cos Cl, cos ~, cos y}. (hen
1he correspondinf..! surface tntegral of the second type is expressed a~ follows:

R = R (x, y,

S~ P dydz+ Q dz dx+ R dx dy=


s+

(P cosa

+Q cos ~ + R cos V) dS.

285

Surface Integrals

Sec. 10]

When we pass to the other side, S-, of the surface, this integral reverses sign.
If the surface S is represented implicitl y, F {X, y, z) = 0, then the direction cosines of the nonnal of th is surface are determ ~ned fron] the form ulas
1
1 iJF
1
cos a = 15 ax ' cos ~ = D ay , cos y = D oz '

of

iJF

where
D=

Y(~~r+(~:r+(~~y.

and the choice of sign before the radical should be brought into agreemenf
the si de of the surf ace S.
3. Stokes' formula. If the functions P = P (x, y, z), Q = Q (x, y, z),
R == R (x, y, z) are continuously differentiable and C is a closed contour bounding a hvo-sided surface S, \ve then have the Stokes' fornzula

\v ith

1
c

P dx

+ Q dy + I? dz =

=55s [(iJR - iJ
iJy

Q ) cosa

iJz

Q
+ (ap
cos ~ + (a - iJP) cos v1 dS
az _ aR)
0'"
ax
iJy
,

\vhere cos a, cos~, cos V are the direction cosines of the norrnal to the surface S, and the dircctton of the nonna) is defined so that on the side of the
nornlal the contour S IS traced counterclockwise (In a right-handed coordinate
~)steln).

Evaluate the follo\ving surface integrals of the first type:


2347. ~ ~ (x' -+ y', dS, where S is the sphere x + y2 -1- Z2 = a 2
~

2348. ~ ~ V x 2 -I- y' dS where S is the lateral surface of the


s
~\ 2
t/!' z'.!
__

cone 2-1-'2--l2=-=0 [O~z~b).


Evaluate the follo\ving surface integrals of the second type:
(I

2349.

(4

')

H
yzdydz+xzdzdx-l-x!Jdxd!J,
s

where S is the external

side of the surface of a tetrahedron bounded by the planes x = 0,


Z = O. x -I- y -t- z =-.:= a.
2350.
zdxdy, where S is the external side of the ellipsoid
s

y = 0,
Xl

yl

Z2

Qi-I- b2+ CZ = 1.
2351.

H
s

Xl

dy dz

-+ yl dz dx + Zl dx dy,

where S is the external

side of the surface of the henlisphere x" + !It. + Zl = a 2 (z ~ 0).


2352. Fi nd the l1lass ot the surface of the cu be 0 ~ x ~ 1
o ~ y ~ 1, 0 ~ z ~ 1, if the surface density at the point M (x, y, z)
is equal to xyz.
J

Multiple and Line Integrals

286

[Ch.7

2353. Determi ne the coordi nates of the centre of gravity of a


homogeneous parabolic envelope az=x' +y2(0~z~a).
2354. Find the mOinent of inertia of a part of the lateral
surface of the cone z = V x 2 + y2 [0 ~ Z ~ h] about the z-axis.
2355. Applying Stokes' formula, transform the integrals:

a) p (Xl - yz) dx (yl - zx) dy


c
b)
y dx + z dy -+- x dz.
c

+ (Zl -

xy) dz;

:ft

Applying Stokes' formula, find the given inte~rals and verify


the results by direct calculations:
2356.
(y + z) dx + (z + x) dy + (x + y) dz, where C is the circle
c
x Z+y2+ z2=a 2, X-i"-Y+z==O.

:I

2357.

:Ic (y-z)dx+ (z-x)dy+<x-y) dz, where C is the ellipse


Xl

-1- yZ = 1,

x -1- z = 1.

2358.pxdx+(x+Y)dy+(x+y+z)dz, where C is the curve


c
x=asint, y=acost, z=a(sint+cost)[O~t~2Jt].
2359. ,cP y 2dx-t z2dy-t-x 2dz, where ABCA is the contour of
ARf;.4

ABC with vertices A (a, 0, 0), 8 (0, U, 0), C (0, 0, a).


2360. In what case is the line integral
1=:1 Pdx+Qdy+Rdz
c

over any closed contour C equal to zero?


Sec. 11. The Ostrogradsky-Gauss Formula
If S is a clo'ied smooth surface bounding the volume V, and P -= P (x, y, z),
Q = Q (x, Ij, z), R = R (\', y, z) dre functions that are continuous togpther with
th-'ir first partial denvatlves in the closed region V, then we have the OstrogrlJdsky-Gauss forlnula

SS (P cos a+ Q cos ~ + R cos


oS

where ers a, cos


S

surrac~

y) dS=

55S(::

~~ + ~:) dxdydz,

(V)

p.

cos yare the direction cosines of the outer normal to the

Applying the Ostrogradsky-Gauss formula, transform the following surface Integrals over the closed surfaces S bounding the
\

;ec. 11]

The Ostro[!radsky-Gauss Formula

287

/olume V (l'osa, cos~, cos)' are direction cosines of the outer


lormal to the surface S).
2361.

H
xy dx dy -I- yz dy dz -+ zx dz dx.
s

2362. ))

Xl

dy dz

+ yl dz dx -I- Zl dxdy.

2363.

2364.

rIll xc('sa+~~
~~c()S Y dS.
V + y2 +

J.S

x2

Z2

55s (~cosa + :~ cos ~ + ~cosy) dS.

Usin~ the Ostrogradsky-Oauss formula, compute the following


surface integra Is:

2~63. ~ ~

Xl dy dz -I- yl dz dx + Zl dx dy, where S is the external


s
side of the surface of the cube O~x~a, O~y~c, O~z~a.
2366. ~~xdydz+ydzdx+zdxdy. where S is the external side
s
of a pyranlid bounded by the surfaces x-+-y-l-z=a, x=O, y=O,
z =- O.
2367. ~ ~ Xl dy dz -1- yl dz dx = Zl dx dy, where S is the external
~

side of the sphere x!_t-y2--1-z2=a2.


2368 ~ ~ (Xl cos a -j yl cos ~ -I- Zl cos y) dS, where S is the exters
na I total surface of the cone
[O~z~b).

236ft Prove that if S is a closed surface and


direction, then

is any fixed

H
cos (n, I) dS = 0,
s

where n i5 the outer nOftTIal to the surface S.


2370. I)rove that the volunle of the solid V bounded by the
sL1rface S is equal to

V = -}

55(x cos a + y cos ~ + z cos v) dS,

s
where (~oSC, cosp, cosy are the direction cosines of the outer
normal to the surface S.

28R

Multiple and Litle Integrals

[Ch, 7

Sec. 12. Fundamentals of Field Theory


10. Scalar and vector flelds. A scalar field is defined by the scalar function
of the point u = f (P) = f (x, y, z), wh~re P (x, y, z) is a point of space. The
surfaces f (x, y, z) = C, where C = const, are called level surfaces of the scalar
field.
A vector field is defined by the vector function of the point a == a (P) ==
=: a (r), where P is a point of space and r= xi + yJ+ zk is the radius vector
of the pOint P. In coordinate form, a~axi+avJ+azk, where ax == ax (x, y, z),
ay = ay (x, y, z), and az = at: (x, lj, 2) are projec.tions of the vector a on the
coordInate axes. The vector lines (force lines, flow lines) of a vector field are
found from the following system of differential equations
dx dy dz
-=-=ax ay az

A scalar or vector field that does not depend on the time t is called
stationary; if it depends on the time, it is called nOttstationary.
2. Gradient. The vector

au

grad U (P) = ox l-~

au. au

ay J + az k.=== \jU,

where \l=i ~ +j:+kO is the Hamiltonian orerator (del, or nabla), is


vX'

dz

vy

called the gradient of the field U = f (P) at the given point P (cf. ChI VI, Sec. 6).
The gradient is in the direction of the normal tl to the level surface at the
roint P and in the direction of Increasing function U, and has length equal
to

au === -./"'(aU)2 + (aU)2 + {UU):!.

an

ax

ay

\dz

If thr directIon is given by the unit vector I {cos a, cos ~, cos V}, then

au
au
au cos~+dz
au cosV
ar=gradU.I=gradtU=ax cosa+
dy

(the derivative of the function U in the direction I).


3. Divergence and rotation. The divergence of a vector field a (P) == a>.i t.
k tl
I
d
iJel x day iJa z
~"7
+ayJ+a
IS
lescaar
Iva=-iJx+ay-+az==:va.
z

The rotation (curl) of a vector field a(P)=axl+ayJ+azk is the vector

rot a=

y) l + (aa. _ aaz ) J + (oay _ aa x )


(iJadyz _ aa
~
~
ax
ax ay

== \!Xa.

4. Flux of a vector. The flux of a vector field a (P) through a surface S


in a direction defined by the unit vector of the normal 11 {cos U, cos p, cos V}
to the surface S is the integral

~~
s

an dS =

~~
s

an dS =

SS(ax cos a -I- ay cos ~ -+ az cos y) dS.


s

If S is a closed surface bounding a volume V, and n is a unit vector of the


outer normal to the surface S, then the OstrogradskyGaus:s formula holds,

289

Fundamentals of Field Theory

Sec. 12,]

wh ich in vector form is


tan dS=
s

H~

diva dxdy dz.

(V)

5. Circulation of a vector, the work of a fteld. The line ultegrai of the


vector a along the curve C is defined by thl: formula

~ adr= ~ Qsds= ~ axdx+Qvdy+azdz


C

(I)

and represents the work done by the field a along the curve C (as is the
projection of the vector a on the tangent to C).
If C is closed, then the line integral (I) is called the clrculuilon of the
vector field a around the contour C.
If the closed curve C bounds a twosided surface S, then Stokes' IOrtrlula
holds, which in vector form has the form

1c a dr= ~s~

n rot adS,

where n is the vector of the normal to the surface S; the direction of the
vfctor should be chosen so that for an observer looking in the direction of n
thf circulation of the contour C should be counterclock\vise in a right-handed
coordinate systelll.
6. Potential and solenoidal fields. The vector tiel d a (r) is called potentlal if
a=grad U.
where U =1 (r) is a scalar function (the potellttu/ of the field).
For the potentiality of a field a, given in a sirnpl yconnected domain,
it is necessary and sufficient that it be non rotational, that is. rot a=O. In
that case there exists a potential U defined by the equation
dU = ax dx + a v dy + a z dz.

If the potential V is a single-valued function, then ~ a dr

= V (B)-V (A);

AB

in particular. the circulation of the vector a is equal to zero:

1c

a dr=O.

A vector field a (r) is called solenoidal if at each point of the field div
a =0; in this case the flux of the vector through any closed surface is zero.
If the field is at the same tinle potential and solenoidal, then div (grad U) ==-0
and the potential function U is harnlonic; that is, it satisfies the Laplace
8 2 U a2 u a2 u
a2 a2 a2
equation O~I + ay 2 + az" =0, or tlU =0, where tl = V2 = ch,a+ ay2+ az 2 is the
Laplacian opera"tor

2371. Determine the level surfaces of the scalar field U = f (r)t


-=
y 2 -t-Z 2 What will the level surfaces be of a field
U = F (Q), where Q =
y2?

where

10-1900

r Vx +

Vx. +

Multiple and Line Integrals

290

(Ch. 7

2372. Determine the level surfaces ot the scalar field

.. r z
U = arc sin

y x2 + y2
2373. Show that straight lines parallel to a vector c are the
vector lines of a vector field a (P) = C, where C is a constant
vector.
2374. Find the vector lines of the field a = - royl-~ roxj, where C1l
is a constant.
2375. Derive the formulas:
a) grad(C I U+C 2 V)=C 1 gradU+C a gradV, where C1 and C1
are constants;
b) grad (UV) = U grad V -t- V grad U;
c) grad (V 2 ) = 2U grad U;
grad V .
d) gra d ( V - V grad U -U
V2
,
e) grad cp (V) = (P' (U) grad U.
2376. Find the magnitude and the direction of the gradient
of the field U = Xl + y3 --1- Z3 - 3xyz at the poi nt A (2, 1, I). Determine at what points the gradient of the field is perpendicular to
the z-axis and at \vhat poi nts it is equal to zero.
2377. Evaluate grad U, if U is equal, respectively, to: a) r,
b) r', c) J.,
d) f (r) (r = V x 2 -t- y2 + Z2).
r
2378. Find the gradient of the scalar field U = cr, where c is
8 constant vector. What will the level surfaces be of this field,
and what will their position be relative to the vector c?
2379. Find the derivative of the function U = a + Yb-; +:;.
at a
c
given point P (x, y, z) in the direction of the radius vector r of
this point. lIn what case will this derivative be equal to the
magnitude of the gradient?
2380. Find the derivative of the function U = ~ in the dir
rection of I {cos a, cos~, cos V}. In what case will this derivative
be equal to zero?
2381. Derive the formulas:
a) div (C1a l + C2 a 2 ) = C l div a l + C2 div a 2 , where Cl and Cz are
constants;
b) div (Uc) = grad U c, where c is a constant vector;
c) div(Ua)=grad Ua+ Udiva.
2382. Evaluate div (

l:!-) _

x:

f) .

2383. Find diva for the central vector field a (P) = f (r)!...
where r = VXl + y2 + Z2.
r

Fundamentals of Field Theory

Sec. 12]

291

2384. Derive the formulas:


a) rot (Cta, + C 2a 2) = C1 rot at + C,. rot aI' where C 1 and C2 are
consta nts;
.
b) rot (Uc) = grad U c, where c is a constant vector;
c) rot (Va) =grad Ua + U rot a.
2385. Evaluate the divergence and the rotation of the vector
a if a is, respectively, equal to: a) r; b) rc and c) f (r) c, \vhere c
is a constant vector.
2386. Find the divergence and rotation of the field of linear
velocities of the points of a solid rotating counterclockwise with
constant angular velocity (0 about the z-axis.
2387: Evaluate the rotation of a field of linear velocities
ttJ = (t}r of the points of a body rotating with constant angular
velocity ro about some axis passing through the coordinate origin.
2388. Evaluate the divergence and rotation of the gradient of
the scalar field U.
2389. Prove that div(rota)=O.
2390. Using the Ostrogradsky-Gauss theorem, prove that the
flux of the vector a = r through a closed surface bounding an
arbitrary volume v is equal to three times the volume.
2391. Find the flux of the vector r through the total surface
of the cylinder X2+y2~R2, O~z~H.
2392. Find the flux of the vector a=x 3i+y3j-t-z sk through:
x 2 + 11 2
Z2
a) the lateral surface of the cone ~ ~ H2' 0 ~ z ~ H; b) the
total surface of the cone.
2393*. Evaluate the divergence and the tlux of an attractive
force F = - r of a point of mass ln, located at the coordinate
origin, through an arbitrary closed surface surrounding this point.
2394. Evaluate the line integral of a vector r around one
turn of the screw-l ine x = R cos t; Y = R sin t; z = hi from t = 0
to t = 231.
2395. Using Stokes' theorem, evaluate the circulation of the
vector a == X 2 y 3; + i+zk along the circumference x2 + y2 = R2~ Z == 0,
taking the hemisphere z == VR 2 _x2 - y2 for the surface.
2396. Show that if a force F is central, that is, it is directed
to\vards a fixed point 0 and depends only on the distance r from
this point: F=f(r)r, where f(r) is a single-valued continuous
function, then the field is a potential field. Find the potential U
of the fiel d.
2397. Find the potential U of a gravitational field generated
by a material point of mass nz located at the origin of coordinates: a = - ~ r. Show that the potential U satisfies the Laplace
equation ~U = o.

Ill:

10*

292

Multiple and Line Integrals

[eh. 7

2398. Find out whether the given ,vector field has a potential U.
and find U if the potential exists:
a) a= (5xy-4xy) I + (3x l -2y)j;
b) a=yzi+zxj+xyk;
c) a = (y+ z) 1+ (x+ z)j+ (x + y) k.
2399. Prove that the central space field a = f (r) r will be solenoidal only when f (r) = r~ t where k is constant.
2400. Will the vector field a = r (c x r) be solenoidal (where c
15 a constant vector)?

Chapter VIII

SERIES

Sec. 1. Number Series


It). Fundamental concepts. A number series
~

al +a2 + +an + = ~ an

(1)

n=1

is called convergent if its partial sum

Sn=al+ a2+
has a finite limit as

n --+ 00.

.. +an

The quantity S

of the series, \vhile the number

lim Sn is then called the

n -+

Rn == S-Sn ==a n + 1 + all + 2

Sllln

00

+...

is called the remaInder of the series. If the limit lim Sn does not exist (or is
n~ lX)

infinite), the series is then called divergent.


If a series converges, then Iirn an = 0 (necessary condition for convergence).
n-+<r>

The converse is not true.


For conv~rgencc of the series (I) it i-; necessary and sufficient that for
any positive nUI11ber e it be possible to choose an N such that for n > lV
and for any posi tivE' p the following inequality is fulfilled:

\a n +l +a n + 2 + ... +an+pl <8


(Cauchy's test).
The convergence or divergence of a serit's is not violated if we add or
subtract a finite nunlber of its terms.
20 Test~ of convergence and divergence of positive series.
a) Comparison test I. If 0 ~ all ~ bn after a certain n = no, and the series
00

bl +b 2

+ +bn + = n=1
~ bn

(2)

converges, then the series (1) also converges. If the series (J) diverges, then
(2) diverges as well.
It is convenient, for purposes of cOlnparing series, to take a geometric
progression:
{a 0).

294

Series

(Ch.8

which converges for I q I < 1 and diverges for I q I ;=:: 1, and the harrnontc sert es
00
1

L. n'

\vhich is a divergent series.


Example t. The series
1
1
1
t:2 + 2.2 2 + 3.2 3 -1- ...

+ 11 2n +

converges, since he re
1
atZ == n2 n

< 2'i'

while the geometric progression

whose ra t10' IS q = 21 ' converges.


Example 2. The series

In2+~+
2
3
... +~+
11

d iv~rges, sInce
t
I s genera I t ertn In n.IS grea t er th an th e correspon d ing ternl

.!.11

of the hannonic series (which diverges).

b) Comparison test II. If there exists a finite and nonzero linlit Hln an
H ~ 'X btl
(in particular, if an "- bn), then the series (I) and (2) converge or diverge at
the sanle time.
Example 3. The series
1

o Iverges,

SI nce

+ 73 + "5 + . + 2n-1 ~- ..

1) =-:1=0
12 '
11m (1
--:2n -1 n

n -+

aJ

whereas a series \vith general term

.-!.n

diverges.

Example 4. The series

1
1
1
2-1 +2 2 -2 +2 3 -3
converges, since
lim

n -+

QO

(_1_
..-!.-) =1
2" - n 2
n

while a series with general terln

+ ... +2 n -n + ...

I e
,. .

cOllverges.

Sec. J1

295

Number Series

c) D' Alembert's test. Let an

a limit

>0

(after a certain n) and let there be

lim an+t=q.
n -+

an

(X)

Then the series (1) converges if q < 1, and diverges if q > 1. If q = 1, then

it is not known whether the series is convergent or not.


Example 5. Test the convergence of the series

2n-1

2" + 22 + 23 + + ~ + ..
Solution. Here,

2n+ 1

2n-l

a,,=--pr-'

0n+l= 2n + 1

and
1

lint an + t =
t1 -.

atl

lim
t1 -. 00

(2n -t- 1) 2

2n ~- I (2n - 1)

I + 2n

- hnt - - = 2 n-.oo 1-.!2


'2n

Hence, the given series converges.


d) Cauchy's test. Let a" ~ 0 (after a certa in n) and let there
lim

a =q.

n/

rz-.::sJ V

be a Ii mif

tz

Then (1) converges if q < 1, and diverJ.!:es if q > 1. When q= 1, the question
of the convergence of the seri~s relnains open.
c) Cauchy's integral test. If atl = f (n), where the function f (x) is posi ti ve,
rnollot0111Cally decreasin~ and continuous for x:;:::a~l, the series (I) and the
integral
(X)

~ f (x) dx
a

converge or diverge at the saine time.


By l11eans of the integral test it may be proved that the Dtriclllet sertes
(3)

if p > 1, and diverges if p ~ I. The convergence of a large nllIuber


of senes l11ay be tested by conlparing \vith the corresponding Dirichlet
series (~3)
Example 6. Test the following series for convergence

convcn~es

r:2 +3 4 + 5 6 + .. + (-2'-1--1)-2-n +
Solution. We have

an :.:: -(2-n---1)-2-/~

4n 2 - - 1 - 4n 2

1-2n

'

Series

296

(Ch.8

Since the Dirichlet series conver~es for p=2, it follows that on the basis of
comparison test I I we can say that the given series likewise converges.
3. Tests for convergence of alternating series. If a series

I aJ '+' a2 1+ ... + I an 1+ ... ,


(4)
composed of the absolute values of the terms of the series (I)t conver~es,
then (1) also converges and is called absolutely converJIent. Bu t if (1) converges and (4) divE:'rges t then the series (1) is called conditionally (not absolutely) convergent.
For investigating the absolute convergence of the series (l)t we can make
use (for the series (4)] of the familiar convergence tests of positive series.
For instance t (1) converges absolutely if

lilll t1 n + 1
n -+ (X) an

I<

1 or

;/1 a I <

lim

n -+

ri

1.

(Xl

In the general case, the divergence of (1) does not follow from the diver
an , > It then not only does
gence of (4). But if Jim an +1 I > 1 or lim
n -+

(X)

an

n -+

VI

(X)

(4) diverge but the series (1) does also.


Lei bniz test If for the aIterna t i ng series
bt -b 2

+ ba-b~+...

(5)

(btl ~O)

the following conditions are fulfilled: 1) b 1 ~ b2 ~ b,~ ; 2)

lim b'J = 0,
-+

then (5) converges.


In this case t for the remainder of the series Rn the evaluation
IRn'~bn+1

holds.
Example 7. Test for convergence the series
1-

n )n
"5 + (4)4
7" + ... +(-1) ~ (211,-1
+ ...
(32)2 - (3)3
2

Solution. Let us form a series of the absolute values ot


this series:

th~

terms of

, (3"2)2 + (3)'
(4)4
(n)n
"5 + -::;- + .. + 2n -- 1 +

1+

Since

lim
n -+ (Xl

V(_n_)n
=
211, - 1

lim _11,_= Jim _l_=~


ex> 2n - 1
11 -+ ~ 2 _ ~
2'

n -+

the serieR converges absolutely.


Example 8. The series

l-~+~- ... +<_l)n+I.*+ ...


converges, since the conditions of the Leibniz test are iulfilled. This series
converges condi tionally, since the series
I
I
1

1+"2'-3++/1+

diverges (harmonic series).

Sec

Number Series

I)

297

Note. For the convergence of an alternating series it is not sufficient that


Its general term should tend to zero. The Leibniz test only states that an
alternating series converges if the absolute value of its general term tends
to zero monotonically. Thus, for example, the series
1

1-5"+"2- 52 +"3- ... +k- 5k + ...


diverges despite the fact that its general term tends to zero (here, of course,
the monotonic variation of the absolute value of the general term has been
violated). Indeed, here, S2k=S~+S~, where

11
1
I)
+ 3 ++ k1, Sk=- (15+S;++5
II

Sk=1+ 2

and tiln S~ =
k -.

00

'

(S~ is a partial SUITI of the harrrtonic series), \vhereas the

7J

limit litll S~ exists and is finite (S~ is a partial sum


k -.

Of

the convergent geo-

Xl

metric progression), hence, lim


k -+

S2k = 00.

00

On the other hand, the Leibniz test is not necessary for the convergence
of an alternating series: an alternating series may converge if the absolute
value of its general ternl tends to zero in nonmonotonic fashion
Thus, the senes

1
1
1
I - 22 + 3i - 4i + ...

+ (2n -

1)3

converges (and it converges absolutely), although the Leibniz test is not fulfilled: though the absolute value of the general term of the series tends to
zero, it does not do so monotonically.
4. Series with complex terms A series with the general term cn =an

00

-1- Ib n (i2 == - I) converges if, and only if, the series with real terms ~ an
n=l
:')

and ~ b'l converge at the same time; in this case


11=1
00

00

C"

n=l

r:IJ

= ~ an + i ~ bn .
"=J

(6)

n=1

The series (6) defini tely converges a nd is called absolutely convergent, if the
series

~ I en I = ~ V a~ + b~l'

I1=J

n=J

whose trrlllS are the moduli of the terms of the series (6), converges.
50. Operations on series.
a) A convergent series may be multiplied terlnwise by any number k;
that lc), if

a,+ a +... +an + ... =S,


2

then
ka.

-t- ka 2 +... + ka n + ... = kS.

298

Series

(Ch.8

b) By the sum (diUerence) of two convergent series

+..

(7)
(8)

a J +a 2 + +a n
=8 1 ,
bl +b 2 + +b,J + ... =8 2

we mean a series

+ +

(at btl (a 2 b2)


(an bn ) -1- = 8 1
c) The product of the series (7) and (8) is the serifs
c1

S2

+ c + ... + cn + ,

(9)

\vhere cn=albn+afGbn_t+ ... +anbl(n=l, 2,


).
If the series (7) and (8) converge absolutely, then the series (9) also converges absolutely and has a sunl equal to SIS2.
d) If a series converges absolutely, its SUITI remains unchanged \vhrn the
terms of the series are rearranged. This property is absent if the series COIlverges conditionally.

Write the simplest formula of the nth term of the series using
the indicated terms:
1

+ 7I + ...

2401.1-+-3"-1- 5
2402.

2+4+6+ 8+
234

2403. 1+2+4+8+ ...


1

2407. 2" ..t- "6

+121 -1- 20I ,- 301 + 42I +


I

13
135
1.357
2408. 1+r:4+1.4.7+1.4.7.10i -

2409. 1- 1 1- 1 - 1 -t 1 - 1 -~-

2410. 1+ .~ +3+++5+~ +-

.
...

In Problems 2411-2415 it is required to write the first 4 or


5 terIllS of the series on the basis of the known general term an.
3n-2

2411. an =n 2 +1

2414. an =[3+(_1) /J J".

2412. (-l)nn
-2n -

2415. alJ=

2-t-(-I)n

2413 an =

n2

.
2+8111

nn) cos nrt

n!

Test the following series for convergence by applying the cOlnparison tests (or the necessary condition):
2416. 1-1 + 1-1 + ... +(_I)n-J + ...
n

(2)2 +"31 (2)3


I (2)n
5 + ... + Ii "5 +..

~417. 5" + 2" 5"

S c. I]

- - - - - - - - -Numb:!r
- - -Series
- - - - - - - - - - -299
2

n+l

2419.

+ .. + 2n t- 1 -t-
(_1)" r
1
1
1
VTo- VlO + Vllj- -j- VTO + ..

2420.

1
1
1
2+4+6+

2421.

h+~ +~I +

24 t 8. 3" -1- 5 -:1-. 7"

'1+

1
-l-2n

-+- ...
1
+.10/1 +1 +

+ V~+.)34 + ., -I-.
I +
23
. r (Il + 1)
2
2
2
2 -J- 2 -~- 3 -1- -r- n +
1-+- ';_+'; + ... + .;-_+ ...
2
3

2422. ) -

12

2423.
2424.

...
f 11

1l

1 ,1
2425. 22 -r 52

f tt

+ 82 -1- . + (:3n _

V2

~3

1 t- 3
2427. y"2
- 2

. +- - . ; r -1(Il 1- 1) r 11

\ Using d'Alenlbert's test, test the


gence:

-~

VIl

V3

I'
1
2426. 'T -J- ---V -J- ~ -~...

1)2

follo\vin;~

+-2 Y-25
+
,2r1 - 1
.. -r (V2)n

~crles

,
j

2 I 25 + 2 5. 8 1
I 2 5 8 r31l - 1) I
2 4_') 8. -.-- 15
r:S:-9 -- .. -- 159 (411-3) - - ..
l'est for convergence, llsing Ca uchy 's test:
2
( ;3 ) 2
( 4 ) 3
( 11 -1- 1 ) It
2429. T+ :3
5" -f- ...
2n -1
t-.
I

2430.

+
3 )'
21 -}- ( 52 )3 + (-8-1-

fl
)2/1-1 + ..
... -1- ( 311-1

Test for convergence the posit ive series:


1

2431. 1 + 2'.
1

2432.
2433.

+ -3' + . -t- -, -1- .


....

II

3 -1- R + is -t- -}- (n -f- 1)2 -

1 I"

1~4+4\+/1O+ '"+(311-2)\311+1)+'''
1
4
9
11
3 + 9 -t- 19 + + 2n + 1 -t-
3 +
'21 + 52 + 10
-r nn+1-r ...
2

2434.
r:
243 n.

2436. 22.33

2n,- 1

+32.42 + 42.52 + + (Il + 1)2 (11 +2)2 +

for conver-

300

(Ch.8

SerifS

+( ~ ).+( l~ ). + + (3n~ I ) n +..

2437 :

2438.

2439.

(i)I+~+(l~)t+ ... +G:~:ri+ ...


1
8
27
n'
e + e + e' + + en +
2
4
2n 1 +2 +3' + ... + 7 + - .
11
2!
3!
nl
2+1 +2 +1 +2'+1 + ---+2 n + 1 + ...
2
4
2n 1 +rr +2f + .. -+ (n-l)! + --.
2

2440.
2441_

2442.

1
13
13-5
1-3-5. _. (2n-l)
44
23"4+4.8+4.8.12+-+
4-8-12 .. _4n
(21)2

(U)2

2444.

(31)2

"21+41" +61+ ... +(2n)1 + .

2445. 1000 + lOO~:~002 + lOOO11~~.2; 1004 +

2446.

+ ...

(n!)!

2-58

2-5-8

T+ 1.5-9+
1

-. -

+ 1.5-9

10001002-1004 ... (998+ 2n)


14-7 ... (311-2)

(fin -7) (6n-4)

(an-Il) (8n-7)

+. _.

1-5 ... (4n -3)

1.5

2 447 2 2-4-6 + -- + 2-4-6 .. . (4n-4) (4n -2) + --


+1-1l-21 ... (10n-9) +
24 48 ~-I_~+1.11-21+

- I!

24 49.

3!
5'
--(2n -1 ) r
-
1-4
14.9
149
n2
1 +1-3-5+ 1.3-5-7-9+ +1-3-5-1-9 (4n-3)

2450_

L arc sin

+...

'IJ

n=l

(X)

.;._.
,

2455.

~.

2456_ ~

L In ( 1 +{ ) .

2457_ ~

2451. L sin
n=J

(X)

~n-lnn-lnlnn

n=J

n=a

(X)

co

2
,;;

2458.

n=1

~rnn
n=2

n:;:;2

00

2454_ ~ 1

~nln2n

n=2

2453. Lin n

co

00

2452.

L n lin n I.

n ~2

n'

2459_

L. I +
n=l Y n
(n

1)

Sec. 1)

Number Series

301

CJ)

2460.

L
n=1 Yn(n+l)(n+2)

2461.

I:
1

n=2nlnn+Ylnln

00

n1
I:
nn
n=1
1: 2"nl
nn

2465.

CJ)

CJ)

2466.

n=1

CJ)

2462.

eX)

L n V n 1eX)

2463.

n =1 (2n -

2467.

YiI-

n=J

L. 3nil"nl

'1=1

Vii
V-
1) (5 ~ n -1)

~e"nl

2468*.

n=1

fIn

eX)

2464.

I: (1-

~)

cos

n=1

CJ)

2469. Prove that the series ~


'\.~ fiPTiiQIi
1

n=2

1) converges for arbitrary q, if p>l, and rorq>l, if p=l:


2) diverges for arbitrary q, if P < 1, and for q ~ 1, if P = 1.
Test for convergence the following alternating series. For convergent series, test for absolute and conditional convergen~e.
1

(_1)"-1

2470. 1-3"+5-f- 2n-1


1

2471.

+ ...

(_1)1l-1

l-yr2 + Y3- ... + Yn1

+ 91 -

2473.1- 7

+ 13 -

2474

2472. 1- 4

+ (-1)"-1
n"l.
+ ...
(-I)n-I n
+ 6n-5 + ...

...

[:2-2.3

+...

+ 3.4-
7

L(

~r

l)n-1 2n+1

n(n+1)
11 2

1234

2475. - "2 - 4

-to 1f + 16 -

+11

-z-n,
(-1 )
2" -r

234
2 yf-I
3 3-I
4 Y4- 1

2476.

+ Y

+ ... +

+(_l)n
3
2477. - 4"

+ ( ""15 )

(7 ) a

10

2478. ~_3.5+3.5.7_
2

2 5

1 4

2 5 . ~
1 4. 7

+ ... +(-1 ) n

n+l
(n+l) Vn+l-l
( 211 -t 1) n
311

-t- 1

+ ...

+(_I)n-l~~~':+I)+
2 5 8.. (In - 1)

2479. 1-1.9+7.9.11- +(-1)

n_

I 4 7
l.U.1I

(~in -

2)

(2n+5)

+ ...

[Ch. 8

Series

302

00

~(_I)nlnnn.
2481. ~

2482 ~
,~( - l)n - It an ..1r-.

n=1

n=1

J' It

2483. Convince yourself that the d' Alembert test for convergence does not decide the question of the convergence of the
(X)

series ~ an' where


1l=1

(k=l, 2, ... ),

whereas by means of the Cauchy test it is possible to establish


that this series converges.
2484*. Convince yourself that the Leibniz test cannot be
applied to the alternating series a) to d). Find out which of
these series diverge, which converge conditionally and which converge absolutely:
1

a)

Y2~1-Y2+1+Y3-1-Y3+1+Y4(
1

b) 1- 3

Yk+l-l'

2k-l-

_
2k--

(
1

2k-l -

c) J -3 +~-32 -t-

(a
1

zk -

_1_

2k -

1 '

1
1
5-33
1

a _ _
2k -

1).

1).

32k -

'

+ ..

2k~1'

l-Y4+1+

Yk+l+l '

1
1
I
1
+ 2-33
+ 2 -35" + ...

2k = -

~k) ;

d) 3-1--1-7-5+TI-g+ ...

( a2k - 1 = 4k l ' a2k = - 4k~3 )


Test the following series with complex terms for convergence:
1

L n (2+i~~
2
f7J

2485.

n=J

00

2488.

L n (2/-1)n

2486:

n=1

3'Z

f7J

2489.

L n (3~i)n.

n;;:l

Ly~n + I

I1=J

2487.

L~'

n=J

00

f7J

2490.

L (n+i)1 Yn

n=l

Number Series

Sec. 1]

303

4 1 ~
1
29. ~[n+(2n-l)i]2.

2492

n=1

~ [ n (2 - i) + 1 ]

n{3-2i)-3i

n=1

2493. Between the curves y = da and y =

II

~ and to the ri~ht

of their point of intersection are constructed segments parallel


to the y-axis at an equal distance from each other. Will the sum
of the lengths of these segments be finite?
2494. Will the sum of the lengths of the segments mentioned
is replaced by the
in Problem 2493 be finite if the curve y = -;
x
I

curve y==-?
x
2495. Form the sum of the series

Does this sum converge?

rSJ

r:n

n=1

n=J

L 1t nand L <-11:- n
~

2496. Form the difference of the divergent series ~

~ 21l-1

n=l
(L

and

L 2~

and test it for convergence.

II=J

2497.

Does the series forlned

by subtracting the

series

00

'ill ~ I from the series

n=l

~ converge?

ft=-J

2498. Choose two series such that their sunl converges


their difference diverges.

L
Il=J

ile

a;

r:r:

2499. Form the product of the series

\\yh

.~-- and

11 Y 11

L LJ .
2

n=1

Does this product converge?


2500. Form the series (1 + ~ +{ + ... + 2
+ ... Does
this series converge?
1
2501. Given the series 1 -t- 2 , - 3\ + ... + <_:>n + ... Estimate
.
11.

LJ

)2.

the error committed when replacing the sum of this series with
the sum of the first four ternls, the sum of the first five terms.
What can you say about the signs of these errors?
2502*. Estimate the error due to replacing the sum of the
series

~ -I- 2 , ( ~ ) -I- ~! ( ~ ) a + ... + ~ ({ )n +...


1

by the sum of its first n terms.

Series

304

[Ch. 8

2503. Estimate the error due to replacing the sum of the

5eries
1

1 + 2! + 3! + ... + ;Ii +
by the sum of its first n terms. In particular, estimate the accuracy of such an approximation for n= 10.
2504**. Estimate the error due to replacing the sum of the
series
1

+ 2i + 3 + .. -I- n2 +
2

by the sum of its first n terms. In particular, estimate the accuracy of such an approximation for /1, = 1,000.
2505**. Estimate the error due to replacing the sunl of the
5eries
1+2 ( ~ + 3 (
+ ... +n ({
+ ...

r i- r

yn-I

by the sum of its first n terms.


00

2506. How many terms of the series

(_l)n-l

does one have

n=l

to take to cOInpute its sum to t\VO decilnal places? to three


decimals?
.

~:~

2507. How many terms of the series ~

(2n

+n I) sn does one

n=l

have to take to compute its sum to two decimal places? to three?


to four?
I
1
1
2508*. Find the sum of the series r:2~-2.3+3.4 + ... +
1

+n(n+l)ot ..

2509. Find the sum of the series

Vi+(Vx-Vx) +<Vx- Vi)+ ... +(2k+ VX_ 2k - Vx)+ ...


Sec. 2. Functional Series
1. Region of convergenc~. The set of values of the argument x for which
the functional series
fl(x)+f2(x)+.+ftl(x)+...

(1)

converges is called the region of convergence of this series. The function


S (x) = lim Sn (x),
n-+~

where Sn (x) = f I (x) + f 2 (x) + ... + fn (x), and x belon~s to the region of convergence, is call(lld the surn of the series; Rn (x) = S (x) -Sn (x) is the remainder
of the series.

305

Functional Series

Sec. 21

I n the simplest cases, it is sufficient, when determiping the region of


convergence of a series (1). to apply to this series certain convergence tests.
holding x constant.

Converges

Diverges

Diverges

i,(1f(~9

-J

-1

Fl~.

1~4

Example 1. Determine the region of convergence of the series

+ + (x + 1)2 + (x + 1)1 +

x I
1 2

2 . 22

3 2'

Solution. Denoting by u" the general

...

+ (xn+. 21n)n + ..

t~rm

(2)

of the series, we will have

n
lim Itln+11_lim Ix+ll + 2 n _lx+ll
n-+ etJ I un' - n-+ etJ 2n+ J (n + 1) I x In 2
J

Using d' Alembert's test, we can assert that the series converges (and converges
I 11< 1, that is, if -3 < x < I; the series diverges, if

X!

absolutely), if

IX!Il> I,

that is, if -co

<x<-3 or I <x< co (Fig. 104). When x=1

i ii - i-

+ + + ... , which diverges, and when X= - 3


-I +
+ ... , which (in accord with the Leibniz

we get the harmonic series I


we have the series

test) converges (conditionally).


Thus, the senes converges when - 3 ~ x < 1.
2. Power series. For any power series
Co

-1- C1 (.\ -

a)

+C

(x-a)2 + ...

+ Cn (x-a)n + ...

(3)

(C n and a are real nunlbers) there exists an interval (the interval of convergence) I x-a I < R with centre at thr point x= u, with in which the series (3)
converges absolutely; for I x-a I> R the series diverges. In special cases, the
radius of convergence R may also be equal to 0 and 00. At the end-points of
the interval of convergence x=a :1: R, the power series may either converge
or diverge. The interval of convergence is ordinarily determined with the
help of the d'Alembert or Cauchy tests, by applying them to a series, the
terms of which are the absolute values of the terms of the given series (3).
Applying to the series of absolute values

I Co I + I c1 II x-a 1+ ... + I cn II x-a In + ...


the convergence tests of d' Alembert and Cauchy, we get, respectivel y, for the
radius of convergence of the power series (3), the formulas

R= lim

jYlc" I

and

R=liml~1
n
n-+oCD

+1

n~(f)

However, one must be very careful in using them because the limits on the
right frequently do not exist. For exanlple, if an inflniture of coefficients clI

Series

306

(Ch.8

vanishes [as a particular instance, this occurs if the series contains terms
with only even or only odd powers of (x-a)], one cannot use these formulas.
It is then advisable, when determining the interval of convergence, to apply
the d' Alembert or Cauchy tests directly, as was done when \ve investigated
the series (2), without resorting to general formulas for the radius of con
vergence.
If z=x+ ty is a complex variable, then for the po\ver series

CO+c 1 (z-zo)+c 2 (Z-ZO)2+ ... +c n (z-zo)n+...

(4)

(cn=an+ib n, zo=xo+iyo) there exists a certain circle (circle of convergence)


I Z-Zo 1< R with centre at the point z=zo' inside which the series conver~e5
absolutely; for I z-zo I> R the series diverges. At points lying on the cir
cumfprence of the circle of convergence, the series (4) may both converge and
diverge. It is custoInary to determine the circle of convergence by nlcans of
the d' Alembert or Cauchy tests a pplied to the series

I Co I + I ct '1 z-zo I+ I c2 11 Z-Zo 12 +... + I en 11 z-zo ,n+ ... ,


whose terms are absolute values of the tenns of the given series. Thu~, for
exaInple, by means of the d' Alelnbert test it is easy to see that the Circle of
convergence of the series

z+ 1 +(Z+ 1)2+(z+ 1)3+


1.2
2.2 2
3.23

+(Z+ l)n+
n2 rl

is determined by the inequality I z+ 11 < 2 [It is sufficient to repeat the cal~


culations carried out on page 305 which served to deterIlline the interva I of
convergence of the series (2), only here x is replaced by z]. The centre of
the circle of convergence lies at the point Z === -1, while the radius I? of thl~
circle (the radius of convergence) is equal to 2.
3. Uniform convergence. The functional series (1) converges unifonnly on
some interval if, no Inatter \vhat e > 0, it is possible to find an N such that
does not depend on x and that when n > N for all x of the given Intervdl
we have the inequal ity I Rn (x) I < 8, where RIl (x) is the renlai nder of th~
given series.
If Ifn(x)l~cn (n=l, 2, ... ) when a~x~b and the nunlber series
00

'

~ en converges, then the functional series (I) converges on the Interval


n=t

lat h' absolutely and uniformly (Weierstrass' test).

The power series (3) converges absolutely and uniformly on any interval
lying within its interval of convergence. The power series (3) may be ternlwise differentiated and integrated within its interval of convergence (for
, x-a I < R); that is, if

Co

+c (x-a) +
1

C2

(x-a)!

+ ... +en (x-a)n + ... = f (x),

(5)

then for any x of the interval of convergence of the series (3), we have

c1+ 2c 2 (x-a)
x

~ codx + ~ c1 (x-a) dx+ ~ c2


%0

Xo

+... + nCn (x_a)n-l + ... =1' (x),


x
(x-a)2 dx+ . + ~ cn (x-a)n dx+ ... =

(6)

Xo

Ko
00

= ~ en
n=:o

(x_a)n+t (x o-a)n+l

+1

x
S
= f (x) dx
Xo

(7)

Functional Series

Sec. 2]

307

(the nunlber X o also belongs to the interval of convergence of. the series (3).
I-Iere, the series (6) and (7) have the same interval of convergence as the
series (3).

Find the region of convergence of the series:


rn

00

2510.

t~x,

n=1

2518.

"f)

2511. L(-l)"+I~.
X
n

11=1

L (2tl~

2519.

. . VOn

YJ

_1_
Inx
n

(_1)"+1

11=1

L. sin(~-l~
(211-1)2

2520.

L (x--2),i'
.L. (11 2a+
1
-t- 1)5

IZ .= 1

'Y)

"f)

2521.

11=1

X 2"

II='J

L.

YJ

.L. 2 sln:)ijx .
n '

2522.

11=1

'r.

L c:;,.\~ .

OCJ

~.,~

2523 .

......

"rz

11=1 X

11=0
Cfj

00

2516. L.(-l)l1tJe-Il~ln.~.

2524*.

1l::0

L (XII +2'!X II )

11=1

Cfj

2517.

(_1)"-1

1l:3 1l (x-5)"

n=o

2515**.

I) X"

11=1

YJ

~~
2512. ......

2514.

n! x n

'1=1

00

2513.

00

L~
x"

2525.

11=-1

11=1

Find the interval of convergence of the power series and test


the convergence at the end-points of the interval of convergence:
Yl

2526.

2531 .

-n

2532.

00

x~n-I

2533.

211-1

L. 2
00

2529.

00

1 W

x - 1
(4n-3)2
-

2534.

L(_I):-I

L n!x

n==1
00

00

n=l

x"

11=1

n=1

2530.

L (_l)n (2n + 1)2


L n!

11=0

'J:)

11=1

+ 1)5 X2n
2n -t- 1

00

L n2"
~"

11=1

2528.

(tl

11 =0

'1=0

2527.

L
Y.)

~.., "
.... X.

XI i .

2535.

L'~
nn
n=l

Xr&.

308

Series

(Ch.8

co

2536. ~ 2nl

)2n-.

II=J

2537.

L 3/1 x
2

n1

2553.

ao

nn

co

2554. ~ nl (x+ 3)11


~

lI

L n!

2557.

n=l

~558.

2559*.

3/

ng"

~561.

(_1)/1-1 (x_2)zn
2n

2562.

co

n=1

(x - 1)".

i: (-

1)"

V~2 X
n

n=o

+1

co

2549. ~~ (x+3)n

t:1 ( + ~ )"I

X (x-2)n.

CD

2550.

nn

2560.

n=l

n=J

(x+2)n'

(X
n S

GO

CD

00

2548.

(n-r I) In (n+ 1)

fl.=J

~ (X_I)211

n=l

(_l)n+1

CIJ

L. (_1)"-1 (x-S)"
n3

(x-2)n

L x::.
co

(x-3)Z"

n=l

2547

L (n+l)ln(n+I)
n=J

crJ

'1 ==J

(x+ I)n

(n+ I) In 2 (11+ I)

n=l

x nl

L 2n~n:n"
n=J

CX)

2556.

crJ

2546.

L
n=1

x U1

n=1

2545.

co

n=l

2544*.

nn

n=1

2555.

2543*.

(2n_I)2n (x_I)n
(3n-2)2fJ

11=2

2542**.

L (_I)n+1 X
X

x ~ n3 n lnn

2541. ~

n=l

n=J

2540. ~

~n4n

(x- 2)11
~ (2n-I)2 n

2539. ~ nlx

X+5)211-1

2552. ~

~ n~l (~
~

2551.

CX)

co

co

n=l

/1=0

2538.

L(
co

~~

~ (3n-2) (X-3)1I
~ (n+l)!2/J + 1
ClO

nn (x + 3)n.

2563. L(-I)"
n=0

t2n

(x-3~

+1) Y

II

-t

1.

Functional Series

Sec. 2]

309

Determine the circle of convergence:


ex

2564.

L inzra.
n=o
ao
,..,

2569.

ao

2570.

n=l
ao

(z-2i)"

n3 n

~ Z211

+ ni) zn.
2567. ~ 2n
n=o
n=o
(1 +2i) + (1 + 2i) (3 + 2i) z + ... +
+ (1 + 2i) (3 +2i) . .. (2n + 1 + 2i) zn -I- .
Z
Z2
1 + l - i + (I-i) (1-2i) -t-

2565. ~ (1
2568.

ao

2566. ,...,

+ (I -

z"
i) (1 - 2i) (I - ni)

L Cn~2~ir z",

n=o

2571. Proceedi ng from the definition of uniform convergence,


prove that the series
1 +x+x2 + ... +xn + ...
does not converge uniformly in the interval (-1, 1), but converges uniformly on any subinterval within this interval.
Solution. Using the fortnula for the sum of a geometric progression, we
get, for I x I < 1,
xn + 1
Rn'(t) --X'I+I + ,xfJ + 2 + -I-x
--Within the interval (-1, 1) let us take a 5ublnterval {-I +a, I-a), \\'here
a is an arbitrarily small positive number. In this subinterval I x I ~ I-a,
Il-x'~a and, consequently,
(l-a)n+1
I R,. (x) I c=;
.
u

To prove the uniform convergence of the given series over the subinterval
(-l+a, I-a], it (TIust be shown that for any >0 it is possible to choose
an N dependent only on e such that for any II > N we ",'ill have the ine
quality I R n (x) 1< e for all x of the subinterval under consideration.
(I_a)n+l
Taking any e>O, let us require that
< e; whence (l-a)1J+ I<ea,
a

that is, n + I > h~~l(~~) [since In (I-a) < 0) and


In (ea)
.
In (ea)
.
n>ln(l_a)-l. Thus, puttIng N In(l-a)-I, we are convinced that
when n > N, I Rn (x) I is indeed less than e for all x of the subinterval
[-I a, I-a] and the uniform convergence of the given series on any sub
interval within the Interval (-I, 1) is thus proved.
As for the entire interval (-I, I), it contains points that are arbitrarily
.
. xn + J

closetox=l.andsince hm Rn (x)=lIm -1 -=ClO, no nlatteJ how large n IS..


%-+1
x .... J -x

(n

+ 1) In (I-a) < In (ea),

Series

310

[Ch.8

points x will be found for which Rn (x) is grea ter than any arbi trarily large
number Hence, it is impossible to choose an N such that for n > N we
would have the inequality I R" (x) 1< 8 at all points of the interval (-1, 1),
and this means that the convergence of the series in the interval (-1, 1)
is not uniform.

2572. Using the definition of uniform convergence, prove that:


a) the series
X

x2

1 +Ti -t- 21+

xn

.. -1- 11T+ ..

converges uniformly in any finite interval;


b) the series
x2

X4

,,\6

T-2"+3- -tconverges

uniformly

throughout

(_l)"-J x ::n

the

+ ...

interval of convergence

(-1, 1);

c) the series
1

1 ++3x+

.. -t;;X+ ...

converges uniformly in the interval (1 -I-i), (0) \vhere B is any


positive nUlnber;
d) the series
(X 2 _X 4 )

+ (x<l-x )-t-(X6 _X8) + ... + (X21J_X211+2) +_ ...


8

converges not only within the interval (-1, I), but at the extremities of this interval, however the convergence of the series in
(- 1, 1) is nonuniform.
Prove the uniform convergence of the functional serIes in the
indicated intervals:
CD

2573.

l: ::

on the interval [-I, I].

n=l
00

2574. ~~
sin nx
~ 2n

over the entire number scale.

1l=1
00

2575. ~ (_l)n-J

;nn on the interval

lO,

11.

Applying termwise differentiation and integration, find the


.sums of the series:
2576.

x2

x2

2577. x- 2

2578.

x3

xn

x+ 2 +3"+ +n T
x3

...
n

+ 3- .. + (-l)n-l nx + ...
Xl

XS

x:. n -

x+3"-rs-l- +2n-l-t- ...

Taylor's Series

Sec. 8]

x 2n -

x5

Xl

311
J

+ ..

2579. x- 3 + 5 - .. +(_l)n-J 2n-l


2580. 1+2x-l-3r + ... (n-t-l)x n + ...
2581. 1-3x2 -t- 5x 4 - +(_l)n-J (2n-l)x 2n - 2 + ...
2582. 12-t-23x+34x2
+n(n+ l)x"-I+ ...
Find the sums of the series:

+
+ ...

2583. -+
2-13+
x
x
X
x9

XS

-1- 9

2584. x+ 5

11
.. + xn+
..
X 4n _ 3

+ + 4n-3 +

(_l)n-J

2585*. 1- 3 . 3 + 5.32-7.3:1+ ... +(2n-l)3 1l 1

2586. 2+22

+ ...

2n-l

+2"3+ .. +2'-z+ ..

Sec. 3. Taylor's Series


to. Expanding a function in a power series. If a function f (x) can beexpanded, in son1e neighbourhood I x-a I < R of the point a, in a series of
po\vers of x-a, then this series (called Taylor's series) is of the form

._,

r (a)

,(nl (a).

1(.t)-f(a)+1 (a) (x-a)+2t(x-a) ++-n,-(x-a)

+...

(1)

\Vhen a=-O the Taylor scri~s is also called a Maclaurin's series. Equation (1)
holds If \vhen I x-a I < R the rellzainder ternz (or sin1ply remainder) of the
Taylor series

RrI (x) = f (x) -

f (a)

L 1 kl(a) (x-a)k1
tl

(k)

-+

k=J

as n --+ 00.
To rva I uate the r C111 ain der,

Rn (x) =

(x

a)"+J

(~t-l )1

0 ne

+ J) [a

can n1 a k e

USC 0 f

+ 0 (x-a),

the forin uIa

\vhere 0

<0<1

(2)

(Lagran!e's lo"n).
Exanlple t. Expand the function 1(x) = cosh x in a series of powers of x.
Solution. We find the derivatives of the given function 1(x) = cosh x.
I' (x) =-= SInh x, f" (x)Ul=:: ccsh x, {'" (x) = sinh x, ... ; generally. f(n) (x) = cosh X,
if n is even, etnd , ) (x) == sinh x, if n IS odd. Putting a= 0, we get 1(0) = 1,
1'(0)=-=0,1"(0)=-1,1"'(0):.=0, ... ; generally, [<")(0)=1, if n is even, and
I(n) (0) =- 0 if n is odd. Whence, from (1), we have:

coshx=1

x2

x4

x2n

+21+41+ + (2n)! +...

(3)

To determine the interval of convergence of the series (3) we apply the:


d' Alembert test. We have

. I

11111

2n

x +2

n ~ 00 (2n

,\ 2n

+2)1 : (211
)t

= ltm

n -+ etJ (2n

+1) (211 +2)

Serie~

312

[Ch. 8

for any x. Hence, the series converges in the interval - 00


remainder term, in accord with formula (2), has the form:

< x<

00. The

xn + 1

Rn (x) = (n + 1)! cosh e x, if n is odd, and


..\n+1

Rn (x) =

(n

sInh ex, if n is even.

+ 1)!

e > 1, it follows that


e~x+e-6x
I cosh e x 1=
2
~ e' x I,

Since 0>

I sinh e x 1=

le~x_e-~xl

<; el x I,

Ixl n + 1

~,.

and therefore I R n (x) J.,;;;; (n + 1)1 e' x I. A series with the gen~r81 term nl
converges for any x (this is made immediately evident with the help of
d' AleInbcrt's test); therefore, in accord with the necessary condition for
convergence,

. I xl n + 1
11m - - - = 0
lZ -+> 00

(n

+1)!

'

and consequently lim Rn (x) = 0 for any x. This signifies tha t the sum of the
n-+oo
sert~s {3) for any x is indeed equal to cosh x.
2~. Techniques employed for expanding in

power series.

Making use of the principal expansions


I.

e =1

xn

x2

+fi+2f+'" + iiT+'"

(-00

.
x ~ I x5
n
II. slnx=n-31+5f- ... +(-I)
x

x"

< x<

(2n+l)I+'"
x

2n

11 I. cos X == 1 -

21 + 4f - ... + (_I)n (2n)1 + ...

IV. (1+>mt-l
x -

+ mTI x + m(m-l)
21
x + .

(-00

<X<

00),

(- 00 < x < 00),

... +m(m-I).~.!{I1l-n+l)xn+...
V

00),

x2n + I

~2

Xl

xn

(-I <x<I)*).

In(l+x)::,:::x-~+3- ... +(-l)n-ln+'"

(-I<x~I),

-and also the formula for the sunl of a geometric progregsion, it is possible,
in many cases, simply to obtain the expansion of a given function in a power series, without having to investigate the remainder term. It is sOllletim('s
advisable to make use of termwise differentiation or integration when expanding a function in a series. When expanding rational functions in power
'S~ries it is advisable to decompose these functions into partial fractions.
*) On the boundaries of the interva1 of convergence (i. e., when x = - 1
and x = 1) the expansion 1\9 behaves as follows: for m;:;: 0 it converges abso
lutely on both boundaries; for 0 > m > -1 it diverges when x == - 1 and
~onditlonally converges when x = 1; for m E;;-1 it diverges on both boundaries.

Tuy/or's Series

Sec. 8]

313

Example 2. Expand in powers of x *) the function


3

t(X)=(I_x) (1 +2x)'
Solution. Decomposing the function into partial fractions. we will have
1
2

!(X)=I-x+l+2x'
Since
r:IJ

_1_= 1+x+x 2 -l- ...


I-x

==,~ xn

(4)

n=o

and
tT-

~2X = 1-2x + (2X)2_ =- L

n n
(-l) 2 x n ,

(5)

+ (_1)n 2 n + 1 ] x n .

(6)

n=o

it follo\vs that we finally get


f(x) =- ~ x n +2 ~ (_l)n 2 n x n = ~ [I
n=o

Il=O

n=o

The geometric progressions (4) and (5) converge, respectively, when I x I < 1
1
1
and I x I < "2 ; hence, formula (6) holds for I x I < 2' i. e., when
1

-"2<x<"2'
30 Taylor's series for a function of two variables. Expanding a function
of two variables f (x, y) into a Taylor's sertes in the neighbourhood of a
point (a. b) has the fornl

f (x, y) =f (a,
+(y-b)

+ -it [(x-a) ~ + (y-b) ~] f (a, b) + ~1 [(x-a)

b)

f (a,

b)

+... + ~ [(x-a) ~+ (y-bl ~r f(a. b) +...

If a = b = 0, the Taylor series


notation is as follows:

a
[ (x-a) ax + (y -

a]

b) ify

I (a, b)

IS

then called a ,Haclaurtn's sertes.

at (x, y)
ax

(x-a)

+ at (;~

x=a

iJJ
(x~a)ax+(y-b)ay
a

f(a,b)=

1(x,

ax"

Her~

(7)

the

(y-lJ);

x=a

y=b
2

tx+

y=b

y)

(x-ar~ -I-

x=a
IJ=b

(x-a)(y-b)

x=a

y=b

+ a2f(x, y)

(y-b)2 and so forth.

ay!

x=a

IJ=b

*) Here and henceforward we mean "in positive integral powers".

~14

Series

[Ch.8

The expansion (7) occurs if the remainder term of the series


iJ
a]k {(a, b) } -+0
L kI1 [ (x-a)ax+(y-b)ay

Rn(x, y)=f(x, y)- { {(a,b)+ n

k=1
8S

n --+

00.

The remainder term may be represented in the form


1

a] n+1 f(x,Y)

Rn(x, Y)+(n+l)! (x-a)ax+(y-b)ay

x=a+~ (x-a)
y=b+'J (y-b)

,v here 0 < e < 1.

Expand the indicated functions in positive integral powers


of x, find the intervals of convergence of the resulting series and
iJlvestigate the behaviour of their remainders:
2587. aX (a> 0).
2589. cos (x -t- a).
. (
1(; )
2590. sin! x.
588
2
Sin x +4
2591*. In (2+x).
Making use of the principal expansions I-V and a geometric
progression, write the expansion, in powers of x, of the follo\ving
functions, and indicate the intervals of convergence of the series:
2x-3
2598 cos 2 x.
2592. (x-l)2.
3x-5

2593. x2-4x + 3

2599. si n 3x -t- x cos 3x.

2594. xe- 2X

2600.

X2

2595. e

2596. sinh x.

2601.

x
9+x 2

4-;\2

2602. In 11 +x .
-x

2597. cos2x.
2603.ln(l+x-2x 2 ).
Applying differentiation, expand the following functions in
po\\'ers of x, and indicate the intervals in which these expansions
occur:
2604. (l+x)ln(l+x).
2606. arcsinx.
2605. arctanx
2607.ln(x+VI+x 2 ).
Applying various techniques, expand the given functions in
po\vers of x and indicate the intervals in which these expansions
occur:
2
2608. sin2 xcos 2 x.
2612 x - 3x + 1
2
x

x
-5x+6
2609. (I + x) e-
2613.
cosh'
x.
2610. (I + eX)I.
1
2614. 4
-4
2611.
x.
-x

Vs +

Taylor's Series

Sec 3]

+ 3x + 2).
SSi:X dx .

2618.

2616.

r
x

2615. In (Xl

315

n (I ~X) dx

2619.

2617. ~e-x2dx.

SVI-x".
dx
o

Write the first three nonzero terms of the expansion of the


following functions in powers of x:
2620. tan x.
2623. sec x.
2621. tanhx.
2624. lncosx.
2622. el.O~X.
2625. eX sin x.
2626*. Sho\v that for computing the length ot an ellipse it is
possible to make use of the approxinlate formula
s

~ 2na ( I - ~)

where e is the eccentricity and 2a is the major axis of the


ell ipse.
2627. A heavy string hangs, under its o\vn \veight, in a catenary line y == a cosh ~,
\vhere a =.!i
and H is the horizontal
c.l
q
tension of the string, \vhilc q is the \veight of unit length. Sho\v
that for small x, to the order of x\ it 111ay be taken that the
string hangs in a parabola y=a 1- 2~.
a
2628. Expand the function x 3 -2x 2 -5x-2 in a series of
po\vers of x -I 4.
2629. f(x}===5x 3 -4x 2 -3x+2. Expand f(x+h) in a series of
powers of h
2630. Expand In x in a series of po\vers of x-I.
2631. Expand J..
in a series of po\vers of x-I.
x
2632. Expand -;
in a series of po\vers of x + 1.
x
2633. Expand
2634.
2635.
2636.
2637.

Expand

X2+~X+2 in a
X2+~X+7 in a

+4.
x +2.

series of powers of x
series of powers of

Expand eX in a series of pqwers of x + 2.


Expand Vi in a series of powers of x-4.

x-i.
2638. Expand cos x in a series of po\vers of x- T.
2639*. Expand lnx in a series of powers of :+;.
Expand cosx in a series of powers of
2

Series

316

[ell. 8

2640. Expand .. r"


in a series of powers of 1 +"
, 1+%
x
2641. What is the magnitude of the error if we put approximately
e~ 2

1
1
1
+ 2f+3f+41?

2642. To what degree of accuracy will we calculate the number ~, if we make use of the series
arc tanx=x-

Xl

XS

+5-.'

by taking the sum of its first five terms when X= I?


2643*. Calculate the number ~ to three decimals by expanding the function arc sin x in a series of powers of x (see Example 2606).
2644. How many terms do we have to take of the series
x2

cosx= 1- 21 + ... ,
in order to calculate cos 18 to three decimal places?
2645. How many terms do we have to take of the series

Xl

slnx=x-3f+ ... ,
to calculate sin 15 to four decimal places?
2646. How many terms of the series
x

x
=1+ 11x '-21+
...

have to be taken to find the number e to four .decimal places?


2647. How many terms of the series
x2

In (1 +x) =x-2"+ ... '

do we have to take to calculate In 2 to two decimals? to 3 decimals?


2648. Calculate
to two decimals by expanding the function V 8 -l- x in a series of powers of x.
2649. Find out the origin of the approximate formula
r
Va' +x~a+;a (a>O), evaluate it by means of V 23 , putting
a = 5, and estimate the error.
2650. Calculate
19 to three decimals.

vr
V

Sec. 3]

317

Taglor's Series

2651. For what values of x does the approximate formula


x2

cosx~l-

yield an error not exceeding 0.01? O.OOI? O.OOOI?


2652. For what values of x does the approximate formula
sin x~x
yield an error that does not exceed 0.01? 0.001?
1/2

Sinx

2653. Evaluate

S-x-dot to

I s.
four decllna

2654. Evaluate ~ e- x2 dx to four decimals.


o
1

2655. Evaluate ~ Vxcosxdx to three decimals.


o

r
1

.
I s.
2656. Evaluate .J sin
yXx dx to three declma
o
1/4

2657. Evaluate ~

VI +x

dx to four decimals.

o
lID

2658. Evaluate ~ Vi eX dx to three decimals.


o

2659. Expand the function cos (x - y) in a series of powers


of x and y, find the region of convergence of the resulting series

and investigate the remainder.


WI ite the expansions, in powers of x and y, of the following
functions and indicate the regions of convergence of the series:
2660. sinxsiny.
2663*. In(l-x-Y1-xy).
2
266 t. sin (x + gl).
~664"'. arc tan ; + y
2662*. l-x+ It
l+x-y

2665.

f (x,

y)

-xy

= ax" -1- 2bxy + cy". Expand f (x + h,

Y -f k) in po-

\vers of hand k.

2666. f (x, y) = x' -2y' -1- 3xy. Find the increment Qf this
function when passing from the values x= 1, y=2 to the values

x == 1 + h, !I = 2 + k.
2667. Expand the function eX +Y in powers of x-2 and y + 2.
2668. Expand the function sin (x y) in powers of x and

1(;

Y-2

[Ch.8

Series

318

Write the first three or four terlTIS of a power-series expansion


in x and y of the functions:
2669. eX cos y.
2670. (1 +X)I+Y.

Sec. 4. Fourier Series


to. Dirichlet's theorem. We say that a function f (x) satisfies the Dirichlet conditions in an interval (a, b) if, in this interval, the function
1) is uniformly bounded; that is I f (x) I ~ M when a < x < b, where M

is constant;
2) has no more than a finite number of points of discontinuity and
all of them are of the first kind
[Le., at each discontinuity ;
the function f (x) has a finite limit on the left f (~-O) = linl f (~-e) and a

f (;+0)= lim f (;+e) (e > O)J;

finite limit on the right

8~O

~o

3) has no more than a finite number of points of strict extrenlum.


Dirichlet's theorem asserts that a function f (x), which in the interval
(-xt, n) satisfies the Dirichlet conditions at any point x of this interval at
which f (x) is continuous, may be expanded in a trigonometric Fourter sertes:

f (x)

= ~ +a

cos x + b1 sin x + a 2 cos 2x -1- b2 sin 2x + ... + an cos nx +

--1- bn si n nx -1- . , ( 1)
where the Fourier coefficlents an and bn are calculated frorn the formulas
I
an=n-

-~

-n

S
I
. (n=l, 2, ... ).
f(x)cosnxdx(n=O,
1,2,S
.. ); bn=n
f(x)slnnxdx

If x is a poinf of discontinu ity, belonging to the interval (-1t, Jl), of a


function f (x), then the SUITI of the Fourier series S (x) is equal to the anthl11Ctical mean of the left and right limits of the function:
I
S (x) =2" [f (x-a)

At the end-points of the interval


S (-n)

=S

+ f (x + 0)).

x = - xt and x = xt,

(xt) =2" [f (-xt

+ 0) + f (1t-O)].

2. Incomplete Fourier series. If a function


(l)

f (x)], then in formula

f (x) is even li. e., f (- x)

bn=O (n= 1, 2, . )
Clnd
n

an =

~ St (x) cos nx dx
o

(n =

0, I, 2, .. ).

Fourier Series

Sec. 4)

319

If a function f(x) is odd [i.e., {(-x)=-{(x)], then an=O (n=O, 1,2 . )


and
n
bn =

5f

(x) sin nx dx (n

= I,

2, .. ).

A function specified in an interval (0, xt) may, at our discretion, be continued in the interval (- n, 0) either as an even or an odd function; hence,
it may be expanded in the interval (0, xt) In an incomplete Fourier series
of sines or of cosines of nlliltir-le arcs.
3. Fourier series of a period 21. If a function {(x) satisfies the Dirichlet
conditions in SOlTIe Interval (-I, 1) of length 21, then at the discontinui ties
of the function belonging to this interval the following expansion holds:
a0

{(x) ~-2+al cos

xtx

2nx

xtx

. 2nx

T + b1 sin T +a 2 cosl +b 2 sin -l-+'

nxtx
. nrtx
.. +ancos-l-+bnsln - l

+... ,

where
l

1
an = T

5{

(x)

nnx dx (n = 0, I, 2, . ),
cos 1-

-I

(2)

1
bn=y

5f
-I

. nxtx
1 2 , ... ).
(x) SUl- dx (n=,
l

At the points of discontinuity of the function f (x) and at the end-points


l of the interval, the SUlll of the Founer series is defined in a manner

X=

similar to that which \ve ha've in the expansion in th~ interval (-xt, n).
In the case of an expansion of the functton f (x) in a Fourier series in
an arbitrary Interval (a, a+2l) of length 2l, the limits of integration in
fornlulas (2) should be replaced respectively by a and a-1-2l

Expand the follo\ving functions in a Fourier series in the


interval (- re, n), deterrnine the surTI of the series at the points
of discontinuity and at the end-points of the interval (x = - Jt,
X == n), construct the graph of the function itself and of the sum
of the corresponding series [outside the interval (- re, n) as well]:
CI when -n<x~O,
2671. f(x)= { c2 \vhen O<x<1t.
Consider the special case \vhen c, = --1, c2 = 1.
ax when -- n < x ~ 0,
2672. f (x) = { bx when 0 ~ x < n.
Consider the special cases: a) a = b = 1; b) a = - 1 b = 1;
c) a=O, b=l; d) a=l, b=O.
2673. f (x) = x 2
2676. f (x) = cos ax.
2674. f (x) = eax.
2677. f (x) = si nh ax.
2675. f (x) = sin ax.
2678. f (x) = cosh ax.
2679. Expand the function f (x) = rr 2 x in a Fourier series in
the interval (0, 2n).
t

320

Series

(Ch. 8

=:

2680. Expand the function f (x)


in sines of multiple arcs
in the interval (0, 31). Use the expansion obtained to sum the
number series:
1

a) 1- 3

c) 1-5

+ 51 -

+ ... ;

b) 1+5-7-n+13+17- ... ;

1
1
1
+"7-n+
13- ...

Take the functions indicated below and expand them, in the


interval (0, Tt), into incomplete Fourier series: a) of sines of
multiple arcs, b) of cosines of multiple arcs. Sketch graphs of
the functions and graphs of the sums of the corresponding sel ies
in their domains of definition.
2681. f (x) = x. Find the sum of the following series by means
of the expansion obtained:
1

1 -t- 32 + 52 +
2682. f (x) = Xl. Find the sums of the follo\ving number series
by means of the expansion obtained:
1

+ 3 + ;

1) 1 + 22

2683.

f (x) = eax .

2684.

f (~) = f

1 when 0 < x < ~

2Z'- 32 -

2) 1-

1
42

-r .

l 0 when i o=;;;;;x<n.

x when 0<x~2'

2685. f (x) = J

l n-x when ~ <x<n.

Expand the following


sines of multiple arcs:
2686.
2687.
2688.

f (x) =
f (x) =
f (x) =

functions, in the

when 0 < x 0=;;;;; ~

interval (0, n), in

o when 2 < x < n.

x (n-x).

sin

Expand the following functions, in the interval (0, n), in cosines of multiple arcs:
2689. f (x) = { 1 when 0 < x ~ h,
o \vhen h < x< 31.

Fourier Series

Sec. 41

2690.

f (x) = {

2691.

f (x) = x

2692.

f (x) =

~21

I _..!- when 0 < x ~ 2h,


02h

when 2h<x<n.
sin x.
COS

x when 0 < x ~
n

i- '

-cosx when 2"<x<n.

2693. Using the expansions of the functions x and Xl in the


interval (0, n) in cosines of nlultlple arcs (see Problems 2681 and
2682), prove the equality

2694**. Prove that if the function f (x)

1( ;- + x ) = -I ( ~ - x ).

is even and we have

then Its fourier series in the interval

(-n, n) represents an expansion in CaStrIeS of odd lTIultiple arcs,

x)

and if the function f (x) is odd and f ( ~ -I= f ( ~ - x ) I then


In the interval (-n, n) it is expanded in sines of odd multiple arcs.
Expand the rollo\ving functions in l:ourier series in the Indicated intervals:
2695. f (x) = I x I (- 1 < x < I).
2696. f (x) = 2x (0 < x < 1).
2697. f(x)=e x (-l<x<l).
2698. f(x)=lO--x (5<x<15).
Expand the rollo\vin:{ functIons, in the indicated intervals,
in incoInplete Fourier series: a) In sInes of ~llultlple ales, and
b) in cosines of multiple arcs:
2699. f (x) = 1 (0 < x < 1).
2700. f (x) = x (0 < x < I).
2701. f(x)=x 3 (0< x<2n).

x when O<X~ 1,
2-x when I <x< 2.
2703. Expand the following function in cosines of multiple
arcs 10 the interval (~. 3):
2702. f(x)= {

f (x) = I

when 2" < x :e;;; 2,


\ 3-x when 2<x<3.

11-1900

Chapter IX

D1FFERENTIAL EQUATIONS

Sec. 1. Verifying Solutions. Forming Differential Equations of Families of


Curves. Initial Conditions
1. Basic concepts. An equation of the type
F(x, y, y' . ... , y)(n)==o,

(I)

where y=y (x) is the sought-for functi l1 n, is called a diUerential equatton of


order n. The function y=(p(x), which converts equation (1) into an identity,
is called the solution of the equation, while th~ ~raph of this function IS
called an integral curve. If the solution is represented irnplicitly, (D (.-t, y) .:=0,
then it is usually called an lntegral
Example t. Check that the function y = sin x is a solutton of the equation
y"+y=O.
Solution. \Ve have:
y'=cosx, y"=-sinx
and, consequently,
y" -t y == - sin x sin x === 0.
The Integral
(1) (x, y, C t , . . . , Cn) === 0
(2)

of the differential equation (1), which contains n independrnt arbitrary constants Ct , , Cn and is equivalent (in the given region) to equation (I), IS
called the general tntegral of this equation (in the respective region). By ass\~n
ing definite values to the constants Cit ... , CIl in (2), we get fJarLlcullr
tntegrals.

Conversely, if we have a family of curVf\S (2) and elirninate the raranl


eters CIt ' Cn from the system of equations
(1)=0,

d<D
dx=O,

dn<D
... , dxn=O,

we, generally speaking, get a differpntial equation of type (I) whose general
integral in the corresponding region is the relation (2).
Example 2. Find the differential equation of the family of parabolas
y:=. Ct (X-C Z)2.
(3)
Solution. Differentiating equation (3) twice, we get:
y'=2C.(x-C z) and y"=2C t

(4)

Eliminating the parameters Ct and C2 from equations (3) and (4), we obtain
the desired differential equation
2yy" =y,l.

t'ertfyinR Solultons

Sec 1)

323

It IS easy to verify that th~ function (3) converts this equation into an
identi ty.
2. Initial conditions. If for the desired particular solution y ==Y (x) of a
differential eq uatto n
y(n) = f (x, y, y', . , y(n-J))
(5)
the inltlal conditions
y (x o) -= Yo,

y' (x o) = y~,

, y(n - I ) (x o) = y~n -I)

are given and we know the general solution of equation (5)


y=cp(x, Cit ... , Cn)'
then the arbitrary constants CI ,
[roln the system of equations

Cn are determined (if this is possible)

Yo -= cp (x o, CI , , Cn),
y~ == cp~ (x o, Cit ... , Cn),

Example 3. Find the curve of the family

Y= C.e x + C2e- 2X ,
for which y (0) = 1, y' (0)

=:: -

Solution. We have:

Putting x=O in

(6)

2.
y' = Cle x -2C 2e- 2X

fOflnula~ (6)

and (7), we obtain

1=C1 +C 2 ,

(7)

-2~CI-2C2'

\\'hence
and, hence,

Determine \\J"hether the indicated functions are solutions of the


given differential equations:
2704. xy'=2y, y=5x 2
2705 Y '2 =X 2 +y,
2
t
Y=x.
CI

Xi

2706. (x l- y) dx -t- x dy -= 0, Y = 2x
2707. y"+y=O, y=3sinx-4cosx.
2708. :;: + Ct)2 X = 0, X = C. cos rot + C2 sin cot.
2709. y"-2y'+y=O; a) y=xe", b) lJ=x 2 e".
27 to. y" - (AI +- A2 ) y' + A1 A2 Y = 0,
!I = CleA,x -~ C 2e 'A x.
Show that for the given differential equations the indicated
relations are integra Is:
27\11. (x-2y)y'=2x-y, xl-xy ~ y2=::C".
J

11*

DtUerential Equations

324

lCh. 9

2712. (x-1/+1)y'=1, y=x-l-Ce'.


2713. (xy-x)y" + xy'2 + yy' -2y' = 0,

Y= In (xy).
Form differential equations of the given families of curves
(C, C I , C a , C, are arbitrary constants):
2714. y=Cx.
2721. In":'= 1 +ay
2715. y = ex!.
(a !!IS a parame ter).
2
2C
2716. Y = x.
272 (
)2 2
2. y-yo = px
2717. xl+y'=C 1
x
(Yo' P are parameters).
27 18. y= Ce.
2723. -C 2X-t-C e-x
...a
C 2 2)
Y - Ie
2

2719. JI. = (x - y ' .


2724. Y = C\ cos 2x+ C 1 sin 2x.
II
2720. y 2 -t.. .!.=2+Ce- 2 .
2725. y=(CI+C~x)ex+Ca

x
2726. Form the~ differential equation of all straight lines in the
I

xy-plane.

2727. Form the differential equation of all parabolas with

vertical axis in the xy-plan~.


2728. Form the differential equation of all circles in the
xy-plane.
For the given families of curves find the lines that sati5fy
the given initial conditions:
2729. X 2 _ y2 = C, Y (0) = 5.
2730. y=(C I -l-C 2x)e 2X , y(O)=O, y'(O)=l.
2731. y=C l sin(x-C 2 ), y(n)=l, y'(n)=O.
2732. y = Gle- x + C2 ex + C aelx;
y (0) = 0, y' (0) = 1, y" (0) = - 2.

Sec. 2. First-Order Differential Equations


1. Types of first-order differential equations. A differential equation of
the first order in an unknown function y, solved for the derivative y', is of
the form
y'=f(x, y),
(1)
where f (x, y) is the given function. In certain cases it is convenient to
consider the variable x as the sought-for function, and to write (1) in the
form
x'=g(x, y),
(1 ')
1
where g (x, y) = f (x, y).
Taking into account that y' = :~ and x' =:;. the differential equations
(1) and (1') may be written in the synlmetric form
p (x, y) dx+ Q (x, y) dy= 0,
(2)
where P (x, g) and Q (x, y) are known functions.
By solutions to (2) we mean functions of the form y=cp (x) or x='\I' (g)
ahat satisfy this equation. The general integral of equations (1) and (1 '), or

Sec. 21

First-Order DiOerential Equations

325

equa Hon (2), is of the form


<D (x, y, C) =0.

where C is an arbitrary constant.


20 Direction field. The set of directions
tana=f(x, y)

is called a direction field of the differential equation (I) and is ordinarily


depicted by means of short lines or arrows inclined at an angle Q.
Curves f (x. y) =-= k, at the points of which the inclination of the field
has a constant value, equal to k. are called isoclines. By constructin~ the
isoclines and direction field, it is possibl~, in the simplest cases. to give a

rough sketch of the field of int~gral curves, regardin~ the latter as curves
which at each point have the given direction of the field.
Exalnple 1. USing the method of isoclines, construct the field of integral
curves of the equation
y' :=2X.

Solution. By constructing the isoclines x = k (straight lines) and the direction field. we obtain approximately the field of integral curves (Fig. 105).
The family of parabolas
Xl

y=-+C
2

is the ~eneral solution.


Using the method of isoclines. make approximate constructions of fields
of integral curves for the indicated differential equations:

2733.
2734.
2735.
2736.

y' = - x.
y' = - ~
.
y
y' = 1 f- yl.
y' =x+y .

x-y

2737. y'=xl+y

326

Dlfferenttal

3. Cauchy's theorem. If a function

Equations

f (x,

y)

[Ch. 9

is continuous in some region

U {a < x < A, b < y < B} and in this region has a bounded derivative
/~ (x, y), then through each point (xo, Yo) that belongs to U there passes one
and only one integral curve y=ep (x) of the equation (1) (ep (xo)=Yo]'

4. Euler's broken-line method. For an approximate construction of the


integral curve of equation (1) passing through a given point Mo (xo, Yo)' we
replace the curve by a broken I ine wi th vertices M i (xi, Yi), where
Xi+l=Xi+L\ Xi,

Yi+l=Yi+L\Yi'

AXi=h (one step of the process),


AYi = hI (xi, Yi) (i = 0, I, 2, .. ).

Example 2. Using Euler's method for the equation

find y (I), if y (0) = I (h


We construct the table:

= 0.1).

xi
I

0
0.1
0.2
0.3

0
I
2

3
5

.6

9
10

Yi

0.5

1
1
1.005
1 015
1.030
1.051

0.6

1.077

0.7
0.3
0.9
1.0

1.109
1.148
1.194
1.248

o4

L\

xiYi

lJ;= 20

o 005
0.010
o 015
o 021

o 026

o 032

0.039
o 046
0.054

Thus, y (1) = 1.248. For the sake of comparison, the exact value is
1

Using Euler's method, find the particular solutions to the


given differential equations for the indicated values of x:
2738. y'=y, y(O)=l; find y(l) (h=O.l).
2739. y'=x-t-y, y(l)=l; find y(2), (h=O.l).
2740. Y'=-l~X' y(O)=2; find y(1) (h=O.l).
2741. y'=y_2x, y(O)=l; find y(l) (h=O.2).
II

Sec. 3)

DiOerential Equations wlth Variables Separable

Sec. 3. First-Order Differential Equations


Orthogonal Trajectories

with

Variables

321

Separable.

I". First-order eql.lations with variables separab Ie. An eq ua tion wi th variables


is a first-order equation of the type

~eparable

y' =

t (x) g (y)

( 1)

or

X (x) Y (y) dx+X 1 (x) Y. (y) dy=O


Dividin~

(I')

both sides of equation (I) by g (y) and multiplying by dx, we get

dll) = f (x) dx Whence, by integrating, we get the general integral of equag(y


hon (I) in the fonTI

5gdy

(y)

=Sf(X)dX+C

(2)

Similarly, divining both sides of (\quclhon (I') by XI (x) Y (y) and integrating,
we get the general integral of (1') in thp form
(r) dx + SY (Y) dy = C
Y (y)
S,X.X (x)
I

(2')

If for some value y=Yo \ve have R (Yo) =0, then the function Y=~/o is
also (as is directly evident) a solution of equation (I) Similarly, the straight
lincs x -=ll and y -== b will be the intet!ral curves of equation (1'), if a and b
3re, respectively, the roots of the equations XI (x) =0 and Y (y) =0, by the
lcft Sldc'i of whlch we had to divide the inItial equation.
Example 1. Solve the equation
,

Y == -

11

~ .

(3)

In particular. find the solution that sathfies the initial conditions


y(l) =2

Solution. Equation (3) may be written in the tonn


dy

dx=-x
Whence, separating variables, we have
dy =_ dx
y
x

and, consequently,
lnIYI=-lnlxl+lnC"
where the arbitrary constant In C. is taken in logarithmic form. After taking
antilogaritluns we get the general solution
(4)

where C = CII
When dlvldlng by y we could )Olie the solution y=O, but the latter is
contalned in the formula (4) for C=O.

[Ch. 9

DifJerential Equations

328

Uti1izln~ the ~fven Initial conditions, we get C = 2; and, hence, the de


sired particular solutIon is
2
y=-.
x

2 Certain di tfertntial equationc; that reduce to equations with variables


separable. Differential equation~ of the fonn
y'=f(ax+by+c) (b#O)

reduce to equations of the form (I) by means of the substitution u=ax+bu+ e,


where U IS the new sough t-for functIon
3 Ortho~onal traJectoric:s ar~ curves that intersect the hnes of the given
family <1>(x,y,a)=O ta IS a parametpr) at a right angle. If F(x,Y,y')=O
is the diflerentIal equation of the family, then
F (X. y, -

~,) =0

Is the differential equaHon of the orthogonal trajectories.


Exalnple 2. Find the ortho.Jonat trajectories of the family of
x2 + 2// 2 =- a2
Solution Difff'rentiatlng the equation (5), we find the
tion of the famll y
x -f- 2yy' -= O.

ellips~s

dlder~ntJal

(5)

equa-

Fig. 106

Whl\nce. repla("n~ y' by - y"

we get

the differential

equation ef the

orthogonal traJeclones

x_ 2y =O or u,=2y.
1/
.,
x
Integrating, we have y = ex! (lalnlly 01 parabolas) (Fig. 106).

Sec. 31

DiOerential Equations with Variables Separable

329

4. Forming difl'erential equations. \\'hen forming differential equations In


geometrical problems, we can frquently make use of the geometrical meaning
of the deriva tl ve a s the tangent of ~n angle fornled by the tangent hne to
the curve in the pos'tive x-direction. In olany cases this makes it I=ossible
straightway to cstabltsh a rplationship hptween the ordinate y of the desired
curve, its abscls~a x, and the tanllent of the angle of the tangent line y',
that is to say, to obtain the diffe.ent\al equation. In other Instances ~see
Problems 2783, 2890, 2895), u~e is made of the geometrical significance of
the dpfinite integral as the area of a curvihnear trapezold or the length of
an arc. In thi~ cac;e, by hypothesls we have a simple lntegral equation
(since the desired function is under the '\ign of the Integral); however, we
can readily pass to a differential equatlon by dlfterentiatIng both ~ides.
Example 3. Find a curve passing through the pOint (3,2) for which the
segnlent of any tangent line contained between the coordinatE:' axes is divid~d in half at the pOlnt of tangency.
Solution. Let M (x,y) be the mid-pOint of the tangent line AB. which by
hypothesis is the point of tangency (the pOlnts A and B are points of lnter')ection of the tangent line with the y- and x-axes). It is given that OA =2y
and OB = 2x. The slope of the tangent to the curve at M (x, y) is
dy
OA
!I
(jX=- VB=--X
This is the differential equation of the sought-for curve. Transforming, we gel

d\+~=o
x

and, conscquentl y,
lnx+lny -=lnCorxy==C.
Utilizing the initial condition, \ve determlne C =32 ==6. Hence, the desired
curve is the hyperbola >.y =.:. 6.

Solve the differential equations:


2742. tan xsin 2 ydx-t-cos 2 xcotydy=O.
2743. xy' -- !I = yS
2744. xyy' =:-: 1_x2
2745. y-xy' = a (I -t-x 2 y').
2746.3 p x tan ydx-f-(1-e X )scc 2 ydy=O.
2747. y' tan x =!J.
Find the particular solutions of equations that satisfy the
indicated initial conditions:
2748. (I + eX) y y' = eX; !J =- I \vhen x = O.
2749. (xy2.t-x}dx-4-{x 2 y-y)d!J=O; y=l \vhen x=O.
2750. y'sin x=ylny; y:.=l when

x=;.

Solve the differential equations by changing the variables:


2751.
2752.
2753.
2754.

y'=(x +y)2.
[I = (8x -~- 2y + 1)2
(2x + 3y- I) dx -t (4x + 6y- 5) dy = O.
(2x-y) dx+ (4x-2y -1- 3) ely = O.

Dzt!erential Equations

330

[Ch.9

In Examples 2755 and 2756, pass to polar coordinates:


2755. y' =

~-x
y

2756. (X 2+y2)dx-xydy=O.
2757*. Find a curve whose segment of the tangent is equal
to the distance of the point of tangency from the origin.
2758. Find the curve whose segment of the normal at any
point of a curve lying between the coordinate axes is divided in
two at this point.
2759. Find a curve whose subtangent is of constant length a.
2760. Find a curve which has a subtangent twice the abscissa
of the point of tangency.
2761 *. Find a curve whose abscissa of the centre of gravity
of an area bounded by the coordinate axes, by this curve and
the ordinate of any of its points is equal to 3/4 the abscissa of
this point.
2762. Find the equation of a curve that pa5ses through the
point (3,1), for which the segment of the tangent between the
point of tangency and the x-axis is divided in half at the point
of intersection with the y-axis.
2763. Find the equation of a curve which passes through the
point (2,0), if the segment of the tangent to the curve between
the point of tangency and the y-axis is of constant length 2.
Find the orthogonal trajectories of the given families of curves (a is a parameter), construct the families and their orthogonal trajectories.
2766. xy=a.
2764. x 2 + y2 = a 2
2765. y2 = ax.
2767. (x-a)! r- y2 = a2

Sec. 4. First-Order Homogeneous Differential Equations


1. Homogeneous equations. A differential equation
p (x, y) dx+ Q (x, y) dy=O

(1)

is called homogeneous, if P (x, y) and Q (x, y) are homogeneous functions of


the same degree. Equation (I) may be reduced to the form

y'=f

(~):

and by means of the substitution y=.-:XU, where u is a new unknown function,


it is transformed to an equation WIth variables separable. We can also apply
the substitution x == yll.
Example 1. Find the general solution to the equation

Sec. 4)

First-Order Homogeneous Differential Equations

331

Solution. Put y=ux; then u+xu'=eu+u or

dx
e -ud u=-.
x

Integrating, we get u=-ln InE..., whence


x

y=-xlnln-.
x
2. Equations that reduce to homogeneous equations.

If
(2)

and

6=1::~:Io,

y=v+~. where
are found from the following system of equations,

then,puttinginto equation (2) x=u+a,

the conc;tants a and

ala+bl~+cl=O, a2a+b2~+c2~O,

we get a homogeneous ditTerenttal equation in the variables u and u. If


6=0, then, puttIng in (2) a1x-t-b 1y=u, we get an equation with variables
separable.

Integrate the ditTerential equations:


2768. y' =- JL -1.
2770. (x- y) y dx-x1dy = O.
x

2769. y':= _

x+x y

2771. For the equation (Xl +y2) dx-2xydy=0 find the family
of integral curves, and also indicate the curves that pass through
the points (4,0) and (1,1), respectively.
2772. y dx i- (2 Vxy-x) dy = O.
2773. xdy-ydx=Vx 2 -f-y 2 dx.
2774. (4x 2 + 3xy + y2) dx + (4 y l + 3xy + x 2) dy = O.
2775. Find the particular solution of the equation (x 2 -3!1)dx+
-t- 2xyriy = 0, provided that y = 1 when x = 2.
Solve the equations:
2776. (2x-y -+ 4)dy+(x-2y+ 5)dx=O.
,
1-3x-3y
2778'
x+2y+ 1
2777. Y == l+x+y
Y =2x+4y+3
2779. Find the equation of a curve that passe~ through the
point (1,0) and has the property that the segment ~ut off b) the
tangent line on the y-axis is equal to the radius vector of the
point of tangency.
2780**. What shape should the reflector of a search light
have so that the rays from a point ~ource of light are rel1ected
as a parallel beam?

DitJerential Equations

332

(Ch. 9

2781. Find the equation of a curve whose 5ubtangent is equal


to the arithmetic mean of the coordinates of the point of tangency.
2782. Find the equation of a curve for which the segment
cut off on the y-axis by the normal at any point of the curve
is equal to the distance of this point from the origin.
2783*. Find the equation of a curve for which the area conta ined between the x-axis, the curve and two ord inates, one of
which is a constant and the other a variable, is equal to the
ratio "of the cube of the variable ordinate to the appropriate
abscissa.
2784. Find a curve for which the segment on the y-axis cut
off by any tangent Iine is equal to the abscissa of the point of
tangency.
Sec. 5. First-Order Linear Differential Equations.
Bernoulli's Equation
1. Linear equations. A differential equation of the form
y' + p (x). y Q (x)
=.::;

of

( I)

de~ree

one in y and y' is called linear.


If a function Q (x) =:; 0, then equation (1) takes the form
y' + p (x) y = 0

(2)

and is called a homogeneoLls linear differential equation. In this case, the


variables may be separated, and we get the general solution of (2) in the
form

-J

P(x) dx

y=Ce.

(3)

To solve the inhomogeneous linear equation (1), we apply a method that


is called variation of parameters, which consIsts in first finding the ~eneral
solution of the respective hornogeneous linear equation, that is, retationship (3). Then, assumin~ here that C is a function of x, we seek the solution
of the inhomo~eneous equation (1) in the form of (3). To do this, we put into
(1) y and y' which are found from (3), and then from the differential equation thu5. obtainerl we determine the function C (x). We thus get the general
solution of the inhomogeneous equation (1) in the form

-f P

(x) dx

y=C (x).e

Example l. Solve the equation


y' = tan xy+cos x.
Solution. The corresponding homogeneous equation is
y' - tanxy:=:O.
Solving 1t we get:
1
.
Y ::-c.-ccsx

(4)

5J

Sec.

333

Bernoulli's Equation'

Considering C as a function of x, and differentiating, we nnd:

=_1_ de
cos x dx

sin x

+ cos 2 X

.c

Putting y and y' into (4). we get:


I
de
sin x
C
de
- - . -+--.C=tanx.--+cosx
or -=cos 2 xt
cos x dx
cos 2 X
cos X
'dx
whence
C (x) =

cos 2 xdx= } x+ { sin 2x+C\.

Hence, the general soJution of equation (4) has the form

y= (
hon

~ X+{ sin 2t+C\ ) CO~X

In solving the linear fquation (1) \ve can also make use of the substitu ..
y==uv,

(3)

where u and v are functions of x. Then equation (I) will have the form

lu' -t- P

(x) u 1v -f-

v' u -= Q (x).

(6)

If we req U Ire fha t


u'

-1- P (x) u == 0,

(7)

then fronl (7) we find ll, and fronl (6) we find v; hence, from (5) we find y.
2). Bernoulli's equation. A first-order equation of the form
y' + P (.()!J -= Q (x) y7.,

where a ~ 0 and a '# 1, is called Berflolllll'~ equation It is reduced to a linear equatioll by nleans of tlll substitution z == yJ-'1. I t is also lossib1e to
apply dire('tly the substitution y-::=.uv, or the Iuethod
of variation of paranleters.
Example 2. Solve the equation
l

y'==-y+x
x

lIrf .tl.

Solution. This is Bernoulli's equation. Pulting


\ve

~et

U'I1+I1'U=~

UI1 +x IW

or

v (U'-

/I) +v'u=x iii1.

To detennine the function u we require that the relation

u,_i u=Q
x

he fulfilled, whence we have


Putting this expression into {8), we get

V'.,,4=xYvX".

(8)

334

DiOerential Equations

whence we find v:

11=( ; lnx+c y.

lCh.9

and, consequently, the general solution is obtained in the fonn

~ lnx+C

y=x4 (

r.

Find the general integrals of the equations:


2785. dy _J!.. = x.
dx

2786. ~dY

+ 2yx =

x' .

2787*. (1 -+ y2) dx = tV 1 y2 sin y-xy) dYe


2788. y 2 dx- (2xy 3) dy = O.

Find the particular solutions that satisfy the indicated conditions:


2789. xy' + y-e" = 0; y = b when x === a.
2790. y ' - -x
-Ylz-l-x=O; y=O when x~o.
2791. y'-ytanx=_l- ; y=O when x=O.
cos x

Find the general solutions of the equations:

+.JLx =

xy2.

2792.

dc!J!.

2793.

2xY~-!I +x=O.

+ (x- ;

2794. y dx
x1y) dy=O.
2795. 3xdy=-=y(1 +xsinx-3y 3 sinx)dx.
2796. Given three particular solutions y, Yl' Y2 of a linear

equation. Prove that the expression Y2-Y rernains unchanged for


Y-Yl
any x. What is the geometrical significance of this result?
2797. Find the curves for wh ich the area of a triangle formed
by the x-axis, a tangent line and the radius vector of the point
of tangency is constant.
2798. Find the equation of a curve, a segment of which, cui
off on the x-axis by a tangent line, is equal to the square of the
ordinate of the point of tangency.
2799. ,Find the equation of a curve, a segment of which, cut
off on the y-axis by a tangent line, is equal to the subnormal.
2800. Find the equation of a curve, a segment of which, cut
otT on the y-axis by a tangent line, is proportional to the square
of the ordinate of the point of tangency.

Exact DitJerential Equations. Integrating Factor

Sec, 6]

335

2801. Find the equation of the curve for which the segment
of the tangent is equal to the distance of the point of intersection of this tangent with the x-axis from the point M (O,a).

Sec. 6. Exact Differential Equations.


Integrating Factor
1. Exact differential equations. If for the differential equation
P (x, y) dx+ Q (x, y) dy=O

(1)

the equality ~: = ~~ is fulfilled, then equation (1) may be written in the

form dU (x, y) = 0 and is then called an exact dtUerential equation. The general integral of equation (1) is U (x, y) = C. The function U (x. y) is determined by the technique given in Ch. VI, Sec. 8, or froln the formula
x

U=

II

~p

(x, y) dx

+ ~ Q (xo y) dy
Yo

Xo

~sce

Chi VI I t Sec, 9).


Example I. Find the general integr al of the differential equation
(3x 2 + 6xy 2) dx + (6x 2 y + 4y') dy = 0,
.
a(3x 2+0 6xy2) ==
So 1Ut Ion. Th
IS IS an exac t d'ff
1 eren t'la I equa tlon, since
y

a (6(2 y -t- 4y S)

ox

:=

Here,
whence

= 12xy and, hence, the equation is of the form dU = O.

au
- =
ax
u= ~

(3.\1

3x!

+ 6xy

au

- = 6x"y + 4y'
ay'

and

+6xyl) dx+ cp (y) =x l

Differentiating U with respect to y, we find

+ 3X1y 2+ cp (y).

~aU = 6x!y -t <p' (y) = 6x!y + 4y'

(by
Y
hypothesis); from this we get q>'(U)=4tj' and q>(f)=Y"+C. We finally get
U (r, y) =XS + 3X 2y 2 + y4+ CClt consequently, X' + 3x y2 + y4=C is the sought-for

general inte~ral of the equation.


2. Integrating factor. I r the left si de of equation (1) is not a total (exact)
differential and the conditions of the Cauchy theorem are fulfilled, then there
exists a function ,.... =,.... (x, y) (integrating factor) such that
)1

(P dx+ Q dy) =dU.

Whence it Is found that the function ,.... satisfies the equation

ay (JLP)=ax (flQ)

The integra ting factor ,.... Is readily found In two cases:


1
aQ )
1) Q ay - ax = F (x), then,.., = JL (x);

(OP
1 (OP
OQ)
2) -p ay - ox = F

(Y), then ~t = Jl (y).

(2)

336

Dif1erential Equations

(CIl. 9

(2XY+X2y+~)dx+tx l +y2)dll=O.

Example 2. Solve the equation


a

Solution. Here P = 2xy + x 2y + l/3 Q = Xl + y2


and

(OP OQ) = 2x +x!x!+


+ y2-2
y2

Q oy - ox
.
Stnce

o(J..tP) iJ(~Q)
ay=--ax-

or

= 1. hence. J..t= J..t (x).

oP

iJQ

J..t oy = J..t ax

+Qd,....
dx

it follows that

dJL=.!
(ap _aQ)dX=dX
,....
Q iJy
ax

and ln~ .. =x.

JL=r.

Multiplying the equation by IJ. = eX. we obtain


eX (

2xy + x 2y + ~
3

) dx-{-e" (Xl + yl) dy -===0

which is an exact differential equation. Integrating it, we get the general


integral

Find the general integrals of the equations:


2802 (x+ y) dx + (x -1- 2y) dy = O.
2803. (Xl + y2 2x) dx -{- 2xy dy = o.
2804. (Xl -:- 3xy 2 + 2) dx - (3x 2y - y2) dy = O.

2805. xdx-ydy=

x dy-y dx

. \ 2+ Y2
2806. 2xdx+ y2 -3x dy-=O.

y'

y4

2807. Find the particular integral of the equation

(x+e~) dx+ef (1-~) dy=O.


\vhich satisfies the initial condition y (0) = 2.
Solve the equations that admit of an integrating factor of the
form ~ = 11 (x) or I..t = I..t (y):
2808. (x+ y2) dx-2xy dy = O.
2809. y(l -~-xy)dx-xdy=O.
2810. Yx dx + (y'-In x)dy = O.
2811. (x cos y - y sin y) dy + (x sin y -~ y cos y) dx = O.

~ec.

7)

Fir~t-Orde,

D;Oerential Equation, /lot Solved for Derivative

337

Sec. 7. First-Order Differential Equations not Solved


for the Derivative
to. First-order dlft'erential equations of higher powers. If an equation
F

(x, y, y') =0,

(1)

,\ hich for example is of degree two in y', the.l by solving (1) for II' we get

two equations:
y'=f,(x,y), y'= f2(X, y).
(2)
Thus, generally spt'aking, through each pOint M" (xo, Yo) of some region
of a plane there pass two integral curves. The general integral of equation
(1) then, generally speaking, h'1s the fOrln

<I> (x, y, C) === cD. (x, y, C) cD 2 (x, y, C) =0,

(3)

\vhere (D. and cI>2 are the general inte~rals of equations (2).
Besides, there may be a slnRular lllte{!ral for equilt10n ~1). Geometrically,
a singular integral is the cnvelope of a fal1lily of curves (3) and Inay be obtained by elinlinating C frolll the s~'steln of equations
<D~(X,y,C)=O

<D(x,y,C)=O,

(4)

or by eliminating p=y' fronl the system of equatIons

F~ (.r, !J,

F (x, 11, p) =-.=- 0,

p)

= o.

(5)

We note thflt the curves defined by the 1quations (4) or (5) are not
alwavs solutions of equation (I); thert'fore, in each casc, a check is necessary.
Exalnple 1. FJ nd the general and sln~ular integrals of the equation
XII'2- r 2xll'-Y~0.

Solution. Solving for y' we have t\\o hOlllogeneous cquations:


y'

=-

+ 1/ 1+ ~.

y' = -

lit'fined in the region


x(x+y)

1-

1t

>0.

the general integrals of which are

or

(2x+y-C)-2

yx

(2x+ y-C) +2

+xy=0,

Yt

+xy=O.

Multiplying. we get the general integral of the given equation

(2x+ y-C)2-4

(x 2 +xy):::0

or

(y-C)2=4Cx
(a family of parabolas).
DifTerentiatin~ th~ general integral \\'ith respecf to C and eliminating C,
\ve find the singul ar in tegral

y+x=O.
(It ll1ay be verified that y

+x = 0 is

the solution of this equation.)

338

Differential Equations

(Ch.9

It is also possible to find the singular integral by differentiating


xp2+2xp-y-=O with respect to p and eliminating p.
2. Solving a differenti al equation by introducing a parameter. If a firstorder differential equation is of the form
x= cp (y, y').
then the variables y a'ld x may be determined from the system of equations
l=aq>+aq>dp
p
ay a dy' x = q> (y, p).

\\'here p=y' plays the part of a parameter.


Similarly, if y='I' (x, y'), then x and yare deternlined from the system
of equations
a", a"'dp
p= ax + ap dx'
y='I' (x, p).
Example 2. Find the general and singular integrals of the equation
2

y=y'2_ xy'

+x2'

Solution. Making the substitution y' = p, we rewrite the equation in the


form
2

y=p2- xp

+x2

Diffelentiatiog with respect to x and considering p a function of x, we have


p=- 2pdp _p_x dp +x
dx
dx

or ~~(2P-X)-(2P-X). or~=l. Integrating we get p=x+C. Substituting


into the original equation. we have the general solution
x2
x2
y=(X+C)2- x (x+C)+2 or Y="2+CX+C2.
Differentiating the general solution with respect to C and eliminating C, we
J

obtain the singular solution: g=X (It may be verified that g=T is the
4
solut ion of the given equation.)
If we equate to zero the factor 2p -x. which was cancelled out, we get
p= ~ and, putting p into the given equation, we get g=~. which is the
same singu lar sol ution.

Find the general and singular integrals of the equations:


(In Problems 2812 and 2813 construct the field of integral
curves.)
2y,
1 0
28 t 2 y ,2 -x
U ;- =
2813.4y,Z-9x=O.
I

Sec 8]

The Lagrange and Clairaut Equations

339

2814. yy,2_(xy+ l)y' tx=O.


2815. yy,2 -2xy' + y = o.
2816. Find the integral curves of the equation y'!-t-y"=1
that pass through the point M (0, ;).
Introducing the paranleter y' =p, solve the equations:
2820.4y==x,,+y,2.
2817. x=siny'+-lny'.
2 + '2
2818 Y=Y ,2 p )',
2821 eX-~
2819. y=y'" +2Iny'.

- 2y'
Sec. 8. The Lagrange and Clairaut Equaticns
to. Lagrange's equation. An equation of the forrn
y = xfP (p) -1- 'I' (p),

(1)

\vhere p==y' is called Lagrange's equatIon Equation (1) is reduced to a linear


equation in x by differentiation a nd taking into con'Sideration that dy -= p dx:
p dx == cr (p) dx -1- [xcp' (p) + '1" (p)] dp.
(2)
If p ~ cp (p), then from (1) and (2) we get the general solution in parametric
form:
x= Cf (p)

+ g (p),

y --= [Ct (p)

+ g (p)] q) (p) + 'I-~ (p),

where p is a parameter and f (p) .. g (p) are certain known functions. Besides,
there [nay be a Singular solution that is found in the l1~ual \vay.
2. Clairaut's equation. If In equation (1) p =-.: ql (p), then we get Cla;raut's equ.atlon

y::= xp -1- 'P (p).


Its general solution IS of the fornl y;-: Cx -t- 'I' (C) (a farnl1y of ~tralght lines).
There IS also a parttcular solutiO'} (envelope) that results by clilninating the
parameter p fronl the systenl of equ~t1011~
x==- - 'P' (p),
{ y==PX+~lP)

Example. Solve the equation


)

Solution.

Puttin~

--2y'x+,.
y

y' 7-= P we have y = 2p\

ing dy by p dx, we get


p dx == 2p dx

or

Solving this lineal

+ 2x dp- ~

dx
2
1
dp=-px+ pl
~quation,

we will have
I
p

*;

x= 2 (In p+C).

(3)

different; llting and replac-

DiOerential Equations

340

(Ch. 9

Hence, the general integral will be

f ;2
X=

(In p+C).

y=2px+

To find the singular integral, \ve form the system


1
1
y=.2px+-, 0==2x- 2
p
p
In the usual way. Whence
1

x= 2p z'
and,

Y=p

con~equently,

y=

2 V2x.

Putting y into (3) we are convinced that the function obtained Is not
a solution and, therefore, equation (3) does not have a singular integral.

Solve the Lagrange equations:


2822. y =..!- x (y' +
2824. Y == (1
2
-'

P,)
.
Y

1/-1-'2-

""

2825 Y

+ y') x + y".

=-

I,
,
2 Y (2x-}- y).

y-y + J'
y.
Find the general and singular integrals of the Clairaut equations and construct the field of integral curves:
,2
'
2826 . y=xy +y .
2827. y=xy' +y'.
2828. Y = xy' + l/rl 1---(y-')-2.
2823

2829. Y = xy'

~- J,
.
y

2830. Firld the curve for which the area of a triangle formed
by a tangent at any point and by the coordinate axe\) is constant.
2831. Find the curve it the distance of a given point to any
tangent to this curve is constant.
2832. Find the curve for which the segment of any of its
tangents lying between the coordinate axes has constant length l~

Sec. 9. Miscellaneous Exercises on First-Order Differential Equation')


2833. Determine the types of diflerential equations and indicate methods for their solution:
e) xy' -t- y = sin y;
a) (x + y) y' =-= x arc tan 1!.. ;
x
f)
(y_xy')2 = y'd;
b) (x- y) y' = y2;
g) Y == xeY ';
c) y' = 2xy + Xl;
h) (y' -2xy) Vy=x l ;
d) y' = 2xy -t- y';

Sec. 9]

Miscellaneous Exercises on

Fir.~tOrder

DiOerential Equations

341

1) (x 2+ 2xy 8) dx -1+ (y2 + 3X2y2) dy = 0;


m) (x ' -3xy) dx + (x 2 + 3)dy=O;
n) (xy' + In x) dx = y2dy,

i) y' = (x +_ y)2;
j) X cos y' + y sin y' =-= 1;
k) (x 2-xy) y' = y4;

Solve the equations:

(x-ycosf)dx+xcos~dy-=O;
b) x In ~
dy-ydx == O.
y

2834. a)

(x; _yl) dy.

2835. xdx =

.+

(2 xy 2 - y) dx x dy =- O.
xy' .~. Y =-= xy2 In x.
y = xy' -l- y' In y'.
Y == xy' -1- V -ay'.
x 2 (y + I) dx+ (xS_I) (y-l) dy== O.
2841. (1 -t- y2) (e 2.-.; dx-e Y dy) -(I -1- y) dy =-= O.
2
2842. y'_y2x 2 1 =1.
2845. (l-x )y'+xy=a.

2836.
2837.
2838.
2839.
2840.

2843. yeY = (yS -t 2xeY ) y'.


2846. xy - x+ I-x = O.
2844. y'+y cos x == sin x cos x. 2847. y' (x cos y -i- a sin 2y) = I.
2848. (x 2 y-x 2 -l-y--l)dx -t-(xy+2x-3y-6)dy=O.

2849. y' = ( 1 +~;:lr


2850. xy3 dx (x 2y 2) dye
'
3.\.2
2851 .y=-= .\ I\- Y -tl
-

7=';

V11-x dx = O.

2852. 2dx+ ... /!... dy- ...

V!I

286 t. eY dx + (xeY - 2y) dll = O.


2862
=2 ' _
,2
Y, :y -j
~y
2863. Y ="A(l-+-Iny-Inx).

2853. y' = Y -I- tan ~ .


x

VI

2854. yy' + y' = cos x.


2855. xdY-I ydx=-~y!dx.
2856. y' (x -J-- sin y)

2864. (2e X + y4) dy-

== 1.

2857. Y t}E = - p -J- p2


dy
2858. x 3dx-(x 4 t- y3).1,y=-=.O.
1

2859. x y'2_l-3xYY'-l-

-r
X 2+y2

2860. x dx+ y d~

+2y 2=O.

xdy--ydx
11

=.

X
- ye dx = O.

(y + 2

)2

2865. y =2 \x+y-I
2866. xy(xyl-t-l)dy-dx=

=0.

2867. a (xy' 2y) = xyy'.


2
2868. xdy-ydx=y dx.

342

lCh.9

DiOerential Equations

2869. (x'-l )3/2 dy


2870. tan

+ (x + 3xy l/ Xl 3

1) dx = O.

dy
x dx -y = a.

+x

2871. Va 2 x 2 dy --I- (x+ y- Val


2872. xyy'8_(X 2 +y2)y'+xy=O.

dx= O.

2873. y = xy' y~
2874. (3x 2 2xy_ y 2) dx+ (x 2-2xy- 3y 2) dy =-- o.
2875. 2yp :: = 3p.,. 4y B.

+ .

Find solutions to the equations for the indicated initial conditions:


2876. y,=y+l; y=O for x= 1.
x
2877. eX-Yy' = 1; y= 1 for x= 1.
2878. cot xy' + y = 2; y == 2 for x == O.
2879. eY (y' + 1) = 1; y == 0 for x == 0.
I
2880. y' +y=cosx~ Y=2 for x===O.
)

2881. y'-2y=-x 2 ; Y=4 for x==O.


2882. y' +y=2x; !J=-l for x==o.
2883. xy' =y; a) y= 1 for x= 1; b) y=O for x=O.
2884. 2xy'=y: a) y=l for x=--= 1; b) y=O for x==O.
2885. 2xyy'+X I _ y 2=O; a) y=O for x=O; b)y==l forx=O;
c) y==O for x=l.
2886. Find the curve passing through the point (0, I), for
which the subtangent is equal to the sum of the cooldinates of

the point 'of tangency.


2887. Find a curve if we know that the sum of the segnlent~
cut off on the coordinate axes by a tangent to it is constant and
equal to 2a.
2888. The sum of the lengths of the normal and subnorlna\
is equal to unity. Find the equation of the curve if it is kno\vn
that the curve passes through the coordinate origin.
2889*. Find a curve whose angle formed by a tangent and the
radius vector of the point of tangency is constant.
2890. Find a curve knowing that the area contained between
the coordinate axes, this curve and the ordinate of any point on
it is equal to the cube of the ordinate.
2891. Find a curve knowing that the area of a sector bounded by the polar axis, by this curve and by the radius vector
of any point of it is proportional to the cube of this radius
vector.
2892. Find a curve, the segment of which, cut orr by the
tangent on the x-axis, is equal to the length of the tangent.

Sec. 91

Miscellaneous ExercIses on First-Order Differential Equations

343

2893. Find the c~rve, of which the segment of the tangent


contained between the coordinate axes is divided into half by
the parabola y2 =- 2x.
2894. Find the curve whose normal at any point of it is
equal to the distance of this point from the origin.
2895*. The area bounded by a curve, the coordinate axes,
and the ordinate of sOlne point of the curve is equal to the
length of the corresponding arc of the curve. Find the equation
of this curve if it is known that the latter passes through the
point (0, 1).
2896. Find the curve for which the area of a triangle formed
by the x-axis, a tangent, and the radius vector of the point of
2
tan~ency is constant and equal to a
2897. Find the curve if we know that the mid-point of the
segnlent cut off on the x-axis by a tangent and a normal to the
curve is a constant point (a, 0).
When forming first-order differential equations, particularly in ph\sical
problelns. it is frequently advisable to apply the so-called method of differentials, which con~ists in the fact that approxirnate relationships between
infinltesinlal i"crenlent~ of the desired quantities (these relationships are
accurate to infinitesinlals of higher order) are replaced by the corresponding
relationships between their dlfftr~nttals. This doe,; not affect the result.
Problenl. A tani< contains 100 litre,; of an aqueous solution containing
10 kg of salt. Water IS rntcnng the tank at the rate of 3 htres per minute,
and the mixture is f1owin~ out at 2 Idres per Jllinute. The concentration is
maintained unlfonn by stirring. I-fo\v tTIuch salt \vill the tank contain at the
end of one hOll r?
Solution. The concentration c of a substance is the quantity of it in
unit volume. If the concentration is uniforlTI, then the Quantity of substance in voltllTIe V is eV.
Let the quantity of salt in the tank at the end of t minutes be x kg.
The quantity of solution in the tank at that instant \vill be 100 t litres,

and. conscquent\ y. the t'Oncentralion c = 100 + t kg per Ii Ire.


Duril1~ tinle dt, 2dt litres of the solution 110\\'S out of the tank (the
solution contains 2c dt kg of salt). Therrfore, a change of dx in the quantity
of salt in the tank is given by the relationship
2x
-dx=2c dt = 100 + t dt.
This is the sought-for differential equation. Separating variables and integrating, we obtain
In x= - 21n (100+ t) In C
or
C

x = ( 100

+ t)

I .

The constant C is found fronl the fact that \\ hpn t = 0, \ -= 10. tha tis,
C= 100.000. At thr expiration of one hour, the tank will contain
X=

lOf~~~ ~ 3.9

kilograms or salt.

344

DiOerential Equations

(Ch. 9

2898*. Prove that for a heavy liquid rotating about a vertical


axis the free surface has the form of a paraboloid of revolution.
2899*. Find the relationship between the air pressure and the
altitude if it is known that the pressure is 1 kgf on 1 cm 2 at
sea level and 0.92 kgf on 1 em! at an altitude of 500 metres.
2900*. According to Hooke's law an elastic band of length
1 increases in length klF(k=const) due to a tensile force F.
By how much will the band increase in length due to its weight
W if the band is suspended at one end? (The initial lenglh of
the band is I.)
2901. Solve the same problem for a weight P suspended fr0l11
the end of the band.
When solving Problems 2902 and 2903. make use of Newton's
law, by which the rate of cooling of a body is proportional to the
difference of temperatures of the body and the ambient 1l1edium.
2902. Find the relationship between the temperature T and
the time t if a body, heated to To degrees, is brought into a room
at constant temperature (a degrees).
2903. During what tirne will a body heated to 100 cool ofT
to 30 0 if the temperature of the room is 20 and during the first
20 minutes the body cooled to 60?
2904. The retarding action of friction on a disk rotating in
a liquid is proportional to the angular velocity of rotation_ Find
the relationship between the angular velocity and tillle if it ic;
known that the disk began rotating at 100 rpm and after one
minute was rotating at 60 rpm.
2905*. The rate of disintegration of radium is proportional
to the quantity of radium present. Radium disin1egrates by one
half in 1600 years. Find the percentage of radium that has disintegrated after 100 years.
2906*. The rate of outflow of water from an aperture at
a vertical distance h from the free surface is defined by the
formula
0

v=c2gh,
where c ~ 0.6 and g is the acceleration of gravity.
During what period of time will the water filling a hemispherical boiler of diameter 2 metres Aow out of it through a circular opening of radIus 0.1 m in the bottom.
2907*. The quantity of light absorbed in passing through
a thin layer of water is proportional to the quantity of incident
light ann to the thickness of the layer. If one half of the original
quantity of light is absorbed in passing through a three-metrethick layer of water, what part of this quantity will reach a depth
of 30 metres?

Sec. 101

IJigher.Order DttJerentlal Equations

345

2908*. The air resistance to a body falling with a parachute


is proportional to the square of the rate of fall. Find the limiting velocity of descent.
2909*. The bottom of a tank with a capacity of 300 litres
is covered with a nlixture of salt and some insoluble suhstance.
Assunling that the rate at which the salt dissolves is proportional to lhe ditference between the concentration at the given time
8nd the concentration of a saturated solution (I kg of salt per 3
litres of water) and that the given quantity of pure water dissolves 1}3 kg of salt in 1 minute, find the quantity of salt in solution at the explraiion of one hour.
2910*. lhe electromotive force e in a circuit with current i,
resistance R and self-induction L is made up of the voltage drop

Ri and the electromottve force of self-i nduction L :;. Determi ne


the current i at tilne t if e ==- E sin rot (E and (J) are constants)
and i = 0 when t = O.
Sec. 10. Higher-Order Differential Equations
to. The case of direct integration. If
y(/.)

= f (.\),

then

Y=~dx~ .,. ~f(.\)dX+CIXII-I+C1Xn-I+... +Cn'

'----..--" 1 'me's
2. Cac;es of reduction of order. 1) If a dJ1Yerential equation does not
contaJn y cxpl1cltly, for instanc~,
F(x,

i/,

y")=O,

then, assuming y' = p, \\'e get an equation ot an order one unit lower:
F (x, p, p'):=O.

Example 1. Find the particular solut;on of the equation


xy" +y' +x=O,

that satisfies the conditions


y=O, y' =0 \vhen x=O.

Solution. Putting y' =p, we have !I' = p', whence


xp'+p+x=O.

Solving the latter equation as a linear equation In the function p.


we get

DtUerenltal Equations

346

[Ch 9

From the fact that y'=p=O when x=O, we have O=C.-O, Le., C1=O.
Hence,
x
p=-2

or
dy
x
dx=-2'

whence. integrating once again,

w~

obtain
x:!

y=-4"t- Ct
Putting y=-O when x= 0, we find C 2 -= O. Hence. the desired
solu lion is
1
X2
Y = _-4
.
2)

particular

If a differential equation does not contain x explicitly, for instance,


F(y, y'. y")=O

then, putting y' =p, y" =p dd P , \ve get an equation of all order one unit
y

lower:

F(y, p, p~~)=O.
Example 2. Find the particulnr solution of the pquation
yy"_y'2=y4

provided that y= 1, y'=O when x=O.


Solution. Put y'=p, then if=pddP and our equation becomes

IJP : : -p"=

~/.

We have obtained an equation of the Bernoulli type in p (y is considered


the argument). Solving it, we find
p=

VCI +y2.

From the fact that y' =p=O when y= 1, we have C1 = -1. lienee,
p=

y V y2-1

or

:~= y Vy"-1.
Integrating, we have
1

arc cos -

t}

x = C2_

I
Putting y=l and x=O, \ve obtain C2 =O, whence -=cosx or
y

y=s~cx.

Higher-Order DiUeTential Equations

Sec. 10]

347

Solve the following equations:

!-.
x

2920. yy" = yly'

+ y,l.

y"= -2~S'

2921. yy" - y' (1

+ y') =

2911. y" =
2912.

2913. y"

2922. y" =

= 1 _ y,2.

2914. xy"t y'

==

2923. (x

O.

O.

x, .

+ 1) y" -

(x -t- 2)y' + x+
+ 2 = O.

yy"=y,2.
2924. xy"=y' In y'.
x
yy,,+y,z=O.
1
,,2
"
'
(ll-x 2 )y"+-y,2+ I ==O. 2925 y +-.r y =xy.
y' (1 + y'2) = ay".
2926. xy'" + y" = 1 + x.
x1y" -t-xy' = 1.
2927. y",Z + y"Z = 1.
Find the particular solutions for the indicated initial conditions:
2928. (1 +x 2)y"-2xy'=O; y=O, y'=3 for x=O.
2929. 1 + y,Z = 2yy"; !I = 1, y' = 1 for x = 1.
2930. yy" + y,J = y,:i; y =:= 1, y' = 1 for x = O.
2931. xy"=y'; y=O, y'=O for x=O.
Find the general integrals of the following equations:
2932. yy' = V y2 + y,Z y" _ y' y".
2915.
2916.
2917.
2918.
2919.

V +

2933. yy" = y,Z -t- y' yZ y,2.


2934. y'" _ yy" = y2 y '.
2935. yy"
y,2 _ y,ol In y =

o.

Fi nd sol utions that satisfy the indicated conditions:


2936. y"y' = 1; y

= I, y' = 1

for x =

-} .

2937. yy"-t-y'2=I; y=I, y'= I for x=O.


2938. xy"=

VI

.~ y,Z; y=O for x= 1; y= I for x=e J


1

2939. y"(1 +lnx)-+-x y'=2+lnx;y=2' y'=l tor x=l.


2940. y" =

~ ( 1 + 1n ~'); y =

i ' y' = 1 for x = 1.

2941. y"_y'2+ y'(y_I)=O; y=2, y'=2 for x=O.


2942. 3y'y"=y+y,i_~1; y=-2, y'=O for x=O.
2943. y2+ y '2_2yy"=O; y=l, y'=1 for x=O.
2944. yy,+y,J+yy"=O; y=l for x=O and y=O for x=-l.

348

DiOerelltial Equations

[Ch.9

2945. 2y' + (y,2 -6x). y" = 0; y = 0, y' = 2 for x = 2.


2946. g'y. yg" -y'. = 0; y= 1, y' = 2 for x= O.

2947. 2yy"-3y,2=4y l; y=l, y'=O for x=O.


2948. 2yy" -t- yl_y,a = 0; y = 1, y' = 1 for x = O.
",.
1,
1 f
1
2949

= Y - y; Y = - 4"' y

2 or x = .

2950. Y"+;2eJ'2 Y'-2yy,2=O; y=l, y'=e for

xO-=-;e'

2951. 1 -t- yy" -1- y,Z = 0; y = 0, y' = I for x == I.


2952. (1 -t- yy') y" = (1 y,2) y'; y == 1, y' = 1 for x = o.
2953. (x+l)y"+xy,2==y'; y=-2, y'=4 forx=l.

Solve the equations:

2954. y' = xy,,2 -t y,,2.


2955. y' =xy" y" _y"z.
2956. y",1 = 4y".

2957. yy'y":=: y,3 -t- y"l. Choose the integral curve passing through
the poi nt (0, 0) and tangent, at it, to the straight line y + x = O.
2958. Find the curves of constant radius of curvature.
2959. Find a curve whose radius of curvature is proportional
to the cube of the norma 1.
2960. Find a curve whose radius of CUIvature is equal to the
normal.
2961. Find a curve whose radius of curvature is double the
normal.
2962. Pind the curves whose projection of the radius of curvature on the y-axis is a constant.
2963. Find the equation of the cable of a suspension bridge
on the assumption that the load is distributed uniformly along
the projection of the cable on a horizontal straight line. The
weight of the cable is neglected.
2964*. Find the position of equilibrium of a flexible nontensite thread, the ends of which are attached at two points and
which has a constant load q (including the weight of the thread)
per unit length.
2965*. A heavy body with no initial velocity is sliding along
an inclined plane. Find the law of motion if the angle of inclination is a, and the coefficient of friction is Jo'.
(Hint. The frictional force is J1N, where N is the force of reaction of the
plane.)

2966*. We may consider that the air resistance in free fall


is proportional to the square of the velocity. Find the law of

motion if the initial velocity is zero..

Linear Differential Equations

Sec. 111

349

2967*. A motor-boat weighing 300 kgf is in rectilinear motion


with initial velocity 66 mjsec. The resistance of the water is proportional to the velocity and is 10 kgf at 1 metre/sec. How long
will it be before the velocity becomes 8 m/sec?

Sec. 11. Linear Differential Equations


1. Homogeneous equations. The functions Yl=q>l (x), Y2=q>2(X), .
. . . , Y = q>n (x) are called linearly dependent if there are constants C., CI , , Cn
not alj equal to zero, such that
C.y. + C1Y2 + ... + CnYn e= 0;
otherwise, these functions arc called linearly t11dependent.
The general solution of a homogeneous linear differential equation
y(1I)+Pl (x) y(fJ-1)

+ ... -1- P n (x)y=O

with continuous coefficients P,.(x) (i=l, 2, ... , rz)

IS

(I)

of the fornl

y = C1Y. + C~Y2 + . + CnYn,

where Y., YI' .. , lin are linearl y independent sol utions of equa tion (I)
(fundamental system of SO/Ii/tons).
2. Inhomogeneous equations. The general solution of an inholnogeneolLs
ltnear different ial eq ua tion
y(n)
PI (.v) y(II-I)+ + Pn (x) Y =f (x)
(2)

\\,ith continuous coefficients P,. (x) and the right side

f (x) has the fonn

Y=Yo+Y,

where Yo is the general solution of the corresponding homogeneous ~qua lion (I)
and Y is a particular solution of th~ given inhomogeneous equation (2).
If the fundam~ntal system of soluHons 'il, y", ... , Yn of the homogeneous
equation (I) is kno\vn, then the general solution of the corresponding inhornogeneous equation (2) Inay be found from the formula

y= C. (x) Y. +C 1 (x)

Y.+.. +C n (x) Yn'

where the functions C;(x) (i=l, 2, .. ,11) are determined from the follo\vjog system of equations:

C~ (x) Y. -t- C; (x) Y2


C; (x)

Y; + C~ (x) y~

c~ (x) y~t1-2)

+
+

+C; (x) y~n-I) +

C: (x) y~n-.) + C~ (x) y~n-.) +

+ C;l (x) Y. =

0,

+C~ (x) y~n-I) = 0,

1
~

+ C~ (x) y~'-.) = f (x)

+C~ (\)y~==O,

(3)

(the method of uariation of parameters).


Example. Solve the equation

xy"+y'=xl

(4)

DiOerential Equations

350

Solution. Solving the homogeneous equation


xy"+y'=O,
we get
y=C 1 In x+C 1

[Ch.9

(5)

Hence, It may be taken that


Yl=lnx andY2=1
and the solution of equation (4) may be sought In the form
y =C 1 (x) In x + C2 (x).
Forming the system (3) and taking into account that the reduced fonn of
the equation (4) is y" + y' =x, we obtain
x

Whence
X'
x'
x'
CJ(x)=a+A and C2(x)=-31nx+g+B

and, consequentl y,
x'
Y=g+A

Inx+B,

where A and B are arbitrary constants.

2968. Test the following systems of functions for linear relationships:


a) x, x + 1;
e) x, x!, Xl ;
b) xs, - 2x 2 ;
f) e", e2 " , el x ;
c) 0, 1, x;
g) sin~, cos x, 1;
d) x, x + 1, x -+. 2;
h) sin 2 x, cos! X, 1.

2969. Form a linear homogeneous differential equation, knowing its fundalnental system of equations:
a) Y1 = sin x, Y2 = cos x;
b) Y1= eX, Yt = xe x ;
2
c) YI=X, Yt=x ,
x
d) Yl=e , Yt=exsinx, Ya=e"co5x.
2970. Knowing the fundamental system of solutions of a linear
homogeneous differential equation
Y1=x, Yt=x!, y.=x' ,
find its particular solution y that satisfies the initial conditions
YI.~=l=O)

y'I.~=l=-I)

Y"IX:l=2.

Sec. 12]

Linear DiOerential Equations with Constant Coefficients

351

2971*. Solve the equation


y" + !)' y'

+y =

0,

.
. I ar so It
si n x .
k nowlng
I.ts par t leu
u Ion U, = -x2972. Solve the equation
x! (In x-I) y" -xy' -t- Y = 0,

knowing its particular solution YI = X.


By the method of variation of parameters, solve the follo\ving
inhomogeneous linear equations.
2973. x1y" -xy' = 3x'.
2974*. X2Y"1-Xy'-y=x2.
2975. y'" -t-y'::.=secx.
Sec. 12. Linear Differential Equations of
with Constant Coefficients

S~cond

Order

1. Homogeneous equations. A second-order llncar equation \\'ith constant


coefficients p and q WI thou t the right si de IS of th~ fOrIn
y" + py' +qy =0
(I)

If k. and 11 2 are roots of the characteristic equation


cr (k) === k' + pk +q ==- 0,
then the general solution of
til ree \va ys:

~quation

(2)

(1) is \\ritten in one of the following

1) y -.::- Cle".>" + C~ek.x if k , and k 2 are real nnd k,:l= k~;


2) y -= e'l.X (C. + C 2x) if k, == k 2;
3) y -=e?'x eC , cns PX+C 2 SlIl ~.\) if k. =a-t-~t and k2-=-a-~1 ((3:f:. 0).

2. Inhonlogeneous eljuations. The general solution of a linear


neous differential equation
y"+py' -+qy=f(x)

lllhoI110~e

(3)

01 ay be wntten in the form of a sunl:

Y=Yo+ Y,

where Yo is the general solution of the corresponding equation (I) without


rl~ht side and deternlined from fOrJnulas (1) to (:3), and Y is a particular
solution of the given equation (3).
The function Y may be found by the method of undetermined roefficl~llts
in the following simple cases:
l. f (x) =e ux PII (x), where Pn (x) is a polynomial of degree n.
If a is not a root of the characteristic equation (2), that is, <p (a) ::1= O.
then we put Y =e'J. x Qn (x) \\hcre Q,. (x) is a polynomial of degree n with
undetermined coefficients.
1f a IS a root of the characteristic equation l2), that is. <p (a) =0. then
Y=reuxQn(x), "here r is the multiplicity of the root a(r=l or r=2).
2. f (x)=eGX (P n (x) cos bx+ Q", (x) sin bx].

352

(Ch.9

DifJerential Equation.s

If q> (a bi) :F 0, then we put


Y = eax [SN (x) cos bx -1- TN (x) sin bx),

where SN (X) and T f:I (x) are pol ynomials of degree N-max {n, m}.
But if q> (a btl = 0, then
y = xr eax [S N (x) cos bx + TN (x) sin bx],

where, is the multiplicity of the roots a

bl (for second-order equations,

r= 1).

is

In the general case, the method of variation of parameters (see Sec. 11)
to solve equation (3).
Exall1ple t. Find the ~eneral solution of the equation 2y"-y'-y=4xe2".
Solution. The characteristic equation 2k 2 - k - l =0 has roots III =- I and

us~d

k.= - ; .

The general solution of the corresponding homogeneous equation


x

(first type) is 1/ 0 = Cle" +C 2e z. 1 he right side of the given equation is f {x) =


=4xe2X =tOXp"(x). l-Ience, Y=e 2X (Ax+8), since n=1 and 1-=.0. DiOeren
tiating Y tWice and pu tting the derivatives into the given equa tion, we
obtain:
2e 2X (4Ax+ 48

+ 4A)-e

2X

(2Ax+ 28

+ A)_e

2X

(Ax-j- B)

= 4xe~x.

2X

Can,cplling out e and equating the coefficients of identical powers of x and


the absolute ternlS on the left and right of the equality, we helve 5A =4 and
4
28
7A+58=O, whence A=S and 8=-25.
Thus. Ye f '>'

x- ~~) and the general solution of the given equa tion

IS

Y= CIe X

+C e-7 (+ e~.,x ( S-x-2"5.


4
28)
2

Example 2. Find the general solution of the equation y"-2y' +u=xe"".


Solution. The charactenstlc equation k 2 -2k+] ==0 has a double root
k = 1 The ri~hl side of th(:l equation l~ of the {orin f (x) =xex ; her~, a = I
and n=-l. The particular solution is Y =x 2e" (Ax+ B), since a cOIncides \1Jith
the doubl~ root k=-l and, consequently, r=2.
Diflerentlating Y h\ice, substItuting into the equation, and equating the
coerticil.>nts. we obtam A =

i.

B =0. Hence, the general solution of the given

equation will be \\rltten in the form


y=(C I +Cax) eX

+~

x'e".

Example 3. Find the Jl:en~ral solution of thE:' equa tion y" + I/=X sin x.
Solution. The characteristic equation k 2 + l=-O has roots III = i and
k 2 = - I . The general solution ot the corresponding honlogeneous equation
will Isee 3, where a=O and p = 1J be

Yo=C. cos x+ C.. sin x.

The right side is of the form

I {xJ =eIJx IP n (x) cos bt + Qm (x) sin bxJ.

Sec. /2]

Linear DiUerential Equations with Constant Coefficients

353

where a =0, b = 1, P n (x) =0, QIn (x) =x. To this side there corresponds the
particular solution Y,
Y =x [(Ax+ B) cos x+ (Cx+ D) sin xl
(here, N= I, a=O, b== 1, r= I).
Differentiating twice and substituting into the equation, we equate the
cOE'fficlcnts of bolh sides in cos X, x cos x, sin x, and x S1n x. We then get four
equations2A+2D==O,4C=O, -2B+2C=O, -4A=1, [rom which we deterx2
x
.
1
1
mine A = -4' 8=0, C=O, D=4 Therefore, Y= --:rcosx+-:rsinx.
The general solution is
Xl
x
y= C. cos x+C 2 sin x-"4 ros x+T sin x.

30 The princi pie of superposition of sol utions. If the righf si de of equation (3) is the sum of several funct:ons

I (x) -=11 (x) +'2 (x) + ... + In (x)


and Yi (i = 1, 2, 3, ... , n) are the corresponding solutions of the equatIons
y"+py',-qY=--.fi(x)
(i=I, 2, , n),
then the sum
y=Yl+ Y 2+ ... Y n

is t he so1uti 0 n

0f

equa t ion (3).

Find the general solutions of the equations:


+ 6y = O.
2982. g" + 2y' + Y= o.
2977. y"-9y=O.
2983. y"-4y' +2y=0..
2978. y" - y' == O.
2984. y" + ky = O.
2979. y" -I- Y == O.
2985. Y = y" + y'.
2980. y" - 2y' + 2y = O.
y'-y
2981. y"t 4y' + 13y=O.
2986. 7
=3.
Fi nd the particular sol utions that satisfy the indicated conditions:
2987. y" -5y' -J- 4y = 0; Y= 5, y' = 8 for x = 0
2988. y" + 3y' +- 2y = 0; Y = 1, y' = -1 for x = o.
2989. y" + 4y = 0; y = 0, y' = 2 for x = O.
2990. y" + 2y' = 0; y = 1, y' = 0 for x = 0
2976. y" -5y'

2991. y"=Y2; y=a, y'=O for x=O.


a

2992. y"+3y'=O~ y=O for x=O and y=O for x=3.


2993. y" n 2 y=O; y=O for x=O and y=O for x= 1.
2994. Indicate the type of particular solutions for the given

i Ilhomogeneous eq uat ions:

a) y"_4y=x 2 e2X ;

b) y"+9y=cos2x;
12-1900

354

DiUerential Equations

[Ch.9

c) y"-4y' -t-4y= sin 2x+elx ;


d) y" + 2y' 2y= eX sin x;
e) y" -5y' + 6y = (xl-t- I) e" + xelX~
f) y" - 2y' + 5y = xe" cos 2x-x1 e" sin 2x.

Find the general solutions of the equations:


2995. y" - 4y' + 4y = Xl .
2996. y" -y' + y = xlt 6.
2997. y" + 2y' + y = e2X
2998. y" -By' + 7y= 14.
2999. y" - y =-=: eX.
3000. y" y = cos x.
3001. y" + y' -2y = 8 sin 2x.
3002. y" + y' -6y = xe2X
3003. y"-2y' -t-y= sinx+sinhx.
. 3004. y" + y' = sin 2 x.
3005. y" - 2y' -+ 5y = eX cos 2x.
3006. Find the solution of the equation y" + 4y= sin x that
satisfies the conditions y = I, y' = 1 for x = O.
Solve the equations:
3007. ~:~+<illx=ASinpt. Consider the cases: 1) P=/=<ilj

2) P =

0>.

3008.
3009.
3010.
3011.
3012.
3013.
3014.
3015.
3016.
3017.

y"-7y'+12y=-e4 ".
y"-2y' =x 2 -1.

y"-2y'+y=2e x .
y"-2y'=e 2X +5.
y"-2y' -8y= eX -8 cos 2x.
y" ~t y' = 5x + 2e x
y"-y'===2x-1-3e x .
y" 2y' y= eX +e- x .
y"-2y' IOy= sin 3x +e x

+
+
y" -4y' + 4y =
+

2e IX

+ ~.

3018. y"-3y' =x+cosx.


3019. Find the solution to the equation y"-2y'=e1x +xl - l

that satisfies the conditions y=-lr. y'=l for x=O.


Solve the equations:
3020. y"-y=2xsinx.
3021. y"_4y=e2X sin 2x.
3022. y" + 4y = 2 sin 2x-3 cos 2x+ 1.
3023. y" - 2y' + 2y = 4e x sin x.
3024. yIP = xe" -t- y.
3025. y" + 9y= 2x sin x+ xelX

Sec. 12)

Linear DiDerential Equations with Constant

Coefficient~

355

3026. y"-2y'-3y=x(1 +e'X ).


3027. y"-2y' =3x+2xe".
3028. y"-4y' +4y=xet ".
3029. y" + 2y' -3y = 2xe-" + (x -t 1) eX.
3030. y" + y = 2x cos x cos 2x.
3031. y" -2y = 2xe x (cos x-- sin x).

Applying the method of variation of paranleters, solve the


following equations:
3036. y" + y = _1_.
3032. y" + y = tan x.
cosx
1
snx

3037. yn + y = _1_.

3033. y" + y = cot x.


3034. y" -2y'
3035. y"

+y= ex.
x

3038. a) y"-y=tanhx.

e-"

+ 2y' + y ="7.

b) g" -2y = 4x 2 exJ

3039. Two identical loads are suspended from the end of a


spring. Find the equation of nlotion that will be p~rfornled by
one of these loads if the other falls.
Solution. Let the increase in the length of the spring under the action
of one load in a state of rest be a and the Jnass of the load, In. Denote by x
the coordinate of the load reckoned vertically from the position of equilIbrium in the case of a single load. Then
d2 x
m dt 2 =mg-k (x+a),

mg
d~
where, obviously, k=a- and, consequently, dt 2
tion is

x=CtCOS

V:

t+Ctsln

V~ ~

t.

=-ag x.

The

~enerat

~olu-

The initial condItIOns Yield x=a

d\':

and dt =0 when t =0; whence C, =a and C,=O; and so


X=QCOS

V:

t.

3040*. The force stretching a spring is proportional, to the


increase in its length and is equal to 1 l<gf when the length
increases by 1 cm. A load weighing 2 l<gf is suspended fron) the
spring. f"ind the period of oscillatory motion of the load if it
is pulled downwards slightly and then released.

3041*. A load weighing P=4 kgf is suspended from a spring


a.nd increases the length of the spring by 1 em. Fi,nd the law
of motion of the load if the upper end of the spring performs
a vertical harmonic oscillation y=2sin30tcm and if at the
initial instant the load was at rest (resistance of the medium is
neglected).
i12

356

DiOerent fat E quat ions

[Ch. 9

3042. A material point of mass m is attracted by each of two


centres with a force proportional to the distance (the constant
of proportionality is k). Find the law of motion of the point
knowing that the distance between the centres is 2b, at the initial instant the point was located on the line connecting the
centres (at a distance c from its midpoint) and had a velocity
of zero.
3043. A chain of length 6 metres is sliding from a support
without friction. If the motion begins when 1 m of the chain
is hanging from the support, how long will it take for the entire
chain to slide down?
3044*. A long narrow tube is revolving with constant angular
velocity ro about a vertical axis perpendicular to it. A ball inside the tube is slidin5 along it without friction. Find the law
of nlotion of the ball relative to the tube, considering that
a) at the initial instant the ball was at a distance a from
the axis of rotation; the initial velocity of the baH was zero;
b) at the initial instant the ball was located on the axis of
rotation and had an initial velocity vO
Sec. 13. linear Differential Equations of Order Higher than Two with
Constant Coefficients
to. Homogeneous equations. The fundamental systelTI of solutions Yl'
Y., .. , Un of a homogeneous linear equation with constant coefficients
y(n)+a1y(n-I)+ .. +an_ty' +any =0

(1)

is constructed on the basis of the character of the roots of the characteristic


equation
(2)

Namely, t) if k is a real root of the equation (2) of multiplicity m, then to


this root there correspond m linearly independent solutions of equation (1):
Yl =ekx , YI = xek" , ... , Y m =xm-'ekx ;
2) if a ~l Is a pair of complex roots of equation (2) of multiplicity nl,
then to the latter there correspond 2m linearly independent solutions of
equation (I):

1I.=e'l.xcospx. y.=e'l.Xslnpx,

Ya=xe'l.Xcos~x, Yf,,=xe'l.Xsin~x, ...


... Yam_l=x m - 1e'l.x cos ~x, Y2m=X m - 1e'l.x sin ~x.

2. Inbomogeneous equations. A particular solution of the inhomogeneou


equation
y(n) -t a1ycn -1) + ... + all - 1y' + anY = f (x)
(3)

II sought on the basis of rules 20 and 3 of Sec. 12.

Sec. 141

Find
3045.
3046.
3047.
3048.

357

Euler's Equations

the general solutions of the equations:


y"'-13y"+12y'=O.
3058. ylv+2y""+y=O.
y'" _y' = O.
3059. y(n) -+!!..} yCn-I)-t_
y'" +y=O.
ylV -2y" = O.
-J- n (n -1) en-I)

3049. y"'-3y"+3y'-y=O.
3050. ylV -f-4y==O.
3051. ylV t- 8y"+ 16y=O.
3052. yIV+y'=O.
3060.
3058. y'V -2y" + y = O.
3061.
-6054. ylV _ally = O.
3062.
3055. ylV -6lJ" + 9." = O.
3063.
':1
3064.
3056. ylV + aly" = O.
3065.
3057. ylv+2y"'+t/'=O.
3066.

Y
+~1 y' +y=O.

)2

...

yIV_2y"'+y"=e x
ylV -2y'" + yIP = Xl.
y'" -y = xl-I.
ylV -t- y'" = cos 4x.
y'" +- yIP = x" + I + 3xe-.
y'" + y" y' + y = xex'
y"'+y'=tanxsecx.
3067. Find the particular solution of the equation
y'" +2y" +2y' +y=x

that satisfies the initial conditions y(O)=y'(O)=y"(O)==O.

Sec. 14. Eul er's Equations


A linear equation of the form
(ax + b)n y \ll) 1- AI (ax -t- b)n-l y (n-1l + ...

+ An

-1

(ax

+ b) Y + AnY -= f (x),

(1)

where a, b. AI' ... , An_I' An are constants, IS called EIller's equation.


Let us Introduce a nc\v Independent vanable t, putting
ax -1- b -=e t .
Then
91
= ae- t r!:!!. y" ==a!e-~t
1_

Y'

(ddIe! d!/)
dt'

dt'

:s -:st (d "
~ d 11
dl l
-ae
dl:J- dt l -l- 2 dt
2

II' _

an d so for th

and Euler's equation is transfornled into a linear equation with constant


coefHclcll ts.
fxarnple 1. ~olve the equation x"y" + xy' y = 1.
Solution. Putting x==-e t , we get

dy= _tdl/
dx edt'

dll/= -2t(d!Y_ d l/\

dx"

\ dt"

dt ) .

Consequently, the given equatio 1 takes on the fonn


d!y

dt,,+y=l,

whence

or

y=C I cos t +C" sin t + 1


g= C1 cos (In x)

+ C. sin (In x) + 1.

DiOerential Equations

358

[Ch. 9

For the homogeneous Euler equation


xny(n) + A1xn-1y,n-u+ ... + An_1xy' + Any=O
the solution may be sought in the form
Y=~.

(2)
(3)

Putting into (2) y, y', ... , y(n) found from (3), we get a characteristic equation from which we can find the exponent k.
If k is a real root of the characteristic equation of multiplicity m, then to it
correspond m linearly independent solutions
Yl =xk. Y1 = /l.ln x, Ys =xk (In X)I, Ym =xk (In x)m-l.
If a ~i is a pair of complex roots of nlultipllcity m, then to it there
correspond 2m linearly independent solutions
Yl =x~ cos (~ In x), YI = xt% sin (~ In x), Ys =xt% In x cos (~ In x),
Ya= x fJ ln xsln (~ In x), ... , Y2m-l = x fJ (1n x)m-l cos (~ In x).
Y2m = x fJ (In x)m-l sin (~ In x).
Example 2. Solve the equation
rt

x 2y -3xy' +4y=O.

Solution. We put

y'=kX k -

y=xk.

y"=k(k-l)xk -

Substituting into the given equation, after cancelling out xk we get the
characteristic equa tion
Solving it we find

Hence, the general solution will be


Y = Ct x2 + Ct,x 2 In x.

Solve the equations:


3068.
3069.
3070.
3071.
3072.

d2 y

X dx l

+ 3x dy
dX + y =

O.

x y"-xy'-3y=O.
x 2 y" -f- xy' +. 4y = O.
xSy'" -3x' y" 6xy' -6y = O.
(3x --1- 2) y" 7y' = O.

3073. y" = 2~.


x

Y' Y
3074. y"+-+"!=O.
x
x
3075. xy"-4xy' +6y=x.
3076. (1 +X)I y"-3(1 +x)y' +4y=(1 +x).

Systems of DiUerential Equat ions

Sec. lt51

359

3077. Find the particular solution of the equation


x 2 y" -xy' + y= 2x
that satisfies the initial conditions y= 0. y' = 1 when x = 1.
Sec. 15. Systems of Differential Equations
Method of elimination. To find the solution, for instance, of a normal
systenl of two first-order differential equations, that is, of a system of the
fonn
dy
dz
(1)
dx=f(x,y,z),
dx=g(x, y, z).
solved for the derivatives of the desired functions, we differentiate one of
them with respect to x. We have, for example,
d2g

of

af

dX 2 =ax+ayf+

af

az g.

(2)

Determining z from the first equation of the system (1) and substituting the
value found,

z=q> ( x, y,

:~)

(3)

into equation (2), we get a second-order equation with one unknown function 1/. Solving it, we find
Y = '" (x, CI' C2 ),
(4)
where C I and C2 are arbitrary constants. Substituting function (4) into formula (3), we determine the function z without new integrations. The set of
fornlulas (3) and (4), where y is replaced by 'i'. yields the general solution
01 the system (1).
Example. Solve the system

:~ +2y +4z= 1 +4x,


{ dz

3 2
dx+ y - z ="2 X

Solution. We differentiat-e the first equation with respect to x:

dly +2~+4 dZ=4.


dx 2
dx
dx
From the first equation we determine

frOln th esecond we
and

:~

Z=} ( 1 + 4x- :~ -2Y )

dz 3 2
1
3
1 dy
havedx=2x+x+4-2Y-4dt(.

WIll

and then

.
Putting

into the equation obtained after differentiation, we arrive at a secord-

order equation in one unknown y:


2

dy

dx'

+ dx
dy -6y=- 6x

-4x+3.

DifJerential Equations

360

(Ch.9

Solving it we find:

and then
1 ( 1 +4x---2y
dY
) = -C e2X +-!e
C _ 3X _ _
1 X2
z=1
4

dx

We can do likewise in the case of a system wi th a larger number of


equations.

Sol ve the syste ms:


3078.

3079.

3080.

~=z.
{

:~ +3y+4z

3085. { dz

:~=Y+5z,

y=O. z=O when x=O.

dx-Y-z=x,

dz
{ dx
-J- Y + 3z = o.

~=-3Y-z.
dz
{ dx
=

dl/

3087.

Z,

dt =Y
dy

y2

:=T'

dx=2 Y .

x-y

+z,

c) ~

y-z

dt=x+z,

dz

dx=x+y+z.

~ + 2y + z =

x+y

z'

dy

=.Ex-y'

z-x

isolate the integral curve passing through the point (I, 1, -2).

:~=y+z,
sin x,

{ dx -4y- 2z = cos x.
dz

d~ +2x--y +2e'=O,

b) ~= dy =dz.

t dt=x+y.

3084.

*
dx
dy
dz.
30 88 a) X 1+-3
2z '
xy 2=21y = -y2

dz

3083.

~; -4x-y + 36t =0,

dy

l :;=x.
dx

3086.

x=O, Y= 1 when t =0.

y-- z.

( ~7=Y'
3081. t dt =

3082.

=2x,

dz
dx=-Y.

d!J+ = 1
d" Z
,
3089. { dz
2
dx + x2 Y = In x.
d2

3090.

d~ +2y+4z = eX,

2
{ d z
dx 2 -

y - 3z = -x.

Sec. 16]

Integration of DiOereniial Equations by Power Series

361

3091**. A shell leaves a gun with initial velocity V o at an


angle a to the horizon. Find the equation of motion if we take
the air resistance as proportional to the velocity.
3092*. A materia I point is attracted by a centre 0 with a
force proportional to the distanc~. The motion begins from point A
at a distance a from the centre with initial velocity V o perpendicular to OA. Find the trajectory.

Sec. 16. Integration of Differential Equations by Means of Power Series


If it is not possible to integrate a differential equation with the help of
elementary functions, then in some cases Its solution may be sought in the
form of a power series:
co

y = ~ Cn (x-xo)n.

(1)

n=o

The undetermined coefficients cn (n = 1, 2, ... ) are found by putting the


series (I) into the equation and equating the coefficients of identical powers
of the binomial X-X o on the left-hand and right-hand sides of the resulting
equation.
We can also seek the solution of the equation
y'-:=f(x, y);

y(xo)=--=Yo

(2)

in the form of the Taylor's series


J

y(x) =

L. y(n~~xo) (x-xo)n,

(3)

n=o

where y (x o) = Yo, y' (x o) = f (x o' Yo) and the subsequent dprivatives y(nl (x o)
(n =- 2. 3, ... ) are successively found by difT(r~ntiating e'luation (2) and by
putting Xo in place of x
Example 1. Find the solution of the equation
y"-xy=O,

If y=Yo, y' =y; for x:=.:O.


Solution. We put

whence, differentiating, we get

y" = 2.lc.

+3.2c.x +... + n (n-I) cn x n - + (n + 1) ncn + Xn - +


+ (n + 2) (n + I) en + 2xn +...
2

Substituting y and y" into the given equation, we arrive at the identity

[21c. + 32cax + ...

+ n (n-I) cnx n - + (n + I) nCn +lX n - 1 +


+ (n +2) (n + I) cn + 2x n + ... ]-x (CO+c1x + ... +cnxn + ... ] ==:0.
I

Collecting together, on the left of this equation, the terms with identical
powers of x and equating to zero ihe coefiiclents of these powers, we Will

DiOerent ial E quat ions

362

[Ch.9

have
Co

32c.-co=O, c'=3.2;

cJ=O;

C,

= ;'14

CJ

5 .4c,-c.=O.

4.3c4 -cJ =O, c4 =4.3'


and so forth.

Generally,
~

cak

2.3.5.6 ..... (3k-l) 3k'


Cak+2=O

3.467 ... 3k (3k + lr


(k= 1,2,3, ... ).
cak+1 =

Consequently f
X'
xci
xk
y=co ( 1+ 2.3 + 2.3.5.6 +... + 2.3.56 ..... (3k-l) 3k +...

x7

X4

+C

1 (

x,kq. 1

+
)

x+3.4+3.4.6.7+-J- 3.4.6.7 .... 3k(3k+l)+ ,

(4)

Y;.

where Co = Yo and C1 =
Applying d'Alembert's test, it is readily seen that series (4) converges
for - 00 < x < + 00.
Example 2. Find the solution of the equation
y' = x y; Yo = Y (0) = 1.
Solution. We put
"
",
,
-Yo 2 Yo a
Y=Yo+Y ox+2f x + 31 x + ...

We have yo=l, y~=O+l= 1. Differentiating equation y'=x+y, we succes.


fiIn d y" = 1+y,
' Yo=
" 1+ 1=2, Y" =y,
' " Yo,,, = 2,etc. Consequent 1y,
sJvely
2

y= 1 +x+2f x

2 a
+31
x + ...

For the examp'le at hand, this solution may be written in final form as
y=1+x+2(eX -l-x) or y=2eX-l-x.
The procedure is sim ilar for differential equations of higher orders. Testing the resulting series for convergence is, generally speaking, COIDplicated
and is not obligatory when solving the problems of this section.

With the help of power series, find the solutions of the equations for the indicated initial conditions.
In Examples 3097, 3098, 3099, 3101, test the solutions
obtained for convergence.
3093. y'==y+x2 ; y=-2 for x=O.
3094. y'=2y+x-l; y=yo for x=l.
3095. y'

=+
yl
I

Xl;

=;

(or

=O.

3096. y'=X _ y 2; y=O for x=O.


3097. (l-x)y' = 1 +x-y; y=O for x=O.

Problems on Fourier's Method

Sec. 17)

363

3098*. xy" + y = 0; Y = 0, y' = 1 for x = o.


3099. y"+xy=O; y=l, y'=O for x=O.
3100*. y,,+J:..Y'-t-y=O;
y~l, y'=O for x=O.
x

+ 2- y' + y = 0; y = 1, y' = 0 for x = O.


dx
dx
dt + x cos t = 0; x = a; dt = 0 for t = O.

3101 *. y"

3102.

Sec. 17. Problems on Fouriefs Method


To find the solutions of a linear homogeneous partial differential equation
by Fourier's method, first seek the particular solutions of this special-type
eq uation, each of which represents the product of functions that are dependent
on one argument only. In the simplest case, there is an infinite set of such
solutions un (n= l, 2, ... ), which are linearly independent among themselves
in any finite number and which satisfy the given boundary conditions. The
desired solution u is represented in the form of a series arranged according
to these particular solutions:
(I)
n=l

The coefficients
condit ions.

en

which reInain undetermined are found from the initial

Problem. A transversal displacement u =U (x, t) of the points of a string


with abscissa x satisfiE:'s, at time t, the eqnation

au
2

att. =a

:2

au
ax
2

(2)

'

To (To is the tensile force and Q is the linear density of the


Q
string). Find the form of the string at ti tne t if its ends x =0 and x= I are

where at.

u
O~--_-..._---~--.-

r'--'

Fig. 107

fixed and af the initial instant, t = 0, the string had the form of a parabola
U

= ~: x

(i-x) (Fig. 107) and its points had zero velocity.

Solution. It is required to find the solution u = u (x, t) of equation (2)


that satisfies the boundary conditions
u (0. t) =0. u (I, t) =0
(3)

Differential E quat ions

364
and the initial conditions

4h
u (x, 0) = j2 x (I-x),

Ut

(Ch.9

(x, 0) =0.

(4)

We seek the nonzero solutions of eauation (2) of the special form


U = X (x) T (t).
Putting this expression into equation (2) and separating the variables, we get
T" (t)
X" (x)
(5)
aIr (t)= X (x)
Since the variables x and tare indeprndent. equation (5) is possible only
when the ~eneral quantity of relation (5) is constant Denoting this constant
by - AI. we find two ordinary dIfferential equations:
T" (t) + (aA)2. T (t) = 0 and X" (x) + ).,IX (x) =0.
Solving these equations, we get
T (t) = A cos aAt + B sin a).,t.
X (x) = C cos ).,x + D sin ).,X.
where A, B. C, D are arbitrary constants. Let us determine the constants.
From condition (3) we have X (0) = 0 and X (I) = 0; hence, C = 0 and
sin Al = 0 (since D cannot be equal to zero at the same tinle as C is zero).
For this reason, Ak =

~n ,

where k is an mteger. It will readily be seen that

we do not lose generality by taking for k only positive values (k = 1. 2, 3...).


To every value Ak there corresponds a particular solution
kan
. kan ) . knx
uk= ( Ak cos-/- t + Bk sin -1- t sin -llhat satisfies the boundary conditions (3).
We construct the series
CJ)

~ (
kant
. kant) . knx
U= ~
Ak cos -1-+Bk s1n -1- Sill -,-

k=J

whose sum obviously satisfies equ3 tion (2) and the boundary conditions (3).
We choose the constants Ali ann Bk so that the sum of the series should
satisfy the initial conditions (4). Since
CJ)

au
~ kart (
at = ~
-/- -

Ak

. kant
Sill - , -

k~nt)
knx
+ Bk cos -1sin -l-

k=J

it follows that. by putting t = 0, we obtain


CJ)

~
. k:rrx
4h
(x, O) = ~ A k sin -1- == j2 x (I-x)

k=J

and

au (x,
iJt

CJ)

0)

~ kan
. krrx
~ -l- B k sin -lk==J

c:::iO.

Problems on Fourier's Method

Sec. 17]

365

Hence, to determine the coefficients All and Bk it is necessary to expand in


a Fourier series, in sines on1 y, the function u (x, 0) =

f unc t Ion

au (x,
at

0)

;2'; x

(I-x)

and

the

==.

From familiar formulas (Ch. VII I, Sec. 4,3) we have

r4h
knx
32h
, dX=n1k
A~=T J lI x (l-x)sinI

'

if k is odd, and All = 0 if k is even:

SOSin-,-dx=O, B,,=O.
I

kart
2
-L- Bk=T

knx

The sought-for solution will be


(2n

00

_32h~

u-

n3

-t- I) ant

cos----l
. (2n+ l)nx
(2n -1- 1)1
sin
I

n=o

3103*. At the initial instant t = 0, a string, attached at


its ends, x = 0 and x = I, had the form of the sine curve
u = A sin 'IT/x, and the points of it had zero velocity. Find the
lorrn of the string at time t.
3104*. At the initial time t = 0, the points of a straight
string 0 < x < I receive a velocity ~~' = 1. Find the form of the
strin~ at titne t if 1he ends of the string x=O and x=l are
fixed (see Problem 3103).
3105*. A string of length 1 = 100 em and attached at its ends.
x=O and x=l, is pulled out to a distance h=2 cm at point
x == 50 cnl at the initial time, and is then released without any
impulse. De,ernline the shape of the string at any time t.
3106*. In longitudinal vibrations of a thin homogeneous
and rectilinear rod, whose axis coincides with the x-axis, the
displacenlent u = U (x, t) of a cross-section of the rod with
abscissa x satisfies, at time t, the equation
iJ2

dt2 =

au
ax
2

where a =.!!(E is Young's modulus and Q is the density of the


Q
rod). Determine the longiturlinal vibrations of an elastic horizontal rod of length 1 = 100 cm fixed at the end x = 0 and pulled
back at the end x = 100 by Iii = 1 CIU, and then released without
Impulse.
l

DiDerential Equations

366

[Ch. 9

3107*. For a rectilinear homogeneous rod whose axis coincides


with the x-axis, the temperature u = u (x, t) in a cross-section with
abscissa x at time t, in the absence of sources of heat, satisfies
the equation of heat conduction

au

at =a

au
2

ax! ,

where a is a constant. Determine the tenlperature distribution


for any tinle t in a rod of length 100 cm if we know the initial
tenlperature distribution
U

(x t 0) = 0.01 x (IOO-x).

Chapter X

APPROXIMATE CALCULATIONS

Sec. 1. Operations on Approximate Numbers


to. Absolute error. The ab~olute error of an approximate number a which
replaces the exact number A is the absolute value of the difference bet\\'een
them. The number ti, which satisfies the inequality
IA-al~L\,

(1)

is called the limIting absolute error. The exact nUlnber A is located within
the litnits a-&~A~a+& or, more briefly, A=a 6.
2. Relative error. By the relative error of an approximate nUlnber a
replacing an exact number A (A > 0) we understand the ratio of the absolute
error of the number a to the exact nurnber A. The number ~, which satisfies
the ineq uality
I A-at ~
(2)
A
1Iilrl::a,
is called the limiting relative error of the approximate number a. Since in
actual practice A =::::a, we often take the number 6=~ for the lilniting
a

relative error.
3. Number of correct decimals. We say that a positive approximate
number a written in the fornl of a decimal expansion has n correct decimal
places in a narrow sense if the absolute error of this number does not exceed
one half unit of the nth decinlal place. In this case, when n > I we can
take, for the limIting relative error, the number

6=2~(I~r-1
where k is the first significant digit of the number a. And conversely, if it
1
( 1 ' n-l
is known that 6E;;; 2 (k + I) 10 )
. then the number a has n correct deci mal
places in the narrow meaning of the word. In particular, the number a

(/0)

definitely has n correct decimals in the narrow meaning if f> E;;; ~


n .
If the absolute error of an approximate nUlnber a does not exceed a
unit of the last decimal place (such, for example, are numbers resulting
from measurements made to a definite accuracy). then it is said that all
decimal places of this approximate number are correct in a broad sense. If
there is a larger number of significant digits in the approxinlate number,
the latter (if it is the final result of calculations) is ordinarily rounded off
so that all the remaining digits are correct in the narrow or broad sense.

368

Approximate Calculations

[Ch. 10

Hencetorth, we shall assume that all digits in the initial data are
correct (if not otherwise stated) in the narrow sense. The results of intermediate calculations may contain one or two reserve digits.
We note that the exanlples of this sectiCJn are, as a rule, the results of
final calculataons, and for this reason the answers to them are given as
approximate numbers with only correct drcimals.
4. Addition and subtraction of approximate numbers. The limiting absolute error ot an algebraic sum of sevE'ral numbers is equal to the sum of
the limiting absolute errors of these numbers. Therefore, in order to have,
in the sum of a ~mal1 number of approximate numbers (all decimal places
of which are correct), only correct digits (at least in the broad sense), all
summands should be put into the form of that summand which has the
smallest number of decimal places, and in each summand a reserve digit
should be retained. Then add the resulting numbers as exact numbers, and
round off the sum by one decimal place
If we have to add approximate numbers that have not been rounded off,
they should be rounrled off and one or two reserve digits ~houl d be retained.
Then be guided by the foregoing rule of addition while retaining the appropriate extra digits in the sunl up to the end of the calculations.
Example 1. 215.21 +14.182+21.4==215.2l1)+14.1(8)+21 4=250.8.
The relative error of a sunl of poc:;itive terms bes between the least and
greatest relative errors of these terms.
The relative error of a d,n'erence is not amenable to simple counting.
Particularly unfavourable in this sense is the ditference of two close nunlbers.
Example 2. In subtracting the a~proximate numbers 6 135 and 6.131 to
four correct decimal places, we get the dIfference 004. The limiting relative

~ 0.00\ ~ 0/'01 I
error is B
0.004
=4=0.25. Hence, not one of the decimals
of the difference is correct. Therefore, it is always advisable to avoid
subtracting close approximate numbers and to trttnsform the given expression,
if need be, so that this undesir~!-Ie operation is omitted.
5. Multiplication and division of approximate numbers. The limiting
relative error of a product and a quotient of approximate numhers is equal
10 the sum of the linliting relative errors of these numbers Proceeding froln
this and applying the rule for the number of correct decilTaals (3), we retain
In the answer only a definite nunlber of decimals
Example 3. The product of the approximate numbers 25.34.12 = 104.236.
Assuming that all dEcimals of the factors are correct, we find that the
limiting relative error of the product is

6=2.20.01

I
01 ~0.003.
+ 4.2.

Whence the number of correct decimals of the product is three and the
result, if it is final, should be written as follows: 25.34 12 = 104, or more
correctly, ~5 34.12= 104 2 (J.3.
6. Powers and roots of approximate numbers. The limitIng relative error
of the mth power of an approxi mate number a is equal to the m-fold linliting
relative error of this number
The limiting relative error of the mth root of an approximate number a
is the

..!.th
m

part of the limiting relative error of the number a.

7. Calculating the error of the result of various operations on approxi-

mate numbers. If lill., ... flail are the limiting absolute errors of the appro-

Sec. I)

Operations on Approximate Numbers

xirnate numbers

all _

369

an' then the limiting absolute error ~S of the result

S === f (at~ . . . an)

may be evaluated approximately from the formula

~S = I:~ I~al + +I:!n IM n


The limiting relative error S is then equal to

6S =

~; I = I :1.1 f;; + .+I:!n It'tr =

= dln'l
da
L\a l
l

+ . + Idln'l
dan tian

Example 4. Evaluate S = In (10.3+ V4_4); the approximate numbers


10.3 and 4.4 are correct to one decimal place.
Solution. Let us first compute the limiting absolute error L\S in the
general form: 8=ln (a+ vb),

~a=M~ ;0; V4."4 =

~S= a+ ~

We

have

2.0976... ; we leave 2.1, since the relative error of

the approximate number 4.4 is equal to


is then equal to

b(~a +~ ;bb)'

~ 2 8~) = ~;

~} ~= ;0;

the absolute error

we can be sure of the first decimal place. Hence,

1
( 1
1
1)
1
(
I)
13
L\S=IO.3+2.1 20+2--20.2.1 =12.4.20 l+U =2604=::::0.005.
Thus, hvo decimal places will be correct.
Now let liS do the calculations with one reserve decinlal:
log (10.3 + y44) ~ log 12 4 -== 1.093, In (10 3+ yU)~ 1.0932.303 = 2.517.
And we ~('t the answer: 2 52
8. Establishing admissible errors of approximate numbers for a given
error in the result of operations on them. Arplying the formulas of 7 for
the quantities L\S or f,S given us and conSIdering all particular differentials

Ia~ I~ak

I I

or the quantities a~ ~,ai equal, we calculate the admissible


errors L\a" ... , L\a n , " of the approximate numbers at ... an'
that enter into the operations (the principle of equal effects).
It should be pointed out that sOlnetimes when calculating the adlnissible
errore;; of the argul11ents of a function it is not advantageous to use the
principle of equal effects. since the latter nlay make delllands that are
practically unfulfilable In these cases it is advisable to nlake a reasonable
redistribution of errors (if thi~ is possible) so that the overall total error does
not exceed a specified quantity. Thus, strictly speaking, the problem thus
posed is indeterm inate.
Example 5. The volume of a "cylindrical segment", that is,''a ~olid cut
off a circular cylinder by a plane passing through the diameter of the base
(equal to 2R) at an angle a to the base, is computed from thp formula
ab~olute

V=

; R'

tan a. To what degree of accuracy should we measure the radius

Approximate Calculations

370

[Ch. 10

R:::::: 60 cm and the angle of inclination a so that the volume of the cylindrical
segment is found to an accuracy up to 1%?
Solution. If l\V, l\R and L\a are the limiting absol ute errors of the
quantities V, R and a, then the limiting relative error of the volume V that
we are calculating is
3L\R
2L\a
1
6=R+sin2a~100

We assume

3L\R

1
2L\a
1
~ 200 and sin 2a. ~ 200 Whence
R
60 cm
l\R E;;; 600 ~ 600 = 1 mIn;
A
sin 2a
1
d
9'
ua ~ 400 <400 ra lan~

Thus, we ensure the desired accuracy In the ans\ver to I % if we measure


the radius to 1 mm and the angle of inclination a to 9'.

3108. Measurements yielded the following approximate numbers


that are correct in the broad meaning to the nunlber of decimal
places indicated:
a) 1207'14"; b) 38.5 cm; c) 62.215 kg.
Compute their absolute and relative errors.
3109. Compute the absolute and relative errors of the following approximate numbers which are correct in the narrow sense
to the decimal places indicated:
a) 241.7; b) 0.035; c) 3.14.
3110. Determine the number of correct (in the narrow sense)
decimals and write the approxinlate numbers:
a) 48.361 for an accuracy of 1%;
b) 14.9360 for. an accuracy of 1 ?1l;
c) 592.8 for an accuracy of 2%.
3111. Add the approximate numbers, which are correct to the
indicated decimals:
a) 25.386 + 0.49 +3.10 + 0.5;
b) 1.2.102 + 41.72 + 0.09;
c) 38.1 +2.0 +3.124.
3112. Subtract the approximate numbers. which are correct
to the indicated decimals:
a) 148.1-63.871; b) 29.72-11.25; c) 34.22-34.21.
3113*. Find the difference of the areas of two squares whose
measured sides are 15.28 cm and 15.22 cm (accurate to 0.05 mm).
3114. Find the product of the approximate numbers, which
are correct to the indicated decimals:
a) 3.49 8.6; b) 25.1 1.743; c) 0.02 16.5. Indicate the possi ble
limits of the results.

Sec. 1]

Operations on Approximate Numbers

371

3115. The sides of a rectangle are 4.02 and 4.96 m (accurate


to 1 cm). Compute the area of the rectangle.
3116. Find the quotient of the approximate numbers, which
are correct to the indicated decimals:

a) 5.684 : 5.032; b) 0.144 : 1.2; c) 216:4.


3117. The legs of a right triangle are 12.10 cm and 25.21 cm
(accurate to 0.01 cm). Compute the tangent of the angle opposite

the first leg.


3118. Compute the indicated powers of the approximate
numbers (the bases are correct to the indicated decimals):
a) 0.4158 2 ; b) 65.2 1 ; c) 1.5 2
3119. The side of a square is 45.3 cm (accurate to I mm).
Find the area.
3120. Compute the values of the roots (the radicands are
correct to the indicated decimals):
a) Y2.715; b) V65.2; c) J,/81.1.

3121. The radii of the bases and the generatrix of a truncated


cone are R=23.64 cmO.OI cm; r-=17.31 clnO.OI cm; 1=
= 10.21 em + 0.01 cm; 1t == 3.14. Use these data to compute the
total surface of the truncated cone. Evaluate the absolute and
relative errors of the result.
3122. The hypotenuse of a right triangle is 15.4 cm + 0.1 cm;
one of the legs is 6.8 cm 0.1 cm. To \\yhat degree of accuracy
can we determine the second leg and the adjacent acute angle?
Find their values.
3123. Calculate the specific \veight of aluminiulTI if an aluminium cylinder of diameter 2 cm and altitude II em weigh5
93.4 gmt The relative error in measuring the lengths is 0.01,
while the relative error in weighing is 0.001.
3124. Compute the current if the electromotive force is equal
to 221 volts 1 volt and the resistance is 809 ohms I ohm.
3125. The period of oscillation of a pendulum of length l is
equal to
T

T=21t

V-

where g is the acceleration of gravity. To what degree of accuracy


do we have to measure the length of the pendulum, whose period
is close to 2 sec, in order to obtain its oscillation period with a
relative error of 0.5%? How accurate must the numbers 1t and g
be taken?
3126. It is required to nleasure, to within 1%, the lateral
surface of a truncated cone whose bas~ radii are 2 m and 1 m,
and the generatrix is 5 m (approximately). To what degree of

[Ch, 10

Approximate Calculations

872

accuracy do we have to measure the radii and the generatrix and


to how many decimal places do we have to take the number n?
3127. To determine Young's tnodulus for the bending of a
rod of rectangular cross-section we use the formula
I

liP

=4 d3bs'
where I is the rod length, band d are the basis and altitude of
the cross-section of the rod, s is the sag, and P the load. To
wha t degree of accuracy do we have to measure the length land
the sag s so that the error E should not exceed 5.50A) , provided
that the load P is known to 0.1 ~u, and the quantities d and b
are known to an accuracy of 1%, I ~ 50 em, S ~ 2.5 cm?

Sec. 2. Interpolation qf Functions


to. Newton's interpolation formula. Let Xo, Xl' , Xn be the tabular values of an argument, the difference of which h=~xj (L\Xj=Xj+l-xi; i=O,I,
.. , n - 1) is constan t (table Inter val) and YOt Yl'
., YtI art' the correspon din~ values of the function y Then the valup of the function y for an intermediate value of the argument x is approximately given by Newton's interpolation formula

Y=Yo+q.,1yo+ q (q2~ I) ,12yO + ... + q (q-I). n~q-n + I) ,1nyo

x-X
where q = -h-o and ~Yo = YI- Yo, L\2yo = L\y.- !lyo,

(1)

are successIve fini te

dU'erences of the furction y. \\ hen x=Xj (t=O, 1,


, n), the ro1ynomial
(1) takes on, accorciingly, the tabular values Yi (l =0, 1, . . , n). As particular cases of l'\ewtc,n's fOrlllula we obtain: for n= 1, linear Interpolation;
for n ~ 2, qupdratlc Interpolation. To simplify the use of Newton's fonnula,
it IS advisable first to set up a table of finite differences.
If Y=f (x) is a polynomial of degree n, then
L\nYi = const and

~n+IYi=-O

and, hence, formula (1) i~ exact


In tile general case, if f (x) ha') a continuous derivative f (n+J) (x\ on the
interval la, b), which includes the points x o, Xl' , x n and x, then the error
of formula (1) is

R ()n

X -

"n....

Y- k..

q(q-l) ... (q-i+l)


If

~'Yo_

;=0

(2)
whE're ~ is some intermediate value between xi (I =0, 1, ... , n) and x. For
practlcal use, the {ollowing approximate formula is more convenient:

Interpolation 0/ Functions

Sec. 2)

373

If the number n may be any number, then it is best (0 choose it so that


the difference ~n+ lyO::::: 0 w,thin the "mits of the given accuracy; in other
words. the difterences 6.'l yo should be constant to wi thin the given places of
decimals
Example 1. Find sIn 26"15' using the tabular data sin 260 =0,43837.
sin 27 ~ 0.45399, sin 28 -:- 0.46947.

Solution. We set up the table


X,

o1

26

261fi'-26

AI/,

0 42837
0 45399
0 46947

27(1
28

I I

Y,

A'II.

I 1562
I --14
1548

Here, 11=60', q=
60'
4 .
Appl ying formula (1) and uSing the first horizontal line of the table, we
have
sin 2615' =0.43837+

.!- ('!--l
~,

0.01562+ 4

' . (-0.00014)=0.44229.

Let us evaluate the error R2 USing formula (2) and taking into account
that if y = Sill x, then I y,nl I ~ 1, we will have:

IR21~

f(f- I )({-2)(3t)3 =12d5rl.333~410.


7
1
I

_I

180

31

Thus, all the decimals of sin 2015' are corr~ct.


Using Newton's formula, it is alsc pOSSible, from a ~iven intermediate
value of the function y. to find the correspolldlng value of the argument x
(inverse interpolation). To do this. first deterrnlne the corresponding value q
by the method of successIve approx,nlatlon, putting
q'O) =Y-Yo
tiyo

and

(I) ( ( I )
1) A2
q(l+l)==q(O)_L:L-=-. -lh._
21
I!J.Yo

(I

= 0, 1. 2. . .. ).

Here, for q we take the common value (to the given accuracyl) of two successive approximations qC1n) '='lcm+l). Whence x=xo+q.h.
Exanlple 2. USln!! the table

I
2 2
2 4

2 6

II=-

,lnl1

4.457
5.466

1.009
1 229

6.695

approximate the root of the equation Sinh x = 5.

0.220

'874

Approximate Calculations

(Ch 10

Solution. Taking Yo = 4.457, we have

5-4.457 0.543 -0 538.


1.009 =1.009-
,
(0) + q(O) (l_q(O A2 yO =0 538 + 0.538.0.462 0.220 =
(0)_

(I) _

q -q

~Yo'

qIZ)=0.538+ 0.565;0.435.

1.009

= 0.538 +0 .027 = 0.565;

~:~:=0.538+0.027=0.565.

We can thus take


x= 2.2+0.5650.2 =2.2 +0.113=2.313.
2. Lagrange's interpolation formula. In the general case, a polynomial of

degree n, which for x=xi takes on given values Yi (i =0, I, ... , n), is given
by the Lagrange interpolation formula
JJ

+ ...

(X-Xl~ (X-X z\ . .. \X-Xn ) ) Yo X-Xo~ ~X-X2)) .(~-Xn) )YI


(X O-x1 (XO-x 1 Xo-X n
X1-X O XI -X 2 X1 -Xn
(x-x o) (X-Xl)" .(X-Xk_l) (X-Xk+l)" . (X-X n )

(Xk-Xo) (Xk-Xl)" (Xk- Xk-I)(Xk- Xk+l)' .. (Xk-Xn) Yk

(X-X o) (X-Xl)" .(X-Xn - 1)


...
(Xn-X o) (Xn-X J ) (Xn-X n - l )

Yn

3128. Given a ta hie of the values of x and y:


x

I
I

2
3

10

15

12

tabl~ of the finite differences of the function y.


3129. Set up a table of differences of the function y=x s _
-5x 2 +x,-l for the values x= 1, 3, 5, 7, 9, II. Make sure that
all the finite differences of order 3 are equal.
3130*. Utilizing the constancy of fourth-order differences, set
up a ta ble of differences of the function y = x 4 - IOx l + 2x -t- 3x
for integral values of x lying in the range I ~ x ~ 10.

Set up a

3131. Given the table

log 1 = 0.000,
log 2 = 0.301,
log 3 = 0.477,

log 4 = 0.602,
log 5 = 0.699.

Use 1inear interpolation to compute the numbers: log 1.7, log 2.5,

log3.i, and log 4.6.

Sec. 2]

375

Interpolation of Functions

3132. Given the table


sin 13=0.2250,
sin 14=0.2419,

sin 10=0.1736,
sin 11=0.1908,
sin 12 = 0.2079,

sin 15 = 0.2588.

Fill in the table by computing (with Newton's formula. for n = 2)


the values of the sine every half degree.
3133. Form Newton's interpolation polynomial for a function
represented by the table

40

85

3134*. Form Newton's interpolation polynomial for a function'


represented by the table
x
y

2
3

I I

10

I 11 I 27

50

83

Find y for x=5.5. For what x will y=20?


3135. A function is given by the table
x

-2

25

-8

I I
2

1-151 -23

Form Lagrange's interpolation polynomial and find the value of


y for x=O.
3136. Experiment has yielded the contraction of a spring (x 111IU)
as a function of the load (P kg) carried by the spring:

I I 10
P I 49 I 105
x

15

20

25

30

35

40

172

253

352

473

619

793

Find the load that yields a contraction of the spring by 14 rom..

Approximate Calculations

376

[Ch. 10

3137. Given a table of the quantities x and y


x

131415
-3 I 25 1 129

381

\A>mpute the values of y for x = 0.5 and for x = 2: a) by means


of linear interpolation; b) by Lagrange's formula.
Sec. 3. Computing the Real Roots of Equations
10 Establishing initial approximations of roots. The approximation of the
roots of a given equation
(1 )
f (x) =0
consists of two stages: 1) separating the roots, that is. establishing the intervals (as small as possible) within which lies one and only one root of equation (1); 2) computing the roots to a given degree of accuracy
If a function I (..\) is defined and continuous on an interval [a. b] and
f(a).' (b) < 0. then on [a. b) thrre is at least one root ~ of equaticn (1).
This root will definitely be the only one if I' (x) > 0 or f' (x) < 0 when
a <x < b.
In approximating the root; it is advisable to use millimetre paper and
construct a graph of the function y = I (x). The abscissas of the r-oints of
intersection of the graph with the x-axis are the roo~s of the equation f (x) =0.
It is sometimes conveni(:'nt to replace the given equation with an equivalent
equation q> (x):.-:: 1f (~). Then the roots of the equation are found as the abscis~as of f,oints of intersection of the graohs l/ = {fJ (x) and y == (x).
2. The rule of proportionate parts (chord method). If on an interval (a. b]
there i~ a unique root ; of the rql1ation f (>.) = O. where the function f (x)
is continuous on [a. b), then by repla( ing the curve y = f (x) by a chord
passing through the points [a, f (a)l and [b. I (b)), we obtain the first
approximation of the root

f (a)

C1 = Q -

I (b)-I (a) (b-a).

(2)

To obtain a second approximation c2 , we apply formula (2) to that one of


the intervals (a, cll or [Cl' b) at the ends of which th(:' function f (x) has
values of oppcsHe sign. The succeeding approximations are constructed in the
same manner. The sequence of numbers en (n = I, 2, ..) converges to the
root ~. that is,

Generally sreaking, we should continue to calculate the approximations CIt


until the decitnals retained in the araswer cease to change (in accord
with the specified de~ree of accuracy!); for intermediate calculations, take
one or two reserve decimals Thi~ is a general remark.
If the function f (.\) has a nonzero continuous derivative f' (x) on the
interval [a. b], then to evaluate the absolute error of the approximate root

c2 ,

Sec. 3]

Computing the Real Roots of Equations

377

en' we can make use of the formula


P:-c 1~lf(cn)1
where J..t =

min

a~.t<.b

f I' (x)

J..t'

3. Newton's method (method of tangents). If f' (x) :1= 0 and f" (x) :A 0 for
a ~x ~ b, where I (a) f (b) < 0, f (a) (a) > 0, then the successive approximations X n (n = 0, 1, 2, ... ) to the root ~ of an equation f (x) =0 are computed
froln the formulas
f (x( n - t) ) ( rz.=,
1 2, ....
)
(3)
xo=a, xn=x - -!'

r'

Xn -

Under the given assumptions, the sequence


tonic and
lim xn=~.

X n (n

= 1,

2, ... ) is nl0no-

n-+e:tJ

To eval uate the errors we can use the formula

where f.1 = min

I f'

a~x~b

(x)

I.

For practical purposes it is 1110re convenient to use the simpler formulas


xo=a, x,,=xll _ t -af(x n - t ) ~n=l, 2, ... ),
(3')
where a =

t ~a)'

which yield the same accuracy as formulas (3).

If f (b) /" (b) > 0, then in fornlula'; (3) and (3') we should put xo=b.
4. Iterative Inethod. Let the given equation be reduced to the form
x==q>(x),

(4)

where' <P' (x) I ~ , < 1 (r is constant) for a ~ x ~ b. Proceeding from the initial value xo' which belongs .to the interval [a, bJ, we butld a sequence of
numbers Xl' X 2 , according to the following law:
X1=q> (x o)' x 2 =q> (Xl)' . , xu=q> (X"_l)'

(5)

If a <. X n ~ b (n = 1, 2, ... ), then the 1itnit


;= lim x n
n

IX)

is the only root of equation (4) on the interval [a, b); that is, xn are succes-

sive ap proximat ions to the root ;.


The evaluation of the absolute error of the nth approximation to xn is
given by the formula

Therefore, if xn and x,,+t coincIde to within e, then the limiting absolute


error for xn will be - 81
-r

In order to transform equation f (x) =0 to (4), we replace the latter with


an equivalent equation
x=x-Af (x),
where the number A:. 0 is chosen so that the function d!! [X-AI (x = I-Af' (x)
.t

Approximate Calculations

.378

[Ch. 10

should be small in absolute value in the neighbourhood of the point Xo (for


example, we can put I-At' (x o) =0].
Example 1. Reduce the equation 2x-ln x-4 =0 to the form (4) for the
initial approximation to the root Xo = 2.5..
Solution. Here, f(x}=2x-lnx-4; f'(X)=2-1.. We write the equivax
lent equation X=X-A (2x-ln x-4) and take 0.5 as one of the suitable
val ues of A; this number is close to the root of the equation

,II-A ( 2

_..!-) I _

=0, that is, close to 1..!-6:::::: 0.6.


x
x-2.5
.
The initial equation is reduced to the form
x=x-0.5 (2x-lnx-4)

or
I

x=2+2"lnx.
Example 2. Compute, to two decimal places, the roof; of the preceeding
equation that lies bet\veen 2 and 3.
Computing the root by thJ; iterative method. We make use of the result
of Example 1, putting Xo = 2.5. We carry out the calculations using formulas
(5) with one reserve decimal.
xt

=2+2" In 2.5::::::2.458,

I
x2 =2 +"2 In 2.458:::::: 2.450,
I

xa=2+ In 2.450:::::: 2.448,


2
I

x4 =2 +2 In 2.448 ~ 2.448.

And so ~ ~ 2 45 (we can stop here since the third decimal place has
'become fixed)
Let us nQw \evaluate the error. Here,
1
lp(X)=2+{lnx and lp'(X)=2 x.
Considering that all approximations to xn lie in the interval [2.4, 2.5], we
get
f=max I lp' (x)

'=2.~.4=0.21.

Hence, the limiting absolute error in the approximation to x. is, by virtue


of the remark made above,

~= 1~~~1
Thus, the exact root

0.0012::::: 0.001.

of the equation lies within the limits


2 447

< ~ < 2.449;

we can take '~2.45, and all the decimals of this apprOXimate number will
be correct in the narrow sense.

Computing the Real Roots of Equations

Sec. 8)

379'

Calculating the root by Newton's method. Here,

f' (X)=2-~,

f(x)=2x-lnx-4,

On the interval 2 ~ x ~ 3 we have:

f (3) 1" (3) > O. Hence, the conditions

l' (x) > 0 and


()f

We take

Q=

1
( 2- 3

3 for

Xo =3

r (xl> 0;

f (2) f (3) < 0;

are fulfilled.

)-1 =0.6.

We carry out the calculations using formulas (3') with two reserve decimals:
x 1 =3-0.6(2.3-ln3-4)=24592;
XI =2.4592-0.6 (22 4592-ln 2 4592 -4) = 2 4481;
x, = 2. 4481-0.6 (2 2. 4481-ln 2.4481-4) = 2. 4477;
x 4 =2.4477 -0.6 (2 2 4477 - In 2.4477 -4) =2 4475.
At this stage we stop the calculations, since _the third decimal place
does not chan~e any more. The answer is: the root. ;=2.45. We olnit the
evaluation of the error.
5. The case of a system of two equations. Let it be required to calculate the real roots of a system of two equations in two unknowns (to a glven
degree of accuracy):
f (x, y) =0,
(6)
{ q> (x, y) =0,
and let there be an initial approxinlation to one of the solutions (~, l)) of
this system x = X o' Y = Yo'
This initial approximation may be obtained, for example, graphically.
by plotting (in the sallle Cartesian Loordinate systenl) the curves f (x, y) =0
and q> (XI Y) ::::::0 and by determining the coordinates of the points of intersection of these curves.
a) Newton's method. Let us suppose that the functional determinant
1= a(I, q
a (x, y)

does not vanish near the initial approximation x=xo' y=Yo' Then by Newton's method the first approxitnate solution to the system (6) has the form
XI = X o + uo' Yl = Yo
~o' where ao' ~o are the solu tion of the syst~ln of two.
linear equa lions
I (x o' Yo)
aof~ (xo Yo) ~of~ (x o, Yo) = O.

+
+
q> (x o, Yo) +o.oq>: (x o, Yo) + ~oq>~ (x o, Yo) = O.

The second approximation is obtained in the very same way:


X 2 =X 1

where a p

~I

+U

l ,

Y2=Yl

+ Pl'

are the solution of the systeln of linear equations

f (Xl'
{

YI) +alf~ (Xl' YI)

q> (x I' !II)

UI

cp~

(x I' Y I)

+ P.f~ (Xl'

+ ~ Icp;, (Xl'

YI) =0,

YI) = O.

Similarly \ve obtain the third and succeeding approximations.

Approximate Calculat tons

380

(Ch

10

b) Iterative method. We can also apply the Iterative method to solving


the system o( equations (6), by transforming this 'iystem to an equivalent one
x= F (x, y),
{ y=cD (x, y)

(7)

and assuming that

IF~(x.y)I+I<D~(x. y)l~r<l; IF~(x.

Y)f+I<D:J(x.

y)l~r< 1

(8)

in some two-dimensional neighbourhood U of the initial approximation (xo' Yo),


which neighbourhood also contains the exact solution (s, 11) of the system.
The sequence of approximations (x n , Yn ) (n === I. 2, ... ), WhlCh converges
to the solution 0' the system (7) or, what IS the sanle thing, to the solution
.of (6), is constructed according to the following law:
XI = F (x o, Yo), YI = cD (x o, 110)'
xz=F (XI' YI)' Yz=<1) (Xl' YI)'
X a = F (x z, Y2)'
Ya-= cD (x 2' Y2)'

If all (x n, Un) belong to U, then lhn xn ==~,


n

-+

o7J

lim y" = 1').


-+

\XJ

The following technique IS advIsed for transforming the system of equal


tions (6) to (7) with condition (8) observed. \Ve consider the system of
equations
at (x. Y) + ~q> (x, y) -= 0,
{

vt (x,

y)

+ 6q> (x,

which is equivalent to (6) provided that

y)

= 0,

I~: ~

:;l:

o.

Rewrite it in the form

+ at (x, y) + ~(P (x, y) ==: F (x, y),


y == Y + vI (x, y) -+- (~tp (x, y).;:; <}) (x, y).

x= X

Choose the parameters a, ~, y, ~ such that the partial derivatives of the


functions F (~, y) -:nd <D (x, lJ) will Ul equal or close to zero In the Initial
arprOXlIllatlun; In other words, we find u, ~, y, ~ as approxImate solutions
o the systeln of equations
1

+ at: (xo'
at~

(x o'

vf~

(x o'

+- ~cp~ (xo,
Yo) + ~q>~1 (x o,
Yo) + ~q>~ (.to'
Yo)

+ yt~1 (xo, Yo) + ~CPII (x o'

Yo)

= 0,

Yo) = 0,

= 0,
110) = O.

lJo)

Condition (8) will h(' ohserved in such a choice of parameters a, p, y, fJ


on the ilSSUlllption that the partial derivatives of the functions t (x, y) and
q> (.\, !J) do not vary very rapidly in the neighbourhood of the initial approximation (x o, !lo>'
Example 3. Reduce to the form (7) the sYliitem of equations
x2+y2_1=O,
{ x3 -li=0

.given the initial approximation to the root xo=0.8. Yo =0.55.

Computing the Real Roots of Equations

Sec. 3]

I (x, y) = X2 + y!-l, q> (x, y) =xl q>~ (xo' Yo) = 1.92, q>~ (x o' Yo) = - 1.

Solution. Here,
"" (x o, Yo) = 1.1;

381
y;

I~ (x o, Yo) = 1.6,

.
Write down the system (that is equivalent to the initial one)
{

a (x

+ y2_1) -t- ~ (x

_y)

=0,

(I a, ~ 1#= 0)
y, l>

V(x2+y2-1)+~(xl_y)=O

in the form
x=x+a(xl+yl-l)+~

y = y +V (x 2 + y2_1)

For suitable numerical values of a,


system of equations

~,

(xl_y),

+ ~ (x

y).

y and l> choose the solution of the

1+1.6a+l.92~=0,
1.1a-~=0,
1.6y -t- 1. 92~ = 0,
l+l.ly-~==O;

~-O.3, ~ ::::::-0.3, V:::::::-0.5, 6 ~ 0.4.


Then the system of equations
x=x-O.3(x2+y2-1)-0.3(xl-y),
{ y == y -0.5 (x 2 + y2_1) + 0 4 (xl_y),

l. e., we put a

which is equivalent to the initial system, has the fornl (7); and in a sufficiently sInall neIghbourhood of the pOInt (x o, Yo) condition (8) will be fulfilled.

Isolate the real roots of the equations by trial and error, and
by means of the rule of proportional parts compute thenl to two
decinlal places.
3138. x 3 - x -I 1===0.
3139. x4-~-05x-I.55=0.
3140. x l -4x --I == O.

Proceeding fronl the graphically found initial approximations,


use Ne\vton's rnethod to compute the real roots of the equations
to two decimal places:
3141. x l -2x-5=O.
3143.2 x ==4x.
3142.2x-Inx-4=0.

3144.logx=-.
x

Utilizing the graphically found initial approximations, use the


iterative method to C01l1pute the real roots of the equations to
two decirna I places:
3145. xl -5x--t 0.1 =0.
3147. x'-x-2=O.
3146. 4x == cos x.
Find graphically the initial approximations and compute the
real roots of the equations and systerns to two decinlals:
3148. x'-3x+ 1 =0.
3151. xlnx-14=0.
3149. x l -2x 2 + 3x-5 = O.
3152. Xl + 3x-0.5 = O.
3150. x 4 +Xl - 2x-2 = O.
3153. 4x-7sin x = o.

382

[Ch. 10

Approximate Calculations

3154. XX +2x-6 = O.
3155. eX + e- 1x -4 = O.
3156.

3157. { x +y-4=O,
y-Iogx-l =0.

{xxl_y=O.
+ -1=O,
l

y2

3158. Compute to three decimals the smallest positive root of


the equation tan x = x.
3159. Compute the roots of the equation x tanh x = 1 to four
decimal places.
Sec. 4. Numerical Integration of Functions
1. Trapezoidal formula. For the approximate evaluation of the integral
b

) f (x) dx
a

[I (x) is a function continuous on

[a, b]] we divide the interval of integratiol1

[a, b] into n equal parts and choose the interval of calculations h = b-a .
11

Let xi=xo+ih (xo=a, xn=b, i=O, 1, 2, ... , n) be the abscissas of the parti tion points, and let Yi = f (xi) be the corresponding values of the integrand
Y = f (x). Then the trapezoidal formula yields
b

Sf(x)dx=::h (Yo~Yn+YI+Y.+"'+Yn_l)

(1)

with an absolute error of

h2
Rn ~ 12 (b-a).M 1 ,

where M2=maxlr(x)1 when a<ax~b.


To attain' the specified accuracy e when evaluating the integral, the interval h is found from the inequality
12e
h 2 ~(b-a)
M

2
2

t )

That is, h must be of the order of Ye. The value of h obtained is rounded
off to the smaller value so that
b-a
-h-=n

should be an integer; this is what gives us the number of partitions n.


Having established hand n from (1), we compute the integral by taking the
values of the integrand with one or two reserve deciJnal places.
2. Simpson's formula (parabolic formula). If n is an even number, then
in the notation of 1 Simpson's formula
b

Sf(x)dx=::~ ((Yo+Yn)+4(YI +Y.+ ... +Yn-I)+


G

+2(Y2+Y,+ . +Yn_2)

(3)

Sec. 4)

Numerical Integration

01

Functions

383

holds with an absolute error of

h4
Rn ~ 180 (b-a) M"

(4)

where M 4 =max I t lV (x) I when a~x<b.


To ensure the specified accuracy e when evaluating the integral, the
interval of calculations h is determined from the inequality

hC

180 (b -a) M 4 <; 8.

That is, the interval h is of the order

(5)

ve. The number

h is rounded off

to the smaller value so that n = b-a is an even integer.


n
Remark. Since, generally speaking, it is difficult to determine the interval h and the nunlber n associated with it from the inequalities (2) and (5),
in practical work h is determined in the form of a rough estimate. Then,
after the result is ohtained, the number n is doubled; that is, h is halved.
If the new result coincides with the earlier one to the number of decimal
places that we retain, then the calculations are stopped, otherwise the procedure is repeated, etc.
For an approximate calculation of the absolute error R of Simpson's
quadrature formula (3), use can also be made of the Runge princIple, according to which

where 11 and ~ are the result~ of calculations from formula (3) with interval
h and II = 2h, respectivel y.

3160. Under the action of a variable force F directed along


the x-axis, a material point is made to move along the x-axis
froln x = 0 to x = 4. Approximate the work A of a force F if a
table is given of the values of its tTIodulus F:
x

0.0

0.5 11.0 11.5

1 2 .0

1 2.5

3.0

F 11.50 10. 75 1 0.50 I 0. 75 11.50 1 2 .75 1 4.50

3.5

4.0

6.75

10.00

Carry out the calculations by the trapezoidal formula and by


the Simpson formula.
I

3161. Approximate

~ (3x l -4x)dx by the trapezoidal formula


o

putting n = 10. Evaluate this integral exactly and find the absolute and relative errors of the result. Establish the upper limit L\
of absolute error in calculating [or n = 10, utilizing the error
formula given in the text.

384

Approximate Calculations

[Ch. 10
1

3162. Using the Simpson formula, calculate

5:~Xl

to fOUf

decimal places, taking n = 10. Establish the upper limit d of absolute error, using the error formula given in the text.
Calculate the following definite integrals to two decimals:
1

3163.

5l+x
dx

3168.

3164.

51+x
dx

3169.

5co;x dx.
2

51+x,
dx

3170.

3166. ~ xlogxdx.
1

5l+xx dx.
2

3171.

cos

3167.

5SI: x dx.
0

3165.

5SI~X dx.

5IO~x dy.

3172. ~ e- X2 dx.

3173. Evaluate to two decimal places the improper integral

5 ~XX2 by applying the substitution x = +. Verifl the calculations


U)

by applying

Simps~~'s

is chosen liO that

formula to the integral

5 l~XI<{. 10-

51 ~\2'

where b

3174. A plane figure bounded by a half-wave of the sine curve


y = sin x and the x-axis is in rotation about the x-axis. Using the
Sitnpson formula, calculate the volume ot the sol id of rotation
to two decilnal places.
3175*. Using Simpson's formula, calculate to two decimal
.
x2
y2
places the length ot an arc of the ellIpse T+ (0.6222)2= 1 situated
in the first quadrant.
Sec. 5. Numerical Integration of Ordinary Differential Equations
to. A method of su(cessive approximation (Picard's nlcthod). Let there
be given a first-order differential equation
y'=/(x, y)
subject to the initial condition y=Yo when x=x o.

(})

Sec. 5]

Numerical Integration of Ordinary DiOerential Equations

385

The solution y (x) of (1), which satisfies the given initial condition, can,
generally speaking, be represented in the form
y (x) = lim Yi (x)
1-+

(2)

r:D

where the suCCeStHVe approximations Yi (x) are determined from the fonn ulas

Yo (x) = Yo,
x
YI (x)

=Yo+

~f

(x, YI_I (x)) dx

Xo

(i =

If the right side

f (x,

0, 1, 2, ... ).

y) is defined and continuous in the neighbo~lrhood

R{lx-xoJ~a, IY-Yo'~b}
and sa t isfies, in this neighbourhood, the Lipschitz cond itlon

(L. is constant), then the process of successive approximation (2) definitely


converges in the interval
Ix-xol~h,

where h =

m~n ( a, ~)

and M = m;x I f (x, y) I. And the error here is

Rn-,y(x)-Yn(x)I~/v'L

I X-X o In+l
(n+I)1

'

If

I x-xo ' ~ h.
The method of successive approximation (Picard's method) is also applicable, with slight nl0difications, to normal systems of differential equations.
Differential equations of higher orders rnay be written in the fonn of systems
of differential equations.
2. The Runge-Kutta method. Let it be required, on a given Interval
X o ~ x ~ X, to find the solution Y (x) of (1) to a specified degree of accuracy 8.
To do this, we choose the interval of calculations h=X-xo by dividing
the interval [x o, Xl into n equal parts so that h4
xi are determined from the formula

xl=Xo+ih

(t

< 8.

The partition points

=0, 1, 2, ... , n).

By the Runge-Kutta method, the corresponding values Yi=Y (x;) of the desired
function are successively computed fron] the formulas
Yi+1 =Yi+ ~Yi.

~Y'-.!.
+2k(i)+2k(t)+k(i
'-6 (k(i)
1
I

..,

13-1900

38.::.6

.-,.;A:..:.!:..p!:....pr:....:o:.....x--.im_at_e_C_a_lc_u_la_t_io_n_s

[ C_"'h_._10

where

(3)

To check the correct choice of the interval h it is advisable to verify


the quantity

e_
-

k(i)-k(t)

2
a
k~t)-k~t)

The fracti'Jn
should amount to a few hundredths, otherwise h has to be
reduced.
The Runge-Kutta method is accurate to the order of hI. A rough estiIn ate
of the error of the Runge-Kutta method on the given interval [x o, Xl Illay
be obtained by proceeding from the Runge principle:

R = I Y2rn--Ym
15

'

Ym

where n = 2rn, Y2tn and


are the results of calculations using the schenle (3)
\vith interval h and interval 2h.
The Runge-Kutta method is also applicable for solving SYStClllS of dlffe~
rential equations
y'=f(x, y, z), z'-=cp(x, y, z)
(4)
with given initial conditions Y==Yo, z=zo when x==x o.
3. Milne's method. To solve (1) by the Mtlne nzethod, subject to the
initial condi\ions Y=Yo when x=xo, we in SOlne way find the successIve
values
YI = Y (XI)' Y2 = Y (x 2 ), Y, == Y (x a)
of the desired function y (x) [for instance, one can expand the solution Y (X)
in a series (Ch. IX, Sec. 17) or find these values by the lTlethod of successi ve
approximation, or by using the Runge-Kutta Inethod, and so forth]. The approximations
and
for the following values of Yi (i =4, 5, ... , n) are
successively found frool the formulas

Yi

Yi

Yl=Yi-.+ ~h (2/; __ -/; __ +21;_.), }


h _

&::

Yi=Yi-2+

"'here fi= f (xi, Yi) and

3" (Ii +4Ii-l + Ii-I)'

fl= f (xi, Yl).


1

(5)

To check we calculate the quantity

1-

=:7

ei=29 YI-Yi

(6)

Sec ..5]

Numerical Integration of Ordinary Differential Equations

387

If 8i does not exceed the unit of the last decimal 10- m retained in the
answer for Y (x). then for Yi we take
and calculate the next value Yi+t,
repeating the pr8cess. But If 8i > 10-tn, then one has to start from the beg1nning and reduce the interval of calculations. The Illagnitude of the initial
interval is detennined approXinlately fronl the inequality hoi < lo-m.
For the case of a solution of the system (4), the Milne formulas are
written separately for the functions y (x) and z (x). The order of calculations
remains the same.
Example 1. Given a differential equation y' =y-x \vith the initial condition y (0)= 1.5. Calculate to two decinlal places the value of the solution
of this equation when the ar~ument is x -= 1.5. Carry out the calculations
by a combined Runge-Klltta and Milne method.
Solution. We choose the initial Interval It frOll) the condl tion hoi < 0.01.
To avoid involved writing, let us take h-=0.25. Then the entire interval of
integration frool x=O to x= 1.5 is divided into six equal parts of length
0.25 by means of points xi (i == 0, 1, 2, 3, 4, 5, 6); we denote by Yi and
the corresponding values of the solution y and the derivative y'.
We calculate the first three values of y (not counting the initial one) by
the Runge-Kutta ruethod [frotH fonnulas (3)]; the remaining three values
- Y4' Ys, Y6 - we calculate by the Milne I11ethod [from formulas (5)]
The value of Ye will obviously be the answer to the problem.
We carry out the calculations with two reserve decilnals according to a
definite schenle consisting of h\'o sequential Tables I and 2. At the end of
Table 2 we obtain the answer.
Calculating the valueYl' Here, f(x, y)=-X+IJ, xo==-O, Uo=1.5

!Ii

y;

~ (k~O) + 2k~O) -I- 2k~O) + k~O =

h = 0.25. !J.Yo =

k~O)

== f (x o,

Yo) h ==

k~O)=f ( xo+~.
h

+2 '

k~O)=f ( x o

k~O) == f (xo-t-/z,

(0.3750+20.3906+20.3926+0.4106) =0.3920;

(- 0 -1- 1.5000)0.25 -= 0.3730;


k(O)

Yo+-+) h=(-0.125+ 1.5000+-0.1875) 0.25= 0.3906;


k(O) )

Yo++

h=(-O 125+1.5000+0.1953)0.25=0.3926;

Yo+ k~O h = (- 0.25

+ 1.5000 + 0.3926) 0.25 =

0.4106;

YI =Yo+ ~Yo== 1.5000+0.3920== 1.8920 (the first three decilnals In thts

approximate number are guaranteed).


Let us check:

8=

k~O) _k~O)

10.3906-0.3926 1

20

k~O)_k~o) = 1 0 .3750-0.39061 "'='156=0.13.

By this criterion, the interval h that we chose was rather rough.


Sim ilarly we calculate the values Y2 and Y. The results are tabulated
in Table 1.
13*

388

[Ch. 10

ApproxImate Calculations

Table 1. Calculating

.vI' YJ' .vI

by the Runge-Kutta Method.


h=0.25

f(x, y)=-x+y;
I

t (Xi+: '

Value of i

1
2

Value of i

== f (xi,

1.5000
1.8920
2.3243
2.8084

0
0.25
0.50
o 75

Yi~

YI

xl

t( Xi+i '

k(i)

k~i) )

k(i)

kIll)

Yi)

1.5000
1 .6420
1 .8243
2.0584

f (xi h,
Yt-j- k~i

k(i)

Y'+T
0.3750
0.4105
0.4561
0.5146

1.5625
1.7223
1.9273
2.1907

0.3906
0.4306
0.4818
0.5477

k(i)
4

~Yi

Yl+1

0.4106
0.4562
0.5148
0.5900

0.3920
0.4323
0.4841
0.5506

Yi+T
0

1 .5703
1.7323
1 .9402
2.2073

2
3

1.6426
1.8251
2.0593
2.3602

0.3926
0.4331
0.4850
0.5518

1 .89-20
2.3243
2.8084
3.3590

Calculating the value of .P4. We have: f (x, y) = - x+ y, h=O.25, x4 = 1;


Yo= 1.5000, YI = 1.8920, Y2 = 2.3243, Y. = 2.8084;
I

y~ = 1.5000,

Y; = 1.6420,

y~ = 1.8243,

y; = 2.0584.

Applying formulas (5), we find


Y4=Yo

"
+ 4h
3 (2Y -Ya + 2y.) =
1

= 1.5000 + 4.0 25 (21.6420-1.8243

if, = f (x it) = 4,

==

h -/
"
Y'=Y2 +"3 (Y4 +4y.

s,

+2.2.0584) = 3.3588;

+ 3.3588 = 2.3588;
+ Y = 2.3243+ 0.25
3 (2.3588+4.2.0584 + 1.8243)=3.3590;
I )

I~Y: 1_,3.3588-3.3590
, _ 0.0002
7 10-' < 1 0 001.
29
29
- 29:::::'
1"'.'

hence, there is no need to reconsider the Interval of calculations.

Sec. 5]

Numerical Integration of Ordinary DiOerential Equations

389

We obtain Y4 = ~ = 3.3590 (in this approximate nu'mber the first three


decimals are guaranteed).
Similarly we calculate the values of y, and Yo' The results are given in
Table 2.
Thus, we finally have

y (1.5) = 4.74.
4. Adams' method. To solve (1) by the Adams method on the basis of
the initial data y (x o) = Yo we in some way find the following three values
of the desired function y (x):

(these three values may b~ obtained. for instance. by expanding y (x) in a


power senes (Ch IX, Sec. 16). or they may be found by the method of suc
cessive approximation (1). or by applying the Runge-Kutta nlcthod (2)
and so forth].
With the help of the nurnbers xO' x.. x 2 Xa and Yo. Yl' Y2. Ya we calculate qo' qt. q2' qat where
qo = IzY~ -= hf (x o Yo).
q2 = h Y~ === hf (XI' Y2)'

qt

= hy; = hf (XI'

YI)'

qa = hy::= hI (x a Yale

We thrn form a diagonal table of the finite differences of q:

li.y=
x

Xo

XI

~!lo

IY\ I

~YI

x2 1 Y2!
XI

x4

~Y2

II/I I

~Ya

Y41

~Y4

XII Y.

y)

q=y'h

aq=Qn+l-q",

li. 2 q=
=li.qU+I-li.q,l

li.1q=-

=li. 2q,.+ .-li.'Jq"

110

1/ =f (x,

;; l/n+ I -lIn

"

XII Y.I

~y,

I qo
f (XI'
I q,
f
I q"
f a,
I qa
t (X Y4) I q4
f (XI' UI) I q.
f (X Ot Yo)

~qo

t:tlqo

~3ql)

YI)

~ql

/).2

QI

,\'q.

(x 2 Y2)

~q2

~2q2

~3q2

Ya)

~qa

&2qa

(X

4t

tiq4

[Ch. 10)

Approximate Calcu lat ions

390

The Adam5 method consists in continuing the diagonal table of differences with the aid of the Adanzs formula
A_I

u y ,,--qn+2 uqn_l

+ 125

A2

3
qn-2+"8

AI

qn-I'

(7)

Thus, utilizing the numbers qa, 6,q2' t1 2qh &a qo situated diagonally in
Ihe difference table, we calculate, by means of formula (7) and putting n::.:.- 3
in It, L\YI=ql+-}

L\ql+~ L\lql + ~

L\'qo. After finding L\YI. we calculate

Y4= Ya + t1Ya And when we know X4 and U,a, we calculate q4= hf (x 4 , Y4)'

introduce Y4' &Ya and q4 into the difference table and then fill into it the
finite differences 6,qa' 6,"q2' !J.3 q1J which are situated (together with q4) along
a new diagonal parallel to the fIrst one.
Then, utilizing the nunlbers of the new diagonal, we use formula (8)
(putting n=4 in it) to calculate ~Y4' Y5 and q, and obtain the next diagonal:
qa, t1q., ~Iqa, 6,aq2' Using this diagonal we calculate the value of y, of the
desired solution y (x), and so forth.
The Adams formula (7) for calculating fiy proceeds from the assumption
that the third finite differences 6,3 q are constant. Accordingly, the quanti tv h
of the initial interval of calculations is determined frol11 the inequality
h 4 < lo-m [if we wish to obtain the value of y (x) to an accuracy of
lO-m].

In this sense the AdalTIS formula (7) is equivalent to the fonnulas of


Milne (5) and Runge-Kutta (3).
Evaluation of the error for the Adams nlethod is complicated and for
practical purposes is useless, since in the general case it yields results with
considerable excess. In actual practice, we follo\v the course of the third
finite differences, choosing the interval h so stnall that the adjacent differences ~3qi and 6,3 qi + 1 differ by not more than one or t\\70 units of the ~iven
decil11al place (not counting reserve desimals).
To increase the accuracy of the result, Adams' fOrlnula may be extended
by terms containin~ fourth and higher differences of q, in which case there
is an increase in the number of first values of the function y that are needed
when we first fill in the table. We shall not here give the Adams fonnula
for higher aceuracy.
Example 2. Using the combined Runge-Kutta and Adanls nlethod, calcu
late to two decinlal places (when x = 1.5) the value of the solution of the
differential equation y' = y-x wi th the Initial condi tion y (0) == 1.5 (see
Exanl pIe I).
Solution. We use the values Yl' Y2' 1JI that we obtained in the solution
of Example 1. Their calculation i~ given in Table 1.
We calculate the suhsequent values Y4' Ys, y, by the Adams method (see
Tables 3 and 4).
The answer to the problerTI is U, = 4.74.
For solving system (4), the Adams formula (7) and the calculation scheme
shown in Table 3 are applied separately for both functions Y (x) and z (x).

Find three successive approximations to the solutions of the


differential equations and systems indicated below.
3176. y'=XI_t-yl; y(O)=O.
3177. y'=x+y+z, z'=y-z; y(O)=l, z(O)=-2.
3178. y" = - y; Y (0) = 0, y' (0) = 1.

Table 2. Calculating J.'4' Y5' )'6 by the Milne Method.


y) == - x h; h = 0.25. (ItalIcised fIgures are Input data)

f (x,

:
Value
ot I

X,

IJI.

U: =/

(Xi'

Y.>

500
/1. 1 1.5000

Y,

I,--

U.=!

(x,.

y,)

=y,

Yi

1.

Y: =/

(X,.

Reconsider interval
of calcula t 10 ns,
following Indications
of formula (6).

Y,)

10, 11.8921 1.6420

1111111111l1li1111111111111111111111111111111111111111111/11111111111111//111111111111 I/JIIIIIIIIIIIIIIIII

/.512.32431

/111111111111111111111111/111111111111111111111111111111111111111111111111111111111111111111111111111111 11/11111111111111/11111111111111111

IIIJIIIIIIIIIII 1111111111111111111/11 111111l1li111/1 111111/111111/1111111111111/1111111111111/11111//111/1//1/11//1///11/1111/1111/111/11111

25

1.8243

111/11111111111[1111111111/11111111

i a /.75/2.80841 2.0584 11/1/1/111111111111111111[1111111111111111111111111 1111111111111111111 I 1111111111111111111111111111111/11111111111111111111111111111111111


i - /1.00 111111111111111111111111111111111111 3.3588 I 2.3588 I 3.3590 I ~71O-5 13.3590 I 2.3590 IDo not reconsider
4

11.251111111111111 111/1/1/111111111111111 3.9947 1 2. 7447 j 3.9950 j

11.50 IIIIIIIIIIIIIIIIIIIIIIJIIIIIIIIIIIII 4.7402

~ 10- 5

3.2402 1 4.7406 j ~ 1.4.10Answer:

13.9950 I 2. 7450

I Do not reconsi,der

j~ll

I Do not reconsider

y (1.5) =4.74

.ve by the Adams Method.


h = 0.25
(Italicised figures are input data)

Table 3. Basic Table for Calculatlni Y4' .vI'

f (x,

=-

y)

....

+ y;

0
Q.I

6.Yl

Yi

t,

=f

Yi=
(xl,

qi=

Y;h

/i:lq,

li,2q,

6.q,

gil

>-

111111111111111111 t .5000 I 0.3750 I 0 0355


o.25 f t .8920 111111111111111111 1.6420 0.4105 0.0456
210 501 2.3243 111111111111111111 t .8243 I 0.4561 0.0585
3/0.751 2.8084
0.5504 2.0584 0.5146 I 0.0751
411 001 3.3588 0.6356 1 2.3588 0.5897 0.0964
511 251 3.9944 0.7450 I 2.7444 0.6861

01

"Il

I t .5000

I
I

I
I

I 0.0101 I 0 0028
I 0.0129 I 0 0037
I 0.0166 10.0047

I 0.0213

Table 4 Auxi liary Table for Calculating by the Adams Method


t1y,=qi+ ; l\qi_l+

Value of

q,

!.2

6.q t-1

t21\2Qi -.+ ~ l\'qi-I


.!.. 6. 2q'_2

.!.
6. 3q.&- 9
S

12

dY,

0.5146

0.0293

0.0054

0.0011

0.5504

o 5897

0.0376

0.0069

0.0014

0.6356

0.6861

0.0482

0.0089

0.0018

0.7450

Approximating Fourier Coefficients

ISec. 6]

393

Putting the interval h = 0.2, use the Runge-Kutta method to


calculate approximately the solutions of the given differential
equations and systems for the indicated intervals:
3179. y'=y-x; y(0)=1.5 (O~x~I).
3180. y' = JL_ y 2; y (I) = 1 (I ~ x ~ 2).
x
3181. y'=z+l, z'=y-x, y(O)=l, z(O)=1 (O~x~l).

Applying a combined Runge-Kutta and Milne method or


Runge-Kutta and Adams method, calculate to two decimal places
the solutions to the differential equations and systems indicated
below for the indicated values of the argument;
3182. y' = x + y; y == 1 when x == O. Compute y when x = 0.5.
3183. y' = x 2 -1- y; y = 1 when x = O. Compute y when x = 1.
3184. y' =2y-3; Y-== 1 when x=O. Compute y when x=0.5.
3185.
= -x +2y-+ z,
z' = x -t- 2y + 3z; y = 2, Z = - 2 when x = o.
Conlpute y and z when x = 0.5.
3186.
= - 3y-z,
z' =y-z; y=2, z=-I \vhen x=O.
Compute y and z when x == 0.5.
3187. y"=2-y; y=2, y'= - I when x=O.
Cornpute y when x = I.
3188. Y3Y"-i-1=0; y=l, y'=O when x=l.
Compute y when x = 1.5.

{y'
{y'

3189.

~2t~+i-coS2t=0; x=O, x'-'l when t=O.

Find x (rt) and x' (n).


Sec. 6. Approximating Fourier Coefficients
Twelve-ordinate scheme. Let Yn =f (x n ) (n = 0, 1, ... , 12) be the values
of the function y = f (x) at equidistant poi nts x n =
of the interval lU,2n),

:It;

and Yo - Yn We set up the tables:

YO Yl Y2 Ya Y4 Ys Y,
Yn YIO Y, Ya Y7

Sums (~)

Uo Us U 2

Uo U1

Differences (l\)

u2

Ua U. Us U,

VI V 2 Va V. Us

ua

Us Us U.

Sums

Dillerences

Sums

Differences

Approximate Calculations

394

[Ch

10

The Fourier coefficients an' bn (n = 0. I. 2. 3) of the function y = f (x)

may be determined approximately from the formulas:


000 = 50 +S 1+ 52 + 5..
60 1 = to +0.866t l -l- 0.5t 2
6a 2 =50 -5.+0.5 (51-51)'
6a l = t o-t 2

6b l = 0.50'1 + 0.8660 2
6b 2 = 0.866 ('t
't
6b a =0'1- 0 a.

1+ 2).

+ Ga.

(1)

ya
1
1
where 0.866 = -2- =:::: 1- 10 - 30
We have
a

/(x)

~ a;

+L

(an cos nx + bn sin nx).

n=J

Other schemes are also used. Calculations are simplified by the use of
patterns.
Example. Find the Fourier polynomial for the function y= f (x) (0~x~2n)
represented by the table
Yo

UI

38

38

Y.
12

14

Solution. We set up the tables:

y 138 38 12
32

4
14
48 - 24 - 27 - 23

u 138 70 20 -

20 - 13 - 19 28
41
27

38

70

20 - 20

20
56

51
89

7 - 20
33

u _ 18 _ 19 _ 13
I
b

v
0
't

18
18

I 276 414 28
I-21-37
33 45 28

From formulas (1) we have

ao=- 9.7; a l = 24.9; a2 = 10. 3; a l


bl =13.9; b2 =-8.4; b.=0.8.

= 3.8;

Co n sequen t I y
f (x) ~ 4.8+ (24.9 cos x + 13.9 sin x) + (10.3 cos2x-8.4 sin 2x) +
+ (3.8 cos 3x

+0.8 sin 3x).

Using the 12-ordinate scheme, find the Fourier polynomials


for the following functions defined in the interval (O,2n) by the

Approximating Fourier Coefficients

Sec. 6]

395

tables of their values that correspond to the equid istant val ues
of the argument.
, 3190. Yo =-7200 Ya==4300
Y6=7400 yg=7600
y, = 300
Yol == 0
Y" = - 2250 Y10 = 4500
Y2 = 700
Ys = - 5200 Ys = 3850
Yl l = 250
3191. Yo=O
Ya=9.72
Ys=7.42
Ye =5.60
YI -== 6.68
Yti = 8.97
Y7 == 6.81
y,o = 4.88
Y2 == 9.68
Ys == 8.18
Ya= 6.22
YII = 3.67
3192. Yo-=::2.714
Ya=I.273
Ys=O.370
Ye =-0.357
YI = 3.042
Y4 = 0.788 Y7 = 0.540
YIO = - 0.437
Y2=2.134
Ys =0.495
Y8=O.191
YII-==0.767
3193. Using the 12-ordinate scheme, evaluate the first several

Fourier coefficients for the follo\ving functions:


a)

f (x) = 2n1

b)

f (x) =

(Xl - 3nx 2 -t- 2n 2 x)

n 2 (x-n)2

(0 ~ x ~ 2n),
(0 ~ x ~ 2n).

ANSWERS

Chapter I
1. Solution. Since a=(a-b)+b, then lal<;la-bl+lbl. Whence ,a-bl~
la I-I bl and I a-b 1=1 b-a I~ I b I-I a I. Hence, I a-b I ~ I a I-I b I
Besides, la-bl=la+(-b)I<;lal+l-bl=lal+lbl. 3. a) -2<x<4;
b) x < - 3, x > 1; c) -1 < x < 0; d) x > O. 4. -24; -6; 0; 0; 0; 6. 5. I;

I .. r - 4 ; , I+x!;
7

..r - -

.. ~

lxI-I, l+x2 ; I/f l+x!.6.11;


13

8. f(x)="6x -6"x+l. 9. 0.4. 10.

11

"2;

5
I
0.7. f(x)=-3"x+"3.

2(x+fxl). 11. a)

-I<:x<+oo;

b)-oo <x<+oo. 12. (-00,-2), (-2, 2), (2, +00).13. a) -00 <x~- 2 ,
V2~x<+00; b) x=O, Ixl~ V2. 14. -1<:x~2. Solution. It should
be 2+X-X2~0, or x 2-x-2<;0; that is, (x+l)(x-2)~0. Whence either
x+l~O, x-2~0, i. e., -1~x~2 or x+l~O, x-2~0,i.e.,x~-1,
x~2,
but this is impossible. Thus, -1~x~2. 15. -2<x~0.
16.-oo<x~-I, O~x~l. 17. -2<x<2. 18. -1<x<l, 2<x<+oo.
1
n
19. -3~x~1. 20. l<;x~IOO. 21. k11~x~krc+2(k=0, I, 2, ... ).
22. <p (x) = 2x4-5x 2 -10,

'1' (x) =

- 3x'

e) odd.24. Hin!. Utilize the identity


28. a) Periodic,

+ 6x. 23. a) Even, b) odd,

c) even, d) odd,

f (x) = {If (x) +f (-x)] +ilf (x)-f (-x)].

T=j n, b) periodic, T= 2~, c) periodic, T ='1:, d)

T='Jt, e) nonperiodic. 27.

y=~x,
c

if O<;x<:c; y=b if

periodic

c<x~a; S=2~X2
C

if O.,.;;x"';;C; S=bx-~ if C < x";;;a. 28. m=q1x when 0 .,;;;x.,.;; 1\; m=


=Q1 l l+q!(x-l l ) when 11 < XE;;I I +l!; m=Qll1+q212+Q.(x-l1-l2) ,vhen
11+1 2 < X~ll +1 2+l.=I. 29. cp [ (x)]==2 2X ; 'I' [q> (x)]=2 X2 30. x. 3t. (X+2)2.
37.

11

-2";

0;

11

38. a) y=O when x=-I, y>O when x>-I, y<O

when x<-I;b) y=O when x=-l and x=2,y>Owhen -1<x<2,


y<O when -00 <x<-l and 2<x<+00; c) y>O when -oo<x<-1-oo;
d) y=O when x=O, x=- Y3and x= Y3, y>O when - V3 < x < 0 and
V3<x< + 00, y<O when - oo<x<- V3 and O<x< V3; e) y=O when x= 1,
1
y>Owhen-oo<x<-l and l<x<+ 00, y<O whenO<x<1 39. a) x=2 (y-3)

(-co<y<+oo);

b)

x=l"Y+l

and

x=- VU+1 l-l~y<+oo);

Answers
e)

x=

=i

V -I-ys

tan y (

(-oo<y<+oo);

-~ <y<~).

O<y<+oo.

41. a)

y=u

10

40.
,

d)

397

x=2IOY (-oo<y<+oo); e) x=

x=y when -co<yo;;;;;O; x="y when

u=2x-5; b) y=2u , u=cosx; c) y=logu,

u=tanv, v=~; d) y=arc sinu, u=311 , v=-x l 42. a) y=sln1x; b) y=


=arctan logx; e) U=2(x!-I) if Ixl<;l, and y=O
43. a) y=-cosx 2 , n';;lxl~ 21t; b) y=log(10-10X ),

if

Ixl> I.

<x<

1;
c) y=-i-when -co<x<O and y=x when OQ<+co. 46. Hint. See Appen
dix VI, Fig. 1. 51. Hint. Completing the square in the quadratic trinomial
we will have y=yo+a(x-x o)2 where xo=-b,'2a and Yo={4ac-b 2)/4a.
Whence the desired graph is a parabola y=ax' displaced along the x-axis by
Xo
and
along
the
y-axis
by
yo.
53. Hint. See . Appendix VI,
Fig.
2. 58.
Hint.
See
Appendix
VI,
Fig.
3. 61.
Hint.
-00

The graph is a hyperbola y= m , shifted along the x-axis by x o and along

the y-axis by

Yo' 62. Hint. Taking the integral part, we have y=i -

(x+ ;) (Cf. 61).65. Hint. See Appendix VI, Fig. 4. 67.Hlnt. See Appendix VI,
Fig. 5. 71. Hint. See Appendix VI, Fig. 6. 72. Hint. See Appendix VI,
Fig. 7. 73. Hint. See Appendix VI, Fig. 8. 75. Hint. See Appendix VI,
Fig. 19 78. Hint. See Appendix VI, Fig. 23. 80. Hint. See Appendix VI,
Fig. 9. 81. Hint. See Appendix VI, Fig. 9. 82. Hint. See Appendix VI,
Fig. 10 83. Hint. See Appendix VI, Fig. 10. 84. Hint. See Appendix VI,
Fig 11. 85. Hint. See Appendix VI, Fig. 11. 87. Hint. The period of the function
is T = 2njn. 89. Hint. The desired graph is the sine curve Y = 5 sin 2x with amplitude 5 and period n displaced rightwards along the x-axis by the quantity
1 { .90. Hint. Putting a=A cos q> and b=-A sin q>, we will have y=A sin (x-q
where A= al+b l and q>=aretan( - : ) - In our case, A=lO, q>=O.927. 92.
Hint. cos l x =} (l +cos 2x). 93. Hint. The desired graph is the sum of the graphs

Yl = x and Y2 = sin x. 94. Hint. The desired graph is the product of the graphs
Yl = x and y" = sin x. 99. Hint. The function is even For x>O we determine
the points at which 1) y=O; 2) y=l; and 3) y=-1. When x~ -1-00,
y --+ 1. 101. Hi n t. See Appendix VI, Fig. 14. 102. Hi nt. See Append 1X VI,
Fig. 15. 103. Hint. See Appendix VI, Fig. 17. 104. Hint. See Appendix VI,
Fig. 17. 105. Hint. See Appendix VI, Fig. 18. 107. Hint. See Appendix VI,
Fig. 18. 118. Hint. See Appendix VI, Fig. 12. 119. Hint. See Appendix VI,
Fig. 12. 120. Hint. See Appendix VI, Fig. 13. 121. Hint. See Appendix
VI, Fig. 13. 132. Hint. See Appendix VI, Fig. 30. 133.Hint. See Appendix VI.
Fig. 32. 134. Hint. See Appendix VI, Fig. 31. 138. Hint. See Appendix VI.
Fig. 33. 139. Hint. See Appendix VI, Fig. 28. 140. Hint. See Appendix VI.
Fig. 25. 141. Hint.

398

Answers

Fornl a table of values=

27

.. I

.. I

I -1

-2
-8

-1

-3

1I

27

Constructing the points (x, y) obtained, we get the desired curve (see Appendix VI, Fig. 7). (Here, the parameter t cannot be laid off geometrically!)
142. See Appendix VI, Fig. 19. 143. See Appendix VI, Fig. 27. 144. See
Appendix VI, Fig. 29. 145. See Appendix VI, Fig. 22 150. See AppendIx VI,
Fig. 28. 151. Hint. Solving the equation for y, we get y= Y25 - x!. It is
now easy to construct the desired curve froln the points. 153. See Appendix VI, Fig. 21. 156. See Appendix VI, Fig. 27. I t is sufficient to construct

i, a. 157.

the points (x, y) corresponding to the abscissas x==O,

Hint.

Solving the equation for x, we have x=..:. 10 log y-y <*). Whence we get the
points (x, y) of the sought-for curve, assigning to the ordinate!, arbl trary
values (y> 0) and calculating the abscissa x froln the formula <*) Bear in
nlind that log y ~ - 00 as y ~ O. 159. Hint. Passing to polar coordinates
r=Vx 2+y2 and tan rp=1!. , we will have r=e'~ (see Appendix VI, Fig 32)
x

160. Hint. Passing to polar coordinates x=r cos <p, and y=r sin cp, we wIll

have r =

3~in ~c~s
all'
SlIl <p

(see Appendix VI, Fig. 32)

cos q>

F = 32

161.

+ I,

Be

ab .
ab
slnx ; Ymax=2
Y=2

Ymax=15 when x=5. 163.


n
1
whenx=T' 164. a) x1 =2' x 2 =2; b) x=O.68; c) x1 =1.37, x2 =10;
d) x=0.40; e) X= 1.50; f) x=O.86. 165. a) x 1 =2, Yl =5; x 2 =5, Y2=2;
b) x 1 =-3, YJ.=-2; x 2 =-2, Y2=-3; xa==2,Ya~3;x.. =3, y.. =2; c) Xl ==2,
162.

y=O.6x(10-x);

Yl=2; x2~3.1, Y2~-2.5; d) xl~-3.6,

Ya::::: 1.8; x4 =:.:::3.4, Y4~-1.6;

x,:::::2.9,

Y2=-

~2.

-1 =N.

a)

:8'

166. n>

a)

171.

2".

:rt

e) x'=T'
b) n> 10; c)

N= 9; b) N=99; c) N=999.

b) 0 002; c) 0.0002. 169. a) log x

x> X (N);

n~4;

YI~-3.1;

c) \ f (x) ,

>

172. 1. 173.

when

-2'

< -N when
I x \ > X (N).

168.
0

x2~-2.7,

Y-2
Y'=T;

n~32.
e

~=5

<x<

(8

Y2:::=29;
5rc

x2

= 4"'

167. n

>~

<

a) 0 02;

1).

(N); b) 2x

>

N when
7

170. a) 0; b) 1; c) 2; d)
3

4".

174. 1. 175. 3. 176. 1. 177.

178.

3.

30 .

Hint.

+...

Use the formula 12 + 22


+ n2 =6"' n (n -1-1) (2n + 1). 179. O. 180. O. 18t. 1.
182. O. 183. 00. 184. O. 185. 72. 186. 2. 187. 2. 188. 00. 189. O. 190. 1. 191. O.
192.

00.

193. -2. 194.

00.

195.

2"1

196.

a-I

3a l

197. 3x. 198. -1. 199."2 .

Answers
200. 3. 201.
208. 2

1
3
203. - 56' 204. 12. 205. 2'

"9'

202.

399

t I l

Vx 209. 3 V x
~

215. O. 216. a)

;2'

2
n'

235.

243.

229.

2'

236.

2
n'

1
Y3'

230. O. 231. -

n'

244.

-"4'

237.

238.

O.

245.

232.

1t.

e- 1

246.

e2
::=

x-+o

~i:

. 220. n. 221.

~.

=-2lim
x--+o

1 ]_2~in2f
.\

---x2sin2 _
2

[(Sin- -i)2 -.X2] =-21


x

2"

.!(see

a ),

I1m (cos x )X2


I

X-.O

= _ 2 lim
x->o

';'e'

4x

4x

Solution.
11m ( - 251:'

= ex-+o

rl(Sin ~ )2 X~2] = _ .:- .


X

2~fn:!.!..)
I

It

f)

As

x
Iitn-=O, it follows

x -+0

in

the

preceding

(_ 2 2i)
Si11

. S'Ince l'1m

x~o

2
253. In 2. 254. 10 log e. 255. 1. 256. 1. 257.

x-+o

-~.

2.')8.

Hint. Put

262. I. 263. a) 1; b)

l..n =
1

"2 .

a, where a-+-O (see Exalnple 2.59)

Hint.

I.

261.

follows that lim (cos X)i> =e

Put eX -1 =a, where a ~ O. 259. In a. Hint. Utilize the identity a=e


260. In a

]
1
that lim (cos x) x = eO == 1. b) .. r- '
x~o
r e
case

e-&.

249.

=e
2

4'.

lim [1-(I-cos x)) x

lim

2sin

l.

234.

e- 1

248.

x-+o

Since Iitn ( - - X'-~o


x

b) I.

a) 0;

227.

x-.o

1- 2sin -i)x = ~i~ l(1 - 2 sln i)


2

0(

"2'

240. 1. 241. 1. 242.

247.

251. e. 252. a) 1. Solution. lim (cos x) x

250. eX.

214.

2
1
(n _m 2 ). 233.2"

4"'

239.

-2'

213.

sin 2; b) O. 217. 3. 218. { . 219.

1.

207.

2"'

211. O. 212.

222. cos a. 223. - sin a. 224. 11. 225. cos x. 226. 228.

-3.

206.

-"3'

210.

1n

a-b.

264. a) -1; b) 1. 265. a) -1; b) 1. 266. a) 1; b) O.

267. a) 0; b) 1. 268. a) -1; b) 1. 269. a) -1; b) 1. 270. a) -00; b) +00.

Answer~

400
271. Solution. If x kn (k =0.

1, 2... ).

then cos! x

<1

and Y =0;
1

but if x=kn, then cos1x=1 and y=1. 272. y=x when O-c;x< I;

Y=2

when x==l; y=O when x> I 273. Y=lxl 274. Y=-T when x<O; y=O
when x=O;
1< x <

n;

Y=T

+ 00.

when

x> O.

61
276. 450' 277.

275.

XI

O<;x~

y= 1 when

-'-1i;

n.

278.

X.-+ 00.

1; y=x when
279.

2n;R.

e
1
Yen + 1 2 4 l' AC
l
ab
280 e-l' 281. 13".282. n
. 8. n~
n=3'
285. 2 286 . k =1.
e 2 -1
I
x'+ 1
b=O; the straight line y = x is the asyrnptote of the curve y = x2 + 1 .
Q\nl = Qo ( 1

287.

+~

r.

where k is the proportionali ty factor (law of

compound interest); Qt = Qoekt 288. I x I>


c) I x , > 1000. 289.
b) Ix-11<0.005;

'x-

1\<

\vhen

..!..,
e
0

<e<

~93

N=~;

c) Ix-l \<0.0005 290. \x-2\<

29S..

3.

a) 1; b) { ; c)

300.

a)

i;

d) 2; e) 3. 295. No

1.03 (I 0296);

yfO=Y9+1=3

VI +{;

2)

3)

1.03 (1.0309);

b)
d)

I x- 1 I < 0.05;

a)

1;

i.

b) 6=0.01; c) 6 = 0.001. 291. a) Second. b) third. }.


c) 3.

\ x I> 100;

a) \ x \ > 10; b)

a)

292. a) 1; b) 2;

296. 15. 297. -1. 298. -1.

0.985 (0.9849); c) 3.167 (3.1623)


10.954(10.954).

0.0095 (0.00952);

4)

~=0.1;

301.

1) 0.98(09804);

3.875 (3.8730);

6) 0 72 (0.7480); 7) 0.043 (0.04139). 303, a) 2; b) 4; c)

~;

Hint.

5)

1.12 (1.125);

d) -}. 307.

Hint.

x>O,
then
when
lL\xl<x we have \Yx+L\x-Yxl=
=1L\XI/(Yx+L\x+yx)~IL\xI/Yx.309. Hint. Take advantage of the

If

inrquality

I cos (x+ L\x) -cos x I ~ I L\x I 310. a) x i= ~

+ kn.

where Ii is an

integer; b) x :. kn;, where k is an integer 311. Hint. Take advantage of the


inequality Ilx+L\xl-,y.l'~\~x\ 313. A=4. 314. 1(0)=1. 315. No
316. a) /(O)=n; b) /(0)=;; c) /(0)=2; d) {(0)=2; e) {(O)=O; f) {(O)=l.

317. X= 2 is a discontinuity of the second kind. 318. x=-1 is a removable discontinuity. 319. x=-2 is a discontinuity of the second kind; x=2 is a removable
discontinuity 320. x=o is a discontinuity of the first kind. 321. a) x==O is
a discontinuity of the second kind; b) x=O is a removable discontinuity. 322. x=O
i~ a relnovable discontinuity, x= kn; (k= 1, 2, ... ) are infinite discontinuities
323.

x= 2n;k ~

(k = 0,

1, 2, ... )

are

infinite

discontinuities.

324. x= kn (k='(), 1, 2, ... ) are infinite discontinuities. 325. x=O is a


discontinuity of the first kind. 326. x= - I is a removable discontinuity;
%= 1 is a point of dislontiiluity of the first kind. 327. x=-l is a discon-

Answers

401

tinuity of the second kind. 328. x= 0 is a removable discontinuity. 329. x= 1


is a discontinuity of the first kind. 330. x = 3 is a discontinuity of the first
kind. 332. x= 1 is a discontinuity of the first kind. 333. The function is
continuous. 334. a) x=O is a discontinuity of the first kind; b) the function
is continuous; c) x= k'Jt (k is integral) are discontinuities of the first kind.
335. a) x=/tt (It is integral) are discontinuities of the first kind; b) x=k
(k ~ 0 is integral) are points of discontinuity of the first kind. 337. No, since
the function y = E (x) is discontinuous at x= 1. 338. 1.53. 339. Hint. Show
that when Xo is sufficiently large, we have P (-x o) P (xo) < o.

Chapter II

Va+h-Va.

341. a) 3; b) 0.21; c) 2h+h 2 342. a) 0.1; b) -3; c)


344. a) 624; 1560; b) 0.01; 100; c) -1; 0.000011. 345. a) a~x; b) 3x 2 Ax
a.
3 2 3
(2.
2x L\x + (~X)I .
2x + L\x .
3 (A 2

+ (L\x)

x ox)

x+L\xd)

f) In
348.

X~i\X;

Yx;

x L\x+ L\x),

1
_ ;
Yx+L\x+Yx

c)

Xl

(x+ L\X)2'

x2(x+ AX)2 '

2x(2~~

2x(2\x_ 1);

-1);

L\x

e)

~X). 346. a) -1; b) 0.1; c) -h; O. 347. 21.


15 cm/sec. 349. 7.5. 350. f (x + L\x) - f (x) . 351. f' (x) = Jim f (x + L\x) - f (x)
Lin (1+

\x~o

L\x

L\x

352. a) L\q>; b) dq> = lim L\q>, where q> Is the angle of turn at time t.
l\t

dt

\t-+o L\t

353. a) L\T ; b) dT = lim


L\t
dt
6/-'+0

~ T, where T is the
~t

temperature

at

time

354. dQ ;;:::: lim ~Q, \vhere Q is the quantity of substance at


dt

355.

a)

c)

\t -'0 L\t

~m;

b) litn L\m

L\x

6x-+oL\x

~l =:::: -0.249;

356.

a)

_.-!.-::::::: -

y~=! = - 0.25.

L\x

X Urn
1
~x ....o cos x cos (x

d)

--=..!- .
sin 2 x

= lin1

AX~o

+ ~x)

359.

Solution.

sin ~
= lim sin L\x X
\x -+0 L\x cos x cos (x ~x) \x-+o L\x
_1_ = sec 2 x. 358. a) 3x!; b) _
c) _1_;
cos 2 X
x'
2 t"

Solution.

.!.;

l' (8) = lim f (8 -t- L\x) -f (8)


~x

I1x-.o

12

.
V 8-l-~x-V8 == 11m
L\x

time t.

-2~1 ~-O 238;

sec' x.

357.

y' = lim tan (x + ~x) - tan x = lim


~x -+0

0.16; b)

t.

Ax-+o ~x

8+~x-8

lV (8 + ~X)2+ V

_
(8 + L\x) 8+ VB!]

= lim
1
= .!.. . 360.
f' (0) = - 8, f' (1) ==: 0,
~x-+o V(8+ L\x)I+2 V8+L\x +4 12
I' (2)=0. 361. x,=O, x2 =3. 2Hint. For the given function the equation
I' (x) = f (x) has the form 3x = Xl. 362. 30m/sec. 363. 1, 2. 364. - 1.
-1
365. f' (xo) = - ! . 366. -1, 2, tan q> .= 3. Hint. Use the results of Exa Inp Ie 3
Xo
and Problem 365. 367. Solution. a)

I' (O) ==

linl

~x-+o

V (L\X)2
Ax

1
11111 J

_.:::

~x.-+0V Ax

OO~

402

Answers

b)

V-1+&x-l= lim

/,(1):: 11m

&x

Ax-+o

V (Ax)e

cos(2k+ 1 n+~x)l
.
2
= lim I sin &x

lim
Ax-+-o

Ax-.+-o

Ax~-o

&x

&x

=00;

lim (sin&xl =1. 36k. 5x -12x!+2. 369.

371 -15x!
- . 372 rna t m -

a
1

375. 2x

385.
389

+ 1 n) =
2

370. 2ax+b.

373 .. 6ax
n
.374
. -2"
rr Q2+b 2
x
3
8 ~
2
8
4b
-fix l -3x-. 376. 3"x l . Hint. y=XIX I =x l 377. 3x1
x1

+ b (m+n ) tTn+n-1 .

-2x 2 -6x+ 25
1-4x
380.
(x 2 -5x+ 5)2
Xl (2x-l)2
1
4-2
y-)I . 382. 5 cos x-3 sin x. 383. - 2- . 384.
z 1l
sin 2x
(sin x-cos X)2

2a
- 3x
----:v-.
V Xi

38t.

+ 1 n)=

(2k

-.!.-+2x-2x l

&x

f'-

i' + (2k

I =_ I;

Ax-+-+

c)

be-ad
(e+dx)!

378.

379.

Y (

tlsin t.

386.

xarc t anx.

387.

y'=O.

390 .

cotx-+.
S111

8 x (7)
xe
x-I- . 391.

388.

arcsinx+

I-xl
5

x 392 ex -xa.
x-2 393 . 5x"-x
xe.
-e-x- .

394. eX (cos x-sin x). 395. xle X. 396. eX (arc sin x+ y_l__) .397. x (2Inx-l) .
1 -x!
In 2 x
2
2
In x 2
21n x l .
398. 3x Inx. 399. -+-2-2. 400. -110--.401. slnhx+xcoshx.
x
x
x
xn
x
2xcoshx-x2 sinhx
- t h2
404 -3(xlnx+sinhxcoshx)
402.
cosh 2 x
403.
an x . .
x In 2 x-sinh! x
.
-2x

405.
407.
411.

x-yX2="farccoshx
2
2 -1

x Yx

12ab

+ 18b y.

V (a +

412.

2
bX I )2

'1r 1arc.
sin I1 x + .. r 1

406.

1-~

bx

415.

416.

rl-~

fl+~

. x.
arc sin

1+2xarctanhx
410. ~(ax+b)2
(l_x2)2
e
c
x 2 -1
-x
2
16x (3 + 2X )3. 413.
- - . 414. V
(2x-l)8
l-x 2

408.

Vr Va2 -1.
x2

418.

l-tan 2 x+tanol x
cos2 x

- ~.
. 420.2-15 cosl x sin x. 42t. -16cos ~ . Hint. x=sin-It +
2 sin 2 x cot x
sin a 2t
+cos-2t. 422.
sin x
423. sin1x
424.
;COsx+2sinx
.
(1-3cosx)a
cos4 x
2 15sin x-l0 cosx
2 cos x + 3 si n x
1
425.
3
S1l1 X
co~ X
426.
2 V I-Xl 'V I + arc sin x
1
3 (arc sin x)!
- 1
427.
Y
428.
2 (1 -t- x 2) arc tan x
y I-x!
(1 + x 2) (arc tan X)2

419.

Answers
429.

eX +xex 1
x +x
2

Vxe

2e V
3
(2e

430.

a
X cos(x'-5x+ 1 ) - - - X2 COS 2

435.

..r
f

_2COS

1
(2x)'
1-(2x)2

2x

5 In4 x .

+ 1)2 + -x- .

-a sin (ax + ~).

-1

sin 2

= ..r 2
f 1 -4x 2

x cos 2x2 sin 3xl

437.

. 439.

- 2
..r
440.
~:'-l

X ,

+x

-ex

sin 2t -t- 2f t cos 2t In 2. 447.

..r
f

453.

-2x
l-x 2

21n x

451.

(1 + 1n

1
x In x

-I- sin! 5x 2 cos ; ( - sin


4x+3
(x- 2)8

469.

475.

441. -- -12 .
1+x

2xl0 2x (1 +x In 10).

cot x log e.

449.

+ S cos x)

1-x 2-4
-_--.J
(eX +5 sin x-4arc sin x) Y l-x 2
1 )

+ 2 (f. r-x + X )

..r--

2x fIn x + I

3 sin!5x cos 5x 5 cos! ~

x2

+x

462.

x-I

V 2t

x2

(1 +

VYx X)3 463.

)'

x5

2x -t- 1

V--a-2
(1 +x)

465.

2ubrnnx"- 1 (a 1- bA 'l )m-l


467.
(a_bx 'l )Tn+J
3x 2 + 2 (a -1- b + c) x + ab + be + ac
2 Y(x+a) (x+b) (x+c)

sin 4 x cos 4 x .

x-x 2

x
.459.
(1_x 2 )5

(x- 3)5

Vat

(eX

--:t-r====:::::l:::;::====::::::::.:::=:: .
V(x-l)3 (x --1- 2)5

471. 2 (7t .-!- 4)

..r--1

-2x 1- 7

+ sin! 5x (cos! ~ )' =

x t4x-6 . 458.

466.

Solution.

438.

i ){ = 15 sin! 5x cos 5x cos! i - f sin! 5x cos i sin i .

460. - : ;2r - - . 461. .. r


f (a -I ~ 2)3
f (1

464.

445.

448.

454.

457.

452.

+
x) X (1 + x 2 ) arc tan x

455. Solution. y' = (sin! 5x)' cos ;

456.

1- e2X

----X

+ q.

x2

450.

sin (2t

434.

.:..

442. 1-1 f . 443. _10xe- 444. _2tS- X2 In 5.


446.

(2x-5) X

432.

436.

sin' x

2x In 2
X

433.

403

a-3x
2 Ya-x

468.

(x+ 2)'

1+2

470.

Yy

6Yy V(y+ y)2


+ 2. 472. ..r y-a
.473 ' ..r X1
474
S1na X COS 2 X.
f t2ay-y2)'
f e +1
477. x cos Xl. 478.
3t 2 sin 2t'.
476. 10 tan 5x sec! Sx.

4
cos 2x
_(U-~) sin 2x
. 483. O.
479. 3 cos x cos 2x. 480. tan x. 481. - - 482. ~.. rr======:::;;:===
2
2

484.

sin'" x
1 arc sin x (2 Jrc cos x-arc sin x)
.. r
485.
2
, I-xi

yf=X2 .

487. xarccosx(1-x 2 ) /2
49t.

-x.
2x - X l

488.

Ya -

492. arc sin

bx l

yx.

x + ~ cos

2 f a sin

2
..r
x , 2x 2 -1

486.

.489." (" ~ .490. 2


V a x

493.

.r
,

- -l
1 +x

a -x.
2

1-2Sxl arc sin 5x

Answers

404

494.

1
Y l-ln

497. 4x ' /6 x

~na

491.

498.

1
5+4sillx

500.

sin 2xe,fn x

1-2xcosa+xl

498. I ~nl ~

V
1

499.

Y eax.

-x
cos x
2
501. 2m p (2ma mx + b)p-Iamx In a. 502. efJt (a cos pt-p sin Pt). 503. ea.x sin px.
504. e- x COSI3x. 505. xn-1a- XI (n-2x 2 In a). 506. 507.

3cot -In 3
(
\).
x sin
I

+ YCOs"'X In a).
y810.
I+
x

y tan x (I

Y a 1+

2ax+b
. 509.
ax2 + bx + c

508.

V'

Xl

-2
I
x-I
2x+11
Hint.
- - . 513. - -2 tan - - . 514. 2
x Ina x
x
x
x -x-2
2
3x - 16x + 19
I
y=5In(x-2)-3In(x+n.515. (x-I}(x-2)(x-3)' 516. sln'xcosx'
. r2- -2
-6x
15a In 2 (ax + b)
517.
r x - a
518.
(3 _ 2xl ) 1n (3 _ 2x ) '
519.
ax + b

511.

520.
524.

I
2ax +x2

+a

yf:t:Xi.
X

527.

525. x + 1 . 526.
xl - I

3 cos bx

x (1

+ In 2 x)

53t.

+ ~ sinh ~x)~

539.

t 2)

[ 2arc sin IX In 2

3
1-9x 2

523.

sin' x

+ 2 (1- arc cos 3x)}.

I+2' x .
SUt

x!
x +x2 -2

. 532.

(I _

Y2 sin In x.

522.

a cos ax cos bX; b sin ax sin bx. 528.


cos bx
530.
Y
1
+ In x -I1- x 2 arc sin x
x x

1
1-1 n 2 x

2
x 2 - 3x
5 3 3 . . .F
534. - cosx r sin x
xoi_1
1R
537 6'
sIn h2 2xcos h 2x. 538. eax (acoslpx+

x{I+ln x)
536. arc sin 3
x /.

1
-1+,.
x

'

mx + n
x 2 -a 2

521.

(3:~: ~:-ln + slnllax )

529.

535

. 512.

/2

6 tanh 2 2x (I-tanh! 2x)

540. 2 coth 2x. 541.

--,.-~'~-.
l' a +- Xl
ol

1
-1
2
542... r
543. - - . 544. . 545.
546. x arc tanh x
2
x r 1n x-I
cos2x
sin x
l-x 2
547. x arc sinh x. 548. a) y' = I when x> 0; y' = - I when x < 0; y' (0) does
t exist; b)
, (2 I
49
'
I
550 f'() { - I when
no
y -= x. 5 Y
x =
_e- x whenx ~
x >0. O.

552.

ya
"2+3'
I

=x .

553. 611.554.

,
-2,
+ (0) =
c) f _ (0) = I,

a;

a)

" ,
f- {O)=-I, f+ (0)=1; b) f- (0)

f + (0) =

0; d)

"

f _ (0) = f + (0) = 0, e) f _ (0)

=(i ,

and

x-3
f + (0) do not exist. 555. I-x. 556. 2+ -4-' 557. -I. 558.0 561. Solution. We have y'=e- x (I-x). Since

e-x=JL.
x

it follows that

or xy' =- y (I - x) 566. (I + 2x) (I + 3x) + 2 (1 + x) (I


567.
_ (x + 2) (5.\2 + 19x + 20)
568.
(x + I)" (x + 3)5

y,=.JL (I-x)
x

+ 3x) + 3 (x + 1) (1 + 2x).
x -4x + 2
Y x(x-I) (x-2)'
2

Answers
3x2 +5
3(x' +I)

569.
571.

574.

3 (x-I)

Vx

577.

V---xr

(x-2)9(x 2 "':"7x+l)

xl+I'

(x

+ 2)

(x-I)(x-2)(x-3)V(x-I)I(X-3)c'

570.

I~XI + X-.~4
2

(x

+ 3)

If 572. xX (l + In x), 573. xx' +1(1 + 2 In x).


2

Y'X-2..
1 ~n x ,575. x
I ( 1+

rio X

(Sl~ x + cos x Inx).

In x ). 576. xXX xX( ~ + In x+ In'x ).


(cos x)&tn x (cos x In cos x - sin x tan x).

578.

(1+~r[ln(I+;)+I~X]'

579.

x[tnaretanx+(l+xltaretanx]'
b)

40&

,
xy

2
'
2-cos x : c) xy

10

= ---:f.
1

t (2 - t I)
585.

1-2tl

'

586. 3

Vi'

580.

581.
3 2

a)

(arctanx)X

x~=3(I~X');

t -1
-2t
582. 2" t. 583. t + 1 . 584. 1_/2.

+ 5e!

t +1

587. t (t l + I)

588.

tan t, 589.

t < 0, 93
590. - -b t an t . 591. - tan 3t. 592. Yx' = ~ -11 when
a
w hen t > 0 . 5
594. tan /. 596. 1. 597.

2x

-2e at

599. No. 600. Yes, since the equality is an Iden-

00.

yY

x2

x(3x+2y)
-.
x2+2y .605. x
607
2y 2
_
l_y3
608
10
606.
x 3(x!-y2)+2xy-l+3 xy l+4 y'
10-3~(jsy
y cos! Y
y 1- x2- y2
2
'
609. -1. 610. 1 -x cos2
612. (x+y). 613. y =
y 611. -x I + x2+~
Y
I
I
~
x+y
x
616.
=eY-I=x+y-I'
614.
f+e
615.
XYy
x-y
2

tity. 601. -5 602.

_Vu

--2- 603. -"2. 604 . y

ay

617. cy + x V~
cx-YX2 +y2

. 618.

x In y -y .!L.. 620. a) 0; b) -.!..; c) O.


ylnx-xx
2

2
arc tan 2 ~ 63 26'. 623. 45. 624. arc tan -

622. 45;

36 21'. 625. (0, 20); (1, 15);

(-2, -12).626. (1, -3).627. y=x2 -x+ 1. 628.

-1 629. ( "8'
1 -16
I)
k=lT

631. y-5=0; x+2=0. 632. x-I =0; y=O. 633. a) y=2x; Y=

-2- x;

b)
x-2y-l=0; 2x+y-2=0; c) 6x+2y-n=0; 2x-6y+3:rt=-=O;
d) y=-=x-I; y=l-x; e) 2x+y-3=0; x-2y+l=0 for the point (I, I);
2x-y+3=0; x+2y-l=O for the point (-1, 1). 634. 7x-IOy+6=O,
n2
IOx+7y-34=0. 635. y=O; (n+4)x+(n-4)y--4-=0. 636. 5x+6y-

Y"2

6x-5y+21 =0. 637. x+y-2=0. 638. At the point (1,0):


I-x at the point (2,0): y= -x+2; y=x-2; at t he pOln
. t
y=2x-2; y=-2-;
-13=0,

(3, 0):

y=2x-6;

3-x
y= -2-.

639.

14x-13y+ 12=0;

13x+ 14y-41 =0..-

Answers

406

640. Hint. The equation of the tangent is 22..

Xo

+ 2YYo = 1.

Hence, the tangent

crosses the x-axis at the point A (2x o' 0) and the y-axis at B (0, 2yo)' Finding
the midpoint of AB, we get the point (x o' Yo)' 643. 40 36'. 644. The parabolas are tangent at the point (0, 0) and intersect at an angle

~ ~88' at

arctan
648.

1~2'

point

(1, I).

647.

8 t =8,,=2; t=n=2Y2.

T=2asin~tan~; N=2asin~; 8t=2asin2~tan~:

652.

Sn=asint.

the

653.

arctanT'

31

2+2cp.

654.

655.

St==431 2a;

Sn=a;

YI +431 n=a Yl +431 tan ~l=2n. 656. St=a; Sn= (Po


-; ;
t=Va +Q:; n=~Va2+Q~; tanll=-q>o' 657. 3ctn/sec: 0; -9cln/sec
658. 15 em/sec. 659. - %- mjsec. 660. The equation of the trajectory isy=x tan u2

t=2na

- - - x2
2v:cos 2 a

The

range

is

v :sin 2a

The

1/
V

velocity,
V o sin

a-gt

v:-2v ogtsina+g 2fl; the slope of the velocity vector is


vocosa
Hint. To detennine the trajectory, elinlinate the parailleter t from the given
systcnl. The range is the abscissa of the point A (Fig. 17). The projections
of velocity on the axes are

V (~~r + ('~r ;

trajectory

66l.

~.

and

The magm tude of the velocity is

the velocity vector is directed along the tangent to the

Diminishes

with

the

velocity

0.4

62.

({, {) .

663. The diagonal Increases at a rate of -- 3.8 cln/sec, the area, at a rate
of 40 cm 2 /sec 664. The surface area increases at a rate of 0 211 In 2 jsec,

the volume, at a rate 01 0.0511: mlJsec. 665. ~ em/sec 666. The mass of the rod
is 360 g, the density at M is 5x gjcrn, the density at A is 0, the density
at B is 60 gJcIn. 667. 56x6 + 210x4 668. ex2 (4xl -1- 2). 669. 2 cos 2x
2
670.
2(1-x )
671.
-x
672.
2 arc tan x+~ .
3 (I +X2)1
Y(a l +x2 )3
I -j-x2
2
2x
arc
slnx
674
1
t x
679
y'"
-_
6.
8
f'"
(3
673. --+
'i/"
- COSal - . ,
6 O.
) ==4 3'20
1 - x 2 ( I - x2 ) 2
a
a
681.

YV=(X~41)5'

682. yVI= -Msil12x

684.0; 1;2; 2. 685. The velocity

is v=5; 4 997; 4.7. The acceleration, a=O; -0.006; -0.6. 686. The law
of motion of the point M 1 is x =a cos wt; the velocity at time t is
-aw sin wt; the acceleration at time t is -aU>2 cos rot. Initial velocity, 0;
initial acceleration: -am2 ; velocity when x=o is -aro; acceleration when
x=o is O. The maximum absolute2 value of velocity
is au>; the InaximUln
absolute value of acceleration is aw . 687. y(n) =nlan. 688. a) n! (l_x)-(tz+l),
1 L
b) (_I)II+1 .: . ~~~3). 689. a) sin (x+ n ~ ; b) 2" cos (2X +n ~ ) ;
2nx

Answers
(_3)n -'x.

c)

(_l)n-l (n-I)!.

d)

(-I)"+I.n !.
(I +X)n-t 1 ,

(1 +x)n'

407
2n!

f)

(l-x)'Z+I'

690
)
x
x
2 n - I51' n ['2x +( n -l)~J.
2 ' h) (-l)n-J(n-l)!a"
(ax b)"
.
a x e
ne .\
1
b) 2"-'e- zx 2 (_I)" x Z 2n (-1)"-'x+ n (n 2- ) (-I )"-Z ] : c) (I-Xl) X

g)

cos

(X + n;) -2nx cos ( x+

f
or

(-l)n-l.l.2... (2n-3) ('-(2 -I)]'


) (-I)n6 (n-4)!
2n + I
X
n
,e
_
xn _ I

d)

2n x
691

I. t . t:
a cos sin

X (I

+3t

b)

).,

4
n~

(O):::-(n-I)!

y(h)

b) 3

699

(n-/) n) -n (n-I) cos ( x+ (n -;2) n) ;

c)

692.

-VI-t

a) 9t'; b) 2t 2 +2; c)

693. a)

-I t ; d) t-: It' 694. a) 0; b) 2ea t. 695. a) (I


4a sin4 2"
a sin

t l+t
(l _t)I

696

-2e-

697

(cos t + sin t)1

-:-~t;

aSln

+t I) X

(d!Y)
1
dx 2 t=o =

2t
700 4e (2sin t - cos t)
701
_ 6 at (I + 3t t 2) 702
"t'"
sint
(sint+cost)s

e . . m
d1x _ -1" (x). d1x 3 [f" (X)]2_f' (x)f"'(x)
p2
b4
dy2 - [I' (X)I ' d yl =
[I' (X)]5
705. - it 706. - a 2 y s
2y! + 2
d 2Y
y
d2x l i i I
1
-----ys-.708. dx2-=(I-y)l; dy 2=Y2 709. 256 710.
16.

2 cot t

703.
707.

3~ b) -

71 t. a)

3a 2 x

yS; . 712. L\y = 0.009001 ~ dy -== 0.009. 713. d (I-Xl) = I when

1
and L\x=-a.

x=1

No.

718.

719.
1[

a dx
--z-+
x
a

725.

2.

dY=-f2~

-0.0436.

dY=2700~0.00037.

720.

dx
dx
.)2
724. ..r--=
-x
r a 2 -XZ
-x 2
-2dx
I + cos cp
726. - 2xe
dx. 727. In x dx. 728. -1- 2 . 729. . 2
d<p.
-x
sin q>

dY=-45 ~ 0.0698.

721.

717. For x=O.

714. dS=2x!:J.x, L\S=2xL\X+(L\X)2.

-mdx

722. ---;;;+T
x

(I

723.

l'tdt
IOx+8y
-1+e2t. 732.- 7x+5y dx. 733.

730.

-Ldx. 734 ~+Ydx.


x-y
x-y

-ye Ydx
x

y2- xe

12

rrdx.737. a) 0.485; b) 0.965; c) 1.2; d) -0.045; e)

735.

4+0.025~O

81.

738. 565 cnl'. 739. Y5~2.25; Y17:::::::4.13; Y70~8.38; Y640==:::253.


740. VlO:::::=2.16; V70~4.13; V200:::::::5.85. 741. a) 5; b) 1.1; c) 0.93~
- (dX)2
-x (dX)2
d) 0.9. 742. 1.0019. 743. 0.57. 744. 2.03. 748.
a/ 749.
'{ .
(I-xl) :I
(l-x 2 ) 2
d 2
Inx+-x---,xr
2 cos x sin X) (x).
750.
-Slnx
75t. 2lnxX-3 (dx)!. 752. _ e-xx

X (x -

ox +
'"

)(

6 d)1
x .753
.

384
(2 _ (dX)4
x)s 754.

3 2n sin (2x + 5

+ 2nn)

(dx) 11 ..

Answers

408

755. excosa sin (x sin a

+na) (dx)n.

does not exist.

(2)

is a discontinuity of the function. 762. 6=0.


14
n
I
765. a) S=g; b) ;=4.
768.1nx=(x-I)-"2(x-I)2+

758. No. The point x=


763. (2,

f'

No, since

757.

4).

-1)1 ,w here
+ 2 (x3!;'

t:

~=

+e(X -I),

e <.1

0<

769
Xl
SlOX=X-3f

+ 4f
x sin
.

'='1'

x
x
1vhere ~1=eJx, 0<8.<1; slnx=x-3f+51 -6fsin~2' where ~2=e2X,
x 2 Xl
xn - t
x n :.
()<O,,<1. 770. eX==I-1-x+2f+3f+ ... +(n_I)I+ii1e", where ;=ex,
.

o < e < 1.

772. Error: a) -16

0<6 < I.

Xl

5/

+~)

b) -

;
2

CI-X

Xl

+~)

In both cases ~ = 8x;

8( ;
3

775.

Solution.

We

have

a-+
x)
-x= ( 1+-

~=~.

773. The error is less than

X ) I .
1-a

Expanding both factors in powers of x,

get: ( 1 + -X) I ::::: I +-I -x


a
2 a

\ve

5
81 (I

X'

(1

-1

- .

Xl

8 a2

1- -X)
a

1 x

3 x
+-8 a
2

::::: 1+-2 a

Multipl) In:,:. we will have: .. / a+x:::::: 1 + ~

a-x

2"

779. I

780. 3.

781.

786. \.

2
788. - .

789. 1.

790: O.

11:

n < 1. 793. O.
803.

-i-,

782. 5.
791. a.

795."5. 796.

12

I
I
804. - . 805. - . 806.

Then, expanding e a in

778 00

802

+ 2a
~.

get the same po 1ynomial ea ::::: I


of ~,we
a

ro\vcr~

o for

00

797. -1.

777.

1
-"3.

784. O. 785.

783. 00.
792.

+ ax + 2ax

n!

2"

>

I; a for n = 1;

799. 1.

800. ea. 801. 1.

for n

807. 1.

808. 1.

810.

Hint.

where S=~ (a-sin a) is the exact expression for the area


bh
2
3
of the seglnent (R is the radius of the corresponding circle).
Find lim
u -~

0 _

Chapter III
811. (-00, -2), increases; (-2, 00), decreases. 812. (-00, 2), decrease~;
(2, 00), increases.
813. (- 00, 00), increases. 814. ( - 00, 0) and (2, 00),
increases; (0, 2), decr~ases 815. (- 00, 2) and (2, 00), decreases. 816. (- 00, I).
increas~s; (1, 00), decreases. 817. (- 00, -2), (-2, 8) and (8, 00), decreases.
818. (0, 1), decreases; (1, (0), increases. 819. (-00, -I) and (1, 00), in

creases; (-1, 1), decreases


.crea!>es; (

~,

820. (- co, co), increases

821. (0, -}-).

(0). increases. 822. (-2, 0), increases. 823. ( -

00 ,2),

de-

decreases;

Answers
(2. 00). increases.

409
.

824. (-00. a) and (a. 00), decreases: 825. (-00,0) and


827. Ymax =

(0, I), decreases; (I, <Xl), increases

x=~.

when

828. No

extremum. 830. Ymin =0 when ~==O; Ymln=O when x = 12; Ymax = 1296 when x = 6.
83t.Ymin~-O.76 when x~0.23; Ymax=O when X:=I;Ymin~-0.05when
x:::=::: 1.43. No extremum when x = 2. 832. No extremum. 833. Ymax =-2
9

when x = 0; Ymin = 2 when x = 2


=-3

lrr 3 when

when x=O

834. Ymax = 16 when x = 3.2. 835. Ymax =


lr-

837. Ymax=-ya \vhen x=-2Y3; Ymin=

838. Ymtn-=O when x= I; Ymax=1 when x==O

i) n;

x-= ( k-

,r-

x= - Va; Ymin=3, 3 when x=}ra 836. Ymax= r 2

Ymax=~ V3"

840. Ym3x-=5 when x=12 kn;

x=

when

(k +{ n)

Ymax=5coS~ when

=-5cos';whenx=12(k})n;

Ymln=1

va when x==2V[
r-

839. Ymin=-2

3 \vhen

(k=O, I, 2, .. ).

X=12(k{)n; Ymin=
x=6(2k+l)n

when

(k-=-=O,

1
1
... ).841. Ymin=O when x=O. 842. Ymin =
when x= e .
4
1
when x=l
843. Ymax == e2 when x = '2
844. Ymin == 1 wh~n
e ; Ymin = 0
4
1
x=O 845. Ymin = - - when x=-l. 846. Ymln=O when x-=O; !lm.lx ~e2
e
when x=2 847. Ymin=e when x= 1. 848. No extremunl. 849. Srl1al1esl
1
value is m=- -} for x=-I; greatest value. M =."2 when x == 1. 850. nz = 0

-e

1. 2,

when x=O nnd x= 10; M =5 for x=5.


M==l

for

krt

x=2 (k=-=-O. 1.

2... ).

851.

m=2"1

when x=(2k

n
+ I) T;

852. m-==O when x=l; A1=J1 \vhrn

853. nl=-l \vhen x=-I; M=27 when x=3. 854. a) fn----6


when x == I; M ~ 236 when x = 5; b) nt = -1579 when x = -10; M = 3745 \vhen

x=-I.

x=12.

856. p=-2. q::..::.4.

861. Each of the terms must be equal to a


2

f.

863. Isosceles. 864. The

side adjoining the wall must be twice the other side

865. The side of tllt:

862. The rectangle must be a square with side

cut-out square must be equal to


base.

867. That whose

altitude

868. Altitude of the cylinder,

~~;

866. The altitude must be half the

is equal to the diameter of the base


radius of its base

vi,

where

1<

869. Altitude of the cylinder. R Y2"


R is the radius of the given sphere. 870. Altitude of the cone, ~ F

is the radius of the given sphere.


where

i-.

Answers

410

where R is the radius of the given sphere.


where R is the radius of the given sphere.

{r,

cone

871. Altitude of the cone,

~ R,

872. Radius of the base of the

where r is the radius of the base of the given cylinder. 873. That

whose altitude is twice the dianleter of the sphere. 874. cp = 1, that is, the crosssection of the channel is a semicircle. 875. The central angle of the sector
is 2n

Vf.

876. The altitude of the cylindrical part must be zero; that

h (I: -d: ):.

is, the vessel should be in the shape of a hemisphere. 877. =


X
878. 2 +2Y = 1. 879. The sides of the rectangle are a V2' and b y~ where
Xo
Yo
a and b are the respective selniaxes of the elli pse.

880. The coordinates of

the vertices of the rectangle which lie on the parabola (; a; 2


88t.

( ~3'
1

.arc cos -k
885.

a)

and

~)-

y~).

882. The angle is equal to the greatest of the numbers


h

arc tan -d .

d
x=y= Vi";

b) X=

883.

.4M

=aV

V3;

y=d

Vii
p

+ Vi

/2
V 3"'

-.

884.

886. X=

r
-

Y2

-. /2aQ
V-q-

V2a'lQ. 887. VMm. Hint. For a completely elastic impact of two


spheres, the velocity imparted to the stationary sphere of (nass m l after
inlpact with a sphere of mass m 2 moving with velocity [) is equal to

Pmin =

2,+n1 v

nZI

n1 2

. 888. n = ... / N R (If this number is not an integer or is not a divbor of


r

N,\ve take the closest integer which is a divisor of N). Since the internal resistance

n;,
2

of the battery

is

the physical meaning of the solution obtained is as

follows: the internal resistance of the battery must be as close as possi ble to the
2
external resistance. 889. Y="3 h. 891. ( - 00, 2), concave do\vn; (2, 00),
concave up; M (2, 12), point of inflection. 892. ( - 00, 00), concave up.
893. (- 00. -3), concave down, (-3, 00), concave up; no points of inflection.
894. l - 00, -6) and (0, 6), concave up; (-6, 0) and (6, 00), concave down;

(-6, -{), 0 (0, 0), (6, {). 895. (- V3') and (0, v3"), concave up; (- va. 0) and (va, 00), concave down;
floints of inflection M
v3: 0) and 0 (0, 0). 896. (4k + I) i '
(4k+3) i), concave up; (4k +3) i, (4k+5) i) ,concave down (k=O,

points of inflection M1

Ml

I, I

00,

i,

I, 2, ... ); points of inflection, (2ft + I)


0)' 897. (2kn, (2k + I) n),
concave up; 2k-l):rt, 2k1t).concave down (k=O, 1, 2, ... ); the abscis-

~as

of the points of inflection are equal to x= kn.

898.

(0, V"ii). concave

41 ]

Answers
,iown; (

~ea'

00),

:e.)

(~ ii' -

concave up; M

is a' point of inflection.

899. ( - 00, 0), concave up; (0, 00), concave down; 0 (0, 0) is a point of
inflection. 900. (-00, -3) and (-I, 00), concave up; (-3, -I), concave

~) and
x == 2, y == l.

M 2 (-I, :). 901. x=2,

down; points of inflection are M1 (-3,


y == o.

902. x == I, x ~ 3, Y == 0 903.
904. Y == x. 905. y == - -",
left, y==x, ri~ht. 906. y==-I, left, y== I, right 907. x= I, y==-x, left,
y==x, right
908. y==-2, left, y-=2x-2, rIght.
909. y=-=2
910. x-::O.
y==l, left, y==O, right.
911. x=O, Y==l.
912. y~O.
913. x-=-l.
914. y=x-n, left; y=x+n, right.
915. y=a.
916. Ymax==O when x==O;
Ymin ==-4 when x==2; point of inflection, M 1 (I, -2).
917. Ymax= 1 \vhen

x=

Y3;

x=O;

when

Ylhin=O

points of

M I , . ( I,

inflection

~)

918. Ymax=4 when x=-I; YmlO=O \\'hen x=l, point of inflection, M 1 (0,2).
919. Ymax=8 when x=-2, Ymin==O \vhen x==2; point of inflection, M (0,4).
920. Ymin=-l \vhen x-:-::O; points of inflection M 1 2( Y5, 0) and

64 )

I,

- 125 . 921. Ymax ==- -2 \vhen x =-= 0; Ymin = 2 \\'hen x -:= 2; asymptotes, x ==-= 1, y =- x-I. 922. Points of inflection M I, 2 ( 1. =f 2); aSylnptote
x==O. 923. YmJx=-=-4 when x-=-l; Ymln~4 \vhcn x==l; as)lnptote. x===O.
924. Ynain = 3 when x= 1; pOint of Inflection, 1\1 (-' 2, 0);
aSyl1lptote,
M a,4 (

V-

x=O. 925. lIlhax={ when

>.=0,

points of II1flectlOn,

MI , .

(l' -{-);

asytnptote, y = 0 926. Ymax --- -2 \vhen x-=-- 0; asynlptotes, x --:: 2 and Y::= o.
927. Ymin-=-=-J when x:=.:--l; YmJx= 1 \vhenx= 1; pOlnt~ of inflection, 0(0.0)
and M I ,2

J-r 3 )

( 2Y3,

~2; aSy1l1ptote, y=:::O


M

point of inflection,
of

inflecti~l1,

\\Then x =--=

"3;

0 (0,

(5, f);

928. !lmax=1

\\'hen x--:4;

O.

929. POint

aSyl1lptotes, x = 2 and y =--=

0); aSy111ptotes, x-=- 2 and y -= O.

nSYlll ptotes,

x --= 0, x.:=: 4 and Y == 0

930. Ym.1X -== -

931.

27
T6

YmJx = -4 \vhen

x=---l; Ymin==4 \\hen x== I; aSylnptotes, x==o and y=3t 932. A (0, 2)
and R (4, 2) are end-points; Ymax=2Y"2" \vhen x=2
933. A (-8, -4) and
B (8, 4) are end-points. POint of inflection, 0 (0, 0).
934. End-polllt,

A (-3, 0); Ymin-=-2 \vhen x=-2. 935. End-points, A(-V-~ 0), 0(0,0)
and B(V~ 0); Ymax= 2\vhen x=-I; point of inflection, M(3+2Y3,

V V + ~;).
6

Ymax = 1 when

points of lI1f1ection,
M I , 2 ( 1, 0). 937. Points of inflection, M 1 (0, I) and M 2 (1, 0); asyrnptote,
y=-x. 938. Ymax=O when x=-I; Ymin=-I (when x=O)
939. YmJx~2
when x=O; points of inflection, M I 2 (1,
asymptote, y=-=O.
940. Ymin=-4 when x=-4; Ymax-=4 when x=4; point of inflection, 0(0,0);
asymptote, y==O.
941. Ymin=
4 when x=2, Ymln=
4 when x=4;
Ymax==2 when x=3.
942. Ymin=2 when x=O; aSylnptote, x= 2.
936.

x=O,

V2);

V-

V-

943. Asymptotes, x=2 and y=O.

944.

Ymin-=

Y3
Vi

when x=Y3;

Answers

412

YmdX=-

Y3
V2

M. (3,

and

when x=-3; points of inflection, M)

~);

asymptotes, X= l

of Inflection, M (12,

x=4;

l~aa)

and

M. ( -a,

of

inflection,

points

M9. Ymax=2 when

V2

~),

0(0,0)

when x=6; point

VI2100 );asy m ptote,X=2 946.Ymax=!e when x= I;

M (2, ~); asymptote, y=O.

of IIlflection,

MI (-3a,

945. Ymln=

(-3, -

point

947. Points of inflection,

); asymptote, Y=?

948. Ymax=e2 when

M I , . ( 82[2 , el ); asymptote,

~).

x=O; points of inflection, MI , . (l.

Y=O.

950. Ymax=l
2

when x=I; !/rnin=O when x=O.


951. Ymax=O.74 when x=e =::::.739:
point of inflection, M (e Mi :::::: 14.39, 0.70); asymptotes, x=O and Y =0.
a

Ymln = - 4e

952.

953. Ymin =
y",O

when

x=

when

x = e;

when

x ...... O.

a...
yi"'
pOInt of Inflection,

954.

Yrnax="T=::::'O.54

Ymin=O when x=O: point of inflection,

(e 2 , e~);

point of inflection, M
4

3a

a
ye;'
-

4e2

asym ptote,

x=

1;

\vhen

x=s--1:::::::-O.86;

(+-1~-0.63; ~~0.37):

Y -; 0 as x ...... -1 0 (limiting end-point). 955. Yrnln = 1 when x = y-2'; points


of inflection, M I , 2 ( 1.89, 1.33); asymptotes, x=- I.
956. Asymptote,
y=O. 957. Asymptotes, y=O (when x -+ (0) and y=- x (as x ~ - (0).

958.

Asymptotes, x=_!, x=O, Y= I; the function is not defined on the


e

I- ~ , 0]. 959. Periodic function with


k
. rn
\vhen x=4 n+2kn; Ymax= Y 2 when x=4+ 2kn
interval

points of inflection,
perIod 211:.
n

X=

Mk (

Yrnln = -

3+ 2kn (k=O,

N k (arc cos ( - {- )

43

~ n + 1m,

.rY

when

I, 2, ... );

+ 2kn, ~ YIS).

0).

960.

period 2n. Ymin = -

Y2

(k=O,

2,

... );

function

with

3 1r '4
Y 3

when

Periodic

5
x ="'3 n + 2kn;
points of

Yrnax =

inflection,

Mk(kn,

0)

and

961. Periodic function with period 2n.

the Interval [-no nl, Ymax=~ whenx=::


x= n; Ymln =0 when x=O; points of inflection, M 1

'On

M a, 4 ( 2 ~O, -0.95).

I,

Ymln=-2

when

(0.57, 0.13) and


962. Odd periodic function with' period 21t. On inter-

-val [0. 2nJ, Ym3ll= 1 when

x=O: Ymln=0.71, when x=i: Ymax=l when

Answers
x

=2;

x =

413

Ymln=-I when x=1t; Ymax=-O.71 when x=4"1t;Ymin=-1 when

n;

x=2n;

Ymax= 1 when

M 2 (1.21, 0.86);

M. (5.50, 0).

M a (2.36,

points

of

M 4 (3.51,

0);

VI

x=4+2k~; Ymax=--2-whenx=-"41t+2kn

~ n + kn

965.

Even

Ymax =

Ymln =

(k=O,

I,

per iodie function

x=arcCOs ( -

~3> Ymln=O

~2, 4~7);

M. (arcsin

~2

when

2,

... );

with

3
I, 2, ... );asymptotes,x=4n+k~.
period 2n

On

the

interval

4
I
,r- when x = arc cos ,r- ; Ymax = 0 when x::.:: ~; Ymin = -

3r 3

0.86);
-0.86);

964. Periodic function with period n; points of

~
V2) (k=O,
-2( 4+k~,

inflection, M k

M s (4.35,

Periodic function with period 2'1:.

963.

asymptotes, x=

M 1 (0.36,

inflection,
-0.86);

r 3

3r 3

when x=O; points of inflection, M 1

M.(

n-arcsin

[0,

.1t

4
.. r- when

~2, _ 4~f).

(; ,

966.

0) ;
Even

periOdIC function with period 2~. On the interval [0, ~] Ymax = I \vhen
2
x==O;
Ymax= lrwhen
x=arc cos - ,r- ; Ymin= ,rwhen
3r6
r6
3r 6

( I)

:6" ;
l

x=arc cos

Ymin = - 1 when x=n; points of inflection, M 1

Y~. ~ y~);

V~),

M.(

,0);

-{ y~}

M. (arc cos
arc cos ( 967. Odd function. Points of inflection, M k (k~, k~) lk =0, I, 2, ... ).
968. Even function. End-points, Al 2 (2 83, -1 57) Ymax == I 57 when x=O
(cusp); points of inflection, M 1 ~ ( 1.54, -0.34).
969. Odd function.
Litnlting points of graph (-I, ' - 00) and (I,
00). Point of inflection,

0(0, 0); asymptotes, x=

=~

+kn;

Mk(k~, 2k~);

I.

970. Odd function. Ymax =

~ -I + ?kn when

n+ 1 +2kn when X= ~ n+kn; points of inflection,


2k+1
asymptotes, x= - 2 - 31 (k=O, I, 2, ... ).
971. Even
Ymln=

function. Ymin=O when x=O; asymptotes, fJ=-

~ x-I (as x -i""-oo) and

y =2""x-1 (as x-++oo). 972. Ymin=O when x=O (node); asymptote, y=l.
973. Ymln = 1

+~

when x= I; Ymax= 3: -I

when

x=-I;

point of

inflection (centre of symmetry) (0, ~); asymptotes, y = x 2~ (left) and Y = x


(right).
974. Odd function. Ymln=I.285 when %=1; Ymax=I.856 when

x=-I;

point

of

inflection,

x-.-oo) and Y=2

(0,

~);

asymptotes, Y=

~ +n

(when

(as x-++oo). 975. Asymptotes, x=O and y=x-ln2.

Answers

414

976. Ymll1=1.32 when x=I; asymptote, x=O.


period

2n.

Ymln=

when x=

i- n + 2kn ;

977. Periodic function with


Ymax=e when x=

+2kn

}IS-I')

lrr5-1
(k=O, I, 2, ... ); points of inflection, M k ( ;lTCSin---+2k31, e
2

Y5-1
YS+l\
and N k ( -arcsin-2-+(2k+l)n,
978. End-points, A(O, I)
and B (I, 4.81). Point of inflection, M (0.28, 1.74). 979. Points of innection~
M (0.5, 1.59); asymptotes, y=0.21 (as x -+- 00) and y=4.81 (as x ~ -1- 00).
980. The domain of definition of the function is the set of intervals (2k31,
2k31 + 31), where k = 0, 1, 2, ... Periodic function wit h period 231.
31
Ymax=O when x=2"+ 2kn (k==O, I, 2, ... ); asymptotes, x=k31.

e')'

981.

The

( 2k +

-4- )

domain

of

is the set of intervals [( 2k - { ) n,

definition

n], where k is an integer. Periodic function with period 2n.

Points of inflection, M k (2kn, 0) (k ==0, I, 2, ... ); asynlptotes,


31
T+2k31.
982. Domain of definition, x> 0; monotonic increasing

X=

function;
(k

asymptote, x=O.

983.

I x-2kn 1< ;

Domain of definition,

== 0, I, 2, ... ). Periodic function with period 2rt


rt

x==2kn (k==O, I, 2, ... ); asymptotes, x==T+kn.

y = 1.57; Y -i""-1.57 as x

-+

(linliting

AI, .(1.3I, 157); Ymln=O when x=O.


I

x == e

0.37; y

-+

I as

-+

Ynllll = I when

984.

end-point).

985.

Asymptote,
End-points,

986. Ymln=(

f)"e ::::::0.69

987. Limiting end-point,

when

A (+0,

O)~

e e =::::: 1.44 when x == e ~ 2.72; asymptote, Y.::= I; point of inflection,


M I (0.58.0.12) and M2 t4 ~5, I 40). 988. xmin == -I \vhen t == 1 (y == 3); Ymtn = - t
when t = -I (x = 3) 989. To obtain the graph it is sufficient to vary t from to 2rt.
xmin = - a when t = 3t (y == 0); xmax ==a when t = 0 (y = 0); Ymin -:: - a (cusp) when

Ymax =

331

t=+T (x=O);
31

when 1=4'
990.

Ymax==+a (cusp) when

3Jt

xmin=--

731

(x==O); pOints of inflection

(
a
a)
x= 2y"2" , Y= V2 .
I

when t = - I (y=-e); Ymax=- when t=1 (x=e); points of


e

inflection when 1 = I.e., (

531

4' T' T

~.

t==2

Y2,

)12'
Y -2 eV -" eva);

Le., ( -

:::~,

V 2eva )

and when 1 =

V 2.

asymptotes, x=o and y=O. 991. Xmin= I and Ymln = 1

when t=O (cusp); asymptote, y=2x when t

-++OJ.

992. Ymin==O "'hen t=O.

Answers
X

a dx,
'993. ds=y
~osa=

1
=Y

Va 4 ---.c2 x2

V p2+y 2 dx;

cosa=

~VXa;

sina-==-

cosa==

bx

;sina-==-

Ya 2 - c2 x!

y ;.
sma=

sin a'-" tanh : . 998. ds = 2a sin

2 2

_C X

dx;

V'

a
-dx,

a
a -x
..r - -2
,where c=-ra"-b 995. ds=

V p2+ y 2

VI.

ya

r
d s=-

994

y.
cos a=-;
Sin a-= - - .

V aI_xi

415

p
V p2+
.

996. ds=

y"

ds=cosh~dx;

997.

cosa=_l_;
cosh ~

~;

dt; cos a =sin

sin a= cos

. 999. ds =

V +

== 3a sin t cos t dt; cos a = -cos t; sin (J,:= sin t. 1000. ds == a


1 cp2 dq>; cos ~
1
a .. r--2
1
a
=
1001.ds= 2 f 1
dcp; cos ~= 1002. ds == - - dcp;
1 -1- cp2
q>
1 <p2
cos l .!

Sill

~ = cos

~,

y 1 -1-

-t

K -= a

1010.

1012. (

1003.

(In a)2 dcp;

1006. K---=36.

V-_.
+

+q>

ds = a cos

sin

p=

1007. K-=3

:'2

_ 1n 2
2'

at

Y2 )
2

_I

. 1013. R -

+ (y -3)2 = 8.
2

+ (bl")

a ==

1024.

1026. py2 =

ds-=

1004.

6x

. 2t 1 .

(X-3)2+

~ (X -

1020.

(y- ~

and

_ (b 4x2

R == I at I.

RIc.,st =
=

~.

-3).

(:'

. 1014. R -

1017.

aSHl

/.

i ' 3)

(1 -1- 9X 4)3/2/

= I r V!+FI . 1019. R = ~ a cos ~


t023. ( _1; a, 1: a ).

~ = cos ~.

101 I.

vertices.

+ 1)21 1016. R = I 23
R =-:: (y2 4y

t015.

sin

1005. ds =-= - d<p; sin p= cos 2q>.


,
a
b
6
1008. KA=b 2 ; K B =(i2' 1009. K=13 V13'

V 1 + (In a)2

Y2

both

~ dcp;

I p I.

+a ab4y!)3/2
4 4

1018.

R=

1022.

1025.

(2,2).

(X+2)2+

.!-.
p)' (scmicubical parabola). 1027. (aX)

C 3 ,

\vhere

c2 ::::: a2 _b 2

Chapter IV
In the answers of this section the arbitrary additive constant C is olnit..
5
x.
ted for the sake of brevity. 103t. 7 a2x7 1032. 2x 3 +4x 2 +3x. 1033. T+

(a

+3b) Xl + abx
2

1l-1

abx4

1034. a x + 2
2

b2x"

-1- 7

1035.

2x '1m=::-

3 '

2px. 1036.

nx 11
n-l'

Answers

416
4

V/ -nx.

1037.

9 -

1038. alx-Sa

Vx

X I

Vx - 63V-x.

3x
3x
1040. --13- 7

..r..r1042. 2a r ax- 4ax + 4x r ax- 2xl


1044.

\x- V
VIOl.

1 In
...rf 10
x+

1047. arc

10

9 -

+7a I

4m

2x'
../"_

1045. In (x

1009.

yX 4xm + n

+1

Yx

2m + 2n + 1 +

Y7

+ .rr
4+
Xl).

arc
arc

1046.

+1
x
7".

4n

tan
sin

Yx

~-In(x+ yX1 +2). 1048*. a) tan x-x. Hint. Put tan1x=

Sin

=sec 2 x-l; b) x-tanhx. Hint. Put tanh 2 x=1

-x; b) x- coth x. 1050.

In(;e~ I'

1051. a In

cos

I I
aC x

=-a5~(a-X)=-alnla-xl+alnc==alnl-C-I.
a-x
a-x

1
h I . 1049.a)-cotxx

Solution.

Sa a x dx =

1052. x+lnI2x+ll

;;~~ =

Solution. Dividing the numerator by the denominator, we get


2
=1+2x+l'

Whence

=x+ ln I2x+ll
a

+ x.

2x 2n Yx

1043.

5 r ax

2x 5Yx
l

Xl

-3.

2x 2m

1041.

X I

1053.

5 + =5 5
2x 3
2x+l dx
3
11

-2x+41nI3+2xl.

ba-a~

2 dx
d (2x + 1)
2x+l=x+
2x+l =
x
a
1054. 7]-bllnla+bxl.

dx+

x2

1055. aX+~lnlax+~I. 1056.2"+x+21nlx- 1 1.


x
x'
4"+3"
+x 2 +2x +31n I x-II.

1057.

Xl

2+2>:+

-~ 1n

I x +31 1058.

b?

t060.~

x-a
dx

+1 1

In 1 x+ll+ x

1059. a2 x+2ab 1n! x-al-

5(x+l)2=
x dx
S(x(x+l)2
+ 1) - 1 dx=

Hint.

2 '1r (

... r -

dx

1062.
-3ij'" (a-bx).
Sx+I-J (x+l)2' 1061. -2b f l-y.
X1
1
t063. V Xl + 1. Solution. S.: dx = 2 S
+ I) = y' Xl + l. 1064. 2 yx +
.
x +1
+1

+ lnlx
-2

t065.

1
-./3
..rarc tan x JI -5'
f

15

Ir

1
Ya-b
1067.
II""
In - .. f"' a+b+x Y
2 f a"-b 2
r a+b-x
a-b
1069.
1071.

- (

~2
2

+
1

~I

In I a2 -x ll )

Y2 1n (2

1
.rIn
4 ,,14

1066.

I.

1070.

.. r.. ~
r 2x+ r 7+8x l ).

5
~r3 __
1073. 3"ln l 3x -21- 2 f6 1n " V3+

Ix

Xl

t068.

Ix YY7-2 V
Y21

x- .rr-2

x-

In (x 2

arc

7 +2

tan

x
1r-'
r 2

+4) +arctan ~

1
... is
Vsarc sin x JI 7'

1072.

Y21
Y2 1074.

Y35 arc tan

/S
V 7"-

...

Answers

i Y5x

-{-In (5xl +7). 1075.

+ 31n I x+
1079.

;5

1n (xY5+

+ I+

1
-8 In (1

1087. -

3" arC tan x'.

1081.

J!

-4 +

I
41n (2x 2 +3).

1084.

1086.

-:31~4 4 2 - ' ' ' .


(aX bX)

Yx

( arc tan

1088.

-~

+ 1).107&.

1078.

Y(arc sm x)'.

(arc tan 2(,'

+ 4x 2)

e-"'''.

ax
1
Xl
arc tan b 1080. 2" arc sin (ii".

} In I x' + Y x6 - 1 I 1083.

1085.

Y5x

21 In lxi - 5 1.

1077.

2U In (a1x' + b2 ) + a

1082.

wr-r x2 -41 .

417

-i

r-------::::----

In (x+ YI
2X

+ X2)~

1089. e1 +e- 1.l090. ; eli +2x2

3-1- Ya 3X +
n a~ n a n
a
In a W/
r aX

--2 ea. 1091. l i b bX--x -2x. 1092.

1093. - 2

1
X2
2 In 7 7.

1094.

X 2 +1.

-,~

1097. Inle"-II. 1098.


- 3 I~ 2 In (2" + 3)
t 102.

'105.

hX

..rx
y 2 ~1l) Y"2

xcos (log x)
S1 n 2(
+-4-

1109.

2a cos 2ax.

hint

-2 tan (x).

. Hint. Put

.
09
In 11

1123.

l 1125.
i t
X I sin (x + I).
n' an x I.
1
1

1131.

1129.

- 9"2 .r
r (1 +3 cosx)

cos (a + bx) .

1108.

-In lOX

1
x ="2 (I-cos 2~). 1110.

I
I
"2
cos (l-x ).

4
4
a sin ax

b1

co t ax
lIlt. -tan(ax+b). 1112. ----x.
a
a

xlnlslna~bl. 1122.5Inlsin~l
1128.

_Y2Inltanx~I.I119. -lnlcosxl.

1104.

.r-

Stll!

i-

1100.

l
1101. In a arctan (a").

3)'

1107. 2 sin r x.

~lnltan(?;+i)l.

1117.

~(ea +1)3.

arc sin et

Sitl 2~.

"2 - - 4 -

ttt3. alnltanil. 1114.


I

( 1- 2}:

1103.

1106. x-

.
S
Hint. ee

1116.

2 5 V-X
li15

1096.

1099.

Y(a-be")'

Hint. 2" ~ 3 """

1 In 111 +e2b
_e- hX

-e x

1095.

ItIS.
1118.

Inltan

x-

axtwr- I.
b

I
V-2
cot x

r 2-

1120. Inlsinxl. 1121. (a-b)X


-2InlcosVXI.1I24.
1126

asin
. I
"2

- -3 In (3 +cos 3x).
32
11..

1130.

3 t an 4 3
x
"4

a
X

~Inx

1 27 SI1)4 6x
1 ~.

I wr--

- -2

Ct S

2x.

1133 . " 3
2 .r-tr an ,-x.

1134.
t4-1900

3CO~ 'x. 1135.;

(tan3x+cos\x). 1136.

{(lnltan~I+2sll1ax)-

Answers

418
1

a .

1137. 3Q1nlb-acot3xl. 1138. 5cosh5x-5s1nh5x.1139-2+4s1nh2~.

I i I

tt40. In tanh

2 arc tan e~.

tt41.

5
tt44. In I sinh x I 1145. - 12
xarc tan

x.

Y5'

-X2

-"2 e

1148.

Y3+ (2+3x

1._ 1n (X

V (5-x

1142.

1 In I ~-4x + 1 . 1147. 4 Y5
1 X
1146. 4"

)'.

(r
11 a)
'2 -

13
V 2" arc tan

...

1149.

a' -

1150.

In 1tanh x I. tl43. In cosh x.

x +x-2In/x+1I1151. -}._.
2
,. eX
1
1
1152. Inlx+cosxl. 1153. 3 (Inlsec3x+tan3xl+~3).
1154. --1sin x
n x- .

2 ).

lnl tanx+ Ytan l x-21. 1156.

1155.

a SI n x
1158.
l"il""a.

1157.

V\X +

Y2

arc tan (x Y2">- 4 (2X:+ I)

1)2
1
1
--2- 1159. 2 arcsin (x 2 ). 1160. a-tanax-x.

I (

1161. x

sin x-, 1162.arcsln. tan-1163.alntan


x
x
n )
2a+T
2
2
2 --

-21n I cos Yx-II.

1165.
l

I.1164."43 V (1+lnx)."

I i /.

{-In tan

1166.

eare tan %+

1167.

x
+ 1n (14+ x ) +arctanx. 1168. -lnlsinx+cosxl 1169. Y21n
tan 2 (2

Y2 cos

- 2x 1172. e

sin 2X

';'2'
1173.

I-

x+ ~21n / :~ ~~ /. 1171. In 1x 1+2 arc tan x.


5 . X Y3
. r--2
ya
arcstn--+ y 4-3x. 1174. x-In(l+e X).

1170.

1175 r 1
a;ctanx,la-+bb.1176.1n(eX+Ye2X-2).1177.-.!.lnltanaxl.
f a2 -b 2
a
a

1178. -2n cos

( 2nt)

r+q>o

1181. - e- tan %. 1182.

- Y 1 -Xl.

arc sin

1185. In (sec x -I-

~2 arc tan (

1187.

1179.

1188.

tanh %
1190 -1.
33
1191. a)

x -I- I). 1186.

X)2

5-sin2x

I.

S1 +dxcos x Ssin! x +dx2 cos---- =-

Hint.

Y2

arc cos -

st

~r- In ~5 -I- 'lill 2x

4 ,. 5

j-Vlln(x-I-Y1-I-X'))"
1/;i
~

arc cos
2

1183. -2cot2x. 1184. (arc

dx

==.\ta~~:2'

(-7i) .

Y sec

t;:;) .

1 1n /2 + In x
2-1nx 1180.

x when x >

1189.

"lry

2;

i-

Jt

sinh (x'-I- 3).

Answers

c)8~(5r-3)';
1192.

(2~lt5)tt].

~2X+T-1 1.
y

1193. 2

(V;- ; +2 yX~2Inll+Vii)

1195. 2arctan Vex-I.

(arc~tlx)l.

yeGs x. 1200. In

+21 arc sinx.

II+Vx
1

1202.

}(tr-2)

1198.
/'

+1

Inx-ln2Inllnx+

1196.

x=-}.

VeX + I.

1199. j.(COS'X-5)X

Hint. Put x=-t . 1201.

x2 .. r - -2 4 .. r - -2
-'3
y2--x -:r y2-x

-a arc cos ~. 1204. arc cos 1.., if x


x
x
Put

In(sinx+Vl+sin'x).

e)

2x+I+1

+21n21. 1197.
X

~ V(x+1)1-2VX+'1;

d)

[(2Xi;5)12 - 5

1194. Inl

419

1205. J"X'i+1-1n 11

>

0, and arc cos ( -

+ ~XZ+TI.

-i Vl-x'+

1203.

'1r-x2 a2
f

-!.)
if x <0 *)
x

1206.

Hint.

V~ . Note. The

substitution x =!. may be used in place of the trigonometric substitution.


z

x .. r2- -2
+"21 arc sin x. 1208. 2 arc sin ..rr-x. 1210. "2
y x -a -+l
a
'1r-I
+2"
In 'x+ y X 2 _ a2 1. 1211. x In x-x. 1212. x arc tan x-'2ln (l-t- x2 ).
x ..r - -2

1207.

2 y

1213.

x arc sin x+

1216.

1 _x

x+1
-ex-.

YI

Xl.

1217. -

1214.

x In 2 + 1

2 xlf1i2

sin x-x cos x.


1218.

e~x

2f (9x

1215.

x Sl; 3x + cn~ 3x .

-6x + 2). Solution. In place

of repeated integration by parts we can use the following method of undetermined coefficients:

~ re1xdx=(Ar+Bx+C)e'X
or, after differentiation,

x2elX = (Ax 2 + Bx + C) 3eIX + (2Ax + 8)e1x


Cancelling out elX and equating the coefficients of identical powers of x, we get:

1 = 3A; 0 = 38 + 2A; 0 = 3C + 8,

1 8=-2 ; C=2j.
2
whence A="3;
In the general form,
Pn(x)eaxdx=-.
ax
= Q (x) e , where P n (x) is the given polynomial of degree nand Qn (x) is
a p01 ynomial of degree n with undetermined coefficients 1219. _e- JC (x 2 +5).
x

Hint.

See

Problem

1218*.

1220.

-3e

a (x'+9x 2 +54x+ 162). Hint. Se~

*) Henceforward, in similar cases we shall some'times give an answer tha'


is good for only a part of the domain of the integrand.

14*

Answers

420
Problem

12IS*.

+ 2X: 5 cos 2x

_ x c~ 2x

1221.

+ SillS2x.

1222.

2x

+ I~X+ II sin 2x+

Hlnl. It is also advisable to apply the method of undeter

mined coefficients in the form

~ Pn (x) cos px dx = Qn (x) cos px + Rn (x) sin px,


where Pn (x) is the given polynomial of degree n, and Qn (x) and Rn (x) are
pol ynomials of degree n with undetermined coefficients (see Problem 1218*).
Xl
x'
In x
1
2
1223. 3 In x- 9" .
1224.
x In x- 2x In x 2x.
1225.
-- 2x l - - 4x 2

1226. 2

V -x In x-4

..rx +I
X
x
1
r x. 1227. -2- arc tan x-"2. 1228. 2 arc sin x-"4 X
2

X arcslnx+ ~ VI-xl. 1229. x In (x+ VI +x2 ) _ VI +xl 1230. -x cot x+

+ In I sin x I
1233

-~
+ 1 It!-.I
sin x
n an 2

1231.

ax (sin x +cos x In 3)

+ (In 3)1

e-

eX (sin x--cns x)

1234 eax (a sin b"t-b cos bx)


al

+b

X2

-cos (1nx)). 1236. --2-(x1 +l). 1237. 2e


Xl

1232.

x 2 -1

Xl

1 35
2

y-

x (Vx-l).

I-x

!- [ .

2 sin

(x

(I

nx

)_

1238.
3-X2+
In 2 x 21n x
2

+3x ) Inx- g +2"-3x. 1239. -2- 1n I+x- x . 1240. --x---x--X-.


Xl
x I I I + x2
1241. [In(lnx)-l).lnx. 1242. aarctan3x-T8+162In(9x2+1).1243.-2-X

1
x(arctanx)2-xarctanx+"21n(l+x2).

X arc sin ..f r

x +2 ..rf x.

1247.

I + V I

x
-1246. -2 yrI-xx
1
I-xl
x
x tan 2 x+ In I cos 2x ,_ ~
1248
e-

arc sin x +In

.
xarcslOx-2x.1245.

.. r - x (arc sin X)2+ 2 r l-x 2 X

1244.

1)

2'

249
+xcos(2Inx)+2xSin(21nx)
1.
2
10

x l .
x dx
1250. - 2 (Xl + 1) + 2 arc fan x. Solution. Putting u = x and dv = (x 2 -t- 1)2 '

COS2X-2Sin2x
X(
5

1
x dx
x
we get du=dx and v= 2(x2 +1)' Whence (x 2+1)2
2(x 2 +1)
dx
x I I
x
2(xl+l)
2(xl+l)+2arctanx+C.
1251.
2a2 iarctan(i+

+S

+ xl~al)'
X

Hint.

..r l- f

(1

at
a -x'+2

Utilize the identity 1....

arc sin

x
tJ'

Solution. Put

;1

[(xl+al)-x l ).

1252.

1-

.. r - -

U= f

ai-xI and dv=dx; whence

andv=x;wehave Yal Xldx=XVa2-xl-S,::::~2dX=


aI-xi
Va 2 -x2
l
l
. , ygl_XI_s(al-xl>-alctx;::x Vaa-x,I-S va -x dx+ a2S
dx
.

dU=-V

XdX

II

Yal-xi

Val-x'

Answers
Consequently, 2

+~
+~
X In

I x+

In

SVa2-x2dx=x a 2_-x2+a2arcsin :

VA +x" I.

i.
I

arc sin

HInt. See

Vll

2 . 1257.

xarc tan V3 1259.

1268. In

l-x 2

2x-1
-4-

+1 I '

1277. In (
1279.

~'5

In ( x V5-

1275.

1276.

x-

1286.

(X-4)51

+ :3)7

-i

1284. 5x

+ In

(X-4)

2--V-

2
Ir,2

(x> V2).

1
3-~1flX
.. r- arc tan
y 3
J

V-

1289.

~b In

0-

11.

x +-1- b I.
x a

,X + 2)4

12

2.

1287.
8

.1285I~X+ln IX~II.

.,,-ml

arc tan x.

W.

161

~x+~lnl(2X_I/~~X+1)81
I

I.

..!.-lnl(X-l)\~+3)~1

1282

(x-I)

___9__

px+q

V5x -2x+ I)

1280.

, 5

2(x-a) 2(x+I)O
49(x-5)
1
1290. -2(X 2 _ 3X+2)2' 1291. %+Inl
1288.

V Xl +

1278. -Inl cosx + 2+ Vcos l x+ 4CI,SX +

V'1-4 In x-Inl x- 2 arc sin 2 -:;~ x .

\x

2x+ 1 .r
- 4 - y 2-x-x 2 +

1274.

IIX - 3 1
-4 In - l - I .

+{ + VI +eX +e2 X

(x-l)4

x+l.r
y x~+2x+5+2In(x+l+, x +2x+5).

1283. In

7
V3

~5 +

~X

1260. x-2'ln (x 2+

x+3In/x-31-3In)x-21

1281.

1269.-arcslD 2-:-:,. 1270. arcsin


Xy 5
(I-x)
1272.

1256.

-2 Vl-x-x2 -9arc sin

1266.

.f r
-- I
.
x-x2+'8arcS10(2x-l)o

2x+l
+-98 arc S111 \i

I.

1261. x + 3 In (x l -6x + 10) + 8 arc tan (x -3).

V5x l -2x+ I + 5

1+

In (r-7x+ I. +

I V \.

+x2+

-i V9-x +

x;

I f

-arCS1tl

1273.

1258.

arc tan

arc sin ~ 1263. arcsin(2x-I). 1264. In x+

1267.

1271.

VIT'

3 Vx 2-4x+5.

1265.

i-

2 1n (x 2-4x+5)+4 arctan (x-2).

9
2x+3
+ 3x + 4) + y"7
arc tan "7 .

/-2

6x-1

arctan

1253.i VA

Problem 1252*. 1254.

Hint. See Problem 1252*. 1255.

Ix+ 2x-7

1262.

421

;-(X~2)i-X~2.

27
~ In 1 ~-5
49 (X+2)+\i4J
x+2

1293. Sl2 1n I X-31-2~ In I x- 1 1+

1292.

x++ln

1;::1-

6~ In lx + 4x +5) + I~O X
2

Answers

422

1
6 In x-x+
.(x + 1)1
1

2
1294.
t
xarcan(x+).

Y2 + 1 Y2
x Y"2
JI- 2+ 1 +-4- arc tan I-xl'

x! + x

X In xl-x

xarctan

+ .;_
r3 arc tan ~r-l
r~ .

x'-I
'
XYa

+arctan(x+I).

1295.

.~_ X
4r2

1
x2 + X + 1
I
Tin xl-x+ 1 + 2 Y3 X

1296.

arc tan x
2

2x-l

2(x2+2x+2)+

1297.

2(1+x2)+

1299.

x+2
5
2x+1
Inlx+II+3(X2+x+l)+3 Y3arctan ya-

1298.

3x-17
I
15
In (x'+x+ 1).1300. 2 (x2- 4x +5) + 2 1n (x 2 -4x+5) + 2" arc tan(x-2).
_x 2 + x
I
I
I
1301. 4 (x+ 1) (x2+ 1) +"2 ln I x+ II-Tin (Xl + 1)+4 arctan x.
1

- 2

IX-II

3
x
3
15x5 +40x s +33x
-Sarctanx- 4 (X 4 _1)+16 In x+l 1303.
48(I+x2 )'
+
. 15
x-I
+48arctanx.
1304.
x x2_2x+2+21n(x2-2x+2)+3arctan(x-l).
1302.

+ 11)
1306.
I
~
I
12X4 + 1- Y
Y51' 1307.
- -4 In I x +x4-II--;y=-ln
2 r 5
2x + I + 5
+2Inl;_~I. 1308. ~ (2InIXlx~II-~I-XI~I)'
4

X- 2 1

+In x-I.
.

21 (81nl xl +81-1n I xl

t305.

1311.

In)xl- 71 In lx7+11.

1310.

1
5
1
In I x I-SIn) x + 1 1+ 5 (xs + I)' 1312.

x+1

X -2- 1313.

-arc tan x.

g (x-I).

V(ax+ b)Zl

13

x-

s Y x+l.l.
I)2+ x

2 arc tan

xarc tan

x+2+

... /x
2. 1322 .

lr-~-

+, x -II 1324.

z=

x+ I

x 1
x-I 1325.

r 3

arc tan 2

Y7
3

l~alX

1316.
1318.
X5 _

I (JrXT~_1)2 1-.;-

- 5x 5 + 3x 3 -X--

V
V -x+2rlr-x-6In(l+ 11h/-)x. 1319. "76 x V"/-x- S6"/;5
+2Vx-3 Vx-6vx-31n II + Vx 1+ 6 arc tan vx.
In

xii +

1=(x7 +1)-x7

-3

1320.

3
+-4
+
x-

4)3

1309.

Put

I
7 (X-I)7' 1314.

1317.

2(

3" arc tan (x+ 1)-6 arc tanx

2Yx=I[(X-;I)"+3(X

1315.

X[2 V(ax+b)S-5b

I
4 (x-I)!

Hint.

2' ln lx4- 11-

+ I . 1321. 2

6
3

V"X -

V-r+

rX-2 V'2x

lrVX~-I
1
-2 arc tan r I-x. 1323.
2
(x-2) +2 In I x
1
Z2+ Z + 1
2
22+ 1
2z
3 1n (Z_V+Y3arctan Y3 +zl-l'

where

V2x+3
2.x+3 .r
1
- - - x - ' 1326. ---8- r x2 -x+l+ In(2x-l+
16

Answers
+2

'If ,
f x

7
8+4x'+3x
-x+l). 132.
IS

4
..

423

r-l-x.
- I 13 8 ( S
5 3 1
2. 16 x -:24 x +"6 x

s) X

..r 5
.. r (1
3 ) ..r2 - 3
1
X y 1 + x2 -16 1n (x+ y 1 + x2 ). 1329.
4x4 + 8x!
r x - 1- 8 arc sln-X
.. r-z--

2x-l .. r

19

1330. 2(x+l)2 r x +2x-"2arcSlnx+l 1331. - 4 - y x -x+l+ S lnx


X (2x-I+2

- -} arc tan

V x- + 1.

(2x! -

1334.

va arc tan 73
2z+ 1
+ -S
1337. - 2

..

-Seas x.

Vx- + 1 + 1
V--x- +1-1

1 1 -b- x'
1
Yx'-x+I). 1332. -2 -;r-----==-.
1333. -4 In
y 1+2x2

where

(x"

1) Y 1 + Xl
1335.
3x s

Z ==

VlXi
+.

,.

TO In

(z - 1)'
z~+z+ 1

1 4 + 3x
- 8" x (2 -t- xa) 2/,
3

1336.

2
1338. sin x- ; sin' x. 1339. -cosx+3" cos s x -

+ 1)1.

1340 stn' x sln l x


1
x
1
1 " 2-:rcOS
-3---S- 1 3 4
-;rcos

s,ft' X

1342. -'2'--

1.
3x
sin 2x
sin 4x
2 sin' x - 21n I Sln x I
1343.
T - -4- + ~
x sin 4x
x sin 4x sin a 2x
5
1
1
1344. 8-32. 1345. 16-64+----:ta 1346. T6x+i2sin6x+64 sin 12x+
1
cotl x
2
1
144 sin a 6x.
1347. -cot x - -3- .
1348. tan x "3 tans x + "'5 tanS x.
cot a x cotS x
tanS x
1
1349. - - 3 - - -S-
1350. tan x + - 3 - - 2 cot 2x.
1351. "2 tan' x+

+
-I

3 In I tan

3 x - 4 tan
1 x 1352. ~
1 + 21n tan 2
xl . 1353. -2Y2 X
x 1- 2 tan2
4

cos "2

I i I+ I i + ~ ) I]

X [In tan
1355.
1357. -

In tan (

sin 4x
16 cos4 4x

3 sin 4x
32 cos2 4x

cot 2 X

2-- 1n I sin x I

s~::: - : ~:~I: +f In
1t )

1358. - 3" cot' x+cot x +x.

I I+x.

1360.

x -l~

V cos

Xl
sin 2\:1
4--8-

I i I
tan

5" tan Sx -x.

1356.

x
x
x
+tanla-3tan"3+31n cos "3

1359.

tan! 3"

coP x
1361. - - 3 - .

' x. 1363. 2 Vtan x. 1364. 2 ~2><


..r1
..
rZI
Xln + l f 2+1_ .. r _ arc tan~. where z=Vtanx. 1365. _C 0 s8x+
Z2_ Z Y2+1
r 2
z2-1
16
cos 2x
sin 25x sin 5x
3
5x
x
3
x
+-4- 1366. ----00- +10 1367. 5 sin "6 +3 sin"6. 1368. 2 tt")s"3-

1362. -

~ V cos x + ~

1354. 4

I (2x +""4 I.

3
32 1n tan

_ .!.2 cos x.

1369 sin 2ax

4a

COSiO

+ x cos22b

1370 t cns <p

sin ~2mt + cp) 1371 ~ +


4uJ.. 2
..

Answers

424

Si '1

5x

S1 f1

7x

tan~-2

2
x

+2J+28 1372. 24cos6x-I6cos4x-acos2x. 1373. 4 1n


tan
1374.

';'2 I

1377.

In

In tan (

i+i) \.

x-tan ~.

1375.

tan ~-5

~).

.13;8. arctan (I+tan

tan2"-3

+ 3cf's x
Solution.
We
+ P(2 ~1I) x + 3cos X)' Whence
12

~=-13

a=T3'
X

(2

S2

~i11

X)

fraction by ces2 x 1382.

. 1383.

51

2 r 2

2tan!--1
2

X --.,-- -

r3

~15 arctan ( V~~n x ).

V2+
V
- c;to 2x
2 - Sill 2x
1
lr- arc tan
r2

x) (J-sln x)

1380. -In Icosx-sinx,.

Hint. Divide the numerator and denominator of the

1
(2-Std

2-S111

\388.

Hint. See Problem 1381.

1 In X
1384. -5

See Problem 1381.

See Problem 1381. 1385. -2(1


Hint.

I
.r- In

1387.

+ 2 cos x ==a (2 sin x + 3cos x) +

x + 3- vG
.
v--1 3 In 122 tan
. I Hint.
tan x + 3 + V IJ

X tant y an x

HX-~lnI2sinx+

2a-3p ==3, 3a+ 2P=2 and, consequently,


3 Slf1 X + 2 Cf'S x
12
5
have
2s11lX+3cosxdx=I3Jdx-13X

+ 3J I~"~ v)' dx= -13


12
5
x -13 In 12 sinx+3cos x I.

x+ c ~ "
1381. } arc tan ( ta~
';ll

We

3 sin x

put

-x+tanx+secx.

1376.

1379.

+2

1
2
-cos x )2 1386. In(1 +5i11 x).

1
5-')in x
-4 In -1- - .

S111

2
r 3

1389.

..r- arc tan

Use

the

3tan~-1
2
.,-

Hint.

2r2

tan~
2

3-S1n

identity

x 1390.

-x+ 21n

tan

2+1

. Hint.

, I-sin x +c 0 s x
2
cosh' x
Use the IdentIty I
+ Sltl x - cos x ==-1+ 1 Sl'l x-cos x . 1391. -a--coshx.
1392. ~ + SIO~ 2x + sin: 4x
1393. sin~4 x
1394.
_
+ sin: 4x .
2
2
x
1
tanh! x
1395. In tanh 2" +coshx. 1396. -2coth2x. 1397. In (cosh X)--2-

cotha x
1398. x-coth x - -- . 1399. arctan(tanhx). 1400. y-garctan
3

or

2
("~)l
V5
arc tan eX , 5

. h - I

-SU1

x-cos

(3

x
tanh 2 +2)

Y5

sInh x sinh 2x
x
..
1401.--- - - 4 - - 2' Hint. Use the Identity
2

== lsinh x + cosh x).

1402.

.;_In
V2
2
,

cosh x

+ Vcosh 2%).

Answers

425

. x+l
x ..r - ..r - 3-2x-x"+2arcsln -2- 1404. 2 ' 2 +x 2 In(~+ r 2+x 2 ) ..
x .. r - -2 9
. . r - -2
x-I '1r l
"2
f 9+x -2"ln(x+ f 9+x ),
1406.
-2-" x -2x+2+

x+l lr

1403. -21405.

Y x 2-2x+ 2)
1407.
Yx 2-4-2In I x+ y;t=4l
..r - 1
I
. r-X-3 .. F
,. Xl +x-Sln 2x+ 1+2 y xl+x I 1409. -2- ,. x 2 -6x-7-

+} In (x-I +
2t+ I
1408. - 4 -

- 81n I x-3+ Vx 2- 6x-71. 1410. i4 (2x + I) (8x 2 +8x + 17) Vx l + x+ 1+


27
+128 In (2x+I+2

Yx +x+I).
1
1413. r_arctan
Vx Y2 1414.
l

14tt.

1- x
2
X (x4-2x -I- 5x -51: + { )
1416.
X cos 3x
Stfl 3x
X Cf'S X
SHI x
1417. - - 6 - + 1 8 + - 2 - - - 2 -

ir (XI

eX
1419. "2

(2 Sl1l2t 5

+l:( S

2~

Y21

1
VI + x +x Y
.r-In.r
2 y 2
y 1 + x2 -x
2

,. 2

x-I
1412. 4Vx2-2x+5'

.. IX-2
V x-I

~2 sin 6x +

2X

1415. eT X

i- cos 6X-~ sin 6x )

2X

e
1418."8 (2-sin2x-cos2x).

S4t)

4 ~ln 4X+Cf
1420 eX ( (.
1 7 ' "2 x sin x

+cosx )-Slnx.
. ]

x I I
.. F 1421. -"2+31nle"-I/+lfln(e"+2) 1422. x-ln(2+e"+2,. e2x +x+I).

~ rXl In ~ +; + In (I-x 2) +x2 ]


-Xln(x+Vl+x 2 )+2x.
1425.

1423.

X V20x-25x l -3.

1426.

sin x

1424. x In 2 (x+ VI +x l )-2 VI +x'X

(X2"-100,
9) arccos (5x-2)-1QO
5x+6
X
2

cosh x-cns x sinh x


2

1427. I

2 (n -

I) at

[(XI+~2)II-1 + (2n- 3) I 11_1] ; 1 = 2~2 (x ~ a + ~ arctan ~ ); I a = 4~a X


3
xJ
cos x sinn X n -1
.
X (3XI + 5a
X [ 2a 2 (Xl + a2)2 + 2a arc tan a
1428. In = n
+
-n- 1n-"
'
3x cos
3 sin 2x
I
4
. 1
8c
I &=8
4
-1-6-;
,=- cos x 5sin x -15cosxslnx-ISosx.
X

'

sin

n-2

1429.1n=(n_l)Cosn-lx+n_l/n-l;

1&= - 3
'+-3 tanx.
cos x

+ IO9x + ... + 1098


8

-1

X""l

~nx

I (

sin x
1
1'=2cos1x+-2 1n tan

1430. In=-xne-" +n1n_t;

... 2x+ 109.. 1).

110 =-e
_1_

-x

1431. V14 arc tan

(x

1t )

10

+ 10x'+

2+"4

V2(x-l)

Y7

1432. In V xl -2x+2-4 arc tan (x-I). 1433. (x 2 1)1 +i In (x1 + x+

~) +

IX+31

1
I
... i - r
1
1
+"2arctan(2x+I). 1434. 5 1n V x"+5 1435. 21n x+2 -x+2-x+3 ~

1436.

~(lnl':x~:II-x~I)'

1437.

~ (XI~2+~aretan;2)

Answers

426

I + I)

1 ( 2x -L
x 1
1438.4" I-xl ,In x - I

1
x- 2
1 2x - 1
1439. 6(xl-x+I)I+6xl-x+I+
x(3+2 Yi)
1
4
I
1440.
..r 1441. - -x- 3x ..yF---2
I
1-2 y x
x
x

2
2x-I
+ 3 .r_arctan
.r-
,. :3
y 3

1442. In(x+{+YXI+X+l). 1443.


1445.

(2X-I.
4. 1-2x+ 1

1448.

1477.1nlx+Jlx -1

x
(xl-I'

--I
z

. x

1
,
.2(x+l)

arc sin 1/-2.

.X arc sin

~-1

1450

X+4.

.. r

,.x~+1

In

-2 (V5-x-I)I-4In (1 + V5-x).
1448.

1 ... tl- x l!
Y l+xl

-2

1(4-

1
-8 In

1451.

2x + 1 + 2 (Xl + X + 1

1)'

3 I
x

-"4 cot 4 x.
1464.

1462.

/.

(x

1455.

+ 2x + 2)

arc tan

1471.

4 ~an

-cotx
3 cos 5x

X In (sin ax

1463.

Itan"25x I

,. 10

i)

2
,-

2x -t 2

y~

2 YtcotX)3

..r- arc tan

1469.

V"2

(x

1456.

5
I2lcos! x- 6)
tans x
1465.

i + ~ ) +21n Icos ( ~ + ~ ) I

2Y_
.

tan

x .. r-2y x - 9-

v- ,

"2 In I tanx + sec x 1- 2

'xarctan (

1
.. r- X
8,3

IV

-3 In

1487. tanl (
x

~ (8x-l) (x-4x' +~ arc sin (8x-l).

C0S 5t"
3
- -20 SiU4 5x - 40 sin 2 5x + 40 In

.1488. {sin 2x.


X

).

21

2x

VI-X a - 1 1
1

1458. --3 In I z -11


I-x' + 1
1
2z + 1
1 + x'
5
.. r- arc tan
where z=

1459. -2 X
f 3
3
x
3x Sill 2x sin 4x
1460. 8+-4-+32.
1461. Inl tanxl-cotlx-

1457.

+ -61 In (z +z + 1) .r - 'Xln(xl + y 1+x

1452.

_(xt l ) V.l.I+2X+2-fln(x+I+Yx'+2x+2).
y'(x

1449.

'

1)

1(1

1453.

-V~+l'

V'(2x)'. 1444.

Hint. xl+4x ="4 -X- x+4'

- i1n I x+ y' XZ -91.


1454.

Y2X-:

(2 ,rtanx)
.
f

cosec x. 1472.

5 -V
cos x.
2

tanS x

+ -5-
1468. - ~3x

-3-

1470. arc tan (2 tan x+ 1).

10

(3 xarc tan

~\
I
V3)- Y2 x

(tan

I
1473. Inltanx+2+ytan l x+4tanx+ll 1474'-a x

+ .r
,. a l +5111

ax).

1475."3 x tan 3x +9 In I cos 3x I

x sin 2x cos 2x
elx
~-4---~- 1477. Tl 2x - 1). 1478

a1

eX3

1479.

xl

1476.

3' -In

Xl

4"-

_-

Vl-x-

Answers
- } In I x-I 1- ;~- ;~-

~.

427

VI +

1480.

arc tan x-In (x.+

Xl

VI + xl).

1
3x
1 5x
I
x
I
I I
-sln---sin--- sln- 1482. -I t
. 1483. n l+cotxl-cotx.
2
2
3
2 10
2
+
anx
sinh 2 x
.. r - I
1481. - 2 - . 1485. -2cosh y I-x. 1486. Slncosh2x. 1487. -xcothx+
1
x
I
I
eX -3
+lnlsinhxl.
1488. 2eX-4+4InleX-21.
1489.
arctan-- .
2
2
4
4
I
I + 2x
10- 2X
X
ln
1490.7"
+I)1- 3 Vtex+l)I. 1491. In4
1-2x 1492. -2InIO X
1481

V(e

x
i_ ) .
X ( x2 _ 1 + _
+_
In 10 21n2 10
1494.lnj

1493.

_x_l_arctanx.

y 1+ x 2

1495.

1496. 2x (coslnx+sinlnx).

1497.

'1r-xVex + I - 1 .
r e +1+ln V--

eX + I + 1
2
1
4 (x.arCSlnJ..+x + 2 Vxl _ 1).
3
X

I ( -x2 cos5x+ 2 xstn5x+3xcos5x+

+ ~COS5X- ~ sin 5x ). 1498. ~ [(X -2) arc tan (2x + 3) + ~ In (2x


1499. {Vx-xl+(x- ;)arCSin V"X. 1500. x 1x 1
2
I

-iJ.

5)~

+ 6x +

Chapter V
T2
210 -1
1502. voT -g -2" . 1503. 3.
1504. lii""2.
1505. 156.
Hint. Divide the interval from x= I to x=5 on the x-axis into subintervals so that the abscissas of the points of division should form a geo1501.

b-a.

nletric progression: xn=-I,


Hint.

See Problem

Sil1a+Sin2a+ ...

Xl

=xoq,

1505.

1507.

X 2=

X n =x(lqn.
1506. In!!...
a
Hint. Utilize the tormula

r(Jq2, . "

I-cos x.

+Shlna=~ [cos~-cos(n+~)aJ.
2S1n

1508. 1)

"2

~=

- __1_. 2) dl __l_
5
I
_ lr--.
2 -X4
-Xl
In a '
db - In b 1 09. n x. 1510.
y 1 +.t . 1511. xe
-e
.
cos x
I
I
3
1512.
.r- + 2 cos 2 1513. x = nn (n= 1,2,3, ... ). 1514. In 2. 1515. - -8
2 y x
x
x
1516.

eX -e- x = 2 sinh x. 1517. sin x. 1518. } . Solution. The sum s" =

1
2
I (I
2
n- I)
=2+"2+
+ n-I
-2-=- + -+
n
n
n
nn
n ... + -n-

may be regarded as the Integral sum of the function f(x)=x on the interval [0,1]. Therefore, lim sn=
n

xdx=-2
S
o

I
1519. In 2. Solution, The sum sn = --I

-+,,, +~)

=*(_1-1 +_1_2
I+nI+n-

I+n-

n+

-+<Xl

I
I
+ --2
+, .. + -n+n
-=
n+

may be regarded as the integral sum of

Answers

428

f (x) = I ~ x

the function

on the interval [0,1] where the division poi nts have


1

the form
1520. p

Xk=I+~n

+1 .

(k=l, 2, , n). Therefore,

"3.

1521.

3 = 33 "3 .

1522.

n~ct>

4" .

1523.

Sn=51 dX+x == In2.


o

16

"3.

1524.

1525. -

In 3' . 1527. In 8 1528. 35 15- 321n 3. 1529. arc tan 3-arc tan 2=
1
4
n
1
n
11:
= arc tan 7 1530. In 3". 1531. 16 1532. 1- Y3 1533. "4. 1534. '2.

1526.

100

lim

.
1
l+YS
1535. 3" In - - 2 -
1540. O. 1541. 9

1536.

~3 + ~

8+ '4.

1542. arc tan

1544. tanh (In 3)-tanh (In 2)=

1537.

e- T'

1543. sinh 1= ;

~+~

. 1545. -

verges. 1548. I I p' if p< I; diverges, if p:;;;:: l.

;5'

1556. Diverges.

1557. Diverges.

1
.
1
1560. In a' 1561. DIverges. 1562. 7i

(e- ~ ).

sinh 211:. 1546. 2. 1547. Di


1549. Diverges.

1551. Diverges. 1552. 1. 1553. _1- ' if p>l; diverges, if


p- 1

1555.

1539. I-cos 1.

1538. In 2.

p~ 1.

1558. I; 2

n
1
1563. 8"' 1564. 3"

1550.

1554.

.
31.

1559. Diverges.

+ T1 ln3

1565. 3

2n

Y3

1566. Diverges 1567. Converges 1568. Diverges 1569. Converges. 1570. Converges. 1571. Converges. 1572. Diverges 157J. Converges. 1574. Hint. B (p, q)=
I/J
1
~ f(x) dx+ ~ f (x) dx, where f (x) =x P-' (l-x)q-,; since lim f (x) x'-P= 1

II

x~o

and lim (I-x) -q f (x) = I, both integrals converge when I-p< 1 and I-q< I.
% -+ 1

that is, when p>O and q>O. 1575. Hint.

I (x) =

(p) =

~ f (x)

CIJ

dx

+ ~ f (x) dx,

wherE

xp-1e- X. The first integral converges when p>O, the second when p i~

arbitrary. 1576. No. 1577.2

Y2 Sy'tdt.

1578.

S YI~Sl11lt' 1579. S dt,


n

' (arc tan


5 I +t

In a

In

QI)

1580.

t)

dt. 1581.

X=

(b-a) t +a. 1582.4-21n3. 1583. 8- 2

1t

1584.2-2

41

1585.

1589. 4-n. 1590.

1t
1/y 5

1586.

n
1 - -
y 1 + a2

n
1587.1-- .
4

1588.

1
~
7+2Y7
1 1t
Sin 112. 1,,91. In - - 9 - 1592. 2+4

V3 31 ,

.. r- n
y 3-- .
3
na2
1593. 8

Answer"
1594.

31

2' 1599. 2 -

e2 +3
t601. -8-

1600. l.

1.

42f)

1
1602."2 (e1r

+ll.

1603. 1.

tIJ

1604. a2 ~ b
1605. a2 ~ b2 ' 1606. Solution. r (p I) =
xPe- X dx. Applying
"
o
the fornlula of integration by parts, we put x P = U, e- x dx = dv. Whence
du =px P - 1 dx, v = -e- X
and

r(p+I)=[-xPe-xI:'+p ~ xP-'e-xdx=pr(p)

(*)

tf P is a natural number, then, applying formula


account that

ll

p times and taking into

r(l)=~ e-xdx=l,
o

we get:

(p+ 1)=p1

... (2k-I)~ 'f


=2k
b
1607 12k = 1.35
2.4.6 ... 2k
2' 1 n
is an even nurn er; I tk + 1 =
2 . 4 . 6 . 2q
.,
2k l '
b
1.::i.5 ... (2k+l),ln=
+ lsano dd numer

128
6331
10 =-= :~ 15; 110 = 512 .
(p-l)!(q-l)!
1 (m+l n + l \ .
"
1608.
(p -t-q-l)!
.
1609. 2 B - 2 ' -2-)' Hint. Put sin x=t.
1610. a) Plus; b) minus; c) plus Hint. Sketch the ~raph of the integrand for
values of the argument on the interval of integration 1611. a) First; b) second;
1
1
3
. 1
c) first.
1612. 3'
1613. a.
1614'"2 . 1615. 8"'
1616. 2 arc Sill "3
1617.

2<I<V5. 1618. 9<1<7' 1619. i3 n <1<7 11 .

Hint. The integrand increases monotonically. 1621.


1624. 1. 1625.

~ </< ~.

n2

O<I<a2.

1623. s=

Hint. Take account of the sign of the function. 1626. 4

1627. 2. 1628. In 2. 1629. m2 1n 3. 1630.


2

1634. lOa' 1635.4.1636.

..r-

1639. ab [2 r 3-1n (2+

32

3.

1637.

11

2-3.

1638 .

1
2 =2(coshl-I)
e+e-

..r3
f 3)]. 1640. 8" nal . Hint. See Appendix VI, Fig. 27.

See Appendix VI, Fig. 23. 1647. al

1648. 2n+'3 and 6n-

nat. 1631. 12. 1632. ~ p2. 1633. 4 ~

1641. 2al e- ' . 1642. : a l 1643. 15n. 1644.

1620.

In 3. 1645. 1. 1646. 3MI Hint.

(2+ ~ ).

16

a 1649. 3

HInt. See Appendix VI. Fig.

va

24.

4 V3
32
4
3
n - - and "3 n + -- . 1650 t nab.

a-

Answers

430

1651. &tal. 1652. It (hi + 2ab) . 1653. 6ltal . 1654.

al . Hint. For the loop, the


parameter t varies within the limits 0 E:; t ~+ 00 See Appendix VI, Fig. 22.
ltal. Hint. See Appendix VI, Fig. 28. 1656. &t'al . HInt. See Appen1655.
l
l
dix VI, Fig. 30. 1657. It: . 1658. al. 1659. It: . HInt. See Appendix VI,

33
60 9
14 -8
F Ig.
16 2"lt. 1661.
3
~r-

1664. n f 2.

Vh

Hint.

V2 a.
I

663

1tp2

1662. (I-el)"'o' 1

'3+-2-

1665. 27 (10 flO-I).

-a2 Hint. Utilize the formula cosh 2 a-sinh 2 a. = 1.


1667. V2 + In (I + 2). 1668. VI +el - Y2 + In (VI+ei-I)(

1666.

1669. 1+

~r-

Pass to polar coordinates.

V3)

1(31

.a

In

~.

V2 +

I) .

1670. In(e + Vel-I). 1671. In(2+ V3") 1672. {(el+I).

e2b -1
sinh b
1
2
3 1675. InW-l+ a - b =ln-.-- 1676. -2 aT.
b
e sin h a
l
Hint. See Appendix VI, Fig. 29. 1677. 4 (a:;-b ) 1678. I6a. 1679. na YI +4n l +
a

1673. aln- .1674. 2a

lrf

+; In (21t+ VI +4ltl) . 1680.8a. 1681. 2a [Y2 + In (Y2 + I)]. 1682.


na s

v:

aVl+m
1
4
+ In 3+V5
2
1683.
m
1684. 2 (4 + In 3). 1685. 30' 1686. :r 31ab
-1
88
31
4 31
1687. aJ31
4 (eI + 4 -e).
16.
83 31 2 1689. vx=T.
1690. v'=7
2

1691.

1696.

31
16rra a
32 a
4
a
3
v X =2"' vv=231. 1692. -5-' 1693. 15 31a . 1694. "3 np . 1695.

n;1 (\5-16.ln 2). 1697.

a.
b ) 6n aa,

c)

6na (931I -16)


.

2ltla. 1698.

32 31aa.
105

1702

1705.

h (
Ab +a B )
3"
AB +
2
+ab.

1709.

I
2nalh.

lt~IH.

1706.

nabh
-3-'

10 31

1699. :: lthla. 1701. a) 5nla';

1703

38 31a.

1707.

128
105 a.

1704
1708.

4 na a
2i

8
2
3Ul o.
'3

16
1710. :ra. 171t.nal Vpq. 1712. nabh

(hI'.
4
I+). 1713. ltabc.

~6nal[5V5-8].

2n[V2+ln(V2+I)].

1714. 8;lVI7'-I];

t7t5.

It (V5 - y'2) +It In 2 (V2 + I) 1717.


n [V2 + In (I + Y2)J.
V5+1
na 2
31a 2
12
n
1718. T (el+e- I + 4)=2 (2+ sinh 2). 1719.
nal. 1720. (e-l) (e 2 +e+4).

17t6.

1721. 4n!ab Hint. Here, y== b Va2 -xl Taking the plus si~n, we get the
external surface of a torus; taking the minus sign, we get the internal sur2
face of a torus. 1722. 1) 231b l + 2nab arc sin e; 2) 2na l 1(b In 11 e, where

Vaa
l

8=

-e

64M
32 2
(eccentricity of ellipse). 1723. a) -3-; b) 1631' a2 ; c)
31a

Answers
1724.

1~8nal.

1~8nal.

1725. 2na l (2-Y2). 1726.

a .. ~
M Y ="2 r a 2 +b 2

ab 2
M a =2;

1728.

43t

a2b
Mb=y.

MX=:

1727.
1729.

Yal+bl ;

a'
M x =M Y ="6;'

a
3
2
x=Y=a' 1730. Mx=M Y =5 a2 ; x=y=Sa. 1731. 2na 2 1732. x=O~
a 2 + sinh 2
- a sin a 4
- 4a
y 4 sinh 1 1733. x =-a- ; y=O. 1734. x=1ta; Y=3 a . 1735. x=3xt:
4b
y -=3xt'

x=Y=20' 1737. X=1ta; Y=r;a.

1736.

(a
)
0,0, 2" .

1738.

Solu-

tion. Divide the hemisphere into elementary spherical slices of area da by


horizontal planes. We have do = 2na dz, where dz is the al titude of a slice ..
a

2n

~ az dz

;Jla

Whence z =

=;.

Due to symmetry, x=y=O. 1739. At a dis--

tance of : altitude from the vertex of the cone. Solution. Partition the

cone into elements by planes parallel to the base. The mass of an elenlen
tary layer (slice) is dmi=y1tQ2dz, where y is the density. z is the distance
M

of

the

cutting

plane

from

of the cone, Q = ~ z. Whence

the vertex

S;:

Zl

z = I0
- xtr 2h
3

dz

= : h.

1740. (0; 0;

+:

a ). Solution. Due to symmetry,

x-== Y=O.

To determine 2' we partition the hemisphere into elementary


layers (slices) by planes parallel to the horizontal plane. The mas~ of such
an elementary layer dm = ynr 2 dz, where y is the denc;i ty, 2 is the distance
of the cutting plane from the base of the hemisphere, r = Ya 2 _z 2 is the
a

radius of a cross-section. We have: z =

~ (al o

2
-na 8
3

Zl) Z dz

3
=aa.
1

1741. I=na 8
1

1742. la=aab' ; I b =3 a 'b. 1743. J=T5hb'. 1744. /a=4"nab 3 ; Ib=T1ta'b.

1745. I =

in (R:-R:).

concentric circles.

The

Solution. We partition the ring into elementary

mass of each such

R2
the moment of inertia J =2nS r l dr=

element dm = y2Jtf dr

and

~ n (R:-R:); (y=I). 1746. J = I~ nR~Hv.

R1

Solution. We partition the cone into elementary cylindrical tubes para lie!

to the axis of the cone. The volume of each such elementary tube is
dV = 2xtrh dr, where r is the radius of the tube (the distance to the axis of
the cone), h = H ( 1- ; ) is the aHi tude of the tube; then the moment 01

Answers

432
R

Inertia I=y

S2nH (1- ~ ) r'dr= yrr~.H where

y is the density of the

'Cone. 1147. 1= ; MaJ. Solution. We partition the sphere into elementary

cylindrical tubes, the axis of which is the given diameter. An elementary


volume dV=2nrhdr. where, is the radius of a tube. h=2a
a

is its altitude. Then the moment of inertia I =4rray

Jor - ( 1 - a

"1/
Y

':

1-

rldr =~ rra5y,
15

,2
2

where y is the density of the sphere. and since the mass M =

~ naly.

it fol.

22
-2
2
2
lowsthatJ=r;Ma
1748. V==2Jtab; S=4n 211b. 1749. a) x=Y=5 a ;

--9
-4r
b) x=y= lOP. 1750. a) x=O, Y:=3 It Hint. The coordinate axes are cho-

sen so that the x-axis coincides with the diameter and the origin is the
centre of the circle; b)

x= ~

Solution. The volume of the solid - a double

cone obtained from rotatin f1 a trIangle about it~ bClc;;e, is equal to V -==-!.. '!'b1z 2
3

f.;,

'

where b is the base. Iz is the altitude of the tnan(!le. By the GuldIn theo-1

rem, the same volulne V-=2Jtx 2bf1. whl're x is the dIstance of


of

gravity

1752.
1755.

trom

the

~
In (1 +~).
2g
11= ~ In (II a bl);
c~

=]y R2H!

base.
1753.

\\ hence

=3

tiL'

centre

175t.

=_!n V o 1754. S =- 10 nz.


[btl-Ia-bt.) In a~bIJ.
1756.
A=

x==

h= ~ x

It

"111 wt;

(I)

v av

Hint. The elementary force (force or gravity) is equal to the

weight of water In the voluIne of a layer cf thlcknpss dx, that IS, dF =


= ynR2 dx, where y is the weight of unit voluml' of \vater. Hence, the elementary work of a force dA =-=. ynR~ (H -x) dx, wht're x is the water level.
1157.

A=l~ yRJHJ.

1758.

A-~'Jl4Y RTM::::::079JOI=079IO'

kgm.

1159. A=ynR'H. 1760. A= mgh ; Aoo=mgR. Solution. The force acting


h

l+ R

on a mass m is equal to F= k
of the earth. Since for

f=

m~
, '

where r is the distance from the centre

R we have F=mg, it follows that kM =gR 2 The


R+h

~~

(~

$ought.for work will have the form A =


k
d, = kmM
- R h) =
mgh
R
. c : : : - --. When h=J we have ArrJ=mgR.
1761. 1.8.104 ergs. Solution.
h

1+1[

Answers

433

The force of interaction of charges is F= eO~1 dynes. Consequently. the work


x

5c;:=

%.

performed

moving charge el

in

from

point

XI

to XI Is A =e'l

x)

(...!.-- 1-)
= 1.810
x

ergs. 1762. A = 800 n In 2 kgm. Solution. For an


XI
2
Isothennal process, prJ =Pov o' The work perfornled in the expansion of a gas
=eoel

VI

from volume

Vo

to volume

VI

Jrp dv = Pov o In VVIo 1763.

is A =

A:::::: 15,000 kgtn.

Va

Solution. For an adiabatic process, the Poisson law pull = Pov~, where
k ::::::: lA,

hold.

true.

Hence

Pov.
' Pov~ dv ~ k-I
Sv

A=

(V

[I _

o ) k-

VI

J] _

Vo

1764. A =3" nJ.LPa. Solution. If a is the radius of the base of a shaft, then

the pressure on uni t area of the support p

==.!!.".
M

The frictional force of a

2JJ.:

ri ng of width dr, at a distance , from the centre, is


rd,. The work pera
formed by frictional forces on a ring in one complete revolution is

S,2 dr = 33tJ..tPa..
4
a

4nJ.LP X
dA = 43lJ.LP
i2 ,2d,. Therefore, the com pi ete work A = (i2

i- MR oo
2

1765.

o
2

Solution. The kinetic energy of a particle of the disk

Qr 2oo 2

mo 2

dK=T=-2dO, where
{ronl

the

axis of

do is an element of area,' is the distance of it

rotation,

is the surface density

Q=

:rr.RI'

Thus.,

Moo 2

Moo25

dK =2JtR2 r2da . Whence K=R2

MR2CJ)2
r 1 dr=--4-

3
1766. K=20,,><MR2W 2.

1767.

K=~ R 2oo 2 =2.31O' kgm. Hint. The amount of work required is equaf
2

to the reserve of kinettcenergy. 1768.

bh
P=T.

(a +2b) h 2
1769. P=--6--~11.3.10' T

1770. P =abVJlh. 1771. P = llR;H (the vertical component is directed upwards).


1772. 633

hb"p

kMm.

1773. 99.8 cal. 1774. M -= -2-- gf cnl. 1775. a (a I) (k JS the


a
a
C
.
gravitatIonal
constant). 1776. npa
8....1 . Solution. Q = v2nrdr = 21tp
-4.... 1 (a 2_,2) rdr==
1tp
=2J.Ll

gm

laT - Tr
B

,2

21,

npa
0 = 8l11' 1777. Q=

Sv a dY:Z::32 P abiLl

15-1900

Hint. Draw thex-axfs

Answers

434

along the large lower side of the rectangle, and the y-axis, perpendicular to

5
V2

it in the middle. 1778. Solution.

s= ~
v.

Mx =- S~ (x-t)dt+ ~ x=x

Jra

dv

=5.

til

1780. Mx = -

a,

t',

whence dt =1. do, and consequently, the acceleration time is t =

1779.

:~ =

dv; on the other hand,

[xt-

~]:+~ x= ~x (1-+)

5(x-t) kt dt +Ax= ~ W-x

),

1781. Q =0.12 T RI: cal.

Hint.

Use the J6ule-Lenz law.

Chapter VI
2
X2)X.
V=3(y2_

1782.

1784.

f ( -2' 3 =-3;

1783.

f(l, -1)==-2.

2
~r 2
S="3(x+y)
r 4z -l-3(x-y)2.
y2_x2
2xy '

1785.

R4

2xy
x -y

-2--2

1786. f(x, x2 )=I+x-r. 1787. z = - R


I2. 1788.f(x)=

Hint. Represent the given function in tile form


replace

X2_ y 2
2xy '

I(

~)=

by x.

x-Y=v.
, u2 -uv

Then

1789.

f(x,

u-t;v

y)=x

-;

u-v

xy. Solution.

u+v

VI-I-x 2
x

V(; r+

Designate

x=-2-' Y=-2-; f(u, v)=-2-

y2_ x 2
2xy '

I and

x+y=u,

-2-+ (U-V)I
-2- =

u-v

=-'-2- It remains to name the arguments u and v, x and y. 1790.

f (u)=

=u +2u; z=x-I+ VY.

Hint. In the identity x=l+f(VX" -I) put


then x =;= (u + 1)2 and, hence, f (u) = u 2 + 2u. t 791. f (y) =
= VI +y2; z= VX 2+y2 Solution. When x= I we have the identity

Vx - I =

u;

y I + y2 = 1 f ( t ), i. e., f (y) = y 1 + y2. Then f ( ~ ) =


1 +( ~ rand
Z=X
1 + (~r = JI x"+y2. 1792. a) Single circle with centre at origin,
including _ the circle (x + y2 ~ 1); b) bisector of quadrantal angle y = x; c) half-

plane loeated above the straight line x+y=O (x+y > 0); d) strip contained
bet~een the straight lines y= I, including these lines (-I ~y~I); e) a
square formed by the segments of the straight lines x= 1 and y= I, including its sides (-I ~x~ I, -1 <aU ~ I); f) part of the plane adjoining the
x-axis and contained between the straight lines y:= x, including these lines and
excludi,ng the coordinate origin (- x ~ y E:; x when x > 0, x <; y ~-x when
x<O); g) two strips x~2. -2~y~2 and x~-2, -2~y~2; h) the
ring contained between the circles x2 +y2=a 2 and x2+y2=2a2, including the
boundaries; i) strips 2nn ~ x~ (2n + I) rt, Y ~ 0 and (2n+ I) n ~ x ~ (2n + 2) 1,
y.-; 0, where n is an integer; j) th~t part of the plane located above the

435

Answers

par~bola Y = - x2 (x2 + Y > 0); .k) the entire xy-plane; I) the entire xy-plane,
with the exception of the coordinate origin; m) that part of th~ plane located
above the parabola y2=X and to the right of the y-axis, including the pOtnts
of the y-axis and excluding the points of the parabola (x ~ 0, y > Y x);
n) the entire place except points of the straight lines X= 1 ~nd y =0; 0) the
family of concentric circles 21tk~x2+y2~n(2k+l) (k=O, 1, 2, ... ).
1793. a) First octant (including boundary); b) First, Third, Sixth and Eighth
octants (excludIng the boundary); c) a cube bounded by the planes X:c:: I,
y =~ 1 and z== 1, including its faces; d) a sphere of radius 1 with centre
at the origin, including its surface 1794. a) a plane; the level lines are
straight lines parallel to the straight line x+y=O; b) a paraboloid of revolution; the level lines are concentric circles with centre at the origin;
c) a hyperbolic paraboloid; the level lines are equilateral hyperbolas;
d) second-order cone; the level lines are equilateral hyperbolas; e) a parabolic
cylinder, the gcneratrices of which are parallel to the straight linex+y:+-I=O;
the level lines are parallel lines; f) the lateral surface of a quadrangular
pyramid; the level lines are the outlines of squares; g) level lines are parap
olas y =- Cx2 ; h) the level lines are parabolas y = C
I) the level Hnes
are the circles C(t 2 +y2)=-2x. 1795. a) Parabolasy=C-x2 (C>0); b) hyper2
bolas xy = C (I C I ~ I); c) circles x + y2 = C2; d) straight lines y = ax + C;
c) straight lines y-=CX (x =1= 0). 1796. a) Planes parallel to the plane
x -t- Y -1- z --= 0; b) concentric sphere~ \vith centre at origin; c) for u > 0,
one-sheet hyperboloids of revolution about the z-axis; for u < 0, two-sheet
hyperbololds of revolution about the same axis; both families of surfaces
are divided by the cone X 2+y2_ Z2=0 (u=O). 1797. a) 0; b) O;c) 2~
d) e'l; e) linlit does not exist; f) linlit does not exist. Hint. In Item(b)
pass to polar coordinates In ltcnls ~e) and ef), consider the variation of x
and y along the straight lines y == kx and show that the given expression
may tend to different lilnits, dependin~ on the choice of k. 1798. Continuous.
1799. a) Discontinuity at x=O, y==O; b) all points of the straight line
x==y (line of discontinuity); c) line of discontinuity IS the circle
X 2 +y2= 1;
d) the tines of discontinuity are the coordinate axes.

Yx;

1800 Hint.

Puttin~

Y = YI

== const,

\ve

~et the function <rl (x) =

2XYI
x2 +

y: ,

which

is continuous everywhere, since for Yt :P 0 the denolninator x 2 + Y~ :P 0, and


when YI ~ 0, (PI (x) === O. Sinl Harly, when x=x 1 = const, the function
,
CP2 (y) = 2x !J is everywhere continuous. Froln the set of variables x, Y, the
-t- y2
function z is discontinuous at the point (0, 0) since there is no lim z. Indeed,

x:

x~o

l/-+O

passing to polar coordinates (x= , cos cp,y = , sin q>l, we ~et z == sin 2'Pt \vhence it is
eviden t that if x ---+ 0 and y -+ 0 in such nlanner that q> = const (0 ~ q> ~ 231),
then z -+ sin 2({). Since these linliting values of the function z depend on the
direction of cp, it follows that z does not have a lilnit as x -+ 0 and y -. o.
dz
2
az
2
az
2y
az
2x
1801. dx=-=3(x -ay), ay=3(y -ax).
1802. ax=(x+y)2' ay=-(X+y)2
1803.

az=_
ax

x2'

az=~

ay

1805. ~=
y2
ax (x 2 -1_ y2)3!2 '
y

1804

az

ay

= Yx2+ y2 (x + Y x 2+ y2)

az_ x
ax- X

xy

(x 2 + y 2)3!:. .

az

1807. ox =

y 2'
1806 dz _
. ax 2_

x2

+ y2'

az

az__ y
ay- Yx2_y2
1

Y x2 + y2'
az

iJz _
ay-

ay = x 2+y2 1808. ax =yx

Answers

436
iJz
..:3

=xY lnx.

~=_1!..,

1809.

ox

Y2x

I - 2y
xy'l
- JY I (xl_yl)'

~% = _

X ~ra_ cot

2y , Y
iJu

uy

sin

.1L

Xl

at!:::a ,. Y

sin

1L oZ::z.!..e
X

ay

yx l 2x l - 2y
I y I (Xl_yl)'

OZ

x.~~.

"'"cos

JL
xcoslL.
1

az

1811.

ox = Yi

1812. iJU -== yz (xy)Z-I. iJau ==xz(xy)Z-I.

~~

1810.

iJx

X +a

cot

yi'

~u = (xy)Ztn(xy).

uz

OU
iJu
_
'
1
'.1813. ox=yzxYlnz,
iJy=xzxYlnz,
oz==xyzxy I.
1814. fx (2,1)=2'
.,
,
1
z
1
'1/(2, 1)=0.
1815. Ix (1,2,0)= 1, ' , (1.2.0)=2'
f (I, 2,0)=2 .

1820. -

+y

x
3/ 1821.
(x 2 +yl + Z2) ,

y
z=arc tan -+<1' (x).

x2

1827. z=-2

1
1
lnx+siny-'2. 1828. I) tana=4. tan~=oo. tanY=T; 2) tana~oo,

p=4.

tan

r. 1826.

tan

Y=4

as

1
oS
1
iJS
1
1829. iJa =2" h, iJb =2 h, iJh =2 (a+b).

1830. Hint.

Check to see that the function is equal to zero over the entire x-axis and
the entire y-axIs, and take advantage of the definition of partial derivatives.
Be convinced that f~(O. O)=f~(O, 0)=0.
1831. L\f=4L\x+L\y+2L\x2 , +2L\xl\Y+~XI~y;
df=4dx+dy;
a) t1f-df=8;
b) L\f-df=0.062.
1833. dz=3{x 2 -y)dx+3(y2_ X )d y. 1834. dz=2xy 3dx+3x 2y 2dy. 1835. dz=
2
1
(.l.2
1/2)2 (xy 2dx-x 2y dy). 1836. dz= sin 2xdx-sin 2ydy. 1837. dz =y xY- dx+

+x)/(l+ylnx)dy. 1838.dz=
t8oW. dz=O.

2
x

2+

(xdx+ydy). 1839.df=-+1 (dX-!..dY)J.


x y
Y

1841. dz= ~ ( dyX S111

J!.

dx ) .

1844. du

1845. du= (x y +:

~) zdx+

X ( xy

+ : ) dz].

1846. du

= 25l5dz-3dx-4dy).

df (1, 1) = dx -

y+

Y Xl +1y2 +Z2 (x dx + y dy + z dz).

(1- ~2) xzdy + (xY+f) In ~<

= X2y:~ Z4 (y dx +x dy- 2:

dZ). 1847. df (3. 4. 5)--.:

1848. dl =0.062 cm; L\l =0.065 cm. 1849. 75

tive to inner dimensions).

2dy.

1843. du = yz dx +lx dy + xy dz.

r-1[ (

1842.

1850.

Cl11 3

(rela-

em. Hint. Put the dilTerential of lke area

of the sector equal to zero and find the differential of the radius froIn that.
1851. a) 1.00; b) 4.998, c) 0.273. 1853. Accurate to 4 lnetres (more exactly,

g
1854. n a -:; ~l
g f 19
e' (t 1n t - I)
du
.=
t In 2 t
1857. i[j=

4.25 m).

1855. da=..!.. (dy cos a-dx sin a).

t
x (
x )
Yu
eot yy' 6- 2y 2

du

1856.

d~t =

1858. dt = 2t In t tan

t+

2
~(ttl+ l)tant+(t +1)lnt 1859 ~=-=O 1860 ~- (. )COSX(
t--;T
t
cos2 t
dt
dx - S1n x
cos x co x

Answers

437

az
y . dz
1862 az . a.<>V-I. dz
-slnx 1n sin x). 1861. a-=-~+
a-=U~
, d-=
x
x Y2' d-=-lx
+x, .
x
x
az,
,
az
,
Y]
Y
=x [ q>'(x)lnx+-X .1863. ax=2xfu(u,v)+yrYfv(u, V); ay z:-2Yf u (U,v)+

;;=1.

+xrYf~(u.v). 1864 ::=0,


:; = (x+ ~ ) f' (x y + ~).
+ f~ (x, Y, z) ("'~ (x, y) + "'~ (x,

:~=Y(1-:2)f'(XY+~);
:~=f~(X. y. z)+<p' (x)f~ (x, y. z)+
1865.

1867.

1873. The

y) <p' (X.

perimeter increases at
1+2ta+3t~

a rate of 2 m/sec. the area increases at a rate of 70 rnB/sec. 1874.

Y2 km/hr.

1875. 20 V5-2

68

C0S

1880. 13. 1881.

1876.

_9~3.

+ C0S3 ~ + C(l5 V .

1891

1886.61

dz
ax 2

abcL/2

~3-.

. 1879. _

. .

b) (0, 0), and (1. 1),

cosa=3", cos~=-3' cosY="3' 1888. cos<p=

cp === 8337'.

~2

1882. a) (2, 0),

4"1 (51-3j).

c) (7, 2, 1). 1884. 91- 3j. 1885.

1877. 1. 1878.

1 + t 2 + te

az
2

+ 3j + 2k. 1887. I grad u 1=6;


3
YIO.

_ _

1889. tan<p:::::8.944;
abcxy

az
2

(b x2 +a!y2)'/.a' QXay- (b 2 x 2 +a 2 y 2)'/2'


ay"=2
abex"
1892 a z _ 2 (y-x 2 ) . ~_ _
2t . a"z
1_
- (b!x2 + a'y2)'/2 .
ax 2- (x 2+ y)2 ' ax ay (Xl + y)2' a y2(x" + y)2
a2u a2u
a!z
XI/
alz
a2,
x2
1893. a-a =
3/ 1894. a-a-- = O. 1895. -a2 = - a - 1896. :i2=-a2=
x y {2xy + y2) I
X Y
x,
v X
y
2u _ alu _~__
_ a"u_.
d
~~
~-J
'3-1 1-1
""-0 Z2 - 0 ,
a x vy
A
-0 y vZ
i~ -aZ a x -I.
1897. ax a y az ::z:a~yx
y.
Z

,2-

1898.

a'z
ax oy2=

-A 2

y cos (xy) -2x sin (xy).

f:y(O, O)==mn; 1;/1/(0, O):=n(n-l).

1899.

1902. Hint.

f"x

(0, 0)

m (m -

1);

USIng the rules of differen-

tiation ~nd the definItion of a partial derivative, verify that f~ (x, y) ~


x 2 - ll2
4, 211 2 ]
,
= y [ - 2 - " I -f-- ( I
! 2
(when,\2 + y2 :;Ie 0), f x (0, 0) -= 0 and, consequently,
~+y
x+y)

f:

1:

that for x =-= 0 and for any y,


(0, y) -== - y. Whence
(0, y):z: -1; in par'1
ticular,
(0. 0) = -1. Sirnilarl y, we find that f;'x \0, 0) ~ 1.
,
"
"
iJ2 z
2 "

':'1

1903.

-a:z=2I u (u, V)+4Xlfuu


a2 z , , ,

ax ay = 'v (u, v) + 4xyf uu (u, v)

"
+ 2 (Xl + y2) f"
uv (u, v) + xYfC'v (u.

aal/z:=21
,

+ 4xYl"uv (u,

u (u. v)

1904.

(u, v)+4xYf uv (u, v) +y I vl,(u, v);

a!u
O~I

"

+ 4y f"uu (u,
l

",,,

v)

,,,

== fxx + 2f~l<PX +I Zl (<Px) 2 + fz<px,

2 "

v) +x f vv (u, v).

v);

438

Answers

a"z
1905. ax"

'"

"""

",

""'"

= fuu (q>x) " + 2fuv<l'x"'x + fvv ("'x) + fu<l'xx + 'v"'xx;


o"z " "
" "
"
I'"
,,,
,,,
ax ay = Iuu CPxq>y+ fuv (<I'x'l'y +'I'x<l'y) + Ivv'l'x'i'u + fuq>xy + fv'i'xy;
iJlz

"

,,,

I"

ay 2 =fuu (<I'y)" + 2fuv<py'l'y+fvv('I'y)"+fuq>yy+fv'l'yy


1914.

,,,

1915. u(x, y)=xq> (y)+'I' (y). 1916. d2z=e XY X


1917. d"u = 2 (x dy dz + y dz dx + 2 dx dy).

u(x, y)=<p(x)+'I'(y).
X dy)" + 2dx dy].

X [(y dx +

1918. d2 z=4cp" (t) (x dx+y dy)1

+2cp' (t)(dx' +dyl).

X (Yln~dx+xln!-dY);

fY x

1919. dz= ( ;

d 2 z= (!..)XY [(ylln" ex+ y )dx2+


\ y
y
x

ey

II

+2 (x y In ex
In ey~+ In":")
dx dy + (X"ln"!...-~) d y2 ] .
y
ey y
y
1920. d"z = a2f~u (u, v) dx" + 2abf;'v (u, v) dx dy + b2f:v (u, v) dy".
1921. d z = (yeXf~ + e2Yf:u + 2yex +Y f;w + y2e!Xf~v) dx" +
2

+2 (eYf~ +eXf~+xe2Yf:u+e x + y (1 +xy) f;tV+ye2Xf~v) dx dy +


+ (xeYf~ +x1e"Yf: u +2xeX+Yf~v+ e"Xf;,v) dy".
1922. d'z = eX (cos y dx 3 - 3 sin y dx" dy - 3 cos y dx dy" -l- sin y dya).
1923. d'z = - y cos x dx 3 -3sinxdx2 dy-3cosydxdy"+xsinydy3.
1924. df(l, 2)=0; d"f(l, 2) =
= 6dx" + 2dx dy + 4. 5 dy 2. 1925. d"f (0, 0, 0) = 'ldx" + 4dy" + 6dz 2 - 4dx dy +
ya
X
+ 8dxdz+4dy dz.
1926. xy +C.
1927. x' Y--3 +sin x+C. 1928.
x-t- y
1
x
x
+In(x+y)+C.
1929. -2 In(x"+y2)+2arc tan-+C.
1930. - + c.

--+

Yx"+y"+C. 1932. a= -1, b= -1, z= ~-+Y2+C. 1933. X2+y2+ Z2_tx y


+ xy +xz + yz C. 1934. x3+ 2xy 2+ 3xz + y!-yz-2z + C. 1935. x"yz-3xy2Z -t+ 4x"y" 2x+ y+3lz +C. 1936. ~+ 1L+~+ C.
1937. Yx 2+y2 -t- Z" + C
y
z x
1938. A= -I. Hint. Write the condition of the total differential for the
xy

1931.

expression Xdx+Ydy.
_

1939. f;=f;.

1940. u=Sf(Z)dZ+C.

1941.

~=

bZx. d"y
b4
day
3b 8x
. . .
-d
.=-2'3; -d3 = - 4 5 1942. The equation definlngy IS the
x
ay
x
ay

---a"y '

equation of a pair of straight lines.

:;=0
1946.

'!1t. =
dx

1948.

1945.

Yy)I'
x

+ ay ;

ax-y

~=x"-yz;

nx

XJJ

-z"

=xsiny-cosz
cos x-y sin z

(:t=1=3

1943. ddY = 1 Y"

or

d"y (a + 1) (Xl + y")


dx"=
(ax-y)3
2

In''~l'

1944. ddY =--L. ;


x y- 1

-xy

-I;

(~:.t=1=8

1947.

c!lt. =

dx

JL ;
x

or

-8.

d"y 2y
dxl=XZ .

~=6Y"-3xz-2.
1949. az=zsinx-cosy.
az
3 (xy -2")
ax cos x- y sin z'
2
1950. oZ=_I; Oz
I
1951. az=_c x; ~=_c:21J.
ox
ay =2'.
ax
a2 z ay
b?z ,.

ay

au

-==.,-

Answers

439
1953.

2v d 2
-(I+y)2 y.

1971.

a)

1965.

dz

dx =

Answers

440

3x+4y+12z-169=O.
1985. x+4y+6z=21
1986. x+y+z=
(2
1987 At the points (1, 1. 0), the tangent planes are
parallel to the xz-plane; at the points (0, 0, 0) and (2, 0, 0), to the yz-plane.
There are no points on the surface at which the tangent plane is parallel to
1983.
=

y a2 + b2 +
O

the xy-plane. 1991.

n . 1994. Projection on the xy-plane:


3

Projection on the yz-plane: {

IY=O
3x 2
\ 4+ z2 - 1 =0.

;:::Z2_

I =0

{Z:=+O!
1-0
x y -xy- - .

Projection on the xz-plane:

Hint. The line of tangency of the surface with the cylin-

der projecting this surface on SOIne plane is a locus at which the tangent
plane to the given surface is perpendIcular to the planf of the proJcclion
1996. f (x + h,
y+ k) =ax 2+ 2bxy + cy2+ 2 (ax + by) h + 2 (b..: +cy) k -t-- ah 2 +
+ 2bhk + ck 2
1997.
f (x, y) = l-(x + 2)2+ 2 (x + 2) (y-l) + 3 (y_l)2.
1998. ~f(x,y)=2h+k+h2+2hk+h2k. 1999. f(x, y, z)=(x-l)!-t-(y-l)2+
+ (z-I)I+2(x-I)(y-I)-(y-I)(z-I). 2000. f(x+h, y+k, z+l)=
= f (x, y, z) + 2 (h (x - y - z) + k 2(y - x - 4 z) +2 I (z - x - y)] + t (h. k, 1).
3"(2 y _ y'
X +y2 x +6x y2+ y"
2001. y+xy+
3!
.2002.1-4!
2003. I+(y-I)+
2

,-+

+tx-I)(y- I).

(x-I)+ (y+I)]'

2005

31

b) "' / (I +a)m

V2

+ (3n

1+[(.t -I)

2004.

(y

I)]

+ [(x-I)

-t-

2!

(y+I)]2

a) arctan I+a __ ~+..!- (u+P)_.!(U2 RI).


1-~ -- 4
2
4
-p,

+
(l + ~)n :::::: I +! (ma +n~) +!
2
4
32

[(3m 2

4m)

2 _

2mna~ +

-4n) ~2]o 2006. a) 1.0081; b) 0.902. Hint. Apply Taylor's formula for

Yx- V;

the functions: a) f (x, y) =


In the neighhourhood of the point (I, I);
b) f (x, y) = yX in the neighbourhood of the point (2,1). 2007. z = 1 2 (x - 1) - (y-l)-B (X_I)I+ 10 (x-I) (y-l)-3 (y-I)!
2008. zmln == 0 when x= 1,
y=O 2009. No ex\remum. 2010. zmin ==- 1 when x= 1, y==O. 2011. zmax= 108
whenx=3,y=2.2012.z min=-8 when x= Y2~'y=and when x=
,r..1.
ab
=- r 2, y= f 2. There IS no extremum for x=y=O. 2013.2 max ==---.;r-at

+ ...

yi

3 r 3

ab

Y3

the FOlots x=Y3' Y=Y3 and x=-Y3' Y=-Y3; 2ml n = - 3


.
a
b
a
b
at the pOints x=Y3t y=-Y3 and x=-Yat Y=YS2014'Zmax==1
when

x=y=O.

2015.

2 m in=0

when x=y=O;

nonrigorous n18XilllUIlJ

(2 = } ) at points of the circle x + y = I. 2016. zmax= Y3 whenx= I, y =-1.


2

2017.

x= ~ .

4 2 1

Umin=-a

when

x=-3"' Y=-3 %=1.


t

2018. u min=4 when

y= I, z= I. 2019. The equation defines two functions, of which one


has a maximum (zmax = 8) when x = 1, y = -2; the other has a minimum
(zmn=-2Jwhenx=-l,y=-2, at points of the circle (x-I)2+(y+2J 2 _.:25,
each of these funcbons has a boundary ext-emum (z =3). Hint. The functions mentioned in the answer are explicitly defined by the equalities

Answers

441

z=3 23-(x-l)2-(y+2)2 and consequently exist only inside and on

the boundary of the circle (x-l)2 + (y + 2l =: 23, at the points of which both
functions assume the value z =3. This value is the least for the first function
and is the greatest for the second. 2020. One of the functions defined by the
2) for x =-= - 1, Y = 2. the other has a
equation has a rnaxim UIn (<'max
mininlum (zmin == 1) for x = - 1, Y == 2, both functions have a boundary extremUlTI
-4":: -

at the points of the curve 4x l -4y2-12x + 16y-33 = O. 2021. zmax =


for
1
x==Y=2' 2022. 2 max =S for x=l, y=2; Zmin-':-:-S for x=-l, y=-2
36

18

2023. zmin = 13 for x = 13' Y ==

12

2024. zmJx =

I3'

2 t-

V2-

7n

x == "8

for

+ ktt.

2 - V2
3n
Sn
Y= 8" kn,
zmin ~ - - 2 - for x =8" + kn,
Y=
krt. 2025. lJ m1l1 =
=-9 f~r x==-I, y==2, z=-2, umax =9 for x=l, y=-2, z=2.
2026. llm.lx --= a for x == a, y = z = 0; umin =.: c for x == y = 0 z -=-= c.
2027. umax=2.42.63 for x=2. y=4, 2=6. 2028. umax=44/27 at the P011ltS
4)
4
3' 3' 3" ; (4
3"' 3' 3"
: 3' 3'
3 ; umtn -=4 at the potnts (2,
2, 1) (2, I, 2) (I, 2, 2). 2030. a) Greatest value z=3 for x=O, y = I;
2
b) smallest value z=2 for x= 1, y=O. 2031. a) Greatest value z=---;r for
3 r 3
,;2
(T
2
.. /2
x= V 3' Y=
3: smal1t'st value z=- 3
for x= V 3'
9n

8+

( 4 4 7)

(7

4)

~-

y~;

Y3-

b) greatest value z = 1 for x =

I, !/--= 0; smallest

V3

value
n

z=- 1 for x=O, Y= I. 2032. Greatest value z=--2- for x=Y=3 (Internal maximulll); slTIallest value z=o for x===y-~O (boundary rTIlnlmum).
2033. Greatest value Z= 13 for x==2. y ==-1 (boundary maXllnUnl)~ snlallest
value 1.:.=-2 for x-=,y== 1 (internal minimum) and for x=O, y=-1 ,bounnary minirn urn). 2034. eu be. 2035.
2V,
2V, 2"1
2V. 2036. I sosceles

V - V- Vtriangle. 2037. Cube. 2038. a= Va-. V;; V; Va. 2039. M (

.
.
3
3
p
2040. Sides of the tnangle are -4 p, -4 p, and -2 . 2041.

11= nl.lh+ m 2lJ2+ n1 aYa.


In.

+ nIl + nla

parallelepi ped are

.~a_.
y

:2

In x

+ nl 2x 2 + rnat a ,
+ 111 + lila

111 1

2042. ~+1L-t-~=3. 2043. The dimensions of the

abc

,2):--,
r 3

axes of the ellipsoid. 2044.


y'=

X=::

-+. +) .

,~_,
r 3

where a, b, and c are the semi-

x=y=2~+ V 2V.

l=

~.

2045. x=

Vf '

2046. Major axis, 2a=6. minor axis. 2b=2. Hint. The square of

the distance of the point (x, y) of the ellipse from its centre (coordinate origin)
IS equal to x 2 + y2. The problem reduces to finding the extrenlum of the function
X 2 +y2 provided 5x 2 +8xy+5y 2=9. 2047. The radius of the base ot the cylinder

Answers

442

is

2+

;5

the altitude R

2-

;5'

where R is the radius of the

spkere. 2048. The channel must connect the point of the parabola
with

the

point

of

204ft. 11 Y2730. 2050.

th~

straight

s~n~=~.

line

~,

-:

Hint. Obviously,

(~, ~)

1; its length is !.-~2=

t~e

point M, at which

SlD p
v2
4
the ray passes frem one medium into the other, must lie between AI and B l ;

AM =_a_, 8M =~, AIM =a tan a, 8 1 M =b tan

p. The duration of motion


cos a
cos p
of the ray is __
a_+ ~. The problem reduces to finding the minimum
v 1 cos a 2 cos p
of the functien f(a, ~)= __
a_+~providedthat
a tan a+b tan~:=:c.
0lCOS a
v 2 cos P
2051.

a=~.

2052. 11 :1.:/.=

~1: ~. : ~.'

Hint. Find the minimum of the

function f(/ 1 , 12 , 1.)=/~Rl'+/:R2+ I:R. provided that 11 +/ 2 +1.=7- 1.


2053. The isolated point (0, 0). 2054. Cusp of second kind (0, 0). 2055. Taenode (0, 0). 2056. Isolated point (0, 0). 2057. Node (0, 0). 2058. Cusp of first
kind (0, 0). 2059. Node (0,0).2060. Node (0, 0). 2061. Origin is isolated point
if a > b; it is a cusp of the first kind if a = b, and a node if a < b.
2062. If among the quantities a, b, and c, none are equal, then the curve
does not have any singular points. If a === b < c, then A (a, 0) IS an isolated
point; if a < b == c, then B (b, 0) IS a node; if a === b = c, then A (a, 0) is a cusp
2
of the first kind. 2063. Y= x. 2064. y2=:.2px. 2065. y-=- R. 2066. x / a +

+ y'/.=l'!a.

2067.

XY=~S.

2068. A pair of conjugate equilateral hyperbolas,

whose equations, if the axes of symmetry of the ellipses are taken as the
coordinate axes, have the form xy =

2~

. 2069. a) The discri minant curve

y=O is the locus Qf pOints of inflection and of the envelope of the givell
family; b) the discriminant curve y = 0 is the locus of cusps and of the envelopE
of the fain i ly; c) the discriminant curve y == 0 is the locus of cusps and is not an en
velope; d) the discriminant curve deconlposes into the straight hnes: x == 0 (locu~
2
00
1
2
of nodes) and x=a(envelope). 2070.y= 2 _gx .2071. 7 .2072. y'9+4n 2 ,
3
g 20:
..r- t
In 10
2073. r 3 (e -1). 2074. 42. 2075. 5. 2076.
X o -l- ze 2077.
11 -1- -92079. a) Straight line; b) parabola; c) ell ipse; d) hyperbola.
2)

ad~O; 3)~ao+add~0.

2081.

2080. I)

'1n aO

:t(abC)=(~~bC )+(ad~c)+(ab~~)

2082. 4t (t 2 + 1). 2083. x = 3 cos t; Y = 4 sin t (ell ipse); for t =0, !' = 4j, tv =. -31; fo
1t
3 Y2
. r-3 y~r
-.
rt
t=4,t1==--2- 1+ 2 r 2J, w=--2-1-2 Y2J;fort="2 ,t1=-31, w==
= -4}. 2084. x == 2 cos t, Y = 2 sin t, Z = 3t (screw-line); !' = - 21 sin t -t
+ 2jcos t-I-3k; v= yf3 for any t; w=-2icost-2jsint; w-=2for any t
for

t=O,

'D=2j+3k,

tv =-21;

[or

t=~,

'D=-2i+3k,

tv=-2J

Answe,~

443

rot

2085. X= cos a cos rot; y = sin a cos rot; z = sin rot (circle); f1 = cos a sin rot -- oo} sin a sin rot rok cos rot; v = I ro I; tv = - ro 2i cos a cos rot - ro 2} sin a cos rot -

-w2ksinrot; w=ro 2 2086.

v=VV~+Vy2
+(v x -gt)2; wx=wy=O;wz=-g;
o a
0

w=g. 2088. 00 Ya + hI, where 00= ~t is the angular speed of rotatien of the
l

:screw. 2089. Va 2u>2+11;-2aoovoslnoot. 2090. 'f=

yet

(i-k).

2091. 'f=

J-3- cos t - sin t) l +

~2(l+k);

(sin t

+ cos t)j +

=-J2(Siut+Cost)l+(Slnt-cost)}); cos ('f-:'Z) =


2092

= -4i-f-5J-8k.

=.i+4j+2k.

y2f'

't

R=
'II

yi05

'Y=-};

~3;

-2i+k

'11

cos('Y~)=O.

2093

Y5

k);

!J=

x-acost =
-a sin t

_y-asint __ z-bt
(t
t)o
x-acost y-asint z-bt (b'
I).
- acost - b
angen,
b sin t
-beast
a
lnorma,
x-a cos t = y-a sin
t =z-bt
I Th e dtrec tIon cosines 0 f
t
0 - (pnnclpa I norma).
cos t
sin
a sin t
t
the tangent are cos a = - y 2
; cos ~ = ya cos
; cos Y= ~r 2b l .
a + b2
a2 + b2
f a +b
The direction cosines of the principal normal are cos a l =cos t; cos ~l = sin t;
cos '\'1 = O. 2094. 2x-z = 0 (normal plane); y-I = 0 (osculating plane);
x-2 y-4 z-8
x+2z-5=0 (rectifying plane). 2095. -1-=-4-=12 (tangent); x+
+4y+12z-114=O (normal plane); 12x-6y+z-8=0 (oscutating plane).
0

t2

y-a-t '

t2

tf.

t~

t3
Y2096. -t2- = - t - = - I - (tangent); t 3 +2t = I - tol

x- 4

x-T

z-2
t3

t"

(bJJ10rmV~~ cos a l =

z-T R
_

Y2

Yi";

(binormal);

y+2 z-2
=-=1=-2-

cos ~I=

Y2 '

MJ

({-,

-t

(prin-

-+,

~);

(tan~ent);

(principal normal);

x+y=O (osculatx-2 y+2 z-2

+1=-1-=-0-

R
x-2'

Y-7[
COSY2=Q. 2098. a) -2-= - 0 - ==

..;x-I y-I z-2


(tangent); x y 2 -z= 0 (normal plane); b) -1-= -1-= -4-

(tangent); x+U+ 4z- 10 =O (normal plane); c)

= -2t

t2
z--

x-- y-4
cipal normal); -1-- = -2}=~
8)
x-2
M 2 ( 4, -3"' 2 2097. -1x-2 y+2
z-2
ing plane); -I-=--=t= - I

z-2

x~,.~ u-2 V3 =

z-:

1
2 r a
-2 3
(tangent); 2 Va x+y-2 y3i=O (normal plane); 2099. x+y=O. 2100. xy-z V2-.=O.
2101. a) 4x-y-z-9=0;
b) 9x- 6U+ 2z- 18=0;
c) b2 x:x-a 2 y:u + (a 2 _b 2 )
= a2 b2 (a 2 -b!). 2102. 6.Y-8y-z + a = 0 (osculat_

ing plane);

z:z

x-I

y-I

z-I

31 =26= -22

(principal normal);

x-I

-6 =

y-l

z-1
-S-==-r

Answers

444

(binormal). 2103. bx-z -= 0 (osculating plane); ; :

g' } (principal

+bz = O,} (bmormal);"


.
i + bk
- bl + k
= y
; p = y--;
Y= O
Itb
l+b
2

+ 3y + 19z-27 =
b) K

--

= T = 2a C~Shlt'
av l

a2 +bl

2111.

2109. a) R = Q =

(y~a)1

1 ,/'19

= 31

; T

K=2,
22

e- t

(P~;:~~)'

; b) R = Q =

t=O,

K=7 JI 14'

t=l,

2106. 2x

Y2

e- t

2108. a) K =

When

2112.

when

w n =2;

yEf

T.

O. 2107. a) Y2; b)

J.

'V

normal);

w-,;=O,

w'1;= Y14'

wn =2

2117. 50.4. 2118.

T.

,/19

JI

14

Chapter VII
2
25
2113. 4"3. 2114. In 24

2120.

~.

11:

4".

2115. 12' 2116.

na 2

2119. 2.4.

2121. x=y; -I; x=2-y; y=-6; y=2. 2122. y=xl ; y=x+9;

x= I; x=3.

2123. y=x; Y= IO-x;

y=O; y=4. 2124.

y=3; y=2x;

x= I; x=3. 2125. y=O; Y= Y25-x 2 ; x=O; x=3. 2126. y=x 2 ; y=x+2;


1

x=2.

x=-I;

o
I

Sdy Sf (x,

2128.

o
1

Sdx Sf (x,

y) dy+

Sdx Sf(x,
J-y

Sdx S f (x, y) dy.

tx+lt

Sdx S f(x,

y) dx =

0
I

2130.

y) dy.

Sdy S f (x,

y) dy. 2129.

2-X

0
1

Sdx Sf (x,
0

y)dy=

IX

.!L.

y) dx=

y)dx=

y.

Sdy Sf(x,

2127.

Sdy Sf(x,
2

y) dx+

Sdy Sf(x, y) dx+ Sdy S f(x, y)dx.


4

II

2131.

_ YI

- y
111-

Sdx S
o

V 2-

x"

f (x, y) dy. 2132.

y2

Sdy S f (x, y) dx + S dy S
o

VI

yl

S dx Sf (x, y) dy= Sdy


-I

IX 2

'V 2 -

X2

S dx S

f (x, y) dx=

f (x, y) dy+

- X

V~

..

-j-

-v f

f(x,

y) dx.

Answers
-

V4="X2

445

V1=Xi

V~ - r

2133. S dx S
{(x, y)dy+ S dx S
{(x,y) dy+ S dx S f(x.gldy+
- 2 _ V 4 - x2
- V 4 - x2
-]
y, - t'2
- 1
J
Yi"'=X2
-1
V4 _y2
1-~

t- SdX

S
- y4

{(x, y)dy= S dy
- X2

{(x, y)dx+ S dy

1/4 IJ2
1/4 - l/~

'V e -

2134. S dx

t SdX

JI"5

JI e -

V, -

2135.

V CJ2

y2

S dx S
- a _

1+

xJ

5dy 5
y + 2a

e) S dy

u Y

5dy
o

f(x, lJ)dll-=

-. x.l

5dx 5{(x, y) dy= S

{(x, y)dx; d)

-J

VI4';i2

dy S {(x, y)dx;

--I-I

{lx, y) dx= S dx

{If
S

fa

{lX, y) dy

{(x, y) dx. 2137. S dy

3...

aa

Vu

- y2

S
VU~-Jay

{(x, y) dx+

Va

JL
a

2 - y2

Sdy 5
a

IQ

f (x. y) dx.

f(x. y)dy.

x- 10

Sf (x, y) dx +S dy S f(x.
a

+5dx 5f (x, y) dy +S tlxS

O!!..

12

2138. S dy

VX

2"

1 ] 1X - x 2

{(x, y) dA; c) SdX

1 -

{(X, y) dy-J-

VJ:4iii

1/'J

- J/J

- ~ .a 2 - lJ2

V. _ yl

1 - X I I -

V a2 - 1/2

{(x, y) dy= S dy

V CJ2

Sdx S {(x, y) dy= SdY S {(x. y) dx~

a)

V.Y2"'=1
S

x2

{(x, y) dx+ S dy

V;;;-:-i

2136.

y) dx+
II',

- 1.1 2

S dy S
1

f (x,

{(x, y)dx.

{(x,y)dy+

lJ2

lJ2

dy

1/ I

5 5

S dy
J

{(x, y) dx +

t- SdY S
_. Vi V y 2

tIL.

1 r 1 + ..(2

- JI i - V 9

V e - lP

y),

-Vy2-1

-I

{(x, y)dy=

f (X,

_11 2

x2

V;-:-xi

_
- 1

b)

{(x,y)dy+ S dx

Ye-x 2

J! 1 +

_ Ve .- x 2

- a

X2

11 2
y2

+ S dy S f (x, y) dx + S dy
-] vJ=Yi
]
- y,
-

f(x.y)d+

Jl 4 V4 -

y)

dx..

Answers

444
(binormal). 2103.

bX-J -

O,} (binormal);
.

x + bz =

= 2a cosh1t 2109. a) R =
av l

a2 +bi .

=2;

2112.

rl+b

yEf

1(=+

2106. 2x +

Y2

e- t

(y + a)2

t = 0,

-V:;,

w~=

; T

Bpcx:'

K = 2,

;14'

e- t

= 3

(pC + 2xl )'

Q = - a - ; b) R = Q =

When

t= I,

when

} (principal normal);

T 2108. a) K =

2107. a) Y2; b)

b) K = T

wn

rl+b

--

+ 3y + 19z-27 = O.

;:~'

..Ir + bk 2 ; p= ..-r bi + k2 ; \' = J.

l' =

Y= O

2111.

0 (osculating plane);

w-,;= 0,

wn =2

-V~;.

Chapter VII
2
2113. 43". 2114.
2120.

2115. 12

4 .

2116.

na2

T.

2117. 50.4. 2118.

2119.2.4.

yl.
2121. x=4- 1, x=2-y; y=-6; y=2. 2122. y=x l ; y=x+9;

6.

x=l;

25

1"24

x=3.

2123. y=x; y=10-x;

y=O; y=4. 2124. Y=3;

y=2x;

x=l; x=3. 2125. y=O; Y= Y25-x 2; x=O; x=3. 2126. y=x2 ; y=x+2;
1

x=- 1;

x=2.

2128.
1

2-y

S dy S f (x, y) dx= S dx S f(x, y) dy. 2129.

S dy

S f (x,

y"

2-

y) dx =

IX + I

S f(x,y)dy.

=SdXSf(X,y)dy+SdX
0

y) dy.

o
1

S dy S f (x. y) dx= S dx S f(x,

2127.

o
I

SdX S f(x,y)dy=

2130.

2X

.JL
4

=SdySf(X, y)dx+SdySf(X, y)dX+SdY S f(x,y)dx.


2

2131.

Sdy S f(x, y) dx+l'"f dy YIY'


o

+ S dx
o

Y2

S
~

- y

_ y

f (x,

y)

dy. 2132.

2 _

f(x, y) dx=

Sdx YIX'f(x,y)dY+

yl

- ]

S dx S f (x, y; dy = S dy
-1

Ix l

-{

+
f (x, y) dx.

.-j-

-V

Answers
Y~

-I

2133.

5 v"5dx

+ 5dX

dx

5 5

f(x, g)dg+

y~

-Y~

x2

2 _

dg

2134.

dx

dx

JI 8

11a 2 - x 2

S dy

- a _ Va
I

f (x,

V .. -

S dy
_1/ 2

11.. _ yl

J -

e)

5dy
o

y+ 2a

X I I -

f (x, g) dx; c) S dx
y~

x2

{(x, lJ)dll

S
11 x --

5f (x, y) dy = S

-I

a::

x~
1/

S dx

VX

dy S '(x, y) dx;

--1-1

fa

aa

S f (x, y) dx = S dx S f (x, y) dy + S dx S f (x, y) dg + S tlxS 1(x, y)dU.

48 II

2136. S dy
o

.!!.
2
2138. S dy

VUl

S
Vu~ -

la x - la

f (x, y) dx. 2137. S dy S f (x, y) dx + S dy S f (x, y) tU~

S
.!!..

O.!!..

2.1L

12

f(x,g)dg-J,..

S dx S f (x, gl dg= SdY S f (x, g) dx:

{(x. y) dx; d)

YY2="i

fIX, y)dx+ S dg
y2

V a 2 - '/~

I-V~

1/). ttL

y) dx+

V,

f (x,

- lJ 2

11 2

a)

y) dg = S dy
S
- V al
- a

1/ 2

2135.

x
+ Vt:4ij2
2 -

y" _ y2

f (x, g) dg +

- V9

f (x,

S dx S

y2

-V'y2-l

S f (x, y) dx.
Vii'2=J

1 r 1 + x2

S dg

114.

dx

- V,"

- 1/ 2

f(x.!I)d~ +

JI" _ y2

f (x, g) dx + S dg

-1

f (x, g) dy =

f(x.gldg+

t'2

J! I + x 2

+S

x2

1
2

_ y2

Sdg S f(x, y) dx+ S dy


_. Jli JI y2 - 1
- 1
- 'V.. JI i
JI.. - lJ2
1

b)

f (x, y) dg
-_

V;-:-xi

- 1

1-

S
'V It

Ye-x 2

Y8="Xi

'V r

J-Y1='Yi

'1 2

S
VI

dx

f (x, g) dx+ S dg

11" -

5 5
-

V" _. lJ2

f(x,g) dg+

V'" _y2

~-r

'V" - x 2

f (x, g) dy= S dg

- 'V4=X2

445

y1.

.E..

f (x, g) dx + dy
2ay

Ya

2 - y2

'lx, y) dx.

446

Answers
aY""'I
a

S dy S1 (x, y) dx + 5 dy S

2139.

a
2140.

a'V a

Jla;-::y;

a-

a-l'a 2

S
~

y) dx.

2a

1(x, y) dx+ Sdy

S dy
o

f (X,

_y2

JI"ia 2a

1(x, y) dx+ S dy S 1 (x, y) dx.

S
a+V a 2 _ y 2

4a

ta
J

'V 1 - x 2

S dx S

2141.

-I

Y"-;:-

'V'-

dxSI(X,y)dY+

.!...

1-

1(x, y) dy+ SdX

dx

RV"I

Y"i'=X2

1/ R2 _ y2

- 2-

5 5

f(x, y)dy. 2143.

VI

f (x,

y) dx. 2145.

5dx S 1(x, y)dy+

2142.

5 5

'V

-;:

S I(x, y)dy.

dy

I(x,y)dx.

n - arc sin y

5dy 5

2144.

arc

2149. 6.

2150.

~ln

2146.

21 5.
2

a)

2.

2151.

In 2
I

Y2 P5 .
-21-

2153. 8

~nRI.

5dx S

2154.

Hint.

55

13t

R (I-cos t) dt

i-.

2148.

(I -

b) 15n - 16
150

3";

) 2 2

XYdy=i

~ a Y2a.

2155.

231R

ydxdy=

(8)

~ a.

2147.

Vl-(X -2)2

2156.

Y=f (t')

5 5
dx

y dy=

cos t)

y dy, where the last integral Is obtained from

the preceding one by the substitution x= R (t-SiD t). 2157.

R
80

"61 .

2158.

3t
4

2159.

2160.

5 S rl
cos q>

dq>

(r cos '1', r

sin '1') dr+

o
n

'2

sin q>

+S dlp
n

5
0

.:!..

5
4.

r/(rcos<p, rsluijl)dr.

2161.

dlp

cos q>

S rl (r
0

dr,

Answers

2162.

441

'31

4.

sin cp

4.

5 5
dcp

rf (r cos cp, r sin cp) dr.


1

sin cp

+S f (tan cp) dcp


-n"

C8,2

S r dr + Sf (tan cp) dcp S r dr.


an

4)

S dcp S
n

S r dr+

(tan lp) dcp

sin cp
cp

aY~

4.

ces 2 cp

an

2164.

5f

2163.

. sin cp

uta'V~

rf (r cos cp, r sin cp) dr +

5d cp S
an

rt (r cos cp, r sin q dr.

4.

31:

a cos cp

5dcp 5
2168. (~+ i )

2166.

2165.

2171.

32

Y2 -20) ~
( ~_16
3
9
2

2170.

2169.

ai,

2167.

nab. Hint. The Jacobian is 1= ubr. The limits of integration are


c

f\

l+fJ

o.;;;; III E;;; 2rr,

0.;;;; r E;;; I. 2172.

I-V

S dv S f (u -

uv,

uv) u duo

Solution.

We

l+a

have x = u (1- v) and y === uv; the Jacobian is 1= u. We define the


limits u as functions of v: when x=O, u (l-v)=O, whence u=O
(since 1- v

i:. 0); when x=c,

= - c1 .

-v

of variation of v:

Linlits

since

y=ax, it follo\\s that llv=au (I-v), whence v=-1


a ; for y=px we find

+a

2173.

+ ~ d<
+ ~(~ dv

t>

2 -V

5f
II

(U

v,

I=~ (~ du ~t f (U~V,

2V) dV] = ~ ( _~ dv

f (U

~ v,

u2

V) dv +

U2V) du +

(u+V
u2 V) dU.] Hint. After change of variables, the equa-2-'

tions of the sides of the square will be u=v; u+v=2; u-v=2; u=-v.

2174. ab [(~- ~)arc tan

~~l +::].

Solution. The equation of the curve

448

Answers

r4 .. r' (:: cosacp- :: slna cp). whence the lower limit for r will be 0 and

Y ::

.ahe upper limit. r=

cosl cp- :: sin l cp. Since r must be

real. It

b2

a2

follows that hI cos 2 q> -Ji2 sin l q> ~ 0; whence for the first quadrantal angle
ak
we have tan q> ~ bh . Due to symmetry of the region of integration relative

4"1 of the entire integral, confining ourselves

10 lhe axel, we can compute

.. /~ a~

ak

arc tan bii

10 Ihe ftrst

SS dx dy = 4

Quadrant:

(S)

~171 ) 4~;

Sdy
o

.178

~183.

16 ,r-

"3

2180.

{naa 2184. 6.

b+4,1=u.
tll8. v =

15.

IOn.

Hint.

J&

2179 n

I
H nt.

4:t

3- ..r-;f 3 .

Change the variables

x-2y = u,

n;a.

2193.

2194.}. 2195.

mt

as

a-

l i b
"3(b-a)(~-a).
2187.
3(~-a)lna.

10
"3 a I .

(' n
1)
3\ 4+2 . 2182.

SdYSO-X) t!x= SdX 0 - X)dy.


o

17
2 8.

7a
120

2181.

2185.

2186.

217 7.

aya;-::.-;z

~I_a;; SdX S dt/.

b)

Y- 2

(2 +4n) a.

) 29.' b)

-l<x<;l.

Vy

abr dr.

S dx+ S dy S dx;
_

ki sln2 cp

dcp

Yg

b2

cP -

COS2

V II

hi

48

V6

88

al

abc

"3. 2197.4Q. 2198. - 5 - 2199. 105 2200. 18 . 2201. T


2202.
na' (a-~).
2203.
34 nal (2 ,rf 2 -1).
2204.
'34 real (r.. r-2 -1).

2196.

2205.

na'
T

2209. JUI(l-'-

tllnt.

Change

2206.
RI

na
3

4
3 nabc .

2207.

variables

lr-;rf

3 -5).

2208.

32

"9 a3

va2
V2 lr-2
.2212. -2-(2 r 2 -J).

3xab
3
). 2210. -2-. 2211.

the

(6

xy=u. ~ =v.

2213.

~ Valbl+blcJ+clal

Y2

2214. 4 (m-n) RI. 2215. -2- a2 Hint. Integrafe in the uz-plane. 2216. 4a l .
b
1
,r2217. Sallfe alna-. 2218. "'3 nal (3 , 3 -I).
Pus to polar coordinates. 2221.

0=

:rra 2

[(

2219.
I

8a2

2220.

+ ~: )';- -1 ].

3M2

Hint.

Hint. Pass to

polar coordinates.

~ a'

2222.

Y52

Hint.

449

aaa.

Hint. Pass to polar coordinates.

and

a
2223. 8a l arc tan

Answers

0'=5' dx5! y a -x
a dy
i-Sa 51 arc sin 2 .. r~~.
-Y
r a -x
2

Integrate by parts, and then change the variable x= a

~ 3 sin to

lransform

31 (b Yb 2 +cl -a a l +c1+c l In b+ ~~) .


Hint.
4
a + al+cl
2
.
2nfJR
a'b a2b~
12-312
Pass to polar coordInates 2225. -3-. 2226 .12; 2T' 2227. x=3(4_n);

the answer. 2224

31
5
2~ sin a 2
Y 6(4-31) 2228. x=1fa; y=O. 2229. x==3<l; y=O. 2230. ~==5;
n
n
y =0.
2231.
Ix=4
2232. a) I O Z:::
(D4_d 4 ); b) IX==64 (DC-dt ).
32

'Vax

tJ

8
5 aC

2234.

161n 2-9

2235.

Hint.

1-

5 5
dx

i.

Hint.

(y+a)1 dg.

-JI'ax

Th~ distance of the point (x, y) from the

straight lin.

x=y is equal to d=V-2~ and is found by means of the normal equation

of the straight line. 2236. l=~ ka 5 [7 Y 2 +3 In (Y2 + I)), where k is the


proportional ity factor. Hint. Placi ng the coordinate origin at the vertex, the
distance froln which is proportional to the density of the lamina, we direcf
the coordillate axes along the sides of the square. The mOlllent of inertia i5
detennlned relative to the x-axis Passing to polar coordinates, \\'e have
n

a sec cp

I" =

J J

kr (r sin <p)! r dr

d<p

a cosec cp

+~

dq>

nac
35
2238. 10 =2' 2239. 12nac. Hint. For the variables of infeiratlon take Land
1

II (see Problem 2156). 2240. \ dx


R

2241.

2242.

Jl'RI_X:il

dx
b

__

-a-JI'a 2 -xl

dy

I (x,

5 5 5f

5 S

0 0 '

dx
dll
-R -V'RI-x1
b
_ _0
y
(J
G a2 - x 2
-a

l-X-y

I-X

dy

(x, y, ,) d,.

f (Je, 1/, .) d.

II. ,) da

Answers

450

Sdx S
-x2

-1

5
2"1 In 2- 16

=25 dx

X2

dy

la cos q>

't'

.. r -

0
3t

=.!a

cos cp

2a

4
="3'

4)
n- '3
[1=

la

rdr 5 dh =

60 -:rna.
3
3 SIr
22.
e u Ion.

S(2aCOSljl)4 dm=!nal.

4
226.

x = 0; y
x

8 r8
22 56 ."3

2261.

't'

2na

V2

. Hint. Pass

aI (3jt-).
4
2263. "9

a.2

xtabc'

nR. 2251. -4-

,2

S dz=2 5dq> S
~9 n.

to spherical coordinates. 2262.

2268.

8
2255. 9

32
2h
ga.

22 59.
n

2a

59

2250. 480

1
2247. 720

:llal

2246.

r'dr
2a

2267.

97)

-"6

n Ra

2254

Y2

4xt

-3-

2245.

18 f 3

xt
2258.
10.
x2 +/p

3t

--25 dm

5na

'V 2QX -

2a

2249.

RI
2253. xth
-4-

4 31 RI .
15

2257.

z) dz.

.. r .. rf 2 -27 f 3).

+ 12

4
bc.
2252. 5111

f (x, y,

dy

- 'VI

8
2244. 15 (31
2248.

YI-x2 _y2

Yi'='Xi

2243.

= 0;
-

rcabc.

z=
-

22 65.

"52 a.

Hint. Pass to cylindrical coordinates.


a be (a+ b -I-c.
)
T

Hint.

y =0, z =0. 2269.

Introduce

ab (6c 2 -a 2 - b2 )
22 66 24

spherical

coordinates.

na h
12
(3a 2 +4h 2 ).

Hint. For the axis of

the cylinder we t~ke the z-axis, for the plane of the base of the cylinder,
the xy-plane The moment of inertia is computed about the x-axis. After
passing to cylindrical coordinates, the square of the distance of an elelnent
r dip dr dz from the x-axis is equal to r a sin a <p

+za.

2270.

n~~al (2h Z +3a2).

Hint. The base of the cone is taken for the xy-plane, the axis of the cone,
for the z-axis. The filoment of inertia is computed about the x-axis. Passing
to cylindrical coordinates, we have for points of the surface of the cone:
r

=f

(11- z); and the square of the distance of the element r dq> dr dz fronl

the x-axis is equal to r2 sin 2 <p+z2. 2271. 2rtkQh(l-cosa), \vhere k is the


proportionality factor and Q is the density. Solution. The vertex of the cone
is tal<en for the coordinate origin and its axis is the z-axis. If we introduce
spherical coordinates, the equation of the lateral surface of the cone will be

'" = ~2 -

a, and the

equation of the plane of the base will be r

=--/!:- .
Sin '"

From the symmetry it follows that the resulting stress is directed along the
z-axis. The nlass of an element of volume dm=Qr 2 cos", dcp d", dr, where Q
is the density. The component of attraction, along the z-axis, by this element
of unit mass lying at the pOint 0 is equal to k

~m

sin "I' = kQ sIn", cos '1' d'1' dq> dr.

Answers
n
'2 - a

2n

The resulting attraction is equal to

451

5 5
dlp

h cosec

11'

d'ljl

kQ sin 'Ijl cos 'Ijl dr.

0 0 0

2272. Solution. We introduce cylindrical coordinates (Q, cp, z) with origin

at the centre of the sphere and with the z-axis passing through a material
point whose mass we assume equal to m. We denote by ~ the distance of
this point from the centre of the sphere. Let r= Y QI+(;-Z)2 be the distance from the element of volume dv to the mass m. The attractive force of
the element of volume dv of the sphere and the material point m is directed
along r and is numerically equal to -kym

~.

where y= ~ is the

-1tR8

density of the sphere and dv = Q dcp dQ dz is the elelnent of volume. The projection of this force on the z-axis is
kmydv
A
E- z
dF = - - - I- cos (rz) = - ktny _. -3- Q dq> dQ dz.
r
r
Whence
R

231

F=-kmy5 d lp

YRJ - zJ

S(~-z)dz
-R

Qr~Q=kmy4n:RI~~.
ex>

4 ynRI:.=.:: M, it follows that


But since "3

kMm

F=~. 2273. -

Se

y2 -XJ

dy _e- X3 .

2275. a)

2276.

-.!.
P

(p> 0); b) _1_ for p


p-a

_..!...
n
2

a; c)

2277.!.
Hint. DilJerentiate
p8
~

a
m

2279. arc tan--arctan-.

2282. arc cot

>

a
If

2283. 1.

polar coordinates. 2287.

r
J

(p

e- pl dt

> 0); d) 2,P


P

=...!..P

R2

-t"'

twice.

(p> 0)

2278. In

~.

2280. -2 In(l+a).

2284.

~"it.

z~

RZ
PI-I-'

2"

2288.

2285.

1.

2286. 4a 2

HInt. Pass to

~z 2289. Converges. Solution. Eliminate

from S the coordinate origin together with its e-neighbourhood, that is,
consider II = ~ ~ In V Xl + yZ dx dy. where the eliminated region is a circle 01
(8 )

radius e with c~ntre at the origin.


ut:

1.=

,n;

Pa5sing to polar coordinates, we have


1

S eSr Inr dr= 5[r~ In r /; -~ eSrdr]


dlp

dlp= 2n:

(~-~

In

e-f)

Whence lim Ii =- - ~. 2290. Converges for a > 1. 2291. Con,oerges. Hint. Sur8-.0
2
XdY
round the straight line g=x with a narrow strip and put S\~
Z
(5)

(x-g)

Answers

452

x-e

== lim
8-+0

> "2 .

2297.

6-+oJ

.!]

-1). 2301.

+b

. 2298.

ab

dy

V(X-y)2

Y-52 + 3 .
In

2294.

(1 +411 2)1 -1

x+~

2293. O.

+ linl r dx

(X-y)1

Sdx Jr V dy

arc tan

21Tb

a . 2302.

ab (a l

+ ab + b

3 (a + b)

2295.

Y5m
1+ m

a"

Converges

2292.

2296.

.2299. a 2
2

231a. 2303.

Y2.

2300.

54 (56

for

2Cl6 a

15 a

Y7-.

16
.. r27
(10 y 10-1).

Hint.

~ f (x. y) ds may be Interpreted geometrically as the area of a cylindrical sure


face with generatrix parallel to the z-axis, with base, the contour of integra
tion, and with altitudes equal to the values of the integrand. Therefore,
S = ~ x ds. where C is the arc OA of the parabola y= ~- xl that connects the

points (0, 0) and (4,6). 2304. a


2306. Y a2+b 2

--

f;

2312. a)

Y3.

2305. 2

b) 0; c)

21rb

-2

kMmb

2310.40

O. 2318.

b) 12; c)

a) 8;

(ia.; a)-

2307.

19

30 ,

2311. -2na

d) -4; e) 4. 2313. In all cases 4. 2314. -2n. Hint.

Use the parametric equations of a circle. 2315.


2317.

+ V"-:-2-+-4-n b-2).

2309. Y(at+bl)I'

~;

(n Ya2+4nb2+~ln

2na Ya 2 +b 2

2308.

(b + .r aa b-b arc sin YQi=bi)


.
a
2

2; d)

~;

ab 2

2316. -2 sin 2.
%2

~ q> (x) dx +

e) In (x+ y); f)

XI

liz

+~""(Y)dY.

2319.

a) 62; b) I; C){+ln2; d) 1+Y2. 2320. YI+a 2_

YI_-

- V 1 + b2

2322.
c) eX-Y(x+y)+C; d)
2325.

a)

(i+ n 1~2) RI.

= SSy2 dx dy.

2328. -

2329.

n:

b) xl -x2y xy2_ yl C;
2323. -2na(a+b). 2324. -nR2cos2a

2326. a) -20; b) abc-I; c) 5

~.

x2+ 3xy-2y 2 C;

Inlx+yl+C.

V2;

d) O. 2327. 1=

t
.

2330. - { .

2331. O.

2332.

a)

0;

(5)

b) 2nn. Hint In Case (b). Green's formula is used in the region between the
contour C and a circle of sufficiently small radiuc;; with centre at the coordinate origin 2333. Solution. I f we consider that the direction of the tangent
coincides with that of positive circulation of the con1our, then cos (X. n) =

= cos (Y.

t)=1!-.S
hence. 1 cos (x.
ds= 11t ds= 1 dy=O
ec s
c
n)

2334. 28. where

S is the area bounded by the contour C. 2335. -4. Hint. Green's formula is
not applicable. 2336. nab. 2337.

~ nat,

2338. 6nat

2339.

a l Hint. Put

Answers

453"

y = tx, where t is a para meter. 2340.

a
00.

2341. n (R

+ r) (R +2r);

6nR2 for

R=r Hint. The equation of an epicycloid is of the form x=(R+r)cost_ r cos R +r t, y= (R + r) sin t-r sin R +r t, where t is the angle of turn 01
,
r
the

radius of

a sta tionary

circle

drawn

to

the

point of

tangency.

11 (R - r) (R - 2r).
llR2 for r = ~ Hint. The equation of the hypocycloid is obtained from the equation of the corresponding epicycloid (see
Problem 2341) by replacing r by - r
2343. FR.
2344. mg (Z.-2 2 ).

2342.

2345.

(a l -b2 ). where k

is a proportionality factor. 2346. a) Potential,

U =1:... work.
~l
~
r
a2 + b2 +C2
k
k
8
c) potential, U=-T(X 2+y2+ Z2), work, 2" (R 2- r l ).
2347. 3 M .

U =mgz.

mg (Zl-ZI);

work.

b)

potential.

V3a" + b

2na2

2348.

25

2349. O.

V5 + 1

V2

(y-- _ ) a. 2354. - 2 - h

2353.

10 5

2356. O. 2357.

iJP
= iJy 2361. O. 2362. 2

-lla2

2361.

.1. 2

2366.

2.

2367.

12

2360.

~: = ~~ .

iJ U)
Jr~5 (dox2U + iJ2U
oyl + OZ2 dxdydz.
,

as

2359. -al .

rr
J J (x + y +z) dx dy dz.
(V)

(V)

2365. 3a4

2352.

rr (cos a + cos p +
JJ

2355. a) 0, b) -

2358.

411.

rr V d1C +dyy2dz+Z2'
.\J

2 .

(5)

+ cos yl dS.

nat

2351.

oP oR
oQ
iJz = ox' ox
2363.

3" nabc.

2350.

na 2 b2
2363. -2-

5" na.

2371. Spheres, cylinders.

2372. Cones. 2373. Circles, X2+y2=C~, z=c,. 2376. grad U (A)=91-3j-3k;

yo; Z2=xy;

\ grad U (A) 1= Y99 =3

I' (r) !...


r

d)

2378. grad (er)

the vector e.
ens (', r)

2379

iJU

or

au
-iJl =0

= e;
2U

x= y=z.

2377. a) !.-;
r b) 2r. c)- ,~ ;

the level surfaces are planes perpendicular to


iJU
au
a;-=/gradU/whena=b=c,
2380.

ar=

=-'-'

2 .
2382. - .
2383. dlV

a=- f (r) + f (r).


r '
2385. a) div r=3, rot r= 0; b) div (re) -= ~
, rot (re) = rx, c ; c) div (f (r) c) =
r
= -

,2

I' (r) (c,


r

r),

rot

for I

J..r.

(I (r) e) = I'

,,0

(r) ex r.
r

2386. div!'::.:: 0;

rot 'D = 2m, where

m = ook 2387. 2oon. where


is a unit vector parallel to the axis of rotation.

iJ2U o"U 02U


dlV grad U = iJ>. 2 + iJ 2 +OZ2;
rot grad U = O.
2391.
31tR 2 H.

2388.

2392. a) kllR2H (3R I +2H 2); b)

~nR2H (R2+2H2).

2393. div F=O at aU

points except the origin. The flux is equal to -4nm. Hint. When calculatine

Answers

454

the flux, use the Ostrogradsky-Gauss theorem. 2394. 2n1hl

2395.

-~R

'2396. U =

Srf (r) dr.

2397. ; .

a)

2398.

No

b) U =xyz +c;

polential;

r.,

c) U=xy+xz+yz+C. 2400. Yes.

Chapter VIII
1

11

2402. 2n 2403. 2n-1

2401. 2n-l

1
2407. n(n+l).

135
1.4.7

2408.

2404.

(2n-l)
(3n-2)

1
n2'

n +2

2405. (n

2n

+ 1)2 2406. 3n + 2 .

2409. (_I)n+l.

2410.

n+ 1
n(-I)

2416. Diverges. 2417. Converges. 2418. Diverges. 2419. Diverges. 2420. Diverges.
2421. Diverges. 2422. Diverges. 2423. Diverges. 2424. Diverges. 2425. Converges. 2426. Converges. 2427. Converges. 2428. Converges. 2429. Converges.
2430. Converges. 2431. Converges. 2432. Converges. 2433. Converges. 2434. Diverges. 2435. Diverges. 2436. Converges. 2437. Diverges. 2438. Converges.
2439. Converges. 2440. Converges. 2441. Diverges. 2442. Converges. 2443. Converges. 2444. Converges. 2445. Converges. 2446. Converges. 2447. Converges.
2448. Converges. 2449. Converges. 2450. Diverges. 2451. Converges. 2452. "-Diverges. 2453. Converges. 2454. Diverges. 2455. Diverges. 2456. Converges.
2457. Diverges. 2458. Converges. 2459. Diverges. 2460. Converges. 2461. Diverges. 2462. Converges. 2463. Diverges. 2464. Converges. 2465. Converges.
2466. Converges. 2467. Diverges. 2468. Diverges. Hint. all+ 1 > I

an

2470. Con-

verges conditionally. 2471. Converges conditionally. 2472. Converges absolutely 2473. Diverges. 2474. Converges conditionally. 2475. Converges absolutely.
2476. Converges conditionally. 2477. Converges absolutely. 2478. Converges
absolutely. 2479. Diverges. 2480. Converges absolutely. 2481. Converges conditionally. 2482. Converges absolutely. 2484. a) Diverges; b) converges absolutely; c) diverges; d) converges conditionally. Hint. In exaillpies (a) and (d)
00

consider the series ~ (a 2k-l +a2k)

and

in examples (b) and (c) investigate

k=1
00

00

-separately the series ~ a2k-l and ~ a2 k'


k=1

2485. Diverges.

2486. Converges

k=1

absolutely. 2487. Converges absolutely. 2488. Converges conditionally. 2489.


Diverges. 2490. Converges absolutely. 2491. Converges absolutely. 2492. Con(IJ
1 (_I)n
verges absolutely. 2493. Yes. 2494. No. 2495.
an ; converges. 2496.
(IJ
1
n=1
(2n-l); converges. 2497. Diverges. 2499. Converges. 2500. Converges.

L2n

n=1

1 .

an

"2501. I R4,1 < 120' I R., < 720' R4,<O, R,>O. 2502. R n < 2n+ 1 =2 n (2n
1) n!
Hint. The remainder of the series may be evaluated by means of the sum of
1
a geometric progression exceeding this remainder: Rn = an [2 _1_

+(i ) (n + 1)1(It +2) +.]


2

< all

ri ~ 1 +( ~ )
n

n+l

(n

~ 1 + .]
)1

455

Answers
2503. Rn

< (n+ n+2


I) (n+ 1)1

= __1

RIo

<

I
310-'. 2504. n+ I

1_

<

i
< Ii

Rn

Solution.
_

(n+ 1)2+ (n+2)2+ ... > (n+ I) (n + 2) + (n +2) (n+3) + ... -

_(_I
I) (I I)
I
n + 1- n + ~ + n + 2- n + 3 + ... = n + 1 ' R

<

I+ I) +

n (n

+ (n + I)1(n + 2) + .. =(iI

2505. For the given series it is easy to find the


exact value of the remainder:

Rn=fs (n+ ~~) ( }


Solution.

Rn=(n+ I) (}

We multiply by

(f

~Rn=(n+l)

rn

yn-z.

+(n+2) ({- rn+Z + ...

(f yn+z+<n+2) ({-yn+4+ ..

Whence we obtain

16 R"

(~)2n
4

+ (!.)2n
4
+ (!)2n+2
4
+ (!.)2n+.&
4
+ ... -_

_ (!)2n
_(-}
4
-t

-n
Fronl this

\ve

1-

r.

_ (

I -

n+ ~)
15

16

find the above value of R n Putting

the series S -= (

(.!.)2n
4

2506. 99; 999.

11

=0, \ve find the sunl of

2507. 2; 3; 5.

2508. S = I.

Hint.

I
I
a'J~----1 2509.8::::1 when x>O, S=-I \vhen x<O; 8==0 when
n
n-tx == O. 2510. COllvcrge~ absolutel y for x> 1, diverges for x ~ 1. 2511. Converges
absolutely for x > 1, converges conditionally for 0 < x ~ I, diverges for
x ~ O. 2512. Converges absolutely for x> e, converges conditionally for
l<x~c, diverges for x~1. 2513. -oo<x<oo. 2514. -oo<X<():).
2515. Convergls ahsolutely for x> 0, diverges for x~O. Solution. I) Ian I~

..;;;; ~x; and when x > 0 the series with general term ),; converges;

2)

)x ~ 1

for x ~ 0, and cos nx does 110t tend to zero as n ---+ 00, since from cos ,1X -~ 0
it would follow that cos 2nx -+- -1; thus, the necessary condition for convergence is violated when x ~ O. 2516. Converges absolutely when 2kn < x <
< (2k + I) n (k == 0, I, 2, ... ); at the remaining points it diverges. 2517.
Diverges everjwhere. 2518. Converges absolutely for x :1= O. 2519. x > I, x ~-l.
1
2
2520. x>3, x<l. 2521. x~l, x~-I. 2522. x~53'
x<4"3' 2523.

x> I, x < -I.

2524. - I

< x <- ;, ; < x <

of

x, bol h the series

L xfl and
k=l

the series

L 2x

1l k

k=l

I. Hint. For these values


converge.

When

I x I ;;;::= I

Answers

456

-and

when

I
'x I ~ "2'

the general term of the series does not tend to zero

O<x<l.

-1<x<O,

2525.

2526. -l<x<L
I

2527. -2~x<2.

2529.-Y2~X";;;;V2' 2530.-1<x~1.2531.-1<x<1

2528. -l<x<l

< x < 1.
< x < 4.
-3 ~ x < 3.

2532. - I

2533. -

.2536. -4

2537.

<x<

00

-3I

< x<

2535. - 00 < x < 00.


< x < 2. 2539.-e < x <e.
-I < x < I Solution. The diver-

2534. x -= O.

00.

I
3'

2538. -2

2540.
2541. - I < x < I 2542.
gence of the series for I x I ~ 1 is obvious (it is interesting, however, to note
that the divergence of the series at the end-points of the interval of convergence x= I is detected not only with the aid of the necessary condition
of convergence, but also by means of the d' Alembert test). When I x I < 1 we
have

lim
n -+ :t>

l(n+l)IX~n+I)J

1= lim

n! x

-+ '"

l(n+l)xnfn'~lim(n+I)lxln= lim n+~==O


n -+ '"
n -+ ~ ~

II

(this equality is readily obtained by means of l'Hospital's rule).


~ x ~ I Hint. USing the d' Alelnbert test, it is possible not only
to find the interval of convergence, but also to investigate the convergence
of the given series at the extremities of the interval of convergence. 2544.
-I ~ x ~ 1. Hint. USIng the Cauchy test, it is rosslble not onl y to find the
interval of convergence, but also to investigate the convergence of the given
series at the extremities of the interval of convergence. 2~45. 2 < x ~ 8.
2546. -2~ x < 8. 2547. -2 < x < 4. 2548. 1 ~ x ~ 3 2549. -4 ~ x ~-2.
2543. -I

2550. x =

2551.

- 7< x< - 3

-2~x~O.

2554. -e-3<x<e-3. 2555.


2558.

-3~x~-1
.
d'Iverges,

.
sInce

l'1m

n~C7J

)n

13

-"4 < x <"4 .

2556.2<x<4 2557.1

1--!.<x<I+J.

2559.

e I

.
series

2552. 0 ~ x < 4. 2553.

Hint. For x=1

( 1+- - nn- - = l rI eye

<x~3.

the

# 0

2560.

-2

< x<

2561. 1 < x~3 2562. 1 ~x < 5. 2563. 2~x~4. 2564. I z 1< 1 2fi6:>.' z 1< 1

2566.

I z-2t 1< 3 2567. I z 1<

V22568. z=O 2569.1 z 1< 00.2570.

I z 1< {

-In(l-x)
(-I~x<l)
2577.
In(l+x)
(-I<x~l).
1 tx
I
i-x (I x f < I) 2579. arc tan x (f x I ~ 1). 2580. (x- 1)2 (I xl < I).
l-x 2
2
x
2581. (1+x2)2(1xl<l)
2582. (l_x)I(lxl<I). 2583
(X_I)2(Ix l >I).
2576.

2578.

"2 111

2584.

I I-X)
+

1 ( arctanx-2"ln
the series x-

sum of
2586.

3.

= ~2

2587.

x'

x
x

Va
(Ixl<l). 2585. n- 6
- . Hint. Consider the

3 +5"-...

aX=l+I. xn~~na.

(see

Problem

2579)

for

-oo<x<oo. 2588. sin (

[l+X_~_~+~:~_ ... _H_l)n.:n ~+ ...].

X=

I
V3.

X+~)=

451

Answers
cos (x + a) = cos a- x sin a -

2589.

x2

x Sln
. r
-l-nr
a+ (n -t-21) nl +... ,-oo<x<00.2590.s1n
n

2:. n - J x2n
2 )' + ... , - 0 0 <x<
\ n .

... -l-'(-l)"-'

x'

x,2

-2.22 +3.2 3-"


gating

the

relnainder,

xn

n.2 n + ....

2
2t
2S 4 25 x'
x=2T-4f-l-m-

In(2+x)=ln2+--2

2591

00.

-2<x~2.

Hint. When investi-

use the theorem on integrating a power series

2x-3

3x-5

00

(X_W=-L(n+3)x".lx 1<1. 2593.

2592.
~

=-~
~

n=o
-

+{_I)n-I

x;4

Xl

2f cos a + 3f sin a +'4f cos a -f- ...

00

( 1+3 2) x
n +1

< xa<

n==o
~

n,

L(-I)n-12n-Jxn

Jxl< 1. 2594. xe-1X=x+

(n-l)l

2n

(SJ

00

<

<

00

00

n=1

~..,

2L(-ll

(n

+ 2) 32n.x21l+1
(2n+I)'

,~

2596.

I+~~

(-oo<x<oo) 2597. I-t-'''' (_I)"2"x " . 2598.


~
\2n)1
2599.

'

n=2

00. 2595. eX'::=: I -1- ~~, ' ~ n.


n=1

-oo<x<oo.

x2 -4x+3

x%n t

~(2n+I)'
n=o

(-1)"(2x)2"
(2n)'

11=1

(-oo<x<oo).

n=o

~
X 2n + 1
2600. L(-I)n gn + J
n=o

1.3.5 x'
-1- 2.4.627 1-

2602.

~x~n+,
2~
211 + I

(-3<x<3).

135 . . (2n -I) x1n


2.4.6 2ft
22n+1

(I

xI

<

I) 2603

l-2 < x < 2)

~(-1)n+12n-l.n(
x

1)

"2 < . < 2

n=1

x,"

00

x+L.<-l)n(n_l)n (lxl~I). 2605.


n=2

2606.

+ ...

. ~

n=o

2604

260).

X2n+1

QCI

L(-J)"2n+1

('x,~).

n=o

1 x' 13 x'
13.5 ... (211-1) X 2n + 1
x-f-"23+2.4S++ 2.4.6 . . 2n 2n+l+

(I.I~)

2n

1 x' I 3 X
11 13 5 . . (2n I) x + 1
2607. x- 2 3"+2.4S-+(-1)
2.4.6 ... 2n
2n+I+'" (lxl~I).
f7J
24n-1 x2n
go
1
2608.
(_1)"1'\ (2n)1
(-oo<x<oo). 2609. 1 t(-l)"-lnnl x"
n=1
n=2
00
1 2n + 3n-,
(-oo<x<oo). 2610. 8--t- 3
nl
X,n (-oo<x<oo).

n=1

2x 2
25x'
l)n-l 258 .. . (3tl-4) xn
2611.2-1-22.3.11-25.32.21+28.:33.31+ ... + (--23n-l.3~-I- t,
X

Answers

458
2612.

(-00 <X<oo).

3
L (I +(2n)1

x2n

2n - l )

"26t3. 1+"4

x
L 4n+1
4n

Q)

I<

<I x

2614.

00).

n=1

<

(-I

x..;;; I).

2616.

X2n + 1

+ I) (2n + I)!

L (_I)n X

Q)

x!n+1

(0). 2617. x+
(_I)n (2n -r- 1) ti! (lxl<oo).
n=l

<x<

( - 00

2618. L(-W+l~1

2619.

(I xl".;;; I).

I s
1.3.
+
+ 2.5
x +2 .9.2r x + ...
2

n=l

~2n-l) 4n+l +
+ 1.3.5...
2n (4n+l)n! x . . .

2x5

2621. x- 3 +15-'

(1-2"+6"-'.'
X 2 X4
)

. 2625.

x3

x+ 3

2620.

(lxl<I).

2622. e

(~+ ~+ ~+ ... )

2624. -

2).

n=o

n=1

<

< Y2).

(.I xl

a:>

n=o

26tS.ln2+L(-I)n-l(I+2- n) :

X (2n

<

(-2

2x
+U-+
...

5x4

xl

.2623.1+2"+24+.

x+x + ~ x+ ... 2626. Hint. Proceed


of the elli pse x = a cos eft Y = b sin CPt com
l

ing from the parametric equations


pute the length of the elli pse and expand the expression obtained in a series
of powers of 8. 2628. x'-2x2 -5x-2= -78+59 (x +~4)-14 (x+ 4)2
+(X+4)' (-00 <x< (0).
2629. f(x+h)=5x ' -4x2 -3x+2+
+(15x 2 -8x-3) h+ (15x-4) h2 +5h l
(-00 <x<oo; -oo<h<oo).
1n
1
2630.
(X n ) (0 <x";;;2). 2631.l:(-W(X-W (Q<x<2).
n=l
n=o

ct)

ct)

L(-WIX)

2632.

L (n+ 1) (x+ I)n

(-2

<x<

0).

2633.

n=o

L (2- n-l_3- n -

(x+4)n

n=o

Q)

2n

(-6<x<-2). 2634. L(-I)n(\-t!

(-2-3<x<-2+3).

n=o

2835. e-' [ I

+~ (x~12)n]

(I x 1<00).

-J- ~ {X-4)8 _ 135 {X-4)4 +


4.6

2'

46.8

28

(0~x~8).

2637.

+ x '}/ - ~

(x 244 )1

+ (_I)n-l I 3.5 ... (2n-3) (x-4)n +


Ql

2636. 2

~(_l)n

468 .. . 2n
x-~ yn-l
(2n-=-l)!

(Ixl <

22n

2638.

00).

2+

n=l

+ \1
n.fd (-I)
(0 < x

<

4n 1
n -

ClO).

X-T

yn-l
(2n-l)!

i,

1I x 1< 00). 2639. -2 ~

2n

+1

(I-X)2n+l
1 +x

Answers

Hint. Make the substitution

~640.

+=

4590

t and expand In x in powers of t.

+}( I :-x +~:~C ~x + ... + ~:~:~:: :~~:=~~( I :-x +'"


... ( - ; ,,;;;;; x < <Xl ) . 2641. I R I < ~ < ~. 2642. I R I < 1\ . 2643. ~ ::::::
I I (~r 1'3(~r.Hint. To prove that the error does not,
::::="2+2
I :-x

-3-+~-5-:::::0.523.

exceed 0.001, It is necessary to evaluate the reillainder by nlcans of a geo..


metric progression that exceeds this renlainder. 2644. Two ternlS, th3t is,
I

+ I: ~I

~.

I -

2645. Two terms, i. e., x-if. 2646. Eight terms, i. e . I

n=1

2647. 99; 999. 2648. 1.92 2649. 4.8 I R 1< 0.005. 2650. 2.087.2651. J x 1< 0.69;
x

1< 0.39; / x / <

2654.

0.~68.

I x 1< 0

0.22.2652./ x 1<0 39;

2655.

0.608

00

(2n

(211)!

2656.

2659.1-t-~(-1)" x-y)

0 621

18 2653. }-23 ';'3!:::::: 0.4931.

2657.

0.2505

2658.

0.026.

(-oo<x<oo; -co<y<oo).

11=1

2660.

~(_I)II(X_y)3n_(x+y)2n

(-co<x<oo; -<Xl<y<co).

2(211)'

11=1

00

( 2
X

2661. ~(_l)n-I

I-

2)'2n-1

-oo<y<oo).

(-oo<x<oo;

-!I ----(2n-l)!

11=1
oJ)

2662.1-1-2

~ (y-x)n;lx-YI<1

.
I-x+y
2
Hint. I
=-1+1 (
. Use
-J-x-y
- y-x)

tt=1

(X)

a geometric progression 2663. - "':""').'


~

+11 'I

(-1

~ x < 1;

tt=1

I: (-1)/1
JJ

Hin:. I-x-y+xy=(l-x) (i-g). 2664.

X 2n + 1

-1

2n+l

211:1

~y <

I).

(-I~";;;;;I;

n=o

-1

~y ~ 1). Hint. (lrc tan

2665.

f(x-t-h,

IX+ Y =arc tan x+arc tan y (for


-~\y

I x I ~ 1, I y J ~ 1).

y-t-k) =ax2+2bxy+cy2_r-2 (ax-j-by) h+2(bx+cy) k+ah 2

1-2bh+ck 2 2666. {(1+", 2-j-k)-f(1, 2):=9h-21k+31z 2 -t-3Izk-12k 2 +h!t1'

-2kl. 2667. 1+~,,[(x-2)+(Y+2)]


~

It'

x2 _y!

x8 -3xy2

n=l

2669.

-,-+

1+x+2

,,00

.2668.1-L~(-1)n
I

i.J

[ x + ( y - ~) 1zn
2

(2n)!

n=1

3'

+ ...

2670.

2671. c1 + c2 _ 2 (CI-C2~ ,..., sin (2/1 + l) x; S (0) = c\


2
11:
~
21l 1
n=o

-t- X -I-XY+"2

-L c2 ;
21

S ( n) =

xlJ +
(1

+.~ C2
-

Answers

-460

b-an_2lb-a)~~COS(2n+l)x+(a+b

'2672.

b-a

.s ( n)= -2- n.

n=o

nl

'3 + 4

2673.

(2n

+ 1)2

~(_l)n-I sinnx.

)~

n=1

cos nx
2
L (-1 )nfi"l
; S ( n) = nl. 2674. 1t sin h anx
CD

n=1

1 ~ ( - 1)11
]
2 sin axt
X [ 2"a+ ~a2+nl(aCosnx-nsinnx) ; S(n)=coshan. 2675. - n - X
R=I

:1"

ClIO

X ~ (_l)n n
n: if a is nonintegral; sin ax if a is an integer; S ( n) =0.
~
a-n
R=1

2 sin axt [ 1
CD
Cf'S-nx]
.If a .
.
.
2676. - 2- + ~
(_l)n -a 2I
IS nonlntegral; cos ax If a Is an

a-n

~
n=1

CD

integer; S ( xt) =cos an.

2677. 2 sinh axt


no

2Si"ha:rr[2~+~(_Wa~('+sn:];
xt
a.....
a n

2678.

~
n=1

(_l)n-l n

~i+"n~; S'l n) =0.

S(:n:)=coshan.2679.

n=1
CD

~.., ~t'1 (211 - 1)


2680. ~
2rt- 1

x . ) ~.
,a

4'

b) ~.

3' c

2681

xt

V3

a)

2~

stnnx.
n'

b)

~_.! ~c0s(2n-l)x.
2

xt

t2a-l)1

'

xt

"8.

~bnsinnx,

2682. a)

211

where

n=1

n=1

blk -

1n

~ (- ) - X

n=1

n=1

~slnnx.
n

~
n=l

8
xt
=2k-l-n(2k-l)' and b2k =-/i; b)

011

~
" cos nx
I) n
3+ 4 .L(-1)"11"2;
6'

xt

n=1

2)

~121 .

2683.

'I)

~;..
n

L...

[1-( _.I)n ea1t] n Jstn nx.

a +n2

b)

n=1

nxt

rt>

rt>[(
2~ ~~

+ 11 ~
n=1
(X)

-)Inan
e - I)'ens nx
a +n
l

2684

2 .
) _2IJI-COS-

xt

SIn

nx,

b)

~+
2

n=1

nn

2 \.1 sin 2
cos nx. 2685.
11......
n

+-

'J --

n=1

_~ ,~ crs 2t2'1-1) x 2606.


11

n=1

(2n-l)"

.J

"(~
k
1
1...bnSlnnx, where b2k =(-1) -12k,b2k+l=
n=1

Answers

461

~ ({ +i: SI::h cos nx). 2690. ~ [~+ i: cl~:hr cos nx].

2689.

n-I

1-~+2
~ (-1)"-1 COlnx.
2
~
n
1

2691.

2892.

..!.
1'

[i

n=1

n~1

~ (_l)n-I ens
21tX]
4n - l .

~
n-I

Solution.

2694.

I)

alII

= ~ Sf (x) cos 2nx dx = ~

+ ~ Sf (x) cos 2nx dx.

n
I

Sf (x) cos 2nx dx +


0

If we make the substitution t - ; -x in the first

11:

Integ:al and t = x - ;

f( ;

identity

in the second, then, taking advantage of the assumed

f(~ - t ),

+t ) = -

(n = 0, I,", ... );
bill

0.11 =0

that

3t

2)

It wl\l readily be seen


n

Sf (x) sin 2nx dx = ~ Sf (x) sin 2nx dx +- ; Sf (x) sin 2nx dx.

=;

The same substitution as in Case (I), with account taken of the assumed
Identity

f (~

+- t

) = f (~- t ) leads to the equalities

~ _ i. ~ cos (2n +- 1) 1lX

2695.

n2

~-

(2/1

sinh 1

~ ~~
;t......

= 0 (n = l, 2, ... ).

sin 2nnx .
n

n=l

11=0

2697.

2696. 1_

1)2

bZII

Icos n;x -lttl SIIlIl;X]

Ii.,
[T f-2 l.J

l2+ n2Jt2

(-1)"

1l=1
00

10 ~

nL

2698.

lJ

(_I)n

""

2699. a)

. fl nx
S1Il -1-

n;.1

8 [

=n

n2

nx

cos -

n=1
ctJ

_ i. ~

n ~
2

n=o

2k+I-\ 2k 1- 1)'

-16 ,,( -1)"-1 _ _


2 . 2702. a)
2

I.J

1)

21~~1 n~:

b)

2700

Il'J
(2'1 - I) nx
oc
41 ~ cos - - / - - ..
fl \'
.2701.a) bn Sll17)'
(2n - 1)2
....
'"

(_1)n+l _ _ ; b) - - -l
11
2 n

blk +- 1
a)

SIn

n=l

11=1

where

nL

4' 2(

n=1

21

a) n

IlJ'tX

SlI1
- ' - 1-

cos (2n + 1) fiX


2703,
(2n+ 1)2

11=1

4n

b2k =-/i;

(2'1 1- 1) 3lX
sin - - - : - - (_I)n
')

b)

00.

~
n2 1"J

n=o

{2.1

+ 1)2

b)

4 n::!

-S--

Answers

462

Chapter IX
2704. Yes. 2705. No. 2706. Yes. 2707. Yes. 2708. Yes. 2709. a) Yes; b) no.
2710. Yes. 2714. y-xy'=O. 2715. xy'-2y=0. 2716. y-2xy'=0. 2717.
xdx+ydy=O. 2718. y'=y. 2719. 3y2_X!=2xyy'. 2720. xyy'(xy2+1)=1.

2721. y=-xy' In~. 2722. 2xy"+y'=O. 2723. y"-y'-2y=O. 2724. y"+4y=O.


Y
2725. y"-2y'+y=O. 2726. y"=O. 2727. y"'=O. 2728. (l+y'2)y"'-3y'y"2-=O.
2729.
y2_ X 2=25.
2730.
y=xe2X 2731. y==-cosx. 2732. y=
1
X
X
2X
=(f(-5e- +ge -4e ). 2738.2.593 (exact value y=e). 2739.4.780 [exact
value y =3 (e-l)]. 2740. 0.946 (exact value y= 1). 2741. 1.826 (exact value

,ry 3)

Y=

2742. cottl/=tantx+C. 2743.


.

= In Cxt. 2745.
IJ

2748. 2e 2

~
x=y__
; y=O.
1 + 1/2

X2 +y2.=

2744.

y =a+ I ~ax' 2746. tan Y =C (I-eX)I; x=O. 2747. y = C sinx.

= Ye (l-t-e x ).

2749.

2
1 +y2=-1- 2

-x

y= I.

2750.

2751.

2752.
8x+2y+l=2tan(4x-I-C).
2753. x+2y+
+3InI2x+3y-7J=C. 2754. 5x+lOy+C=3InllOx-5y-t-61. 2755. Q=

arctan(x+y)=x+C.
I

C
- cos (J>

or y2=2Cx+C2. 2756. InQ=-2-.--1nlcosq>I+C or Inlxlcos" <p

-2y22 =C. 2757. Straight line y = Cx or hyperbola y=... Hint.


x

The seg-

x '

ment of the tangent is equal to Yyt+(f.-f.2758.yt-xt=C.2759.y=


x

~ xydx
o
3
-x--=4
x.

=Ce a . 2760. y2~2px. 2761. y=ax". Hint. By hypothesis

) ydx
o

Differentiating twice with respect to x, we get a differential equation.


2762. y2 = } x.
..r 2-Y4-x 2
2763. y == f 4-x 2 21n
. 2764. Pencil of lines y = kx. 2765. Fax
2
roily of similar ellipses 2X'-t- y 2=C 2766. Family of hyperbolas X 2 _ y 2::=:.C.

2767. Family of circles x 2 +(y_b)2=b 2 2768. y=x In


x

2770._x=Ce y.

2771.

,ix
V y+ln1yl=C.

(X-C)2_ y2=C2;

2773.

y=2 X2 -2C;

s..
2769. y=.E._!-2 .
x
x
y=

(x-2)!_y2=4;

x=O.

2774.

x.

2772.

(X +y2)3(X+y)IC.

2775. y=XY1-: ;c. 2776. (;c+y-l)'=C(x-y+3). 2777.3x+y+2X

Xln 1x+u-1J =C. 2778. In 14x +8y +51 + 8y-4x =C. 2779. xl = 1-2y.

463

Answers

2780. Paraboloid of revolution. Solution. By virtue of symmetry Ule sought-

for mirror is a surface of revolution. The coordinate origin is located in the


source of light; the x-axis is the direction of the pencil of rays. If a tangent
at any point M (x, y) of the curve, generated by the desired surface bein~ cut
by the xy-plane, forms with the x-axis an angle cp, and the segment connecting the origin with the point M (x, y) forms an angle a, then ta-n a = tan 2cp =
I 2t~n ~ . But tan a. =JL ; tan cp = y'. The desired diiTerential equation is
- an q>
x
y-yy'!= 2xy' and its solution is y2:= 2Cx +C2. The plane section is a parabola. The desired surface is a paraboloid of revolution. 2781. (X-y)2_Cy=O.
2782. xl =C(2y+C). 2783. (2 y 2_ X ')I.=CX 2. Hint. Use the fact that the area
x

~y

is equal to

2784. y =Cx-x In I x

dx.

I.

278li.

y=Cx +x l .

y=

2786.

tl

,-::i-X4+~I' 2787.
t~x

respect

I
eX ab-e fJ
and d-. 2788. X=C y 2 _ - . 2789. y=--f---.
y
y
x
x

.. r -

x VI+yl+cosy=C. Hint. The equation is linear with

dx

=-2 (x r l - x 2 +

yl+X

.
arc Sill
x)

-1- .

-x

2793. y2=xln-. 2794. x 2 :=: +C

2.

2791. y =

~Cy2+a2. 2q98. y2+ X +ay==O. 2799. x=y

,\2_t- y 2-Cy+a 2=O. 2802.

x4

yS

"4-2"X 2y 2_1-2x+ 3 =C.


x2

?807. -2

x!

-x . 2792. -II (x 2 -t- ex) = 1.


cos x

y'(3+CeC09X )=x

2795.

a
y
In - . 2800. -

+ -by = I.

Y
2+yl_2 arc tan-:t=C.
2806.

+ ye l/=2
-

+ C)I =x'; there is

RO

1
y

-lnx+

1- 2Cy) X

Qeneral

singular integral. 2814. General integral (

f-

Inte~ral

y+C ) X

+ C ) =0; there is no singular integral. 2815. General integral

UI+C1=-=2Cx; singular integral X1_yl=0. 2816.

x=sinp+lnp,
{ Y = P sin 191- cos P + p +
Sin~ular

2801.

y 2=Cy.

II"
X Xl
2808. Inlxl-~=C. 2809. --1-- =C. 2810.
x
y
2

X ( x-

X 2_

(.t C 2 +
'

XlJ=

2804.

+ '2I y2 :--::: C. 2811. (x Sill Y + y cos y -sin y) c"(::::L C. 2812.


X (x + C2- 2Cy) = 0; singular integral :.;2_ y2= O. 2813.
(y

2797.

Xl
"3+xy2+X2=C.

+xy+y2=C. 2803.

2805.

y=

2790.

solution:
__

c.

2818

y=O.

{x=eP+peP+c, 2819.
y = p2eP .

2820.
p

y=~cosx ~-3Sil1X. 2817.

4Y=X +pl.

y2+ p 2

JX=2P--p~ +C.
\

y=p'+21np.
x

Inlp-x/=C+--.
p-x

2821. In yp2+y2+arctan-=C, x=ln-- . Singular solution: y=e".


y
2p

Answers

464
2822.

Y=21 Cx I + C2.
t

y::;g

2x.

x=Ce-P-2p+2,
{ y=C (1 +p) e- P - pZ+2.

2824.

--

x=- (Cp 2 -p),


x=
In
I
p
I-arc
sin
p
+
C,
2825
3
I
Hint. The differential
2823. { y=p+ VI-pl.

1
_
y="6 (2Cp I
p2).

equation from which x is defined as a function of p IS honlogeneous. 2826.


x2
y=-CX+C2; Y=-"4. 2827. y=Cx+C; no singular solution 2828. y=--Cx+
2829. y = Cx + ~ ; yl = 4x. 2830. xy = C 2831. A ci rcle
and the family of its tangents. 2832. The astroid x 2 /1+ y2JI=a 2 ;1. 2833. a) Honlogeneous, y=xu; b) linear in x; x=uv: c) linear in y; y=uv; d) Bernoulli's
equation; y= uv; e) with variables separable; f) Clairaut's equation; reduce
to y =xy' y g'l; g) Lagrange's equation; differentiate with respect to x; h)
Bernoulli's equation; y = uv; i) leads to eq uatIoll with variable~ separa ble;
u =x+ y; j) Lagrange's equation: differentiate with respect to x; k) Bernoulli's equation in x; x= uv; 1) exact differential equation; m) linear; y = uv;

+ y I + CI;

Xl + yl

= I.

n) Bernoulli's equation; y=uv. 2834. a) sin 1t.

= -In Ix, + C;

2835. xl+yC=Cyl. 2836. y= xl+C. 2837. xy(C-

b) x= y.eCY+1.

1n 1 x)= 1.

2838.

y=

=Cx+C In C; singular solution, y=t-(x+l). 2839. y=Cx+ V -aC; singular


a
I x'-l I
1
solution, Y=4i. 2840. 3y+ln (y+l)'=C. 2841. 2etX-eY-arc tany1

-'2 1n (1+y')=C.

2842. y=xI(l+Ce X

).

2843. x=yl(C-e-Y ).

2844. y=

=C,-slnx+Sinx:-l. 2845. y=ax+CVl-xl 2846. y=X~l (x+1nlxl+C).


2847. x=Ce

slny

-2a (1 + sin y). 2848.

y-l
x

x2

2 +3x+y+ In [(X_3)IO I y-11 3 ] = c.


2

--

2849. 2arc tan -2-=lnCx. 2850. x 2 .=1--+Ce y. 2851. x'~CeY-y-2

2852.

V~ + In I x I =C.

2853. y=x arc sin (Cx). 2854. y'=Ce- 1X +

~ sin x+

+:cosx. 2855. xy=C(y-I). 2856. x=CeY - - } (siny+cosy). 2857. py __


333
c::C(p-l). 2858. x4 =CecY -y'-4 yl-SY-32

2860.

x
Yx-+y2
__ =C. 2861.xeY l

y2=C. 2862.

2859. (xy+C) (x 2y

+C):.:O.

VI +p2
1
C
x=t+.In(p+
..~ _ _ 2p
2p
( + f 1+p2),
Y = 2px + V 1 + pl.

2863.IJ=xeCx 2864. 2eX _1t=Cy l. 2865. In 1Y +21+2arctan;+:=C.2868.

465

Answers
~~
, 2

-t- Ce - 2
4(

1 2 = 0.
+-x

2867 . x 2 'Y= C'I


ew

x
x+-=C.
y

8
286.

y=

2869.

C-x"
.
a 2 In(x+Ya 2 +x 2 )+C
2
1)3,2.2870.y==CS1l1X-a.2871.Y=
x X+ y a2 +x2

2872.

(y -Cx) (y2_x 2

+C):.= 0.

2873. Y=-= Cx

I
+ C2'

Y==-

2" Vi
\ 2x 2

x'

2874.

+ x y2

_y2x _ y3===C. 2875. p2+4y2:-=Cy 3. 2876. y=-=x-l. 2877. y-=-=x. 2878. y=2.

2879. y=O. 2880. Y=~(SiI1X+COSX). 2881. Y={-(2x 2 +2x+I). 2882. y=


=_-::e- x -i-2x-2. 2883. a) y=x; b) y=Cx, where C is arbitrary; the poinf (0,0)
is a siugular pOint of the differential equation. 2884. a) y2==X; h) y2== 2px;
(0,0) is a singular pOint. 2885. a) (x -C)2 + y! = C!; b) no sol ution; c) x 2 + y2 =--= x~
x

(0,0) is a singular point. 2886. y -=-e". 2887. y=-=(Y2a YX)2. 2888. y2=-=
--::l-e- x . 2889. r=Cea~. Hint. Pass to polar coordinates. 2890. 3y2_2~=O
2891. r--kcp 2892. x 2 -1-(y-b)2=.-b 2. 2893. y2--l-16x=O. 2894. Hyperbola
!/ _x 2 -~ "C or circle x 2 1- y2 --- C2 2895. Y = 21 (eX -)- e- x ). Hint. Use t h~ fact
x

Ilut the area

IS

equal to

Sydx

and

the arc leni:tll, to ~ I

u
2

2896. X~:.!.-+ CII.


II'

l-ry'2

1r..

2897. 1/2_4C (C-;-a-x). 2898. Hint. Use the fact th:lt th~

re"ultant of the force oi ~ra\'lt~ and the centrIfugal force is nornlJI to thl' surface.
'1 akll1~ the y-axl~ as the :tXIS of rotation and denotlIlI1: by (J) the ,11lgular velocity -of rotatll)l1, \\ e ~t't for the plane aXIal cross-section of the deSired surface the differential equation g ~~! ={t)~x. 2899. p -=e- IJ .I)QIl1671l. Hint. The pr<:s..
dr

sure at each level of a vertical CUIUlll11 of air IBay be consHlcrcd HS due solely
to the pre~"1I1C uf the upper-lyIng layers Usc the la\v of Boyh~-.J\LI!" utte, ~c
cordll1~ to \\ 11Ich the dCllsIty IS proportional to the pressure. The sought-for
dtflefl\J1tlal equation

_'k'WI-~xdx.

IS

dp -- - kp dll. 2900. s -:.: 2"

Hint. EqUation ds-=

2901. S=(Pl-{W)kl. 2902. T=a-j-(To-a)e- kl 2903. In

one hour. 2904. w'-' 100 (

rpm. 2905. 4 2% of Ihe initial quantity Qo

will decay in 100 years. Hint. Equation

=::: 35.2 sec. Hint. Equation


dQ =-kQ dll. Q = Qo

1t

dd~ =kQ.

(h'-2h) dh =

(2"I)!!'

I.

tionality factor). Hint. Equation

2908. v ~

1t

Q -= Q. (

(~r rJ dt.

.V. / grn
T as t

m~~ =mg-ku

2909. 18.1 kg. Hint. Equation ~~ = k (


16-1900

kl~.

~ - 3~O)

; U=

)00.

2907. Id24'
-+- 00

(k

g;; tanh

2910. i =

2906.

IS

t =:::
HInt.

a propor-

(t V~ ).

R' :L2W2 [(R sin wt-

Answers

466
R

- Lro cos rot) + Lroe

-- t
L

].

di
Hint. Equation Ri + L dt = E sin rot.

=xlnlxl+C l x+C 2. 2912.

1+Cly2=(C2+~;r.

Y ==

2911.

2913. y=ln/e

2x +C I

!-

-x+C 2. 2914. y=C 1 +C 2 In/xl. 2915. y=Clec~x 2916.y=YCtx-l-C~


2917.

y=(l+C~)lnlx+Cll-C1X+C2'

2919.

y=

Clee," +

(In I x 1)2+ Cl ln Ixl +C2 2920.X=~ In II -:C "1 C2 ; Y= C. 2921. Y =

~2'

2922. Y =

c=(C 1ex +I)x+C 2

[x V C~ _x + C~ arc Sin~] + C2

x
-+J
e
2924. y==(C1x-C:)e Ct +C 2 ; y=2X2+C

=12+"2 + C1x In Ix I +C 2x+C.


=xl +3x.

1
2929. Y-=="2 (x 2

+ I).

'2935.

+C

2927. Y=

2923. Y =

(singular salu-

(singular solution).

sin (C, x) + C x-t- Ca.


2

Y=

2926.
~

2928. Y---=

2930. y=x+ 1. 2931. y=Cx2. 2932. y--=C.X

+ C2e y=C.
X 1
l-C~x;

tion). 2925. y=C,x(x-C 1 )+C 2 ; Y= ~

=aln\sinY~C21

(x-C I )

2918.

2
1 In \ y+C
!I
x==C 1 +ln y-C
y-t-C \ 2934. X==Cl-~

2
2
X=C ty2+ y Iny+C 2 2936. 2y 2_4x 2=:: 1. 2937. !J=-x-~-l. 2938. y:-=.

x 2 _1
2 (e 2 -1)

2933.

e2 -1

I-x!

-4- 1n I xl or y=2 (e2+ 1)

e2 +1

+ 4 - 1n I x I.

2939.

!I -

2 x

1 3
x

2940. y == 2 x2
2944.

2941. Y =-= 2e

e- x

y2="e_I+I_e

2942.

2945.

x ==

-"2

(y -~. 2)

JI2

2943.

+-1.

Y-== -:~-x-

Y - eX.

461

Answers
xcosh x-t-C +C 2, where H

IS

The differential equation

a constant hOrizontal tension, and

VI -/- (:~r

::~=1i

ds
dt 2 =g(sina-l-tcosa). La\v of nl0tlon,

gt

!!..=a.
q

2965. Equation of motion,

s~2(sin(L-l-tcosU)

s=/i

2966.

yg ~).Hint.EqUatiOnOfmotJon.m:;~=mg-k(~r.

Xincosh(t

Hint_

2967. In

6.45 seconds. Hint. Equation of nlotlon, 300 dd:!~ =-10 v. 2968. a) No, b) yes.
g

d) yes, e) no, f) no, g) no, 11) yes 2969. a) t/, + y = 0; b) y" -2y' + y -= 0;
c) x 2 y"-2xy' +2y=O, d) y"'-3y"-J-4t/-2y-=-=O 2970. 1I:::.=3x-5x 2 +2x 3 2971. Y-=

c)

Yl~S,

=~(CISlnx-J-C2COSX).
x

+C

Hint. Use Ule substitution Y=YIU, 2972. y=C1x+

x2
B
In x. 2973. Y = A -t- Bx 2 +Xl. 2974. y == -3 -f- Ax + -. Hint. Particular so-

lutlons~

the honlof1:cneolls equation

YI~=X' .tI2-=--=~'
x
x

variation of paranlcters we find: C I :-:: ~

+ A,

C2 =

By the 1l1cthod of the

x:s
6 -t- B

2975.

y= A

-I B Sl1l x., C cos x -t-In/ sec x + tan x 1-1- "in x Inl cos \" I-x cos x. 2976. Y == C 1e -12X

-1-C2e3.\: 2977. !I:....:Cle-3~ j-C2,e 3 ';. 2978. y--=CI-1-C2e~ 2979. y=C.Losx+C 2 sinx.
2980. !J --=. eX (C I cos x -1- C 2 Sill x) 2981. !I =-= e- 2X (C l co~ 3t -1- C:! 5111 3x) 2982. Y =

- (CI-!-C~x)e-x. 2983. y~-:..e2.\ (Cle t J -; -f- C2e-X~'2).

=-

Cle J Ii

C 2e-d -1/;

If

<

y-~CI cos

O.

.' -5" x
-Jr-:- x
--t
2 (Cle 2 -1-C 2e 2 ) 2986.y=-e 6

x
--

298t).y=e

If

2984.

V -llX

Y-=

Il> 0,

-t- C 2

Slll

(~rJ' 11

V -kx.

r- )

l 11

Clcos~x-t-C2Sill-6-X

.
2987. !I =- 4e x -1- elX". 298R.!I ~ e-". 2989. Y:=-;: sin 2x. 2990. y = 1. 2991. y

== a cosha-

II -== 0 2993. !I =- C sin rrx


2994. a) xe (Ax:! + Bx -t- C); b) A C()~ ~~ t-1- n "in 2x; c) A co~ ~\" -t-1J Sill 2\' +- C\"2e2X; d) eX (A cos x-I- B "in \), c) e\:;<
).' (A \:! -1- Bt -t- C) 1- xe:!'; (!)r.,- E); f) xe X [(Ax 2 -i- B\ + C) cos 2x+ (Dx 2 + Ex t-F) X
2X

2992.

2995. II=(C I -!

XSln2x)
,-

Y3)

,. x 2C 2 s1n

c.x)eX-/-~(2x2+4X+3).

'~ 2 2997
+x 3 -1- .\.

2998. 11_ Cle x 1- C 2e7X


0

+2

C1e:!''\ -t-C 2e- 3X

~e-x

lC, cos 2x

16*

3004.

+C

__ (C I-f-:!X
C ,) e -x -19
L 1

lj---

2999. Y ~ C .ex

+C2SiIlX-l-ixSinx. 3001.

2996.
l

Y=e:(clcosX~3+

,2.'\:

+ C .e-\ + xe~.

3000. y

Y=C,l!x+C2e-2X-i(:~S1l12X+COS2X). 3002.

+ x l 10Xl)
- 25 e
I

2X

TeX

Y=

3003. y = (C 1 -J- C 2x)

eX

+ 2"1 cos x + x4" e

y=c,+c2e-x+~x+~(2COS2x-Sin2X).
sin 2x) -/-

= C I cos x+

sin 2x.

3006.

-= cos 2x +

3005.

y=eXx.

(sin x -/- sill 2x).

Answers

468

-8007. 1) x== C. cos rot + C. sin rot + -.--. sin pt;


ro -p

2) x= C1 cos rot + CI sin oot-

A
x Xl x
- 200 tcos rot. 3008. y= C1e'x + Cae4X-xe4X. 3009. y =C 1 +Cae"x + 4-4-"6
1
5
-8010. y:::eX(C 1 +C 2X+X 2). 3011. y=Cl+C2e2X+2xe2X-2"x.
3012. y:::::l
=Cle-IX+czeU'_~ eX+ ~ (3cos2x+sln2x). 3013. y=C1+Cze-X+eX +
5
+ 2XI-5x.

3014.y=C 1+C.ex -3xex -x-xl . 3015.y= ( C1+C1 X+ 1x1)X

1
xe- x + 4 eX.

3017.

II = (C 1 cos 3x + C2 sin 3x) eX

3016.

y=(Cl+caX+x~elx+xtl.

+ 37 (sin 3x + 6 cos 3x) + e~}x

3018. y=C1+CiI'X-f6(COsx+3sinX)-

x3 x 2 X
X
_X
(4x+ 1)- 6- 4+"4 3020. II=C 1e +C 2e e2X
- x sin x-cos x. 3021. y = e1e- 2X + C~e2X -20 (sin 2x 2 cos 2x). 3022. Y ==
Xl

- 6-9

3019.

1 2X

Y=g e

c:

C1 cos 2x+ Casin 2x -

f (3 sin 2x+2 COS2X)++.

+ C 1 sin x-2x cos x). 3024. y= C1e x + Cze-x +


+C a sin3x+

)( (2-3x) +

(xa-x)eX. 3025.y =C1cos3x+

1
3
3
16 (2x 2-x)e3X 3027.y=Cl+C2e2X-2xex-4x-4"xl 3028.y:::::a

)( (2xl + 3x) eX.

~ sin 3x.

y=eX t~l cos x +

xSinx-~cosx+514 (3x-l)e lX 3026. y=C1e,x+cae-x+{x

-=(CI+Clx+~)eIX.
+

3023.

3029.

Y=Cle-,x+cr-i(2xl+x)e-'x+~x

II=C1cosx+C 2 sinx+

8030.

Xl

cosx+"4slnx-acos3x+

Hint. Transform the product of cosines to the sum of cosines.

8031. Y= C1e- x 'VI; +Cr 'VI +xeX sin x+e x cos x. 3032. y=C 1 cos x+C I slnx+
+COSXln/cot

(-i-+~ )/.

3033.

y=C1COSx+Caslnx+slnx.tnltan;

I.

3034. y=(C 1 +C 2X)


In I x I.
3035. 11= (C 1 +C 2x) e- x +xe- x In I x ,.
8036. y=C 1 cosx+C 2 sinx+xsinx+cosxlnlcosxl
3037.
y=-=C 1 cosx+
+ Clsin x-xcos x+sin x'VIn I sin x I. 3038. a) y=C1ex +Cst- X+ (r +e-X)x
yxarc tanr; b) y=C 1ex I +C~-x I +exi . 3040. Equation of motion,

eX +xex

i-(~a:')=2-k(X+2);

-=

(k=l);

T=2n

vi

sec.

3041.

x-

2gsin30t-60 Visin Vit


.
g-900
cm. Hint. If x Is reckoned from the position of

rest of the load, then

x"=4-k (X.+X-II-I), where X o Is the distance of


the point of rest of the load from the Initial point of suspension of the spring.
4 dlx
I Is the length of the spring af rest; therefore, k (xo-l) = 4, hence, - dt l ~
dx
g
---k(x-y), where ~:a:4, g=981 cm/secl. 8042. mdil=k(b-x)-k(b+x)

Answers
and

x=ccos (t

= ~
is

(e

:;~

lUt

V;

V2~). 3043.6 ::~=gs; t=

e- wf );

=ro 2r.

469

b) r = ;~

(elllf _e- mt )

Yi5). 3044.a)r=

Hint. The differential equation of motion

y=C.+C~x+Cael2~.

3045.

In (6+

y--

X(
3047. y=C.e-X+el c 2 cos-+x+C,sin

y=C.+C~-x+Carr.

3046.

y;--3 )
2 x

3048. y=-=Cl+C2X+CaexV;: -f-C 4 e- x lr 2 - 3049. y-=-e x tCI1-C2X+Cax!).


3050. y =-- eX (C I cos x -1- C 2 sin x) e- x (C. cos x -f- Col sin x)

3051. y::= (C I + C 2 x) cos 2x

3052. y=--CI-rC~e-x
I

-1- e~

+ (Ca-l- C

x) sin 2x

(caCOSTX1-C4SlIl-2-x.
yJ
. Y3 )

3053. y = (C I -t- C 2x) e-'Y: --1- (C a -1- C4~) eX.


3054. y === Cle QX -t- C2e-a:~ -1- Ca ("('S at Col sin a~
1ra
1
3055. y =- (C I 1- C 2x) e
t -t- (C a -I C 4 x) e- 1";- t'

+-

+
+

3056.
Y = CI
C 2x
3057. U _::.- CI 1- C 2x -1- (C a C4 t) e- x
3058. y:-::: (C I
-f- C2 x) cos x -t- (C a + G\x) sin x.
3059.
y -- e-o,; (C I -t- C2 "< -!- ...
C'l"(n-I).

+ Cs cO~J'K -1- Col sin a~.

3060. y=C. +C 2 x 1- ( C, -I- C~X+~) eX,


3061 y==C I -l-C 2 X_II
3062. y==Clex-!-e

12r:!_L:3X3---l_~
r
"I
2'\4_L~.\5_;
I 20

(C S -l-C
t)c'Y:
I
-l

3
(rtf
C cOS-i-X-1 Ca<;':lT

-~
2

}'F

-x 3 -5.

c~e-'>;+1~88(4CnS'lx-sin4X)

3063. y=C.+C 2 X+C aX 2 -t


3064. y =c C,e- x -I- C. -1- CaX

-I-{ x

+Xl

' n x --,I
.3065
.
Y:- - C Ie -x + C 2 C( IS x -t' C
3 "1
e~

-I-

I~ \:~ -I- e~ ( {X-~) .

~3 ) .
"4x - ~
C I -t- C2 cos x 1- C3 sin x -1 sec x -1- cos x In I cos t 1- tan x sin x +x sin .'(.
(

3066. Y =.3067.

V"3
1
}/3) -1-x-2
y=-e-X+-e - ~2 ( COS--X1--sin-x

Y3

3068. Y :-- (C 1-t- C2 In

1
x) . - . 3069. Y =-= CIXS
X

_2

3070. !I =-= C J cos (21n x) -{-- C 2 sin (2111 x).


3071. y:=.: C IX -1- C 2 X 2 -1- Cax s . 3072. Y ==- CI -1- C2
3073. y

3075.

~ CIX~ +- Cx

(:~x

-1- 2) - 4/3

3074. Y == C 1 cos (In x) + C 2 sin (In x).

y=C.XI+C~2

-I-{ x.
2

3077. y=x(lnx+ln x).

3076.
3078.

3079. y =e- x (C. cos x + C2 sin x),


3080. Y= (C 1 -C 2 -C.x)e-

2X

y=(x+ I)" IC. -I-C 2 1n (x -I-

1)1 tex -I- I)'.

y=C1COSx+C 2 sinx, z=C 2 cnsx-C t sinx.


x
Z= ~ e- [(C:-2C.) cos x- (C -I- 2C ) sin xl.

z=(C.x+C 2 )e-

2X

Answers

470

t
-~ ( C CosTt+Casln-3081. x=Cle +e
2
2

. ya- t ) ,

ya

C t+ -{- (C s
Y = Ie
e

Y3-C
Y3 t
2
2
cos -2-

C2

t) ,

3
Y3-I-C
y~r
2
SID -2-

s ,. 3 t)
t+ e--~(-Cs Va-c
:3" t+ C2 Y3-C
2
cos 2
2
n 2
t
t
2t
t
2t
x= Cle- + C e , Y =-=Cae- + C e , z = -(C + Cal e- + C2e~t.

-C

Z -

3082.

Ie

"-f (x!+x). z=C e


2

3083. y=c t +C 2eU

81

2X

d-f (x -x-I).

3084. y = C I C2x+ 2 sin x, z = -2C I -C 2 (2x + I) -3 sin x-2 cos x.


3085. y=(C 2 -2C l -2C 2 x) e- x -6x+ 14, z==(C 1 +C 2 x) e- x +5x-9;
CI =9, C2 ==4,
y = 14 (1-e- X )-2x (3 + 4e- X ), Z == -9 (1 _e- X ) + x (5 + 4e- X ).
3086. x = lOe 2t -8e at -et + 6t -- I; y = - 20e2t + 8e at + 3et -I- 12t + 10.
2C 1
C1
*
(X 2 +y2)y
Z _

_
3087.y (C
)2' z - C - - '
3088. a)
=C ,
---C 2 ,
2- X
2- X
x , (I O Y
2+y2.=arc tan JL +C , lr z
b) In
==C 2. Hint. Integrating the horno1
x
y x2 + y2

Yx

geneous equation
= arcX tan

~=~,

x-y

1L
+ CI'
x

.. r z
y

x2+ y2

\ve

find the first

x~+y~

+ y2

Whence 1n z == -2 111 (x

y X +p2-:-:-:
2

+ y 2) -1- InC 2 and,

=C 2 ; c) x+y+-z-==O, X2 +y2+ Z2==6. flint. Applying till'"

. t'Ive propor t'Ions, we h ave


0 f derlva
proper t les
whence dx

tlltegral In

Then, uSing the properties of derivative proportions, \ve havl'

y~
-~z = x (x~
x-y ) = y (x + y )

hence,

x+y

dx
dtj
dz
dx+dl/-I-dz
--=----=-==--==
0
y-z z-x x,-y

-r dy -t- d.J~--== 0 and, consequentl y, x + y + z == C


+

S, nlilarl y,

(x dx )-

x Y-Z

z dz
x dx II dy z dz
Y dy
- - - - - - . xdx-'-ydtj-t-zdz-=--Q and x 2
fy(z-x) z(x-y)
0
'
I.
2
2
+Z2 == C2 Thus, the integra I curves are the circles x +!I + z -:: C1, x
y -I- Z2 - C~
From the initial condition<;, x=l, y==l,z=-=-2, \VC will haveC 1 =O,C 2 ={).
C
x2
3089. y==C l x2 +-f-T8 (31n 2 x-2ln x),

+y0)

z= 1-2C,x + :~ +i (31n 2 x+ In x- \).

C 2e- ~ V'2 Cs cos x C" sin


. x ex - 2x,
3090 . y = C Ie ~ V 2x Vi- C -xV;- C s
C4
1 x
z=- C Ie
- 2e
-4-cosx-TslIlx-2e+x.

3091.

X=

v m cos a (
_.!!0 k
l-e m

t) 'Y=/i2(kv
m
(_!- t) - T
mg t
'
o sina+mg) l-e m

Solution. m d;t = -kv g ; m ~;: = -kvy-mg for the initial conditions: when

Answers

t=o,

l'XO=vocosa, Vt' =uosina.

xo==Yo==O,
k

== V o cos ae

til

t,

YIll

d!t

+mg == (kv o sin f1 + mg) em.


k'2~l
nzv 2
o

x2

-:;+-~- -=-

u"

d2~

dt 2 =-= - k"x,

III

kV y

JIm sin --:..-::


k

t'

3096

2" -f-"4 X -1- ~

1
Y =- : 3"

y=

dt;- --= - k y.

III

1.

3092. x ==a cos ~r-' i,


r In

l. Hint. The differential equations of motIon:

3093. y= -2-2t-x 2 3094. y=


5
_
309. y -

Il

-- t

Integrating, we obtain v x

- -- t

=_0_ _

471

-7.9 x

x2

-1- 16

(Yo+{) e (X-I)_~ x+~ .


2

+ 32 X

-t- 7. 112 .27 X II -

x3

21

-1- 320 X -1-

x4

3097. y~x.+T:2-1-2.3+a.4+ ... ; thc series converges for -l~x<l.


r2

x4

XS

3098. lj-~X-(1f)2.2-i-(2!)2.3-(3!)q 1- ; thc series conven~es for -00<

<x< -1- 00.

Hint. Use the 111ethod of undeternlloed coefficients.

3!

3099. Y --= ) -

)4

1 47

-t- 6! x 6 -

~}-,-

x' . .. ; the

sene~

converges for-

00

<X< +

CX).

I (/.::

0).

3100. 11-== sin x . Hint. U"e the Illethod of undctcnnincd corfficH:nts.

x~

xG

~'I

3101. y;:-=. 1-22-t-22.4~-2l ..P.6~-1- ... ;


Hint.

U~C

2
-1- ~r!

t 4 - ()!

senes converges for

the IlIl,thod of UI1(ktrrnlinl'd coefflclents. 3102.


U t 6 -1- 8!
55 t 8 -...

tlon~: Ll

the

(0, t)-::-::o,

L 1 (I-cos
no-a

(I-~
t 2 -t2!

. T
:rl X
31 03 u =-= A cos art
-l-t S1I1
HInt. l'Jse t 1h~ condi-

t)-:--o,

U (1.

x -- a

(x, 0) --.
(x, 0) --= A'
sin nx
T' iJu dt

00

3104. U -=

21

-2

-f)-

It

. l11tut

rt=1

u (0, t) =-= 0,

Ii

(l, t) --= 0,

rzrx

l1n) Sill - l - sin -1-.

(x, 0) -= 0,

au (x,
dt

0)

Hint.

Usc

the

conditions:

= 1.

00

3105.

Biz

== 312

11:rl

1t2 SlIt 2

ll:Tat.

IUtX

cos -l- SUI -l- Hint. Use the conoitlOl1s:

11=1

iJu

~/

211t
-1-

0) _-=-0,

It

(0, t)=O,

It

(I, t)=O, u (x, 0)=

Jt ( .\
1-3.- \
2h

for O::::x ~ :l" ,


I
for- < x
1J
2

< I

00

,-.,
3106 u ~ ~

n=o

An Cos --2-[-(2n -1- I) ant . (211 + 1) nx


wllcrc tile
fn
t
1
sIn
2t'
coe 11Clf11l S / II =:

Answers

472
1

x
= T S-1

. (2n+I)1tx
dx. Hint. Use the conditions
21

Sill

au

(1, t)
x
u (0, t)=O, ---ax=0, u (x, 0) 1

3107. u =

au (x,
0)
iJt
o.

a 2 n:!j't2t

CJ)

400

'

na ~ fi3

. nnx -1""002
(I-cos n1t) Sin 100 - e

n=l

Hint. Use the conditions: II (0, t) == 0, U (100, t) == 0, u (x, 0) = 0.01 x (laO-x).

Chapter X
3108. a) ~ I"; ~0.0023!o;
b) ~ I mm; ~0.26/0; c) ~ I gm; ~0.OOI6%.
3109. a) ~ 0.05; ~ 0.021%~ b) ~ 0.0005; ~ 1.45% ; c) ~ 0.005; ~ 0.16/0.
3110. a) two deciInals; 48.108 or 49.10 3 , since the number lies between 47,877
and 48,845; b) two decimals; 15; c) one decinlal; 6.10 2. For practi~" purpo"es
there is sense in writing the result in the fonn (5.90.1).102. 31 I t. a) 29.5;
b) 1.6-102 ; c) 43.2. 3112. a) 84.2; b) 18.5 or 18 47 0.01; c) the result of

subtraction does not have any correct decimals, since the difference is equal
to one hundredth with a possible absolute error of one hundredth.
31131&. 1.80.3 cm 2. Hint. Use the fornlula for increase in area of a square.
3114. a) 30.00.2;
b) 43.70.1;
c) 0.30.1.
3115. 19.90.1 In 2
3116. a) 1.12950.0002; b) 0.120O.006; c) the quotient nlay vary behveen
48 and 62. Hence, not a single decinlal place in the quotient may be con~id
ered certain. 3117. 0.480. The last digit 111ay vary by unity. 3118. a) 0.1729;
b) 277.10 3 ; c) 2. 3119. (2.050.01)103 cm 2 3120. a) 1.648; b) 4.0250.001;
c) 9.0060.003. 3121. 4.01.10 3 cm 2. Absolute error, 6 5 crn!. Relative error,
0.16/0.3122. The side is equal to 13.8+0.2 cm; sin a=0.440.01, (1,=---=2615'
35'. 3123. 2 7 O.l. 3124. 0.27 ampere 3125. The length of the pendululll
should be measured to within 0.3 enl; take the nurnbers :t and q to three
decinlals (on the principle of equal effects). 3126. Measure the radii and the
generatrix with relative error 1/300. Take the number 1t to three decimal places
(on the principle of equal effects). 3127. l\\easure the quantity 1 to within
0.20/ 0 , and s to within 0.7/0 (on the principle of equal effects).
3128.

/).y

&2y

/).3 y

/).4 y

ti 5 y

-2

-6

14

-23

10

-8

-9

15

-3

-1

12

-3

-I

-4

473

4nswers
J129.

&y

/).2 y

flSy

-4

-12

32

48

-16

20

80

4,Q

fi

100

12R

48

104

228

176

332

40t

11

7:36

3130.
.\

.\ :l

IJ

I
- - I- - -

,--------

I
Hint. Compute the first live values of y and, after obtaining L\4 yo 24, repeat
the nUlllber 24 throughout the C01Un1l1 of fourth differences. After this the
remaining part of the table is tilled HI by the operation of additlo.1 (nl0ving
-=..:;

from right to left).

Answers

474

3131. a) 0.211; 0.389; 0.490; 0.660; b) 0.229;0.399;0.491;0.664.3132.0 1822;

0.1993; 0.2165; 0.2334; 0.2503. 3133. 1 + x + XZ + x s 3134. y = :6

+~~ XZ-~x+8;

x~-~~ Xs +

y=::::22 for x=5.5; y=20 for x=::::5.2. Hint. When computing

x for y=20 take Yo== 11. 3135. The interpolating polynolnial is y =-= x 2 - lOx -t- 1;
Y= 1 \vhen x==O. 3136. 158 kgf (approxilllately). 3137. a) y (0.5)...:;;;- 1,
y(2)=1l; b)

Y(0.5)=-~,

y(2)=-3.

-1.325

3138.

3139.

1.01.

-1.86; -0.25; 2.11. 3141. 2.09. 3142.245 and 0 019. 3143. O.:~I and 4
2.506. 3145. 0.02. 3146. 0 24. 3147. 1 27
3148. -1.88; 0 35; 1 5:)
1.84. 3150. 1.31 and -0.67. 3151. 7.13. 3152. 0.165. 3153. 1.73 and O.
1.72. 3155. 1:38 3156. x=0.83; y-==O 56; x~ -0.83; y== -0.56
x== 1.67; Y== 1 22. 3158. 4 493. 3159. 1 1997 3160. Bv the trapezoidal formula, 11.625: by Sinlpson's formula, 11 417. 3161. -0 995; -1; 0.005;
0.5/o~ 6. -=0.005. 3162. 0.3068; 6. == 1.:~10-5.
3163. 0 69 3164. 0.79.
3165.0.84. 3166.0.28. 3167.0.10. 3168. 161. 3169. 1.85
3170.0.09.
3171. 0.67. 3172.0.75. 3173. 0.79. 3174. 4.93. 3175. 1 29. Hint. Make use
of the parametric equation of the elli pse x ~ cos t, Y -== 0.6222 sin t r'::11d tran~-

3140.
3144.
3149.
3154.
3157.

1t

fOlm the formula of the arc length to the form ~ VI-Ezcos z tdt, where E

IS

. .

x3

the eccentnclty of the ellipse. 3176. Yl (x) ==:3 'Y2 (x)


x7

2x JJ

x2

XIS

XS

x3

x7

x3

== 3" +. 63 ' Y3 (x) -= ""3 -tx4

3x 2

+ 63 +2079 + 59535 3177. Yl (x)==2- x + 1, Y2(X)==6 + 2 -x + 1, !/:s(x) =-:-- 12x 3x


x'
7x
-"6-l-2- x + 1; Zt(x)=-:3x-2, z2(x)-==1f-2x2+3x-2, z3(x)-==T3

x3
x3
XS
-2x2 +3x-2. 3178. Yl(X)=X, Y2(X)=X-tf' Y3(x)~x-6+1~0
31 79. Y (1) == 3. 36 . 3180.
Y (2) == 0 .80 . 3181 . Y (1) === 3 . 72; Z ( 1) == 2 . 72
3182. Y:::: 1.80. 3183. 3.15. 3184. 0.14. 3185. Y (0.5) --:-:3 15; z (0 5) --.: - 3 15.
3186. Y (0.5) = 0 . 55; Z (0 5) = - 0 . 18. 3 t 87. 1. 16. 3188. 0 87. 3189. x (It) - 3. 58~
x' (rt) --=0.79.
3190. 429+ 1739 cos x-l0:37 SUI x-6:321 ros 2x -t- 126:J Sill 2x- ]242 cos 3x-33 S111 3x.
3191. 6 49-1 96 cos x -t- 2.14 SUI x-l.6H Cos 2x -t-t- O. 53 ~in 2x - 1. 13 cos 3x 0.04 sin 3x. 3192. 0.960 0.851 cos x 0.915 Sill x -I0.542 cos 2x 0.620 sin 2x 0.271 cos 3x -I- 0.100 sin 3x. 3193. a) () 608 ~In x 1+ 0.076 sin 2x 0.022 sin 3x; b) 0.338 0.414 cos x 0.111 cos 2x 1- 0.056 cos 3x.

+
+

+.

APPENDIX

I. Greek Alphabet
Iota-It
Kappa-}(x

Alpha-An

Beta- Bri
Ganll11a - ry
Delta-8b

Rho-PQ
Slgnla -~O'
Tau- Tt
Upsl1on- r"
Phi -(Dcp
Chi-Xx
Psi - 'l'~'
Oll1ega-Qw

LaInbda-AA
Mu-l\f~l

Nu-Nv

Epsilon-Ep,

Xi-3~

Zeta -Z~
Eta - II.,

Ornicron-Oo
Pi-Iln

Theta-eO

II. Some Constants


x

Quantity
1

log x

QU3nt It)

log x

0.79818

I
e
el

0.36788
7.3890fi

1.57080

0.19612

Ve

I.G48i2

O.R6859
0.21715

0.78540

I .89509

V-;

1 39561

0.14-176

--

0.31831

t .50285

1\1 = tog e

0.4342)

t .6:)778

n2

9.86n60

0.99130

-=lnl0

1
M

2.30258

O.~3G222

Vn

1.77245
1.46459
2.7182R

0.24857
0.16572
0.43429

1 radian
arc 1

5717'45"
0.01745
9.81

2.24188
0.99167

:t

2:t
j(

"2n
4
1
rt

V;
e

14159

0.49713

G.28318

:~

r 56571

476

Appendix

III. Inverse Quantities, Powers, Roots, Logarithms

xl

1.0 1.000
1. 1 0.909
1.2 0.833
1.3 0.769
1.4 0.714
1.5 0.667
1.6 0.625
1.7 0.588
1.8 0.556
1.9 0.526
2.0 0.500
2. I 0.476
2.2 0.454
2.3 0.435
2.4 0.417
2.5 0.400
2.6 0.385
2.7 0.370
2.8 0.357
2.9 0.345
3.0 0.333
3.1 0.323
3.2 0.312
3.3 0.303
3.4 0.294
3.5 0.286
3.6 0.278
3.7 0.270
3.8 0.263
3.9 0.256
4.0 0.250
4.1 0.244
4.2 0.238
4.3 0.233
4.4 0.227
4.5 0.222
4.6 0.217
4.7 0.213
4.8 0.208
4.9 0.204
5.0 0.200
5.1 0.196
5.2 0.192
5.3 0.189
5.4 0.185

X2

X3

1.000
1.210
1.440
1.690
1.960
2.250
2.560
2.890
3.240
3.610
4.000
4.410
4.840
5.290
5.760
6.250
6.760
7.290
7.840
8.410
9.000
9.610
10.24
10.89
11.56
12.25
12.96
13.69
14.44
15.21
16.00
16.81
17.64
18.49
19.36
20.25
21.16
22.09
23.04
24.01
25.00
26.01
27.'()4
28.09
29.16

1.000
1.331
1.728
2.197
2.744
3.375
4.096
4.913
5.832
6.859
8.000
9.261
10.65
12.17
13.82
15.62
17.58
19.68
21.95
24.39
27.00
29.79
32.77
35.94
*39.30
42.88
46.66
50.65
54.87
59.32
64.00
68.92
74.09
79.51
85.18
91.12
97.34
103.8
110.6
117.6
125.0
132.7
140.6
148.9
157.5

I I Iv: IVloxl V-IIOgX I


Vx

VlO x

1.000
1.049
1.095
1.140
1.183
1.225
1.265
1.304
1.342
1 378
1.414
1.449
1.483
1.517
1.549
1.581
1.612
1.643
1.673
1.703
1.732
1.761
1.789
1.817
1.844
1.871
1.897
1.924
1.949
1.975
2.000
2.025

3.162
3.317
3.464
3.606
3.742
3.873
4.000
4.123
4.243
4.339
4.472
4.583
4.690
4.796
4.899
5.000
5.099
5.196
5.292
5.385
5.477
5.568
5.657
5.745
5.831
5.916
6.000
6.083
6.164
6.245
6.325
6.403
6.481
6.557
6.633
6.708
6.782
6.8G6
6.928
7.000
7.071
7.141
7.211
7.280
7.348

2.~9

2.0 4
2.098
2.121
2.145
2.168
2.191
~!. 214
2.236
2.258
2.280
2 302
2.324

lOOx

1.000
1.032
1.063
1.091
1.119
1.145
1.170
1 193
1.216
1.239
1.260
1.281
1.301
1.320
1.339
1.357
1.375
1.392
1.409
1.426
1.442
1.458
1.474
1.489
1.504
1.518
1.533
1.547
1.560
1.574
1.587
1.601
1.613
1.626
1.639
1.651
1.663
1.675
1.687
1.698
1.710
1.721
1.732
1.744
1.754

2.154
2 224
2.289
2.351
2.410
2.466
2.520
2.571
2.621
2.668
2.714
2.759
2 802
2.844
2.884
2.Q24
2.962
3.000
3.037
3.072
3.107
3.141
3.175
3.208
3.240
3.271
3.302
3.332
3.362
3.391
3.420
3.448
3.476
3.503
3.530
3.557
3.583
3.609
3.634
3.659
3.684
3.708
3.733
3.756
3.780

4.642
4.791
4.932
5.066
5.192
5.313
5.429
5.540
5.646
5.749
5.848
5.944
6.037
6.127
6.214
6.300
6.383
6.463
6.542
6.619
6.694
6.768
6.840
6.910
6.98e
7.047
7.114
7.179
7.243
7.306
7.368
7.429
7.489
7.548
7.606
7.663
7.719
7.775
7.830
7.884
7.937
7.990
8.041
8.093
8.143

(t.nantlssas)

0000
0414
0792
1139
1461
1761
2041
2304
2553
2788
30rv
3222
3424
3617
3802
3979
4150
4314
4472
4624
4771
4914
5051
5185
5315
5441
5563
5682
5798
5911
6021
6128
6232
6335
6435
6532
6628
6721
6812
6902
6990
7076
7160
7243
7324

In x

0.0000
0.0953
0.1823
0.2624
0.3365
0.4~55

0.4 00
0.5306
o 5878
0.6419
0.6931
0.7419
0.7885
0.8329
0.8755
0.9163
0.9555
0.9933
1.0296
1 0647
1 0986
1.1314
1.1632
1.1939
1.2238
1.2528
1..2809
1.3083
1.3350
1. 3610
1.3863
1.4110
1 4351
1.4586
1. 4816
1.5041
1.5261
1.5476
1.5686
1.5892
1.6094
1.6292
1.6487
1.6677
1.6854

Appendix

471
Continued

5.5
56
5.7
5.8
5.9
6.0
6.1
6.2
6.3
6 4
6.5
66
6 7
68
69
7.0
7.1
7.2
7.3
7.4
7.5
7.6
7.7
7.8
7.9

S.O
8.1
8.2

8.3

8 4
8.5
8.G

8.7
8.8
89

1+1
o 182
0.179
0.175
0.172
o 169
0.167
0.164
o 161
o 159
0.156
~,~~ 154
0.151
0.149
0.147
o 145
0.143
0.141
0.139
o 137
0.135
0.133
0.132
o 1:\0
o 128
o 127
o 125
o 12~
0.122
0.120
o 119
0.118
0.116
0.115
0.114
o 112

9.0 O. III

9.1 0.110
9.2 0.109
9.:3 0.108
9.4 0.106
9.5 0.105
9.6 0.104
9.7 0.103
9.8 0.102
9.9 0.101
10.0 0.100

t''J

x"

30 25 166 4
31 36 175.6
32 49 185 2
33.64 195.1
34 81 205.4
36.00 216.0
37 21 227.0
38.44 2a8 3
39.69 250 0
40 96 262.1
42.23 274 6
43.56 287 5
44 89 300 8
46 24 314 4
47 61 ~i28 5
49.00 343 0
50.41 357.9
51.84 373.2
53.29 389.0
54 76 405.2
56.25 421.9
57.76 439 0
59 29 456 5
60.84 474 6
62.41 49~3. 0
64.00 512 0
6G.61 531.4
67.24 551. 4
68.89 571.8
70.56 592.7
72.23 614.1
7:3.96 636.1
75.69 658.5
77.44 681.5
79 21 705.0
81.00 729.0
82.81 753.6
84.64 778.7
86.49 804.4
88.36 830 6
90.25 857.4
92.1~ 884.7
94.09 912 7
96.04 941.2
98.01 970.3
100.00 1000.0

v"x

2.345
2 366
2 387
2 408
2 429
2.449
2.470
2 490
2.510
2.530
2.550
2 569
2 588
2 608
2.627
2.646

2 665
2.683
2.702
2.720
2.739
2 757
2 775
2.793
2.811
2.828
2 846
2.864
2.881
2.898
2.915
2 933
2.950
2.966
2 983
3.000
3.017
3.033
3.050
3 066
3.082
3.098
3.' 14
3.130
3.146
3.162

I IV-lv-/v-/ I
VTOX

x'

7.416
7 483
7 550
7 616
7.681
7 746
7.810
7 874
7.937
8.000
8.062
8.124
8.185
8.246
8 307
8.367
8 426
8.485
8.544
8 602
8.660
8.718
8.775
8.832
8.888
8.944
9.000
9 055
9.110
9.165
9.220
9.274
9.a27
9.381
9.431
9.487
9.539
9.592
9.644
9.695
9.747
9.798
9.849
9.899
9.950
10.000

1.765 3.803
1 776 3.826
1.786 3.849
1.797 3.871
1.807 3 893
1.817 3.915
I 827 3.936
1.837 3.958
1.847 3.979
1.857 4.000
1.866 4 021
I 876 4.041
I 885 4 062
1.895 4.08~
1.904 4.102
1.91a 4.121
1.922 4.141
1.931 4.160
1.940 4.179
1.949 4.198
1.957 4.217
1.966 4.236
1.975 4.254
1.983 4 273
I 992 4 291
2.000 4.309
2 008 4.327
2 017 4.344
2 025 4.362
2.033 4 380
2.041 4.397
2 049 4.414
2.057 4.431
2.065 4.448
2 072 4.465
2.080 4.481
2.088 4.498
2.095 4.514
2.103 4.531
2.110 4.547
2.118 4.563
2.125 4.579
2.133 4.595
2.140 4.610
2.147 4.626
2.154 ,4.642

IOgX
lOOx (~a n-

lOx

t ls~as)

In x

8.193 7404 1.7047


8.243 7482 1 7228
8.291 7559 1.7405
8.340 7634 1.7579
8 387 7709 1.7750
8.434 7782 1.7918
8 481 7853 1.8083
8 527 7924 1.8245
8 57:3 7993 1.8405
8 618 8062 1.8563
8.662 8129 1.8718
8.707 8195 1.8871
8 750 8261 I 9021
8 794 8:325 1 9169
8.837 8388 I 9315
8 879 8451 I 9459
8 921 8513 1.9601
8.963 8573 1.9741
9.004 8633 1.9879
9.045 8692 2.0015
9.086 8751 2.0149
9.126 8808 2.0281
9.166 8865 2.0412
9.205 8921 2.0541
9.244 8976 2.0669
9.283 9031 2.0794
9.322 9085 2'.0919
9 360 9138 ~.1041
9.398 9191 2.1163
9.435 9243 2.1282
9.473 9294 2.1401
9 510 9345 2.1518
9.546 9395 2.1633
9. 58~~ 9445 2.1748
9 619 9494 2.1861
9.655 9542 2 1972
9.691 9590 2.2083
9.726 9638 2.2192
9.761 9685 2.2300
9.796 9731 2 2407
9.830 9777 2.2513
9 865 ge23 2.2618
9 899 9868 2 2721
9.933 9912 2.2824
9.967 9956 2.2925
10.000 0000 2.3026

478
Appendix
---------------=-.....:..-_---------_._-

IV. Trigonometric Functions

,,0

x
(fad lans)

0.0000
0.0175
0.0349
0.0524
0.0698

1
2

3
4
5
6
7
8

41ln .\

0.0000
0.0175
0.0349
o .052:~
0.0698
0.0872
0.1045
0.1219
0.1392
0.156t
0.1736
0.1908
0.2079
0.2250
0.2419
0.2588
0.2756
0.2924
0.3090
0.32G6

0.087~

10
11
12
13
14
15
16
17
18
19
20
21
22
23
24
25
26

27

28
29
30
31
32
33
34
35
36
37
38
39
40
41

42

43
44
45

cos x

0.0000
0.0175
0.0349
0.0524

O.176~

0.1944
0.2126
0.2309
0.2493
0.2679
0.2867
O. ~~057
0.3249
0.~44:3

0.3640
0.3839
0.4040
0.4245
0.4452
0.4663
o 4877
0.5095
0.5317
0.5543
0.5774
0.6009
0.6249

tan x

0.9~O5

0.89R8

0.8910
0.8829
0.8716
0.8660
0.8G72
0.8480
0.8387
o 8290
0.81Q2
0.8090
O. 7~)86

0.7880
0.7771
0.7660
0.7547
0.7431
0.7314
0.7193
0.7071

~in

I
1.5708
1.55:33
1 5:~5q
1. 5184
l.fiOl0
1.48:35
1.4661
1.4-lR6
1 4:H2
1 41:37
I ~963
l. 37~8 .
I ~61~

0.9135
0.906:3

1.5399
1.4826
1.4281
1.3764
1.3270
1.2799
1.2349
1.1918
1.1504
1.1106
1.0724
1.0355
1.0000

0.6745
0.7002
0.7265
0.7536
0.7813
0.8098
0.8391
0.8693
0.9004
o 9325
0.9657
1.0000

cos x

1.0000
0.9998
0.9994
0.9986
0.9<)76
0.9962
O.994S
0.9925
0.9903
0.9877
0.9848
0.9816
o 9781
0.9744
0.9703
0.9639
0.961:3
O. 956;~
0.9511
O.945[)
o 9397
O. 9;t~6
0.9272

I.GOO:3

o 64q4

rot x

00

0.0875
0.1051
0.1228
0.1405
o 1584

cot x

57 29
28.64
19.08
14 30
11.43
9.514
8.144
7.115
6 314
5.671
5.145
4.703
4. :~~~1
4.011
3.732
3.487
3.271
3.078
2.904
2.747
2.605
2.475
2.356
2.246
2.145
2.050
1.963
I.R81
1.804
1 732
1.6643

0.0()9~

0.1047
0.1222
0.1396
0.1571
0.1745
0.1920
0.2094
0.2269
O. 244'~
0.2618
0.2793
0.2967
0.3142
0.3316
0.3491
0.:~420
0.3665
0.3584
0.3840
0.3746
0.4014
0.3907
0.4189
0.4067
0.4363
0.4226
0.4538 .. o 4384
0.4712
0.4540
0.4887
0.4695
0.5061
o 484S
0.52:36
0.5000
0.5411
0.5150
0.5585
0.5299
0.5760
0.5446
0.5934
0.5592
0.6109
0.5736
0.6283
0.5878
0.6458
0.6018
0.6632
0.6157
0.6807
0.6293
0.0981
0.6428
0.7156
0.6561
0.7330
0.6691
0.7505
0.6820
0.7679
0.6947
0.7854
0.7071

tan

90
89
88
87
86
85
84
83
82

RI
80
I

79
78

1.34;~9

77

1.3263
1.3090
1.2915
1.2741
I 2566
I 2:392
I 2217
1.2041
1.1868
1.1691
1.1519
I 1343
1.1170
1.0996
1.0821
l.n647
1.0472
1.0297
I .OI2:l
0.9948
0.9774
0.9599
0.9423
0.9250
0.9076
0.8901
0.8727
0.8552
0.8378
0.8203
0.8029
0.7854

76

(rad~al") I

75
74
73
72
71
70
69
68
67
66
6;)
64
():~

62
61
60
59
58
57
56
55

54
53

52
S!
50

49
48
47
46

45

Appendix

479

V. Exponential, Hyperbolic and Trigonometric Functions


e"'C

o 0
0.1

e-;J;

sinh x

cosh x

tanh x

sin x

cos x

0.4

1.0000
1.1052
I 2214
1.:3499
I 4918

1 0000
o 9048
o 8187
0.7408
0.6703

0.0000
0.1002
0.201:3
0.3045
0.4108

1.0000
1.0050
I 0201
1.0453
1.0811

0.0000
0.0997
0.1974
0.2913
0.3799

0.0000
0.0998
0.1987
0.29n3
0.3894

1.0000
0.9950
0.9801
0.9553
0.9211

o 5
o6
o 7
0.8
o 9.

1 6487
1.8221
2.0138
2.22;)3
2 4596

0.6063
0.5488
0.4966
o 4493
0.4066

o 5211
o 6367
o 7586
o 8881
1.0263

1.1276
1.1853
1.2552
1.3:374
1.4331

0.4621
0.5370
0.6044
0.6640
0.7163

0.4794
0.5646
0.6442
0.7174
0.7833

0.8776
0.8253
0.7648
0.6967
0.6216

0.3679

o 8415

12
1 :~
1 t

2. 718:~
3.0042
:l 3201
:~ 66 3
4 O:l52

0.9320
o 9636
0.9854

0.5403
0.4536
0.3624
0.2675
0.1700

15

4 4817

1.6

t.9':>:lO

-0.0292
-0.1288
-0 2272
-0.3233

0.2
o :l

I 0
1.1

17
I H

1~
2 ()
~ 1
22
2

:~

o :3329

() 22al

~.12n3

2.3524
2 5775
2 8283
:3.1075
3 4177

0.9051
o 9217
0.9354
0.9468

o 9:)62

0.9973
0.9996
0.9917
0.9738
n 9463

:~ 7622
4 .144:~

o. ~640
0.9704
0.9757
D.U8ul
0.98:37

0.9093 -0.4161
O.86~~ -0 5048
o 8085 -0.5885
o 74.:17 -0.6663
(1 6753 -G 7374

0.9866

0.5985 -0.8011
o 5155 -0.8569
0.4274 -0.9041
O. :3:350 -0.9422
0.23H2 -0.9710

o 2019

I;~~):~
122:>
IIn8

() 10n:~
o 0907

24

11

2 ;)
26
27
2.8

1~ 182.)
I:J. 4().~7
11.8797
16 4146
18.1741

0.0821
o 0743
o 06i2

2 3756
2.6456
2.9422
:~.2682

:~

6269

4 0219
4 4571
4.9370
5.46G2

4 567<)

S.0372
5.5569

0.0550

6.0502
6 6(}47
7.4063
8.1919
9.0596

6.7690
7. 4i~l5
8 2f)27
9.1146

0.991U
0.9926
0.9940

3 4

20.085!l
22.1979
24.5325
27.1126
29.9641

0.0498
0.0450
0.0408
0.03()9
0.0334

10.0179
11.0764
12.2459
I~l. 537n
14.9654

10.0677
11.1215
12.23G6
13.5748
14.9a87

0.9950
0.9959
0.9967
0.9973
0.9978

3.5

33.1154

0.0302

16.5426

16 05728\ 0.9982

2 9

:l.O

3.1
3.2
3.3

o 8912

o 8337
0.8617
o 8834

()
()
()

O~;~~

0.8005

1.8107
1.9709
2.1309

7 ~~8~} I
H 1662
()~.so

o 7616

1.5095
1.6981
1. 9043

o 1827
() 1653
o 11<)6

1.5431
1.6685

0.3012
o 2725
o 2166

:>.47:39
6.0496
h 68fi9

n 974~

1.1752
1.3356

o 0608

6.1.12:~

o ~)890

0.1411
0.0416
-0.0584
-0 1577
-0.2555

0007071

-0.9900
-0.9991
-0.9983
-0.9875
-0.9668

--0.3508 -0.9365

480

Appendix

VI. Some Curves (for Reference)

y
y

X
3. Rectangular

2. Cubic parabola,
y == Xl.

1. Parabola.
y=x 2

hyperbola,

Y:_-=x
"''"'
y

-1 0

4. Graph of a fractional

function,
1
y=-X2

-1

1
5. The witch of Agncsi,
1
y;;:. 1

+x

o
6. Parabola (upper
branch),

y=x.

7. Cubic parabola,
11=

a -x.
V

481

Appendix

Bb Sem icubical
parabola.
f x=t'l.
y2=X' or
'

\ y=t'.

Sa. Neile's parabola.


Y :..::

t'

or

x~

is

X: -

t-.

rtf

-f

9. Si llC curve and cosi ne curve,


y - -= SA:l x ;]nd 'I = cos x.

10. Tangent curve and cotangent curve,


y=fanx and y=cotx.

Appendix

11. Gra phs of the f unct ions


y -= sec x and y = cosec A.

y=arc sin :r

12. Graphs of the inverse trigonometric functions


II =

arc sin x and y = arc cos x.

""

Appendix

483

y=arc

-----..,...----'-......,;;.-

tan:l

------0------1[

13. Gra phs of the Inv~rse trigonoInetric functions


y =- arc tan x and y = arc cot x.

-1

14. Graphs of the exponential functions


y=ex and y=e- x .

486

Appendix

x
x

25. Bernoull i's lenlniscate,


(x 2 _I- y2r~ = a 2 (x 2_ y2)
or r2 == a2 cos 2<p.

24. Strophoi d,

a+x
y 2 =x2 -.
a-x

o
26. Cycloid,
x=a (t -sin t),
{ y=a (1- cos t).

28. Cardioid,

r=a (1 +cos <pl.

27. IIypocyclold (astroid),


x=a cos 3 t,
{ y = a sin a t
2

or

.\ a

+ 11 a = a 3-.
y

29. Evolvent (invol ute) of the circle


X = a (cos t + t sin t) t
{ Y =a (sin t -t cos l).

481

Appendix

x
tI--................_ - - " " ' _........._~

X
.30. Spiral of Archinledcs,

31. Hyperbolic spiral,


a
r=-.
q>

r =aq>.

32. Logdrithl11ic spiral,


(-=

33. Th ree-Iea fed rose,

ea'l'.

r =a sin 3tp.

34. Four-leafed rose,


r = a sin 2<p.

INDEX

Absolute error 367


Absolute value
of a real nUlnber 11
Absolutely convergent series 296, 297
Acceleration vector 236
Adams' formula 390
Adarns' method 389, 390, 392
Agnesi
Witch of 18, 156,480
Algebraic functions 48
Angle bet\veen two surfaces, 219
Angle of contingence 102, 243
Angle of contingence of second kind
243
Antiderivative 140, 141
generalized 143
Approximate numbers 367
addition of 368
division of 368
rnul ti plicat ion of 368
powers of 368
roots of 368
subtraction of 368
.Approxil11at ion
successi ve 377, 385
J\rc length of a curve 158-161
J\rc length of a spacE' curve 234
.Archimedes
spiral of 20, 65, 66, 105, 487
.\rea in polar coordinates 155, 256
Area in rectangular coordinates 153,
256
/\rea of a pI ane region 256
Area of a surf ace 166168, 259
Argument 11
Astroid 20, 63, 105, 486
Asymptot~

93

Ifft horizontal 94
l~ft inclined 94
right horizontal 93
right inclined 93
vertical 93

Bending point 84
Bernoulli's equation 333
Bernoulli's
lemniscate 155,
Beta-function 146, 150
B inorm al 238
Boundary conditions 363

Branch of a hyperbol a 20, 480


Broken-line Illethod
Euler's 326

c
Ca rdioi d 20, 105, 486
Catenary 104, 105, 484
Catenoid 168
Cauchy's integral test 295
Cauchy's test 293, 295
Cauchy's theorcln 75, 326
Cavalieri's "leInon" 165
Centrc of curvature 103
Change of variable 211-217
ina de fi nit e in t eg raI 146
in a double integral 252-254
in an indefinite Integral 113
Characteristic equation ~~56
Characteristic points 96
Chebyshev's conditions 127
Chord method 376
Circle 20, 104
of convergence 306
of curvature 103
osculating 103
Circulation of a vector 289
Cissoid 232
of Diocles 18, 485
Clairaut's equation 339
Closed interval 11
Coefficients
Fourier 318, 393, 394
Comparison test 143, 293, 294
Composite function 12, 49

486

Index
Coneave down 91
Concave up 91
Concavity
direction of 91
Conchoid 232
Condition
Li pschi t7. 385
Conditions
boundary 363
Chebyshev's 127
Dirichlet 318, 319
initial 323, 363
Conditional extrelTIUm 223225
Conditionally (not absolutely)
convergent series 296
Contingence
angle of 102, 243
Continuity of functions 36
Continuous function 36
properdes of 38
Convergence
circle of 306
interval of 305
radius of 305
region of 304
unifonn 306
Convergent i111prOper integral 143,
270

Convergent serie~ 293


Coordinates
of centre of gravity 170
~en('rali7ed polar 255
Correct deci n1 aI pI aces i 11 a hroad
sense 367
Correct decilnal places in
a narro\v sense 367
Cosine curve 481
Cotangent curve 481
Coupling equation 223
Cntical point of the second kind 92
en tical points 84
Cuhic parabola 17, 105, 234, 480
Curl of a vector field 288
Curvature
centre of 103
circle of 10:~
of a curve 102, 242
radius of 102
second 243
Curve
cosine 481
cotangent 481
discriminant 232, 234
Gaussian 92
integral 322
logarithmic 484

489

probability 19,484
sine 481
tangent 481
Cusp 230
Cycloid 105, 106, 486
D
D' Alembert's test 295

Decreasing function 83
Definite integral 138
Del 288
Dependent variable 11
Derivative 43
left-hand 44
logarithmic 55
nth 67
right -hand 44
second 66
Derivative of a function
in a given direction 193
Derivative of functions
represented parametrically 57
Derivative of an implicit functicn 57
Derivative of an inverse fUllction 57
Derivative of the second order 66
Deriv at ives
of higher orders 66-69
one-si ded 43
table of 47
Descartes
foliuln of 20, 21, 232, 485
Deternlinant
functional 264
Determining coefficients
first Inethod of 122
second method of 122
Diagonal table 389
Difference of two convergent
series 298
Differential
of a 11 arc 10 1, 234
first-order 71
higher-order 198
pri nci pal properties of 72
second 198
second -order 72
total, integration of 202-204
Differential equation 322
homogeneous linear 349
inhotnogeneous linear 349
Differential equations
first-order 324
forming 329
higher~rder 345
linear 349, 351

Index

490

Differential equations of higher powers


first-order 337
Oifferenti als
met hod of 343
of third and higher orders 72
Differentiating a composite function
47

Differentiation 43
of implicit functions 205-208
tabular 46
Diocles
cissoid of 18, 485
Direction of concavity 91
Direction field 325
Dirichlet
conditions 318, 319
function 40
series 295, 296
theorem 318
Discontinuity 37
of the first kind 37
infinite 38
removable 37
of the second kind 38
Discontinuous function 270
Discriminant 222
Dicriminant curve 232, 234
Divergence of a vector field 288
Divergent improper integral 143, 270
Di vergent seri es 293, 294
Domain 11
Domain of definition 11
Double integral 246
in curvilinear coordinatps 253
in polar coordinates 252
in rectangular coordinates 246
Double point 230

coupling 223
difTerenti al 322
Euler's 357
exact differential 335
first-order differential 324
homogeneous 330, 351, 356
honl0geneous linear difTerential 332,
349

inhomogeneous 349, 351,


Lagrange' s 339
Laplace's 289, 291
linear 332
of a normal 60, 218
of a tangent 60
of a tangent plane 218
with variables separable
Equivalent functions 33
Error
absolute 367
limiting absolute 367
limiting relative 367
relative 367
Euler integral 146
Euler-Poisson inte~ral 272
Euler's broken-line nlethod
Euler's equation 357
Even function 13
Evolute of a curve 103
Evolvent of a circle 486
Evolvent of a curve 104
Exact diITerential equation
Exponential functions 49,
EX\remal point 84
Extremum
conditional 223-225
of a functi 011 83, 83, 222

356

327, 328

326

335
55, 483

F
E

Elimination
met hod of 359
Ellipse 18, 20, 104, 485
Energy
kinetic 174
Envelope
equations of 232
of a family of plane curves 232
E picycloi d 283
Equal effects
principle of 369
Equation
Bernoulli '5 333
charactrristic 356
Clairaut's 339

Factor
integrating 335
Field
direction fiel d 325
nonstati onary seal ar or vector 288
potenti al vector 289
scalar 288
solenoidal vector 289
Field (cont)
stationary scalar or vector 288
vector 288
Field theory 288-292
First-order differential 71
First-order differential equations 324
Flow lines 288
FI ux of a vector field 288
Folium of Descartes 20, 21, 232, 485

Index
Force lines 288
FOrl11
Lagrange's 311
Forn1ula
Adams' 390
Green's 276, 281, 282
Lagrange's 145
Lagrange's interpolation 374
Leibniz 67
t.\aclaurin's 77, 220
Nc\vton-Leibniz 140, 141, ~75
Newton's interpolation 372
()strogradsky-Gauss 286-~88
parabul1c 382
Sln1p~on's

Lagr~lnge

39~3,

:~8

223, 224

sin~lc-vallled

394

G
Gamma-function 146, 150
Gaussian curve 92
General integral 322
General solution 359
General solution (of an equation) 323
General term 294
Generalized antiderivative 143
Generalized polar coordinates 255
Geometric progression 293, 294
Gradient of a field 288
Gradient of a function 194, 195
Graph of a function 12
Greatest value 85, 225,227
Green's forrnul a 276, 281, 282
Guldin's theorenls 171
H

Illultiple-valued 11
periodic 14
11

vector 235
F11nctional detern1inant 264
Functional series 304
Functions
al~ebraic

logarit hm ic 49
.
transcendental, integration of 135
trigonometric 48
trigonometric, integrating 128, 129
Fundalnental system of solutions 349

382-384

Stokes' 285, 286, 289


T a\' I or's 77, 220
tra"'pezoidal 382
Forn1111clc;
reduction 130, 135
Fourier'" c(_'efficlents 318,
Fourier series 318, 319
Four-leafed rose 487
Fraction
proper rational 121
I'unctlon 11
COllI posite 12. 49
contInuous 36
cont inUOll':>, properties of
decreasing 83
DIrichlet 40
dl'icont Inuous 270
('veil 13
(If C1 function 12
inlpl1clt 12
increasi n~ 83

491

48

equivalent 33
('\ ponenti al 49, 55, 483
hy perbol ic 49, 484
hyperbolic, integration of 133
inverse 12
Functions (cont)
inverse circular 48
inv('rse hyperbolic 49
inverse trigonometric 482, 483
Ii nearl y dependent 349
linearly independent 349

Han11ltonian operator 288


Harn10nic series 294, 296, 297
HIgher-order dIfferential 198
l-ligher-order dIfferential equation'.'
Higher-order partial derivative
Hodograph of a vector 235
H0l110geneous equations 330, 351,
l-Ion10geneous Iinear difTerent Ial
equation 332. 349
llyperbola 17, 18,20,485
rectangular 480
l-Iyperbolic functions 49, 484
integration of 133
Hyperbolic spiral 20, 105, 487
Hyperbolic substitutions 114, 116,
Ilypocycloid 283, 486

345
197
356

133

I 111 pI i cit fun c t i on 12

Inlproper integral
conver~ent

270

divergent 270
Improper Jllultiple integrals 269, 270
Incoillplete Fourier series 318, 319
Increasing function 83
Increment of an arguI11ent 42
Incren1ent of a function 42
Independent variClble 11
Indetenn inate forll1s
evaluating 78, 79

492

Index

Infinite discontinuities 38
Infinitely large quantities 33
Infinitely small quantities 33
Infinites 33
Infinitesimals 33
of higher order 33
of order n 33
of the same order 33
Inflection
points of 91
Inhomogeneous equation 349. 351. 356
Inhomogeneous linear differential
equation 349
Initial conditions 323. 363
Integral 322
convergent improper 143
definite 138
divergent improper 143
double 246
Euler 146
Euler-Poisson 272
general 322
improper multiple 269, 270
line 273-278
part icul ar 322
probability 144
singular 337
surface 284-286
triple 262
Integral curve 322
Integral sum 138
Integrating factor 335
Integration
basic rules of 107
under the differential sign 109
direct 107
by parts 116, 117, 149
path of 273, 274, 280
region of 246-248
by substitution 113
Integration of differential equation
by means of power series 361, 362
Integration of functions
numerical 382, 383
Inte~ration of ordinary differential
equation
nurnerical 384-393
Integration of total difTerentials 202204

Integration of transcendental functions 135


Jnterpolation
of functions 372-374
inverse 373
linear 13. 372
quadratic 372

Interpolation formula
Lagrange's 374
Newton's 372
Interval
of calculations 382
closed II
of convergence 305
of monotonicity 83
I nterv al (cont)
open 11
table interval 372
I nverse circular functions 48
I nverse functions 12
Inverse hyperbolic functions 49
Inverse interpolation 373
I nverse trigonometric functions 482,
483

Involute of a circle 20, 106, 486


Involute of a curve 104
Isoclines 325
Isolated point 230
,
Iterative method 377, 378, 380
J

Jacobian 253, 264

K
Kinetic energy 174
L

Lagrange's equation 339


Lagrange's form 311
Lagrange's fornlula 145
Lagrange's function 223, 224
Lagrange's interpolation fornlllla 374
Lagrange's theOrel11 75
Laplace equation 289, 291
Laplace transformation 271
Laplacian operator 289
Lamina
coordinates of the centre of gravity
of a. 261
mass and static nlornents of a 260
moments of inertia of a 261
Least value 85
Left-hand derivative 44
Left horizontal asymptote 94
Left inclined asymptote 94
Leibniz rule 67, 269
Leibniz test 296, 297
Lemniscate 20, 105, 232
Bernoulli's 155, 486
Level surf aces 288
L'Hospital-Bernoulli rule 78.. 82

493

Index,
Lima~on

of successive approxi.mation 38f,

Pascal's 158
Limit of a function 22
Limit on the left 22
Limit on the right 22
Limit of a sequence 22
Limiting absolute error 367
Lilniting relative error 367
Lilnits
one-sided 22
Line
straight 17, 20
Line Integral
application of 276, 28:3
of the first type 273, 274, 277, 278
Line integral of the second type 274,
275, 278-281

Linear differential equations 349, 351


Linear equation 332
Linear interpolation 372
of a hinrtion 13
Linearly dependent functions 349
Linearly independent functions 349
Lines
flow 288
force 288
vector 288
Li pschitz condition 385
Logarithlllic curve 484
Logarithnlic derivative 55
Logarithlllic functions 49
Logarithnlic spiral 20, 21, 105, 106,
487
M

l\\aclaurin's formu] a 77, 220


Maclaurin's series 311, 31:3
}'v\axiUll1I11 of a fUllction 84, 222
MaxinlU111 point
~\ean value of a function 151
1\\ean-valuC' theorellls 75, 150
j\\ean rate of change 42
~\\et hod
l\dall1s' 389, 390, 392
chord lnethod 376
of differentials 343
of elinlinat ion 359
Method (cont)
Euler's broken-line 326
iterative 377, 378, 380
Milne's 386, 387, 390
Newton's 377, 379
Ostrogradsky 123, 125
Picard's 384, 385
reduction 123
Runge-Kutta 385-387, 390

385, 389

of tangents 377
of undetermined coefficients 121, 351
of variation of parameters 332, 349,
352

Minimum of a function 84, 222


Mininlum point 84
Mixed partial derivative 197
Moment
of inertia 169
static 168
Monotonicity
intervals of 8a
Multiple-valued function 11
Multi pi icities
root 121
N
nth derivative 67
Nabla 288
Napier's number 28
Natural trlhedron ~3B
Necessary condition for convergence
29:3
Necessary condition for an extrCJ11Unl
222

Newton
trident of 18
Ne\vton-Leibniz fOrillula 140, 141, 275
Newton's interpolation formula 372
Newton's J11ethod 377, 379
Ne\\ton's serpentine 18
Niele's par abo! a 18, 234. 48]
Node 230
Nonstationary scalar or vector field 288
Nonnal 217
to a curve 60
equations of 218
principal 238
Normal plClne 238
NtlInber
Napier's 28
real 11
Number series 293
NUlllerical integration of functions 382,
383

NUlnerical integration of ordinary


differential equations 384-393

o
One-sided derivatives 43
One-sided lilnits 22
Open interval 11

Index

494

Operator
Hamiltonian 288
Laplacian 289
Order of smallness 35
Orthagonal surfaces 219
Orthagonal trajectories 328
Osculating circle 103
Osculating plane 238
Ostrogradsky-Gauss formula 286-288
Ostrogradsky-Gauss theorem 291
Ostrogradsky method 123, 125
p

Parabola 17, 20, 104, 105, 480, 485


cubic 17, 105, 234
Niele's 18, 234, 481
safety 234
semicubical 18, 20, 234, 481
Parabolic formula 382
Parameters
variation of 332, 349, 352
Parametric representation of
a function 207
Partial derivative
hirheg-order 197
"mixed" 197
second 197
Partial sum 293
Particular integral 322
Particular solution 339
Pascal's limac;on 158
Path of integration 273, 274, 280
Period of a function 14
Periodic function 14
Picard's met hod 384, 385
Plane
normal 238
osculating 238
rect ifyi ng 238
tangent 217
Point
bending 84
critical (of the second kind) 92
of discontinuity 37
double 230
extrern aI 84
of inflection 91
isol ated 230
nlaximum 84
minimum 84
singular 230
stationary 196
of tangency 217
Points
characteristic 96

critical 84
stationary 222, 225
Polar subnormal 61
Polar subtangent 61
Potential (of a field) 289
Potential vector field 289
Po\vcr series 305
Principal normal 238
Principle
of equal effects 369
Runge 383, 386
of superposition of solutions 353
Probability curve 19, 484
Probability integral 144
Product of t\VO convergent series 298
Progression
~eometric 293, 294
Proper rational fraction 121
Proportionate parts
rule of 376
Q

Quadratic interpolation 372


Quadratic trinonlial 118, 119,
Quantity
infinitely large 33
infinitely sl11a11 33

12:~

R
Radi liS of convergence 305
Radius of curvature 102, 243
Radius of second curvature 243
Radius of tor~ion 243
Rate of change
of a function 43
mean 42
Ratio (of a geornetric progreC\sion) 294
Real nunlbers J I
Rectangular hyperbola 480
Rectifying plane 238
Reduction fornlulas 130, 135, 150
Reduction nlcthod 123
Regi on of convergence ~04
Region of integration 246-248
Relative error 367
Remainder 311
Renlainder of a series 293, 304
Remainder term 311
Removab!e discontinuity 37
Right -hand derivative 44
Right horizontal asymptote 93
Right inclined aSYlnptote 93
Rolle's theorem 75
Root multi plici ties 121

495

Index
Rose
four-leafed 487
three-leafed 20, 487
Rotation (of a vector field) 288
Rule
Leibniz 67, 269
I' I-Iospit aI-Bernoulli 78-82
of proport j onat c parts 376
Runge-Kutta nlethod 385-387, 390
I~unge principle 383, 386
S
Safety parabola 234
Scalar field 288
Scheme
twelve-ordin~lte

393-39!i

Second curvature 243


Second derivative 66
Second dJJTerent ial 198
Second-orde~ difl'crential 72
Second partIal derivative 197
Segment of the nonnal 61
Segnlent of the polar BOrIllal 61
Segment of the polar tangent 61
Segment of a straight line 20
Segrnent of the tangent 61
Senlicircle 20
Senlicubical parabola 18, 20, 2.34, 4~1
Series
absolutely convergent 296. 297
with cornplex ternlS 297
condrtionally (not absolutely)
cOllvergent 296
convcrgent 293
Series (cont)
Dirichlet 295, 296
dl vergent 293, 294
Fourier 318. 319
functional 304
harnl0nic 2~}4, 296, 297
IIlC0l11plete Fourier :318, 319
Maclaurin's 311. 313
number series 293
operations on 297
power 305
Taylor's 311, 313
Serpentine
Newton's 18
Simpson's forrllula 382-384
Si oe curve 481
Single-valurd function 11
Singular integral 337
Singular point 230
Slope (of a tan~ent) 43
Stnallest value 225, 227

Solenoidal vector field 289


Solution (of an equation) 322
general 323, 359
part icu Iar 339
Spiral
of Archimedes 20, 65, 66, 105, 487
hyperbolic 20, 105, 487
logarithmic 20, 21, 105, 106, 487
Static moment 168
Stationary point 196, 222, 225
Stationary scalar or vector field 288
Stokes' formula 285, 286, 289
S t ra i g h t 11 ne 17, 20
Strophoid 157,232,234,486
Subnormal 61
polar 61
Substitutions
hyperbolic 114, 116, 133
trigonolTIctric 114, 115, 133
Subtangent 61
polar 61
Successive approxitnation 377, 385
rl1 et hod of 384, 385, 389
Sufficient conditions (for an extrclnum)

222
Sunl

Inte~ral 138
partial 293
of a seri es 293. 304
of two convergent serres 298
Su perposlti on of sol uti ons
principle of 353
Surfncc jnte~ral of the first type 284
Surface iI1t('~ral of the second type 284
Surface integrals 284-286
Surfaces
level 288
orthogonal 219
T

Table
d I ago naI tab I e 389
of standard integrals 107
Table interval 372
Tabular difTerentiation 46
Tacnode 230
Tangency
point of 217
Tangent 238
Tangent curve 481
Tangent plane 217
equation of 218
Tangents
met h od of 377
Taylor's formula 77, 220

496

Index

Taylor's series 311, 313


Term
general 294
remainder 311
Test
d' Alembert's 295
Cauchy's 293,295
Cauchy's integral 295
com parison 143, 293, 294
Leibniz 296, 297
Weierstrass' 306
Theorem
Cauchy's 75, 326
Dirichlet's 318
Theorem (cont)
Lagrange's 75
Ostrogradsky-Guuss 291
Rolle's 75
Theorems
Guldin's 171
mean-value 75, 150
Theory
field 288-292
Three-leafed rose 20, 487
Torsion 243
Tractrix 161
Trajec.tories
orthogonal 328
Transcendental funct ions
integrati on of 135
Transformation
Laplace 271
Trapezoidal fornlula 382
Trident of Newton 18
Trigonometric ~unctions 48
integrating 128, 129
Trigonometric substitutions 114 115
133
'
t
Trihedron
natural 238
Trinorrlial
quadratic 118, 119, 123
Triple integral 262
applications of 265, 268
change of variables in 263

computing volumes by means of 268


evaluating a 265
in rectangular coordinates 262
Trochoid 157
Twelve-ordinate schenle 393 395
U

Undetermined coefficients
method of 121, 351
Uniform convergence 306

v
Value
greatest 85, 225, 227
least 85
mean (of a function) 151, 252
smallest 225, 227
Variable
dependent 11
independent 11
Variables sep.arable
an equation \vith 327 328
Variation of paranleters '332. 349, 352
Vector
acceleration 236
of binormal 238
of principal nOrIllal 238
of tangent line 238
velocity 236
Vector field 288
Vector function 235
Vector lines 288
Velocity vector 236
Vertex of a curve 104
Vertical asynlptote 93
Vertices of a curve 104
Volume of a cylindroid 258
Volume of ~olids 161-166
W

Weierstrass' test 306


Witch of Agnesi 18, 156, 480
Work of a force 174, 276, 271

You might also like